加拿大家园论坛

求学选择个人谈及幼教,私人护理,社工,医师行政助理等热门行业介绍

原文链接:https://forum.iask.ca/threads/178547/

alexwei : 2008-06-08#1
关于公立和私人职业学校的课程和资质,可以到www.edu.gov.on.ca上去查, 所有经过政府审批的学校,以及私立职业学校的经过政府审批的diploma的课程在上面都可以查到。

首先一个概念,加拿大大专(College)跟我们国内的大专概念大不相同,更偏重于职业培训,注重的是技能和动手实践能力。大专的课程设置比较全面,合理,由浅入深易于接受。除了理论基础课程,更多的是职业培训课程、大学转学分课程和热门职业证书培训。同时,许多大专都与企业合作,甚至为大型企业设置特定专业,而且学院还经常调整专业,开设新兴的专业,尽量为学生就业创造机会,因此,大专生普遍比本科生好找工作,但如果想要做管理工作和科研工作,还得再读大学。

再说说多伦多的college, 多伦多的college分为两种,community college和private college,Community colleg就是俗称的公立大专,像Seneca, Geoge Brown,特点是学科非常全,基本上涵盖各个行业,学分可以被各个大学接受。有1年到3年的certifate, diploma和Degree的课程,也有时间更短的培训。可以在[URL http://www.edu.gov.on.ca/eng/general/list/college.html[/url]上找到On省所有Community 大专,一共24所。学生可以向政府申请OSAP贷款上学,但是大部分课程不能够申请EI学费上学,因为学制比较长,其实有的专业实际课时不多,但是由于Community College 假期和大学类似,时间较长,所以拖得比较长。政府可不愿意你拿着EI学费还休假,不过由于竞争的关系,他们也推出一些适合申请EI学费的专业。学生毕业后的就业率在70%到80%之间。

还有一类叫private college, 也可以叫private career college, 故名思义,是私人办的以职业培训为主的college, 一般规模不大,没有专门的校园。他们不可能开设像community college那么全的课程,所以都选择市场上的热门专业,并且可以较短的时间让学生莆眨玫iploma和Certificate(如果这个行业需要专门的certificate).由于规模不大,因此各个学校都有不同的侧重点,有比较强的专业,因此选择起来要困难一些。课程一般不超过一年,提供diploma,还有很多的课程是专门帮你通过一些证书考试的,有的学校学生可以申请OSAP,但是大部分学校不行。他们主要面向自费学习和EI学费学生。由于没有假期,上课时间灵活(一般每个专业都是滚动开课,所以学生在一年当中的任何一个月都可以入学)。这些学校,就业率是命根子,如果没有很高的就业率,就难招到学生,政府也很难批准EI学生入学。因此他们的专业一定是就业趋势很好的,老师一般是专业知识强并在行业有多年工作经验的,不仅负责教授学生技能,还要帮助学生实习和找工作的指导,因此把老师叫instructor,这是和community College教学非常不同的一点,Community教学还是正规的学校式教育,一个专业,7、8门课,每个老师各教一摊。大部分private college的一个班一个老师带到底,像师傅带徒弟,老师是老板雇的,因此压力更大些。学校也积极和各种社会资源雇主联系给学生提供co-op 和就业机会,因此就业率比community college高,好的能够达到90%以上。现在比较热门的,比如IT的一些专业,社工CSW,牙医助理,幼教助理ECA,药剂师PT,Computer Accounting,护工PSW,医生行政助理MOA等。但是private college 良莠不齐,首先大家可以去政府的网站
http://www.gov.on.ca/ont/portal/!ut/p/.cmd/cs/.ce/7_0_A/.s/7_0_252/_s.7_0_A/7_0_252/_l/en?docid=053263 去搜索,确保这个college是政府审批过的正在经营的college. 然后,看看这个学校的网站,看看有没有感兴趣的课程。

关于如何选择学校的问题,我想留到下次再写,希望能够逐渐补充些资料,给大家一个好的参考。也希望大家和我充分交流,提供不同意见。

6月10日续,如何选择college呢,论坛上这类东西可多了,我谈谈自己看法。我觉得核心就一个,快,时间越短越好。为什么呢,第一点,college 就是一个大专,你不管怎么学,拿什么文凭,到社会上也是一个具备初级技能的毕业生。虽说,seneca等在我们圈子名气响亮,可是并不入本地人眼。我有几个银行和政府的朋友,有本地的也有移民。我问他们关于college的问题,他们一致认为,college是非常初级的,只能从事初级工作。而且他们其实不是很清楚或者很在意college之间有什么差别。而到了大学这一级别,才开始注意学校之间的区别,这和国内差不多。第二点,既然college文凭不怎么样,我为什么还需要呢,因为我要本地教育背景阿。移民都是高学历,被这个卡住了,只好搞个本地文凭,其实大家都在学校里呆了不少年,知道学校里学的东西工作中能用得很少,一旦工作了,还得吃自己的工作经验。那这样,我们上college的目的就明确了。搞个本地文凭,既然如此,同样是diploma, 越快拿到得越好。千万不要去学那些什么2年,三年的课程。1年的本地工作经验足以媲美三年的学校教育。其实还有一点,这里的人并不是不认你在国内的工作和教育经历,只是他们认为如果你能够通过本地的学习获得技能,这样你的经验加上本地获得的技能才能把你的才干发挥出来。

其实短的课程未必不好,紧凑,只教实用的。大家如果仔细看各community大学的课程(我那里三所大学的全套),头三个学期都是基础课,一点用没有。那是给这里的高中生用的。有人说了,不对,我确实感觉学到了很多东西。是的,你主要在头三个学期学到的是英语。有些知识,翻译成中文就是普普通通的东西,一看就懂。但是你的口语恐怕提高不大。工作环境是提高口语的最好地方。所以如果集中学一个快的课,省下的时间去工作,是一个好的选择。
总而言之,时间最重要,同样都是一样的文凭,从哪拿都一样,越快越好。但是还是要找质量好的课,好的专业,这个问题下回讨论。

第三回,关于专业的选择,找热门的技能,两个办法,一个是去www.jobfutures.com上去看,那里可以查到各个专业工作的薪资水平,需要的学历,相关的专业,以及就业趋势。另外一个方法更简单,看看报纸上的各个 carrer college和培训学校的广告,那上面列的专业绝对都是比较好就业的专业,因为就业率是各个学校的生命线。没有前景,华而不实的专业绝对不会落入他们的眼睛。不过也不是绝对,career college主要关注初级技能培训和证书考试,对于有些比如医生,律师之类的可能前景很好,但是需要去大学读master. 我对于技工这块不太了解,不过大家可以去www.serviceontario.ca上面看看,在school & work里面,apprenticeship 是政府大力支持的,有很多的服务和培训。有些学校是专门教授这些的,比如常青藤。community college里面也有相关课程。不过这一块更要当心,有些技能淘汰的很快。其他方面,IT方面硬件是没有什么前途的,软件现在的课程可多了,软件测试,数据分析SAS, 网络安全都算热门。不过如果在国内不是IT人士,学这个恐怕已经晚了。还有一些课程,适合不同背景的人学习,就业前景基本可以,在这里可以介绍一下。

首先是电算化会计,其实学出来bookkeeper,不管你学两年的课程,还是6个月的速成,找工作都差不多,因为会计最需要经验,所以没有经验的只有从基层老老实实干起。在国内干过的,还是好找,那些从来没干过的,学了两年,还是难找。不过现在很多小公司找行政员工,如果同时能做些会计工作竞争力会大很多。

平均薪资:会计$15.78左右,bookkeeper 14.8左右,一开始也就拿个10-12块
就业情况:一直都还可以,比平均情况好。但是由于中国移民学这个的太多,所以现在的竞争也非常激烈

性别:女性占了91%,看来男性没什么机会,但是CFA男性多一些。不过要拿时间熬。光想办法考证是不行的,还要有机会进入事务所或大些的公司混些经验。所以如果希望将来能够发展空间大,那就要宁为牛后,不为鸡首。到大的事务所做做义工,合同工可能是必要的。
建议:学这个行业选择一个6-8个月的速成就可以了。

下一部分:幼教助理 Early Childcare Assistant
ECE o和ECA 介绍,
幼儿教育老师和幼儿教育助理,ECE需要两年学习,需要到community College里面学,ECA有很多6个月的短期课程,其实工资差不多,差1、2块钱。这个行业除了diploma外还要有certificate,基本上你在学习期间就可以考过这个证,你就可以工作了。考试费很便宜:50元左右
英语要求:ECA的英语要求不是太高
行业前景:不错,而且自雇比例颇高,达到41%。所以比较适合需要照顾家庭的女性。
平均薪资:12.87.这个工资实在没有吸引力,但是适合自雇,自己当老板赚多少,这里没有统计,但是自在啊。
性别:还是有2%的男性的!,而且男性的竞争力特强!
课程的情况看第二页的帖子

三:私人护理(personal support worker)
其实如果是需要找个挣钱多一些的职业,这个职业比ECA强,首先工资高,其次有了这个专业的diploma一样可以做ECA的工作,因为课程里面有专们如何照顾孩子的。而且学到的知识其实对自己对家庭都很有好处。只是我们中国人有的时候有偏见,觉得去照顾病人,老人,收拾床铺不太好对家中父老交待。说在国外干什么呢,私人护理,都干些啥,收拾屋子,陪老人聊天,照顾病人。人家会觉得你好好的,跑国外干这个。但是我觉得这个职业不错,听我分析一下
就业前景:很不错。整个世界都进入老年社会了。失业率只有1%!。未来中国也很需要,另外,发达社会越来越重视对于老人,病人,disable包括孩子的照顾和护理。其实不仅是照顾吃穿活动那么简单,还有精神上的照顾,属于体验经济的一类,阳光产业。而且这个行业对于自己的性格磨练很有好处,你会变得很有耐心,细心和爱心,听说对于生老病死的看法也会大大改变。自雇和part time的机会也不错,照顾照顾邻里,有40%的人做part time. 平时做义工的也是这个行业比较多。
薪资:16-18也有到20块的,而且,由于从事一些和病人打交道的工作,一般健康福利会比较不错。
工作环境:其实我去过老人院和老人公寓,环境真得很不错。由于工作中动体力的时间较多,所以一般都不会让你连续工作时间太长的。
语言要求:这个工作一般对语言要求较高,不过有一点,会一门外语绝对是一个财富。
发展空间:这个专业实际上是Nurse Aid,发展当然是朝Rigester Nurse了。热门阿。
性别:女性占91%。 但是:听说急需男性阿,虽说有器械帮助,但是还是有些体力活的,所以经常在老人院里和医院里见到五大三粗的女性。还有,男性也有隐私阿,男同志有的时候也想让同性来照顾一下。顺便说一句,男护士也是非常非常受欢迎。
课程介绍,上这个课的教室很有意思,大家有兴趣可以参观。这个课对于实际操作要求很强,经常要演练急救,比如,突然有人晕倒了,怎么处理,学生们七手八脚的上。
这个课必须要西人教师上,因为西人和我们在处理人的问题上首先观念大大不同。西人把安全放在第一位,首先必须安全。所以什么民间土法,没有经过验证的不用。比如脖子不舒服,趁他不注意给他嘎吱一下。所以同样操作,观念不同,处理的方法不同。
课程介绍太长了,包括,心理学的知识,人际沟通,营养学,老人护理,临终关怀,危机处理,家庭护理,收拾房屋,以及相关的法律法规,当然,实习是非常重要的一环
PT: Pharmacy Technician
就业前景:Very Good, 还是和老年社会有关。
薪资水平:如果在零售,10-15块,如果在社区community health care. 10-20块,医院:19-26块起,有些人成为了药剂师,工资约7万/年起,但是要再继续学习获得degree.
性别:男女均衡
语言:要求较高,学习辛苦,因为有大量东西要记,中国人毕业率高,老外经常吃不消。
自雇:自己做生意的占15%。 年收入较高。
学位:diploma, 同时还有行业的几个certificate
学习时间:40周,要学得抓紧,政府马上要把它改成两年的课程了
课程介绍:太长了

Community Service Worker 介绍,请点,就不用费时找了

深圳渔民 : 2008-06-08#2
回复: toronto college介绍,选择,相关问题交流

ddddddddd

playaroundtoCanada : 2008-06-08#3
回复: toronto college介绍,选择,相关问题交流

好帖。正有亲戚想申请SENECA的研究生文凭项目,据说这个学校号称是最大的公立学院,不知道属实否?请问楼主,如果读者个研究生文凭,是否可以在结束之后继续申请DEGREE呢?

zmq480 : 2008-06-08#4
回复: toronto college介绍,选择,相关问题交流

up

承承 : 2008-06-08#5
回复: toronto college介绍,选择,相关问题交流

楼主,请教两个问题。

一:我是4月19日登陆的,我老公将于6月28日登陆,我们俩都想从9月1日起上学,能申请贷款吗?能贷多少?如何申请?

二:我在国内是从事人事部门工资管理的,计算机应用水平较高,到加后我应该申请什么专业的college?原打算申请BUSINESS ADMINISTRATION - HUMAN RESOURCES 专业,但是有朋友告诉我毕业后工作不好找,是这样吗?另外Program Type 中regular和co-op有什么区别?

alexwei : 2008-06-08#6
回复: toronto college介绍,选择,相关问题交流

好帖。正有亲戚想申请SENECA的研究生文凭项目,据说这个学校号称是最大的公立学院,不知道属实否?请问楼主,如果读者个研究生文凭,是否可以在结束之后继续申请DEGREE呢?
Seneca 是大专,读master去university 好了

alexwei : 2008-06-08#7
回复: toronto college介绍,选择,相关问题交流

楼主,请教两个问题。

一:我是4月19日登陆的,我老公将于6月28日登陆,我们俩都想从9月1日起上学,能申请贷款吗?能贷多少?如何申请?

二:我在国内是从事人事部门工资管理的,计算机应用水平较高,到加后我应该申请什么专业的college?原打算申请BUSINESS ADMINISTRATION - HUMAN RESOURCES 专业,但是有朋友告诉我毕业后工作不好找,是这样吗?另外Program Type 中regular和co-op有什么区别?
1、申请OSAP,查一下相关贴子吧,注意到加拿大后的银行存款不能超过3000。申请多少因人情况而异。
2,CO-op就是提供实习。人力资源找不到工作,除非英语和当地人一样。

playaroundtoCanada : 2008-06-09#8
回复: toronto college介绍,选择,相关问题交流

楼主,请看下面的链接 ,这是北外和SENECA合作的一个研究生文凭项目,你帮我看看。我表妹今年本科毕业,英语专业,想通过这个途径赴加拿大读书。费用不高,据说可以拿职业资格证和研究生毕业证。这个学校的网站我也看了,是有研究生文凭项目的,也有大专,也有本科的学时学位授予权。请你帮我看看值不值得,谢谢。http://www.bilingochina.cn/ca/

hnass : 2008-06-09#9
回复: toronto college介绍,选择,相关问题交流

好贴,收藏

游客 : 2008-06-09#10
回复: toronto college介绍,选择,相关问题交流

好帖。正有亲戚想申请SENECA的研究生文凭项目,据说这个学校号称是最大的公立学院,不知道属实否?请问楼主,如果读者个研究生文凭,是否可以在结束之后继续申请DEGREE呢?
degree当然不行。

alexwei : 2008-06-09#11
回复: toronto college介绍,选择,相关问题交流

楼主,请看下面的链接 ,这是北外和SENECA合作的一个研究生文凭项目,你帮我看看。我表妹今年本科毕业,英语专业,想通过这个途径赴加拿大读书。费用不高,据说可以拿职业资格证和研究生毕业证。这个学校的网站我也看了,是有研究生文凭项目的,也有大专,也有本科的学时学位授予权。请你帮我看看值不值得,谢谢。http://www.bilingochina.cn/ca/

Seneca 是大专,研究生文凭在加拿大估计没什么知名度,在国内也许可以用用。

playaroundtoCanada : 2008-06-10#12
回复: toronto college介绍,选择,相关问题交流

今天下午又查了很多信息,研究生文凭就是只有毕业证,没有学位证的,难度高于本科,低于硕士学位。这个文凭拿下来就可以按照技术移民里的学历分25分拿。在加拿大,一些学院提供这个课程,即post graduate programme.而且在UBC,也开设了研究生文凭项目。在我认为,这个就是比COLLEGE高一级别的职业教育,是有利于在加拿大就业的。SENECA是college不假,但是该学院还有若干的学士学位授予项目。所以我觉得我们管它叫大专是不是有点那个? ANYWAY,谢谢各位和楼主。加分分了。

noisy8888 : 2008-06-10#13
回复: toronto college介绍,选择,相关问题交流

有毕业证没学位能有25分,而不是22分吗?

playaroundtoCanada : 2008-06-10#14
回复: toronto college介绍,选择,相关问题交流

LS的,应该是。

小南瓜 : 2008-06-10#15
回复: toronto college介绍,选择,相关问题交流

多伦多的四个公立college都有degree program的,但它们都还是大专。

college的Post Graduate Certificate是给已经有degree的人学习另外一个专业的。跟普通college里的课程比起来就是少了那些公共课而已,难度一样的。我已经有同学上过了。

wzb830128 : 2008-06-10#16
回复: toronto college介绍,选择,相关问题交流

LZ你好.我准备去加留学移民.现在还是在犹豫申请什么样的学校.我专科毕业.自考本科快毕业了.预计明年5月.不知道是直接申请研究生,还是上一个文凭课程的好?

alexwei : 2008-06-10#17
回复: toronto college介绍,选择,相关问题交流

今天下午又查了很多信息,研究生文凭就是只有毕业证,没有学位证的,难度高于本科,低于硕士学位。这个文凭拿下来就可以按照技术移民里的学历分25分拿。在加拿大,一些学院提供这个课程,即post graduate programme.而且在UBC,也开设了研究生文凭项目。在我认为,这个就是比COLLEGE高一级别的职业教育,是有利于在加拿大就业的。SENECA是college不假,但是该学院还有若干的学士学位授予项目。所以我觉得我们管它叫大专是不是有点那个? ANYWAY,谢谢各位和楼主。加分分了。
如果你就是想拿学历分,我想OK,关于就业,还要看你的专业和你想干什么。

alexwei : 2008-06-10#18
回复: toronto college介绍,选择,相关问题交流

6月10日续,如何选择college呢,论坛上这类东西可多了,我谈谈自己看法。我觉得核心就一个,快,时间越短越好。为什么呢,第一点,college 就是一个大专,你不管怎么学,拿什么文凭,到社会上也是一个具备初级技能的毕业生。虽说,seneca等在我们圈子名气响亮,可是并不入本地人眼。我有几个银行和政府的朋友,有本地的也有移民。我问他们关于college的问题,他们一致认为,college是非常初级的,只能从事初级工作。而且他们其实不是很清楚或者很在意college之间有什么差别。而到了大学这一级别,才开始注意学校之间的区别,这和国内差不多。第二点,既然college文凭不怎么样,我为什么还需要呢,因为我要本地教育背景阿。移民都是高学历,被这个卡住了,只好搞个本地文凭,其实大家都在学校里呆了不少年,知道学校里学的东西工作中能用得很少,一旦工作了,还得吃自己的工作经验。那这样,我们上college的目的就明确了。搞个本地文凭,既然如此,同样是diploma, 越快拿到得越好。千万不要去学那些什么2年,三年的课程。1年的本地工作经验足以媲美三年的学校教育。其实还有一点,这里的人并不是不认你在国内的工作和教育经历,只是他们认为如果你能够通过本地的学习获得技能,这样你的经验加上本地获得的技能才能把你的才干发挥出来。
其实短的课程未必不好,紧凑,只教实用的。大家如果仔细看各community大学的课程(我那里三所大学的全套),头三个学期都是基础课,一点用没有。那是给这里的高中生用的。有人说了,不对,我确实感觉学到了很多东西。是的,你主要在头三个学期学到的是英语。有些知识,翻译成中文就是普普通通的东西,一看就懂。但是你的口语恐怕提高不大。工作环境是提高口语的最好地方。所以如果集中学一个快的课,省下的时间去工作,是一个好的选择。
总而言之,时间最重要,同样都是一样的文凭,从哪拿都一样,越快越好。但是还是要找质量好的课,好的专业,这个问题下回讨论。

alexwei : 2008-06-10#19
回复: toronto college介绍,选择,相关问题交流

自己先顶着,还要接着写呢

futurelight : 2008-06-10#20
回复: toronto college介绍,选择,相关问题交流

关注!

candice_fion : 2008-06-11#21
回复: toronto college介绍,选择,相关问题交流

有谁了解post graduate programme的吗?

游客 : 2008-06-11#22
回复: toronto college介绍,选择,相关问题交流

有谁了解post graduate programme的吗?

了解什么?

阿拉蕾 : 2008-06-11#23
回复: toronto college介绍,选择,相关问题交流

楼主,请看下面的链接 ,这是北外和SENECA合作的一个研究生文凭项目,你帮我看看。我表妹今年本科毕业,英语专业,想通过这个途径赴加拿大读书。费用不高,据说可以拿职业资格证和研究生毕业证。这个学校的网站我也看了,是有研究生文凭项目的,也有大专,也有本科的学时学位授予权。请你帮我看看值不值得,谢谢。http://www.bilingochina.cn/ca/

你表妹读这个的目的是什么?呵呵!
拿移民的分数?
提高英语?
还是单纯家里有余钱支持出国镀金?然后回流?

这8个月的专业属于实用性质,本地人或者英语好的移民出来找工作机会还比较多。如果回流,来这边读就没有意义了,因为学的东西多数还是与加拿大实际操作比较相关的基础知识,翻译成研究生毕业证就是表面上好听。到不如在国内找个公司或者修相关的课程来的实在。

alexwei : 2008-06-11#24
回复: toronto college介绍,选择,相关问题交流

Strongly agree with upstairs

alexwei : 2008-06-11#25
回复: toronto college介绍,选择,相关问题交流

6月11日,
第三回,关于专业的选择,找热门的技能,两个办法,一个是去http://www.jobfutures.com上去看,那?..看报纸上的各个 carrer college和培训学校的广告,那上面列的专业绝对都是比较好就业的专业,因为就业率是各个学校的生命线。没有前景,华而不实的专业绝对不会落入他们的眼睛。不过也不是绝对,career college主要关注初级技能培训和证书考试,对于有些比如医生,律师之类的可能前景很好,但是需要去大学读master. 我对于技工这块不太了解,不过大家可以去www.serviceontario.ca上面看看,在school & work里面,apprenticeship 是政府大力支持的,有很多的服务和培训。有些学校是专门教授这些的,比如常青藤。community college里面也有相关课程。不过这一块更要当心,有些技能淘汰的很快。其他方面,IT方面硬件是没有什么前途的,软件现在的课程可多了,软件测试,数据分析SAS, 网络安全都算热门。不过如果在国内不是IT人士,学这个恐怕已经晚了。还有一些课程,适合不同背景的人学习,就业前景基本可以,在这里可以介绍一下。
首先是电算化会计,其实学出来bookkeeper,不管你学两年的课程,还是6个月的速成,找工作都差不多,因为会计最需要经验,所以没有经验的只有从基层老老实实干起。在国内干过的,还是好找,那些从来没干过的,学了两年,还是难找。不过现在很多小公司找行政员工,如果同时能做些会计工作竞争力会大很多。
平均薪资:会计$15.78左右,bookkeeper 14.8左右,一开始也就拿个10-12块
就业情况:一直都还可以,比平均情况好。但是由于中国移民学这个的太多,所以现在的竞争也非常激烈
性别:女性占了91%,看来男性没什么机会,但是CFA男性多一些。不过要拿时间熬。光想办法考证是不行的,还要有机会进入事务所或大些的公司混些经验。所以如果希望将来能够发展空间大,那就要宁为牛后,不为鸡首。到大的事务所做做义工,合同工可能是必要的。
建议:学这个行业选择一个6-8个月的速成就可以了。
课程介绍:会计基础,Quickbooks, Simply Accounting, Payroll, ACCPAC accounting system
下一部分:幼教助理 Early Childcare Assistant

playaroundtoCanada : 2008-06-12#26
回复: toronto college介绍,选择,相关问题交流

你表妹读这个的目的是什么?呵呵!
拿移民的分数?
提高英语?
还是单纯家里有余钱支持出国镀金?然后回流?

这8个月的专业属于实用性质,本地人或者英语好的移民出来找工作机会还比较多。如果回流,来这边读就没有意义了,因为学的东西多数还是与加拿大实际操作比较相关的基础知识,翻译成研究生毕业证就是表面上好听。到不如在国内找个公司或者修相关的课程来的实在。

我表妹读书的目的就是在加拿大留下来,有个工作。呵呵。很简单,不想回流的。因为经验类移民政策马上要出来了,不知道适合不。

playaroundtoCanada : 2008-06-12#27
回复: toronto college介绍,选择,相关问题交流

如果你就是想拿学历分,我想OK,关于就业,还要看你的专业和你想干什么。
专业有两个:国际商务管理 国际物流与供应链管理
毕业之后可以拿研究生毕业证和相应的专业的职业资格证:
加拿大国际贸易协会(FITT)颁发的执业资格证书CITP (Certified International Trade Professional)
加拿大采购管理协会(PMAC)颁发的执业资格证书 CPP (Certified Professional Purchaser)
想毕业后靠着证书留在加拿大,并找到一份工作

playaroundtoCanada : 2008-06-12#28
回复: toronto college介绍,选择,相关问题交流

为楼主顶一个。这个对新移民会有很大帮助。收藏。加分分

阿拉蕾 : 2008-06-12#29
回复: toronto college介绍,选择,相关问题交流

专业有两个:国际商务管理 国际物流与供应链管理
毕业之后可以拿研究生毕业证和相应的专业的职业资格证:
加拿大国际贸易协会(FITT)颁发的执业资格证书CITP (Certified International Trade Professional)
加拿大采购管理协会(PMAC)颁发的执业资格证书 CPP (Certified Professional Purchaser)
想毕业后靠着证书留在加拿大,并找到一份工作

(FITT)
http://www.fitt.ca/home.htm

(PMAC)
http://www.pmac.ca/

去这两个网站看看咯,8个月的课程学完距离拿证书还有很长的距离呢?

不过鉴于你表妹的简单想法,读supply chain management, 来了以后努力适应英语,也可行。

其实自己考托福,弄个ibt100分以上,自己申请,国内的学费就能省了。

alexwei : 2008-06-12#30
回复: toronto college介绍,选择,相关问题交流

ECE o和ECA 介绍,
幼儿教育老师和幼儿教育助理,ECE需要两年学习,需要到community College里面学,ECA有很多6个月的短期课程,其实工资差不多,差1、2块钱。这个行业除了diploma外还要有certificate,基本上你在学习期间就可以考过这个证,你就可以工作了。考试费很便宜:50元左右
英语要求:ECA的英语要求不是太高
行业前景:不错,而且自雇比例颇高,达到41%。所以比较适合需要照顾家庭的女性。
平均薪资:12.87.这个工资实在没有吸引力,但是适合自雇,自己当老板赚多少,这里没有统计,但是自在啊。
性别:还是有2%的男性的!,而且男性的竞争力特强!
课程介绍:粘贴另一份帖子里的
课程包括:这个是比较标准的课程。
Sorry no中文资料,安省规定是必须用英语或法语教学
Orientation
This module covers the requirements to set and achieve educational goals and gain exposure to market requirements and opportunities. The student will acquire tools for dealing with the challenges of being a student such as stress, note taking, studying, financial and time management.
Role of Childcare Worker
This module covers the principles of professionalism in the childcare field. It includes manner, dress, attitude and the importance of the Childcare Assistant in a child’s daycare experience and development.
Child Development
This module covers the multifaceted aspects of a child’s development including physical, social, intellectual, emotional and moral components. An in-depth exploration of the complexity of child development is undertaken.
Observation Skills
Course work covers the skills required to observe children effectively. Written observations are made in a variety of settings and circumstances. Students draw from their practical experiences to add to the module. Pitfalls and problems with observation techniques are discussed.
Guiding Children
This module covers concepts of enhancing child based play, encouraging age appropriate behaviors and using a variety of effective guidance techniques. Based on the day-to-day operations of a childcare facility, the theory and practical application of these concepts are observed and discussed.
Communication Skills
The emphases in this module are on developing the skills to communicate in a clear, concise, and articulate manner in both written and oral form. A variety of topics are included: i.e. two-way communication, listening skills, grammar, spelling, written notes and report completion.
Introduction to Technology
This module is designed to introduce the student to basic computer hardware, software. It provides the basics in using computers to complete assignments and essays. Additionally, it provides a good foundation for the module Children and Technology.
Children and Technology
A variety of children’s software programs are examined to evaluate/assess the effectiveness of this medium for child development. Students have an opportunity to review programs and learn about their appropriate application in an early learning environment.
Program Planning
The students are introduced to the diverse components of effective program planning. Incorporated into the program planning are a variety of activities that aid in a child’s overall healthy development. The importance of dramatic play, circle time and problem solving activities are discussed.
Health and Safety
Health and safety standards and applications are studied and discussed in this critically important module. Students learn about healthy environments and how to educate children in managing their own good health habits. Child abuse recognition and prevention procedures are examined and sample policies are reviewed.
Nutrition
This module covers the theory and application of balanced, healthy nutrition for children 0 to 6. Basic food handling principles are discussed, while recipes are reviewed and assessed. Menu planning and compiling a resource of appropriate recipes and menus are incorporated.
Developing a Business in Childcare
This module deals with the demands and trends in daycare centres, starting a daycare centre, government rules, policies and licensing procedure, strategies involved in operating, income potential, daycare management, and marketing the business
Practicum ? 280 hours internship
Field placement provides the students with an opportunity to practice their new skills in a work setting. While on placement students can gain experience in a wider range of Early Childcare Assistant skills, become more self-confident and in some cases receive offers of employment from the placement site.

教材;
TEXT BOOKS & REFERENCE MATERIALS

Keys to Success: How to Achieve Your Goals: 3rd Canadian Edition by Carter Bishop, Kravits and Maurin, Pearson, 2004. ISBN 0-13-120121-2

The Little Brown Compact Handbook: 2nd Canadian Edition by Jane E. Aaron and Murray McArthur, Pearson, 2003. ISBN 0-321-10205-3

Early Childhood Development: A Multicultural Perspective: 3rd Edition by Jeffery trawick-Smith, Merrill Prentice Hall, 2003. ISBN 0-13-046576-3

Skills for Preschool Teachers: 7th Edition by Janice J. Beaty, 2004. ISBN 0-13-048609-4

银燕轻舞 : 2008-06-12#31
回复: toronto college介绍,选择,相关问题交流

请问toronto有没有可以提供funding的培训啊?我看到卡城有的,读书政府给1500刀的生活费,toronto有吗?

ty_precious : 2008-06-13#32
回复: toronto college介绍,选择,相关问题交流

:wdb10:关注中,楼主继续!

alexwei : 2008-06-13#33
回复: toronto college介绍,选择,相关问题交流

请问toronto有没有可以提供funding的培训啊?我看到卡城有的,读书政府给1500刀的生活费,toronto有吗?
我记得是有一种给移民的就业培训的补贴,可以申请,不过不记得详细信息。你可以到www.settlement.org上查 或者问一下YMCA.
还有一种就是政府funding的一些成人课程,你可以查一下周围成人教育中心,他们可能会有这些课程,费用很低,时间也短很多。

yeriver : 2008-06-13#34
回复: toronto college介绍,选择,相关问题交流

lz帖中有些话还是比较中肯滴。

想问问物流的就业情况怎样?是不是要有客源,新移民不容易做?不想轧堆读会计,可是好像物流的薪水比会计底,发展空间也没会计大。

alexwei : 2008-06-13#35
回复: toronto college介绍,选择,相关问题交流

lz帖中有些话还是比较中肯滴。

想问问物流的就业情况怎样?是不是要有客源,新移民不容易做?不想轧堆读会计,可是好像物流的薪水比会计底,发展空间也没会计大。
不是很了解,不过感觉不是很大就业空间,如果是做delivery倒是可以,长途货运的薪水不错,只是很辛苦,而且要花一、两年时间考牌

alexwei : 2008-06-13#36
回复: toronto college介绍,选择,相关问题交流

三:私人护理(personal support worker)
其实如果是需要找个挣钱多一些的职业,这个职业比ECA强,首先工资高,其次有了这个专业的diploma一样可以做ECA的工作,因为课程里面有专们如何照顾孩子的。而且学到的知识其实对自己对家庭都很有好处。只是我们中国人有的时候有偏见,觉得去照顾病人,老人,收拾床铺不太好对家中父老交待。说在国外干什么呢,私人护理,都干些啥,收拾屋子,陪老人聊天,照顾病人。人家会觉得你好好的,跑国外干这个。但是我觉得这个职业不错,听我分析一下
就业前景:很不错。整个世界都进入老年社会了。失业率只有1%!。未来中国也很需要,另外,发达社会越来越重视对于老人,病人,disable包括孩子的照顾和护理。其实不仅是照顾吃穿活动那么简单,还有精神上的照顾,属于体验经济的一类,阳光产业。而且这个行业对于自己的性格磨练很有好处,你会变得很有耐心,细心和爱心,听说对于生老病死的看法也会大大改变。自雇和part time的机会也不错,照顾照顾邻里,有40%的人做part time. 平时做义工的也是这个行业比较多。
薪资:16-18也有到20块的,而且,由于从事一些和病人打交道的工作,一般健康福利会比较不错。
工作环境:其实我去过老人院和老人公寓,环境真得很不错。由于工作中动体力的时间较多,所以一般都不会让你连续工作时间太长的。
语言要求:这个工作一般对语言要求较高,不过有一点,会一门外语绝对是一个财富。
发展空间:这个专业实际上是Nurse Aid,发展当然是朝Rigester Nurse了。热门阿。
性别:女性占91%。 但是:听说急需男性阿,虽说有器械帮助,但是还是有些体力活的,所以经常在老人院里和医院里见到五大三粗的女性。还有,男性也有隐私阿,男同志有的时候也想让同性来照顾一下。顺便说一句,男护士也是非常非常受欢迎。
课程介绍,上这个课的教室很有意思,大家有兴趣可以参观。这个课对于实际操作要求很强,经常要演练急救,比如,突然有人晕倒了,怎么处理,学生们七手八脚的上。
这个课必须要西人教师上,因为西人和我们在处理人的问题上首先观念大大不同。西人把安全放在第一位,首先必须安全。所以什么民间土法,没有经过验证的不用。比如脖子不舒服,趁他不注意给他嘎吱一下。所以同样操作,观念不同,处理的方法不同。
课程介绍太长了,需要的留下邮件。包括,心理学的知识,人际沟通,营养学,老人护理,临终关怀,危机处理,家庭护理,收拾房屋,以及相关的法律法规,当然,实习是非常重要的一环

Flying Dragon : 2008-06-13#37
回复: toronto college介绍,选择,相关问题交流

LZ说得挺透切的。
LZ所说的PSW和俺老师说的基本一样,LZ是不是。。。????
有什么课本资料,请PM。
花花赞上!!!

Flying Dragon : 2008-06-13#38
回复: toronto college介绍,选择,相关问题交流

请问toronto有没有可以提供funding的培训啊?我看到卡城有的,读书政府给1500刀的生活费,toronto有吗?
阿省给Funding的只是ESL 。安省没有类似ESL的Funding,要获得资助主要还是通过OSAP或EI培训。

银燕轻舞 : 2008-06-13#39
回复: toronto college介绍,选择,相关问题交流

我记得是有一种给移民的就业培训的补贴,可以申请,不过不记得详细信息。你可以到www.settlement.org上查 或者问一下YMCA.
还有一种就是政府funding的一些成人课程,你可以查一下周围成人教育中心,他们可能会有这些课程,费用很低,时间也短很多。

阿省给Funding的只是ESL 。安省没有类似ESL的Funding,要获得资助主要还是通过OSAP或EI培训。

谢谢两位的解答!那什么是OSAP或EI培训啊?

Thumbelina : 2008-06-14#40
回复: toronto college介绍,选择,相关问题交流

楼主能不能多说一下软件中"网络安全"的问题. 我现在在想是不是去学一下. 谢谢先.

守望 : 2008-06-14#41
回复: toronto college介绍,选择,相关问题交流

thanks

Flying Dragon : 2008-06-14#42
回复: toronto college介绍,选择,相关问题交流

谢谢两位的解答!那什么是OSAP或EI培训啊?

OSAP:学生贷款。OSAP全称是Ontario Student Assistance Program. 是指安省学生助学金项目,主要用于安省合法居民(包括移民)在加拿大进一步学习深造。
www.osap.ca

EI培训:[FONT=&quot]Employment Insurance(EI),[/FONT]相当于中国的下岗再就业培训。前提是,要先工作,后下岗。,
[FONT=&quot]http://www.servicecanada.gc.ca/en/sc/skills/skillsdevelopment.shtml[/FONT]

alexwei : 2008-06-14#43
回复: toronto college介绍,选择,相关问题交流

楼主能不能多说一下软件中"网络安全"的问题. 我现在在想是不是去学一下. 谢谢先.
我只能告诉你,行业非常好,前景不错,薪资也不错,努力朝这个方向奋斗。

Thumbelina : 2008-06-14#44
回复: toronto college介绍,选择,相关问题交流

我只能告诉你,行业非常好,前景不错,薪资也不错,努力朝这个方向奋斗。
谢谢, college有没有这个专业呢? 还是要自学考证?

alexwei : 2008-06-14#45
回复: toronto college介绍,选择,相关问题交流

谢谢, college有没有这个专业呢? 还是要自学考证?
College 有软件专业,建议你到各个college去一趟,那里都有免费的课程介绍可以拿,如果你在国内,来了再说。如果是做网络安全的,需要自学或者到一些职业学校学习。我建议你如果要进这一行,就去一些本地的IT网站多看看,这样子会了解行业趋势。www.itworldcanada.com
了解薪资
http://www.itworldcanada.com/calculator/

Thumbelina : 2008-06-14#46
回复: toronto college介绍,选择,相关问题交流

thanks a million :)

银燕轻舞 : 2008-06-15#47
回复: toronto college介绍,选择,相关问题交流

OSAP:学生贷款。OSAP全称是Ontario Student Assistance Program. 是指安省学生助学金项目,主要用于安省合法居民(包括移民)在加拿大进一步学习深造。
www.osap.ca

EI培训:[FONT=&quot]Employment Insurance(EI),[/FONT]相当于中国的下岗再就业培训。前提是,要先工作,后下岗。,
[FONT=&quot]http://www.servicecanada.gc.ca/en/sc/skills/skillsdevelopment.shtml[/FONT]
Thanks!

heipi : 2008-06-15#48
回复: toronto college介绍,选择,相关问题交流

顶!

alexwei : 2008-06-15#49
回复: toronto college介绍,选择,相关问题交流

I met a friend today, he is studying in a PSW program and almost done. He has got a $21/hours work and plan to get Register Nurse degree in the past three years. He offer me a useful info that U.S. hospitals often come to Toronto hiring Register nurses and the salary will be over $100000/year with other benefit such as free apartment. To start a career in nurse you need five years study(including at least one years work experience.

cindy汤包 : 2008-06-15#50
回复: toronto college介绍,选择,相关问题交流

lz可以讲一下药剂师PT么?谢谢,非常非常期待,给你送花

alexwei : 2008-06-16#51
回复: toronto college介绍,选择,相关问题交流

lz可以讲一下药剂师PT么?谢谢,非常非常期待,给你送花
应你要求,先讲这个:
PT: Pharmacy Technician
就业前景:Very Good, 还是和老年社会有关。
薪资水平:如果在零售,10-15块,如果在社区community health care. 10-20块,医院:19-26块起,有些人成为了药剂师,工资约7万/年起,但是要再继续学习获得degree.
性别:男女均衡
语言:要求较高,学习辛苦,因为有大量东西要记,中国人毕业率高,老外经常吃不消。
自雇:自己做生意的占15%。 年收入较高。
学位:diploma, 同时还有行业的几个certificate
学习时间:40周,要学得抓紧,政府马上要把它改成两年的课程了
课程介绍:太长了,需要的给mail.

cindy汤包 : 2008-06-17#52
回复: toronto college介绍,选择,相关问题交流

应你要求,先讲这个:
PT: Pharmacy Technician
就业前景:Very Good, 还是和老年社会有关。
薪资水平:如果在零售,10-15块,如果在社区community health care. 10-20块,医院:19-26块起,有些人成为了药剂师,工资约7万/年起,但是要再继续学习获得degree.
性别:男女均衡
语言:要求较高,学习辛苦,因为有大量东西要记,中国人毕业率高,老外经常吃不消。
自雇:自己做生意的占15%。 年收入较高。
学位:diploma, 同时还有行业的几个certificate
学习时间:40周,要学得抓紧,政府马上要把它改成两年的课程了
课程介绍:太长了,需要的给mail.
谢谢,课程介绍不用麻烦了,小包子可以在COLLEGE的网页上找到,你真是个好人:wdb19:

alexwei : 2008-06-17#53
回复: toronto college介绍,选择,相关问题交流

谢谢,课程介绍不用麻烦了,小包子可以在COLLEGE的网页上找到,你真是个好人:wdb19:
需要做好心理准备,听同事讲说一开始只有学历没有经验的时候,工资不高的。一定要有继续深造拿pharmacist 的目标和动力,五年计划,努力!

sinoroc : 2008-06-17#54
回复: toronto college介绍,选择,相关问题交流

好帖,非常感谢!

四月天 : 2008-06-18#55
回复: toronto college介绍,选择,相关问题交流

很有用的信息,谢啦!

十二少 : 2008-06-18#56
回复: toronto college介绍,选择,相关问题交流

信息非常实用,谢谢/
我9月要去HUMBER读Fundraising Information and Volunteer,应该也属于non-profit机构, 时间是一年半,实际读书只有2学期,最后一学期实习, 所以从时间上讲是不错的, 但是不知道以后的工作是怎样的, 不过听说这个专业整个GTA只有HUMBER开了,所以实习期间会有很多机构上门找人,希望是这样.
另外系主任刚发了个学生名单过来,我好像是唯一的中国人,也可能是唯一的移民,所以很担心自己的英语跟不上,这个专业需要大量writing & presentation

alexwei : 2008-06-19#57
回复: toronto college介绍,选择,相关问题交流

介绍一下社会工作者行业
国内没有社工这个行业,不过大家应该清楚这个是干什么的,我们遇到的settlement Counselor, Employment Counselor都是属于这个行业。这个行业对于我们移民来说有一个优势,我们会1门外语,而且是目前最大的移民群体的外语,各个机构都在扩展对于中国移民的服务。这样子就需要我们了。社工为什么要拓展服务呢,大家知道,no profit机构的钱来自于政府,企业或个人捐助,自我经营收入,其中政府最多,而政府的钱是要根据你做了多少事情来得,比如政府今年有7个亿是帮助新移民的,会分配到各个非营利机构,这些机构就要做一些针对新移民的服务来申请经费,同时还要有效果,如果没有政府的funding, 这个机构就要关门了。所以从事这个行业的人往往有marketing、工商管理,教师的背景,然后获得了本地学历。那么这个行业有多大呢,这个行业非常大,和商业基本上快差不多了。那些机构属于no-profit, 医院,学校,就业、安居服务机构,慈善机构,老人服务机构,绿色和平组织,环境和动物保护组织等。他的行业自成体系,你在一般的招聘网站或者agency那里很少看到招聘的信息。需要什么样的人呢,其实和商业一样,需要sales, accounting, administration, customer service, technician.不一样的地方,核心的服务人员是社工,教师,或者医务人员。大家可以去www.charityvillige.com 上去看,这是加拿大最大的非营利机构的招聘网站。
就业前景:不错
薪资水平:16-20左右。
性别:女性居多
个人奋斗方向:如果成为专业social worker, 工资可以达到70000,但是一般需要有social science master degree. 而 bachelor 毕业实际上和college毕业薪资差别不是太大,我个人觉得可以在拿到diploma后工作两年,直接申请master(用国内的学历)+diploma+工作经验,这样不错。
课程介绍
• Introduction to Community Service Worker
• Social Welfare, Diversity and Awareness
• The Family
• Couples, Family and Interpersonal Communication
• Oriented Case Management
• Environmental Issues and Crisis Interventions and
Prevention
• Bereavement and the Elderly
• Respond to Abuse
• Understanding Human Behavior
• Communication Skills
• Counseling Skills
• Report Writing for the CSW
• Field placement

By the end of the course, students will be able to:
• Demonstrate knowledge of the range of social and community service organizations in their own community as well as detailed knowledge of an agency.
• Practice the role of a team member within a social and community service organization.
• Demonstrate beginning level professional skills
• Apply theory and skills learned in the independent study components of this course.

alex_lz2005 : 2008-06-19#58
回复: toronto college介绍,选择,相关问题交流

Good to know...tks for sharing...

stevenchenxi : 2008-06-19#59
回复: toronto college介绍,选择,相关问题交流

up

fmvictory : 2008-06-19#60
回复: toronto college介绍,选择,相关问题交流

学习

wanderinrain : 2008-06-22#61
回复: toronto college介绍,选择,相关问题交流

谢谢。这个对新移民会有很大帮助。我刚拿到签证,正在考虑学什么专业。

alexwei : 2008-06-24#62
回复: toronto college介绍,选择,相关问题交流

私立学校的选择和信息,请察看http://www.nacc.ca/ National Association of Career College. 里面有详细的分析。

yiyannanjin : 2008-06-30#63
回复: toronto college介绍,选择,相关问题交流

非常感谢!

candice_fion : 2008-06-30#64
回复: toronto college介绍,选择,相关问题交流

介绍一下社会工作者行业

个人奋斗方向:如果成为专业social worker, 工资可以达到70000,但是一般需要有social science master degree. 而 bachelor 毕业实际上和college毕业薪资差别不是太大,我个人觉得可以在拿到diploma后工作两年,直接申请master(用国内的学历)+diploma+工作经验,这样不错。


这个帖子对我们新移民有极大的帮助啊!:wdb10:

请问alexwei, "(用国内的学历)+diploma+工作经验"申请master的这个工作经验是指拿到本地diploma之后的相关工作经验吗?
另外,我在国内是读化学的,国内经验与化学和会计无关,是否可以申请加拿大的会计的diploma呢?毕业之后能否申请master?
谢谢!

zxz96086 : 2008-07-01#65
回复: toronto college介绍,选择,相关问题交流

valuable information. thanks a lot.

游客 : 2008-07-01#66
回复: toronto college介绍,选择,相关问题交流

介绍一下社会工作者行业

个人奋斗方向:如果成为专业social worker, 工资可以达到70000,但是一般需要有social science master degree. 而 bachelor 毕业实际上和college毕业薪资差别不是太大,我个人觉得可以在拿到diploma后工作两年,直接申请master(用国内的学历)+diploma+工作经验,这样不错。


这个帖子对我们新移民有极大的帮助啊!:wdb10:

请问alexwei, "(用国内的学历)+diploma+工作经验"申请master的这个工作经验是指拿到本地diploma之后的相关工作经验吗?
另外,我在国内是读化学的,国内经验与化学和会计无关,是否可以申请加拿大的会计的diploma呢?毕业之后能否申请master?
谢谢!

会让你去社区或组织开工作经验证明的。

alexwei : 2008-07-02#67
回复: toronto college介绍,选择,相关问题交流

介绍一下社会工作者行业

个人奋斗方向:如果成为专业social worker, 工资可以达到70000,但是一般需要有social science master degree. 而 bachelor 毕业实际上和college毕业薪资差别不是太大,我个人觉得可以在拿到diploma后工作两年,直接申请master(用国内的学历)+diploma+工作经验,这样不错。


这个帖子对我们新移民有极大的帮助啊!:wdb10:

请问alexwei, "(用国内的学历)+diploma+工作经验"申请master的这个工作经验是指拿到本地diploma之后的相关工作经验吗?
另外,我在国内是读化学的,国内经验与化学和会计无关,是否可以申请加拿大的会计的diploma呢?毕业之后能否申请master?
谢谢!
college的会计你当然可以读了,和背景无关,只要达到英语的要求就行了。毕业后会计没有master

candice_fion : 2008-07-03#68
回复: toronto college介绍,选择,相关问题交流

谢谢alexwei的回复!
我知道会计是没有,但是其他经济类的是有master的

college的会计和social worker比较的话,哪个前景更好呢?

cindy汤包 : 2008-07-03#69
回复: toronto college介绍,选择,相关问题交流

需要做好心理准备,听同事讲说一开始只有学历没有经验的时候,工资不高的。一定要有继续深造拿pharmacist 的目标和动力,五年计划,努力!
:wdb11:谢谢提醒

alexwei : 2008-07-03#70
回复: toronto college介绍,选择,相关问题交流

谢谢alexwei的回复!
我知道会计是没有,但是其他经济类的是有master的

college的会计和social worker比较的话,哪个前景更好呢?
其他经济类的我觉得还是MBA可能好一些。至于accounting 和social worker.不好回答,会计是一个永远不会太差不会太好的行业,需求稳定。social worker是一个正在发展的行业,机会多些,但是压力大些,要看你是否适合这个工作。
首先你要先评估自己,你是适合和人打交道的工作,还是适合和物打交道的工作。也就是你的职业倾向。www.jobfutures.ca 上有简单的测试,或者去COSTI或者YMCA看有没有职业测试。免费的。
第二,你的过去的工作经验是否对你在相关行业发展有用,比如你以前做教师,当然对于你做Social service worker非常好.
第三, 你自己的兴趣。工作都是boring,找一个不太boring的工作多好。
写下这些,找出交集,确定目标,坚定不移。我觉得选择的时候如果确实结合了自己的长处,兴趣,不会错的,失败的人是不能坚持按照目标前进,遇到困难就在出发点找原因,结果朝三暮四。
时间对于我们最宝贵,早点定目标吧。

candice_fion : 2008-07-03#71
回复: toronto college介绍,选择,相关问题交流

其他经济类的我觉得还是MBA可能好一些。至于accounting 和social worker.不好回答,会计是一个永远不会太差不会太好的行业,需求稳定。social worker是一个正在发展的行业,机会多些,但是压力大些,要看你是否适合这个工作。
首先你要先评估自己,你是适合和人打交道的工作,还是适合和物打交道的工作。也就是你的职业倾向。www.jobfutures.ca 上有简单的测试,或者去COSTI或者YMCA看有没有职业测试。免费的。
第二,你的过去的工作经验是否对你在相关行业发展有用,比如你以前做教师,当然对于你做Social service worker非常好.
第三, 你自己的兴趣。工作都是boring,找一个不太boring的工作多好。
写下这些,找出交集,确定目标,坚定不移。我觉得选择的时候如果确实结合了自己的长处,兴趣,不会错的,失败的人是不能坚持按照目标前进,遇到困难就在出发点找原因,结果朝三暮四。
时间对于我们最宝贵,早点定目标吧。


非常感谢!:wdb6:
非常中肯的建议!:wdb17:

mrs.bean : 2008-07-03#72
回复: toronto college介绍,选择,相关问题交流

好贴!
我有个关于入学申请的问题
究竟需不需要最高学历公正啊?

alexwei : 2008-07-04#73
回复: toronto college介绍,选择,相关问题交流

你申请读什么啊,如果拿不准,就公证了再说,反正以后总有机会用到,又没有多少钱

mrs.bean : 2008-07-04#74
回复: toronto college介绍,选择,相关问题交流

你申请读什么啊,如果拿不准,就公证了再说,反正以后总有机会用到,又没有多少钱

Dental Hygiene 或者 Dental Assistant

我在国内是读国际贸易的

alexwei : 2008-07-04#75
回复: toronto college介绍,选择,相关问题交流

读什么学校。

mrs.bean : 2008-07-04#76
回复: toronto college介绍,选择,相关问题交流

读什么学校。

Cambrian College

mangcang : 2008-07-05#77
回复: toronto college介绍,选择,相关问题交流

Lz,请教:
基本情况:
我在国内本科是法学,刚刚毕业.想去加拿大继续读书,目的就是希望通过读书后能移民.
问题:
1'中介建议读college的一年的研究生文凭课程,但以我的学科背景只能读EVENT MANAGEMENT.这个专业的前景如何?
2'希望还能介绍一下旅游管理就业情况.

谢谢~~

alexwei : 2008-07-07#78
回复: toronto college介绍,选择,相关问题交流

Cambrian College

一般情况下,读college是可以不评估学历的,除非你想少修课。你需要参加他们的英语,或者还有数学的测试,来看看你是否达到学习这个专业应有的水平,如果你评估了你的学历,给他们看一下也无妨,不过还是要参加他们的英语测试

alexwei : 2008-07-07#79
回复: toronto college介绍,选择,相关问题交流

Lz,请教:
基本情况:
我在国内本科是法学,刚刚毕业.想去加拿大继续读书,目的就是希望通过读书后能移民.
问题:
1'中介建议读college的一年的研究生文凭课程,但以我的学科背景只能读EVENT MANAGEMENT.这个专业的前景如何?
2'希望还能介绍一下旅游管理就业情况.

谢谢~~
没有好的建议,如果能拿到VISA,也不妨一试,旅游管理在加拿大算是个成熟行业,竞争激烈,薪水不太高。不过如果中国开放了加拿大旅游,将会出现好的机会。看看Happer总理是否愿意向中国低头了。college是哪个

Aim : 2008-07-07#80
回复: toronto college介绍,选择,相关问题交流

好帖子

doudou8008 : 2008-07-07#81
回复: toronto college介绍,选择,相关问题交流

关注!谢谢LZ。

mangcang : 2008-07-10#82
回复: toronto college介绍,选择,相关问题交流

没有好的建议,如果能拿到VISA,也不妨一试,旅游管理在加拿大算是个成熟行业,竞争激烈,薪水不太高。不过如果中国开放了加拿大旅游,将会出现好的机会。看看Happer总理是否愿意向中国低头了。college是哪个

拿visa 是没有问题,因为我的一个同学和我情况相似只是专业不同也是申请的college上周已经抵达渥太华了.
我心仪的college有humber college,georgian college(在巴厘市).
LZ,这两个哪一个口碑和就业率更好点呢.

谢谢

木尘子 : 2008-07-10#83
回复: toronto college介绍,选择,相关问题交流

搂主能否直接给我们推荐几个短期证书的培训学校呢?比如学幼儿教育助理,护士助理。

alexwei : 2008-07-10#84
回复: toronto college介绍,选择,相关问题交流

需要学什么,虽然很多学校都有相同专业,但是也有很多不同。有一个简单办法,到华人超市,大中报,或者加拿大都市报,或者北美时报,很多很多学校广告,打电话问。如果不想上华人学校,也可以,到地铁站找几份Metro来,打电话问。是西人学校又在中国报纸上做广告的,Yorkville, Canadian Business School, Evergreecollege,基本上在scarborough的都是华人学校。

木尘子 : 2008-07-10#85
回复: toronto college介绍,选择,相关问题交流

免费的最好,比如像yorkdale那样的, 半年就有证书。

alexwei : 2008-07-10#86
回复: toronto college介绍,选择,相关问题交流

你到 TDSB的网站上看一下,我帮你找了几个成人学校,多市成人高中就这么多了,有就有,没有就没有。另外,COSTI也有免费的,www.costi.org.
School
Address
Postal
Code

Burnhamthorpe Adult Learning Centre/SS
500 The East Mall
M9B 2C4
City Adult Learning Centre (CALC)
1 Danforth Avenue
M4K 1M8
Emery Adult Learning Centre
3395 Weston Road
M9M 2V9
Scarborough Centre for Alternative Studies (Adults)
939 Progress Avenue
M1G 3T8
Yorkdale Adult Learning Centre/SS
38 Orfus Road
M6A 1L6


木尘子 : 2008-07-10#87
回复: toronto college介绍,选择,相关问题交流

thanks for ur kindness . u r really helpful . but it seems that only yorkdale adult learning school supplys those programms ,such as eca ....all the others have no practical programmes .

mangcang : 2008-07-11#88
回复: toronto college介绍,选择,相关问题交流

LZ,
你了解georgian college吗(在巴厘市).谢谢~~

木尘子 : 2008-07-11#89
回复: toronto college介绍,选择,相关问题交流

你到 TDSB的网站上看一下,我帮你找了几个成人学校,多市成人高中就这么多了,有就有,没有就没有。另外,COSTI也有免费的,www.costi.org.


School
Address
Postal
Code
500 The East Mall
M9B 2C4
1 Danforth Avenue
M4K 1M8
3395 Weston Road
M9M 2V9
939 Progress Avenue
M1G 3T8
38 Orfus Road
M6A 1L6
我决定去yorkdale学,不懂这个 ECA & PSW 哪一个就业好,好像day care center 要比老人院多 是么? 谢谢。

alexwei : 2008-07-11#90
回复: toronto college介绍,选择,相关问题交流

PSW 更好就业,工资也高。但是看你愿不愿做。

木尘子 : 2008-07-11#91
回复: toronto college介绍,选择,相关问题交流

PSW 更好就业,工资也高。但是看你愿不愿做。
谢谢, 我也在想,这个工作要不怕脏不怕臭,要有很高的服务精神。

alexwei : 2008-07-11#92
回复: toronto college介绍,选择,相关问题交流

要你就狠下心来做两年PSW,有些底子和经验,然后再转行做,因为确实PSW的就业形势现在比较好。

木尘子 : 2008-07-11#93
回复: toronto college介绍,选择,相关问题交流

要你就狠下心来做两年PSW,有些底子和经验,然后再转行做,因为确实PSW的就业形势现在比较好。
好像那个成人学校只要半年就拿个证书哦。

游客 : 2008-07-11#94
回复: toronto college介绍,选择,相关问题交流

好像那个成人学校只要半年就拿个证书哦。
psw 证书很好考,但你收否能干是另一回事。

alexwei : 2008-07-15#95
回复: toronto college介绍,选择,相关问题交流

psw 证书很好考,但你收否能干是另一回事。[/quo
赞同,最好和工作过的人谈一下。

三脚猫 : 2008-07-18#96
回复: toronto college介绍,选择,相关问题交流

我留意到Seneca里面有个专业是兽医(Veterinarian)的.请问兽医的就业情况和薪水是如何的呢?LZ有了解吗?难读吗?

alexwei : 2008-07-25#97
回复: toronto college介绍,选择,相关问题交流

兽医,不知道,完全陌生

相见不如怀念 : 2008-07-25#98
回复: toronto college介绍,选择,相关问题交流

转到这里来问!

楼主说的改成2年的课程是指考出来是证书,没有diploma的那种吧?一样找工作是吗?

如果我想做采购,参加这种就够了吧。但看要求入学门槛比较高哦。我英语很一般,没有雅思成绩,能不能申请到?



本来是做人力资源的,有10多年的工作经验了,但英语口语很一般,所以被迫转行,想来想去,以后吃EI的时候再读照顾老人孩子的那种证书,我的性格只适合短期做这样的工作,以便挣点钱接续生活,长远来看,我以后还是想做商务方面的工作。

所以决定向采购方向进军。

如果以后想转去UNIVERSITY读的,有没有学校可介绍的?能拿到diploma的。请楼主给点意见。谢谢。

annieyouy : 2008-07-26#99
回复: toronto college介绍,选择,相关问题交流

好贴,受藏+SW!

木尘子 : 2008-07-26#100
回复: toronto college介绍,选择,相关问题交流

学psw也可以去当eca ? really ?

alexwei : 2008-07-28#101
回复: toronto college介绍,选择,相关问题交流

学PSW中间也会有一点照顾孩子的内容,但是和ECA差别较大,我纠正。

玻璃糖纸 : 2008-07-29#102
回复: toronto college介绍,选择,相关问题交流

6月10日续,如何选择college呢,论坛上这类东西可多了,我谈谈自己看法。我觉得核心就一个,快,时间越短越好。
==================================================
最赞成楼主这个观点,说的好!

另外想请问下楼主Electronics Engineering Technology 这个专业怎么样,我查来查去还是觉得头昏脑胀的,周围没有朋友学这个,都是在学护理,助理牙医,会计什么的,实在不了解这边情况啊...
是在这个学校看见的http://postsecondary.humber.ca/03511.htm

非常感谢,对了能不能问下楼主是做什么工作的,怎么懂那么多啊?

alexwei : 2008-07-29#103
回复: toronto college介绍,选择,相关问题交流

6月10日续,如何选择college呢,论坛上这类东西可多了,我谈谈自己看法。我觉得核心就一个,快,时间越短越好。
==================================================
最赞成楼主这个观点,说的好!

另外想请问下楼主Electronics Engineering Technology 这个专业怎么样,我查来查去还是觉得头昏脑胀的,周围没有朋友学这个,都是在学护理,助理牙医,会计什么的,实在不了解这边情况啊...
是在这个学校看见的http://postsecondary.humber.ca/03511.htm

非常感谢,对了能不能问下楼主是做什么工作的,怎么懂那么多啊?

给你个数据,看起来找工作不难。我是学校的counsellor,谢谢夸奖

Electrical/Electronic Engineering Technologies (C362) - Career Program, Community College/CEGEP (2 or 3 years)

Earnings | In the Work Force






Average earnings after 2 years working

Graduates earned 16% more than all college graduates.

Graduates earned 24% more than trade/vocational graduates in similar educational programs.

Graduates earned 35% less than bachelor's graduates in similar educational programs.

Graduates earned 43% less than master's graduates in similar educational programs.





Importance of previous work experience

43% of those with previous work experience found a job within one month of graduation.
Labour Market Facts2 Years After Graduating
(%)
This ProgramAll Programs at this LevelIn the work force9495Full-time employed9388Part-time employed712Unemployed
Occupations of Graduates
Electrical Trades and Telecommunications Occupations41.0%Electronics and Electrical Engineering Technicians29.0%Machining and MetalworkingWoodworking Operators11.0%Stationary Engineers and Power System Operators11.0%Contractors and Supervisors, Trades and Related8.0%

What They Experienced

Graduates competed for work with trade/vocational graduates in construction technologies (T332), electrical/electronic engineering technologies (T362), computer science (T720) and in instrumentation (C382).

木尘子 : 2008-07-29#104
回复: toronto college介绍,选择,相关问题交流

mothercraft 已经不开eca 了 。一年的ece需要8000多刀学费!

玻璃糖纸 : 2008-07-29#105
回复: toronto college介绍,选择,相关问题交流

给你个数据,看起来找工作不难。我是学校的counsellor,谢谢夸奖

Electrical/Electronic Engineering Technologies (C362) - Career Program, Community College/CEGEP (2 or 3 years)

Earnings | In the Work Force






Average earnings after 2 years working

Graduates earned 16% more than all college graduates.

Graduates earned 24% more than trade/vocational graduates in similar educational programs.

Graduates earned 35% less than bachelor's graduates in similar educational programs.

Graduates earned 43% less than master's graduates in similar educational programs.





Importance of previous work experience

43% of those with previous work experience found a job within one month of graduation.
Labour Market Facts2 Years After Graduating
(%)
This ProgramAll Programs at this LevelIn the work force9495Full-time employed9388Part-time employed712Unemployed
Occupations of Graduates
Electrical Trades and Telecommunications Occupations41.0%Electronics and Electrical Engineering Technicians29.0%Machining and MetalworkingWoodworking Operators11.0%Stationary Engineers and Power System Operators11.0%Contractors and Supervisors, Trades and Related8.0%

What They Experienced

Graduates competed for work with trade/vocational graduates in construction technologies (T332), electrical/electronic engineering technologies (T362), computer science (T720) and in instrumentation (C382).


谢谢楼主这么快回答我,你的专业很合适额,你人好又有耐心:wdb10:

alexwei : 2008-07-29#106
回复: toronto college介绍,选择,相关问题交流

mothercraft 已经不开eca 了 。一年的ece需要8000多刀学费!
:wdb1:看来ECA的竞争太激烈了。ECE一年8000刀是正常偏贵的收费,但是不离谱。这个课程的内容和学习时间应该相当于community college两年的课程,community college的收费应该在tuition 5500+1500(books and others)=7000 左右。由于community college的教室等资源公用,班级比较大,所以收费平点正常。
看来你又要搜索了,不过你还是可以去瞧瞧的。真是个苦差事。:wdb7:不过你还是很实干的:wdb17:。加油。

木尘子 : 2008-07-29#107
回复: toronto college介绍,选择,相关问题交流

他们一点都不热情 呵呵 ,不像其他私立学校那么抢人。

alexwei : 2008-07-30#108
回复: toronto college介绍,选择,相关问题交流

首先更正一点,他们也是私立学校。我不确定他们是否不在乎,但是一般如果你接触的是本地人,他们的工作方式应该是这样的。西人由于从小就被教育自己选择,自己决定自己的事情,所以一般他们喜欢自己做结论,不喜欢别人给什么意见。所以对待西人客户的时候,一般来说,保持一个专业的态度,回答他需要了解的问题,走完需要的流程,是基本的工作方法。
其实,一个学校做得好不好,靠接待人员的热情起不了什么作用,靠的还是老师教学的质量,学生的质量和就业率。然后口碑相传。作为一个Consellor, 不仅要能够让潜在的学生充分了解学校,老师和课程,还要让学生知道如何度过这宝贵的时间,从而找到满意的工作。还有一个就是学生的质量,Conselor要对学生做评估,对于有些认为不适合学某一专业的学生还要泼冷水。否则到时候学生学不好,或者毕不了业,会怨学校,怨Consellor. 现在确实有些学校只顾招揽生源,不顾其他方面。
你还是可以亲身考察,用自己的眼睛,耳朵去选择合适的学校。

不能再小的豌豆 : 2008-07-30#109
回复: toronto college介绍,选择,相关问题交流

万分感谢楼主的热心, 资料太有用了. 我上星期刚到多伦多, 想找学校念PT, 不知道楼主有没有什么这方面的资料, 怎样念花费最少. 到college报名的时候是不是要测试英语, 如果英语程度不够, 要念他们esl课程吗?

alexwei : 2008-07-30#110
回复: toronto college介绍,选择,相关问题交流

万分感谢楼主的热心, 资料太有用了. 我上星期刚到多伦多, 想找学校念PT, 不知道楼主有没有什么这方面的资料, 怎样念花费最少. 到college报名的时候是不是要测试英语, 如果英语程度不够, 要念他们esl课程吗?
资料有,但是不知道你需要什么。这个课程的学习,你需要
1、英语至少linc 5以上
2、最好有一定医学,健康,药物方面的背景
我想你需要了解的东西不仅这些,包括就业前景,方向,薪资,课程内容,学费,金融资助,学校选择等。如果你需要了解详情,可以PM我联系方式,我可以提供些建议。

alexwei : 2008-07-30#111
回复: toronto college介绍,选择,相关问题交流

PT是一个正在走向规范化的职业,他现在的薪资不高,10-15元之间。但是行业协会对PT的要求却很高。因此PT的学习难度和成本和他的未来的收入不算合适。造成这种局面的原因,是因为传统药房视PT为一般的General的Clerk. 但是,药师的高昂成本和稀缺造成对PT的需求增加。因此,PT的供给和需求矛盾也在扩大。行业协会加紧对PT的规范,以使得PT承担更多专业工作,并具有更加专业的技能,并能够为未来进入药剂师打下良好基础。因此,PT的薪资处于一个缓慢上升的阶段,PT的专业水平也处于一个上升阶段,而且PT的经验可以帮助自己往更高的专业迈进,因此是一个非常不错的方向。

木尘子 : 2008-07-30#112
回复: toronto college介绍,选择,相关问题交流

好像牙医助理也不错哦。

游客 : 2008-07-30#113
回复: toronto college介绍,选择,相关问题交流

看见过牙医助理工作,牙医在那检查牙齿,牙医助理听写记录。

alexwei : 2008-08-05#114
回复: toronto college介绍,选择,相关问题交流

关于公立和私人职业学校的课程和资质,可以到www.edu.gov.on.ca上去查, 所有经过政府审批的学校,以及私立职业学校的经过政府审批的diploma的课程在上面都可以查到。

JING : 2008-08-07#115
回复: toronto college介绍,选择,相关问题交流

楼主可以介绍一下牙医助理和洗牙师吗?我听说这两样比较好找工作,是这样吗?对英语要求高吗?

蔚蓝 : 2008-08-08#116
回复: toronto college介绍,选择,相关问题交流

很好的贴子,加声望先!

alexwei : 2008-08-08#117
回复: toronto college介绍,选择,相关问题交流

楼主可以介绍一下牙医助理和洗牙师吗?我听说这两样比较好找工作,是这样吗?对英语要求高吗?
网上找到一个文章,供参考,是个学校的宣传来着。个人说两点:
关于英语,很多专业需要的工作语言要求都不高。但是我们不能光听学校的宣传。首先,只要是服务人的工作,相对于对着机器,电脑的工作,语言的要求要高很多。你无法设定你的客户(患者)会如何和你交流。你需要向客户解释很多事情。你或你的老板需要你通过交流保持和客户的良好关系。另外一方面,你的同事和老板需要和你沟通,能够自在交流的部下和同事可以使气氛活跃,工作顺畅。所以,就这两个工作或者还有其他工作,我觉得对语言要求不高恐怕要打折扣。当然会讲国语是个优势,不过在这里会几种语言的人多的是。你可以选择为本国移民工作,但是语言不好的人都挤在这里,结果就是工资大打折扣。而且一般诊所即使医生讲国语,也希望助理的英文好。有一个调查,说移民的工资大约平只有加国平均的一半,我看和我们的英语差距有些对应。
如果要学这个课,建议一定找西人教学的,最好的本地人,一年学下来,不会唱歌,至少也会哼哼。一开始听不懂,没关系,还可以看书。不过不要把学习中提高英语期望太高,因为这个课主要是听和动手,交流不多,如果要提高交流水平,学习中间还要建议参加提高口语的班。

工作职责:

虽然牙医助理和洗牙师都在在普通牙医诊所,专科牙医诊所 (齿髓病诊所,牙周病诊所,牙齿矫形诊所和牙医儿科等),医院的口腔科,牙科实验室以及公共健康部门等领域工作,但具体工作职责却完全不同。牙医助理的工作职责主要是辅助牙医Dentist对病人进行诊断和治疗,其工作包括准备相关器械和材料,消毒处理,拍牙片,取牙模等,同时指导病人如何保持口腔卫生和口腔护理。洗牙师的工作除了cleaning洗牙, fluoride抛光和加氟等洗牙具体工作之外,还包括拍摄X光照片,了解病人个人信息,病史,和提供牙齿健康教育。

就业前景:

牙医助理和洗牙师两种工作在加拿大一直以来都保持非常高的就业率。加拿大统计局根据各种统计数字做出对加拿大各类工作的综合评价;其中能够被评为“就业前景好”的行业屈指可数,例如大陆移民朋友非常青睐的会计员和文员等工作,统计局的评价均为“一般”。而牙医助理和洗牙师都被评为就业前景非常好的行业。

工资待遇:

牙医助理和洗牙师这两种工作在工薪和待遇方面差别较大。加拿大统计局提供的数据显示,牙医助理的平均工资在$16.51加元每小时,工作一般都是全职工作 Full Time,工作环境好,工作时间有规律,福利待遇好。另一方面,洗牙师工资就高很多,初入行的洗牙师都可以拿到$35加元每小时,在温哥华甚至可以拿到 $40每小时。洗牙师工作一般都从part time开始,三年以后找到全职带福利的工作的可能性很高。当然,洗牙师一般都回同时选择两份左右的part time工作,保证一周工作在35-40小时左右,并有选择工作时间的自由。

入门要求:

加拿大牙医助理分为几个等级,初级牙医助理只要有承认的专业毕业文凭Diploma就可以上岗;而二级牙医助理就需要持毕业文凭在相关部门进行考试,持专业执照上岗。一般来说,初级牙医助理的课程大概在8-10个月左右,而二级牙医助理的学习时间要再加上4-6个月。而洗牙师必须持有专业洗牙师协会颁发的专业执照才可以上岗工作,如果打算考取洗牙师牌照,必须首先完成洗牙师专业培训,洗牙师专业课程一般在18-36个月左右。

英语高求:

牙科技术工作对英语的要求都是整个医疗健康体系中较低的。工作性质决定了牙医助理和洗牙师都不需要与病人进行大量的英语沟通,简单的解答和服务英语就足够了。但是,因为从业人员必须拥有专业毕业证书,所以在学习当中必须要涉及一些基础的医疗健康知识。具备基本的英语能力的华人移民朋友,即使没有医疗背景,只要肯下功夫,都能在这两个职业方向有所发展。

蔚蓝 : 2008-08-08#118
回复: toronto college介绍,选择,相关问题交流

请教楼主,刚到加拿大二周,已经办好了所有的手续,现在就差着找学校了,由于语言不行,所以现在的想法就是先过语言关,然后再考虑着报college,据说成人高中里的ESL学习会紧张一些,所以就想着是不是去报个ESL,现在的问题就是往哪里找学校的问题,刚才到了你给的网站上去看了一下,有五家学校,看了几家,主要都是英文看着怪累的说,也不知道意思到底有没有搞懂,不知道可不可以请教楼主知不知道哪一家的成人高中较好呢?据说有的成人高中里修的学分还可以转到college或者university里。如果报名的话是要自己去学校里报吧?如果英文沟通不行咋办呀?岂不报名都没法报了?

alexwei : 2008-08-08#119
回复: toronto college介绍,选择,相关问题交流

去TDSB(Toronto District School Board的网站,www.tdsb.on.ca
那里的高中都可以, 会根据你的情况安排的,不用担心

bksbhc : 2008-08-08#120
回复: toronto college介绍,选择,相关问题交流

好帖,关注中.....

shanshanlan : 2008-08-09#121
回复: toronto college介绍,选择,相关问题交流

首先感谢alexwei,寻觅了这么久,终于找到明白人了。我在国内做医生,到加拿大深深了解了做回医生的难度,就想可以做个和医疗相关的工作。您提到的洗牙师在哪培训啊,另外男护士好就业吗,在哪培训啊,您了解心脏灌注师这个职业吗,我在MICHENER COLLAGE 看到介绍,这个职业还是能再回手术室啊,另外有没有什么collage有post-graduate,对以后还能做回医生有帮助啊还有想B 超技师,核医学技师的就业形式如何呢,能直接进行pharmcy的培训吗,不好意思,问题多了点

JING : 2008-08-10#122
回复: toronto college介绍,选择,相关问题交流

太感谢了,回复的真是很详细,还有你的建议。:wdb17::wdb6:

alexwei : 2008-08-11#123
回复: toronto college介绍,选择,相关问题交流

首先感谢alexwei,寻觅了这么久,终于找到明白人了。我在国内做医生,到加拿大深深了解了做回医生的难度,就想可以做个和医疗相关的工作。您提到的洗牙师在哪培训啊,另外男护士好就业吗,在哪培训啊,您了解心脏灌注师这个职业吗,我在MICHENER COLLAGE 看到介绍,这个职业还是能再回手术室啊,另外有没有什么collage有post-graduate,对以后还能做回医生有帮助啊还有想B 超技师,核医学技师的就业形式如何呢,能直接进行pharmcy的培训吗,不好意思,问题多了点
There are many colleges offering Detal Hygiene courses.you can check http://www.edu.gov.on.ca/eng/general/searchpcc.html for Private career college or http://www.edu.gov.on.ca/eng/general/list/college.html for Community College.
I don't have too much knowlegement about your interest. I am sorry about that and sugest you ask for help to others.

椰香公主 : 2008-08-11#124
回复: toronto college介绍,选择,相关问题交流

请问楼主HVAC这个专业怎么样?老公对这个比较感兴趣。谢谢!

beautyfish19726 : 2008-08-11#125
回复: toronto college介绍,选择,相关问题交流

我今天交了钱上一个学校要花5000块半年的PSW,但我在网上看到SENICA是3000多一年,我很犹豫,两个学校都能申请OSAP,我怎么办才好呢?

linlinchen : 2008-08-12#126
回复: toronto college介绍,选择,相关问题交流

谢谢楼主,请问楼主,在加拿大做pharmacist对学历、工作经验有什么要求,对于移民来说,是不是也象做医生那样高不可攀?:wdb17::wdb17::wdb17:谢谢!

我爱我家123 : 2008-08-12#127
回复: toronto college介绍,选择,相关问题交流

谢谢,好贴!

alexwei : 2008-08-12#128
回复: toronto college介绍,选择,相关问题交流

我今天交了钱上一个学校要花5000块半年的PSW,但我在网上看到SENICA是3000多一年,我很犹豫,两个学校都能申请OSAP,我怎么办才好呢?
senica的时间长,中间有假期,实际学习时间我估计在9个月左右,而且开学时间固定,不好选择,批给你的生活费不负责暑假部分,另外,你只看到了学费,公立大学的其他费用较高,一般还有一个统一的杂费,大概在6、7百块钱,还有书本和材料费,估计也要不少,而且这个课程上公立大学也没有多大意思。半年5000块的时间成本较少(早上一个月班工资也有2000块呢),应该每个月都能上课,不过能否批多少贷款未知,因为半年5000+生活费6个月约7200=12000。这个可以去osap网站上去模拟看看自己能拿到多少。对于半年的职业培训,如果经济允许,建议不要申请贷款学习,因为1、半年后就要开始还款,2、凡是申请OSAP贷款学习的学费都贵很多。比如这个课程,如果自费的话包括所有费用3000块左右,现在你申请OSAP,岂不是要多掏2000块! 而且有的学校可以分期付款
3、职业培训的学习一般都比较灵活,你可以选择白天班,晚上班,或者周末班,通过打些工来补贴日常费用。

所以,我觉得贷款上学是不得已的做法。

alexwei : 2008-08-12#129
回复: toronto college介绍,选择,相关问题交流

谢谢楼主,请问楼主,在加拿大做pharmacist对学历、工作经验有什么要求,对于移民来说,是不是也象做医生那样高不可攀?:wdb17::wdb17::wdb17:谢谢!
Yes.

alexwei : 2008-08-12#130
回复: toronto college介绍,选择,相关问题交流

请问楼主HVAC这个专业怎么样?老公对这个比较感兴趣。谢谢!
Sorry, I have no idea about HVAC

iryee : 2008-08-12#131
回复: toronto college介绍,选择,相关问题交流

很感谢您的热心,我准备10月长登卡城,我在国内做软件开发的,但最近两年做些项目经理的工作,对于编程不是很喜欢,但工作做的还行。主要做java相关的企业应用软件,英语一般,雅思5分水平, 我本科成绩 60- 70较多,我的年龄31岁。
1 请问我如果还做计算机相关的工作 要如何安排?需要读个collage吗? 那边对年纪有要求吗?工作累吗,我不喜欢编程的最大原因就是在国内加班很多,很累。
2 如果考虑转行的话,我的成绩是否可以申请读个当地不算太差的本科。当然英语成绩应该是必需的。

多谢了。。。

iryee : 2008-08-12#132
回复: toronto college介绍,选择,相关问题交流

对不起 我上面说的是

2 如果考虑转行的话,我的本科成绩是否可以申请读个当地不算太差的本科。当然英语成绩应该是必需的。

多谢了。。。

alexwei : 2008-08-12#133
回复: toronto college介绍,选择,相关问题交流

很感谢您的热心,我准备10月长登卡城,我在国内做软件开发的,但最近两年做些项目经理的工作,对于编程不是很喜欢,但工作做的还行。主要做java相关的企业应用软件,英语一般,雅思5分水平, 我本科成绩 60- 70较多,我的年龄31岁。
1 请问我如果还做计算机相关的工作 要如何安排?需要读个collage吗? 那边对年纪有要求吗?工作累吗,我不喜欢编程的最大原因就是在国内加班很多,很累。
2 如果考虑转行的话,我的成绩是否可以申请读个当地不算太差的本科。当然英语成绩应该是必需的。

多谢了。。。
不用转行,坚持下去就好了,college也没有必要读

beautyfish19726 : 2008-08-13#134
回复: toronto college介绍,选择,相关问题交流

senica的时间长,中间有假期,实际学习时间我估计在9个月左右,而且开学时间固定,不好选择,批给你的生活费不负责暑假部分,另外,你只看到了学费,公立大学的其他费用较高,一般还有一个统一的杂费,大概在6、7百块钱,还有书本和材料费,估计也要不少,而且这个课程上公立大学也没有多大意思。半年5000块的时间成本较少(早上一个月班工资也有2000块呢),应该每个月都能上课,不过能否批多少贷款未知,因为半年5000+生活费6个月约7200=12000。这个可以去osap网站上去模拟看看自己能拿到多少。对于半年的职业培训,如果经济允许,建议不要申请贷款学习,因为1、半年后就要开始还款,2、凡是申请OSAP贷款学习的学费都贵很多。比如这个课程,如果自费的话包括所有费用3000块左右,现在你申请OSAP,岂不是要多掏2000块! 而且有的学校可以分期付款
3、职业培训的学习一般都比较灵活,你可以选择白天班,晚上班,或者周末班,通过打些工来补贴日常费用。

所以,我觉得贷款上学是不得已的做法。
很感谢你,我已经去了那所私立的学校,半年的课程,但我还是选择了贷款,我这个账有点不清楚啊,政府给我12000,我还7000,我还用了5000,实际上我只花2000读这个课程,而且10年才还清,怎么会不划算呢,我的思维出了问题?

alexwei : 2008-08-14#135
回复: toronto college介绍,选择,相关问题交流

很感谢你,我已经去了那所私立的学校,半年的课程,但我还是选择了贷款,我这个账有点不清楚啊,政府给我12000,我还7000,我还用了5000,实际上我只花2000读这个课程,而且10年才还清,怎么会不划算呢,我的思维出了问题?


1、不知道政府是否已经批给你12000了。
2、是否确认可以减免5000。减免计划中是要给学生债务减免到7000的计划,一般是指针对一年的学习,并且和你的收入相关,根据的报税纪录。如果可以保证,应该也是不错。
3、建议你不要10年还完,那样即使你只还7000,你需要支付的利息已经5000了。
4、为了避免付利息,尽量在一年内还清。因为这个贷款的利率很高。

popjoy : 2008-08-15#136
回复: toronto college介绍,选择,相关问题交流

请问LZ, 对SENECA LAW CLERK 和 SOCIAL WORKERS 两个专业的就业机会怎么看? 另外根据您的了解, COMMUNITY AND PRIVATE COLLEGE 在这两个专业上区别大吗?
谢谢

chongxia : 2008-08-17#137
回复: toronto college介绍,选择,相关问题交流

关于college的介绍太有用了,我还想多了解些,我的邮箱
chongxiaqiu@yahoo.cn

佐伊 : 2008-08-17#138
回复: toronto college介绍,选择,相关问题交流

你好,请问读6个月的eca那种学校,如Yorkdale Adult Learning Centre

需要什么样英语水平呢,谢谢

alexwei : 2008-08-17#139
回复: toronto college介绍,选择,相关问题交流

你好,请问读6个月的eca那种学校,如Yorkdale Adult Learning Centre

需要什么样英语水平呢,谢谢
Go to Yorkdale and ask staff.

alexwei : 2008-08-18#140
回复: toronto college介绍,选择,相关问题交流

关于college的介绍太有用了,我还想多了解些,我的邮箱
chongxiaqiu@yahoo.cn
想了解什么

alexwei : 2008-08-18#141
回复: toronto college介绍,选择,相关问题交流

请问LZ, 对SENECA LAW CLERK 和 SOCIAL WORKERS 两个专业的就业机会怎么看? 另外根据您的了解, COMMUNITY AND PRIVATE COLLEGE 在这两个专业上区别大吗?
谢谢

1.Both have good Job future.
2.什么样的区别呢。基本上,private college 提供时间更短,实践性更强的学习,学生的时间成本少。很多人认为,学费是最大的成本,实际上学习期间不工作的成本才最大,我在另外一篇文章里写过,按照$9/小时的full time工作来计算,一年大概18000,加上学费5000-7000。估计在23000-25000。这就是你一年投资学习的成本,两年就是46000-50000。私人职业培训学校提供的培训在提供同样的技能的基础上,缩短了培训时间,我觉得适用于已经有多年工作经验的成年人。
3、很多人认为私人学校质量差,在加拿大很多私人学校,他们培养了无数很好的专业人才。很多老师有多年工作经验,可以给你最适用的工作技巧。
4、就这两个专业而言,我觉得community college和私人学校各有其优势,community college 基础学习更扎实一些,学习的时间更充分。私人college偏重实用,时间短,实习机会好

popjoy : 2008-08-18#142
回复: toronto college介绍,选择,相关问题交流

1.Both have good Job future.
2.什么样的区别呢。基本上,private college 提供时间更短,实践性更强的学习,学生的时间成本少。很多人认为,学费是最大的成本,实际上学习期间不工作的成本才最大,我在另外一篇文章里写过,按照$9/小时的full time工作来计算,一年大概18000,加上学费5000-7000。估计在23000-25000。这就是你一年投资学习的成本,两年就是46000-50000。私人职业培训学校提供的培训在提供同样的技能的基础上,缩短了培训时间,我觉得适用于已经有多年工作经验的成年人。
3、很多人认为私人学校质量差,在加拿大很多私人学校,他们培养了无数很好的专业人才。很多老师有多年工作经验,可以给你最适用的工作技巧。
4、就这两个专业而言,我觉得community college和私人学校各有其优势,community college 基础学习更扎实一些,学习的时间更充分。私人college偏重实用,时间短,实习机会好


LZ, 请问您能帮我推荐一下有这两个专业且较好的私校吗?

alexwei : 2008-08-18#143
回复: toronto college介绍,选择,相关问题交流

很多,请看我的第一份帖子的最前面的链接,你可以在那里查到。我所在学校有Community and Social Service 专业,但没有law clerk专业。基本上我觉得,大部分西人学校都还是各有所长的。你可以选择性的考察。

诚心请教 : 2008-08-19#144
回复: toronto college介绍,选择,相关问题交流

HI, ALEX, 请问读DENTAL HYGIENE PROGRAM哪个学校好些? 我想选择时间比较快的,但是也可以在业余时间可以打工帮补家用的,谢谢!

alexwei : 2008-08-19#145
回复: toronto college介绍,选择,相关问题交流

HI, ALEX, 请问读DENTAL HYGIENE PROGRAM哪个学校好些? 我想选择时间比较快的,但是也可以在业余时间可以打工帮补家用的,谢谢!
Hi, I would like to give you below link which you can find all colleges with Detal Higiene
http://www.riccpcc.serviceontario.c...rm_value_2=64&parm_type_3=STRING&parm_value_3=
Basically, It will take at least one and a half year to complete this course and internship for full time study and the tuition is very high. If you want to do some part time job, it is OK. I believe all colleges have a good study schedule.

alexwei : 2008-08-19#146
回复: toronto college介绍,选择,相关问题交流

很多人问我,公立College和私立College有什么区别,这个问题其实也困扰了我一段时间,我一直都在想一个问题。从公众舆论上看,在加拿大,似乎公立的College享有很好的信誉,光明的就业前景和雄厚的师资,而且有能力招收更多的学生。为什么私立college还有发展的空间。私立college和公立college的立足点差别究竟在哪里?作为私立college的一个教育顾问,我觉得有义务去探讨这个问题。
公立College是政府Funding建立的College, 同时,通过收取学费和其他费用来支持日常的营运。公立College主要有三个经营目标:
1、提供短期的培训,通常少于一年,满足学生学习专项技能或知识的需求,学生毕业后得到Certificate
2、提供大专文凭课程,通常需要2-3年,满足学生准备从事一种职业学习必备知识,技能的需求和继续进行university深造的需求,并且训练学生为未来的社会生活做好准备,学生毕业可以获得代表教育程度的大专文凭。
3、提供Apprenticeship 培训(专业技工培训?),比如木工,面包师,厨师等等
现在有些学校也提供一些快速毕业的大专文凭课程(1年)或者3-4年的Degree课程甚至研究生课程
公立College需要通过提高知名度,招收更多学生获得更多政府支持,改善员工福利,因此竞争也很激烈。
私立College 是私人根据安省私人职业培训法例投资成立的,并获得安省教育厅(MTCU)颁发许可证。许可证每年更新,MTCU每年会对各私人college进行审查,确保其设施,教学质量,教师资质达到要求,私立college每一项培训的收费,课程安排,时间设置也需要经过审批同意,所以在MTCU颁发的证书上不仅能够看到学校的名称,还列出了审批后的各个课程的名称。
私立College经营目标只有一个:提供紧凑的技能培训满足就业市场需求。私立college一般规模要比公立的小得多,名气也差很多,专注于比较少的课程。除了面临公立College的挤压外,私立College之间的竞争也非常激烈。
选择培训首先是选择学校,选择学校比选择课程更重要。我们大概都知道公立College的优势,因此,这里从私立college的特点来入手进行一些比较,由于很多私立学校有一些短期证书培训课程无法和公立College进行比较
1、由于私立college是私人所有,为了生存发展,他们对于就业市场的变化反应更迅速,并不断提供易于就业的课程或者根据需要对课程进行改进。
2、由于私立college专注于提供技能培训,所以学生不需要花大量时间学习基础学术知识,技能传授+实践是职业培训的基本教学方法。
3、私立college对于教师的选择非常慎重,教师必须具有多年行业的工作经验和培训的经验。
4、学生导向,私立college的生存基础是学生的就业,学生的口碑相传是私立学校发展的基石。因此私立college更注重学生的就业率和对课程的评价,对于教师的要求更加严格。
5、私立college的课堂通常都比公立college小,因此和学生有更多的互动和交流。通常学生在整个学习期间由一位老师执教,因此可以给予学生更多的个人指导。
6、私立college提供非常灵活的注册时间表,对大部分课程而言,学生每个月都可以注册开始学习,除了政府公众假期,没有寒暑假。一般有Full timepart time课程,满足不同需求。
7、私立college允许学生反复听课,来巩固自己的学习。

其实,正如我在另一篇文章里讲到的,培训的最大成本不在学费,很多人认为学费贵,实际上一般一年的课程所有费用不过在5000-7000左右,而真正贵的是机会成本,即如果你不工作,你所挣的钱。按照$9一个小时,应该在$18720。这样你一年的学习成本就是$23000---$25000,而两年的成本在$ 46000-$50000左右。对于承担着家庭重担的成年人来说,是一个很大的成本。因此,职业培训的选择和上学不同。学生应该在获得技能的同时,尽量缩短学习时间,这也是选择职业培训的重要原因。
另外私立college对于学生资质的要求其实更高,他们在学生入学前通过交谈和测试,来判断学生的技能,兴趣和学习的动力是否合适所学的专业。因为在这里,学生也是顾客,如果没有给学生的学习提供正确的建议,就是没有尽到学校的职责,耽误学生的职业生涯。
那么确实我看到很多对于私立职业学校的负面的评论,我想一方面,是大家心底里对于私立两个字的不信任,而且市场上也很难找到一个标准来衡量私立学校。另外一些私立学校确实存在不重视教学质量的现象。我在这里需要说的是,大部分私立college非常重视教学质量,教师质量和就业率。以最短的时间获得专业的技能和工作,是选择职业学校最重要的理由。
我希望有时间接下来告诉大家私立college长得什么样,怎么样教学的,以及选择私立学校的问题清单,我去过很多这样的学校做调查,总结了一些,希望对大家有所裨益。(待续)

alexwei : 2008-08-19#147
回复: toronto college介绍,选择,相关问题交流

和Cindy 讨论了一个关于学费退税的问题,觉得对大家有所裨益,经cindy同意,放到我的帖子里。
Cindy:
你好!
我想报名读XX药物学院(私立的)fulltime 6个月的Diploma班,我本来在药厂工作.我在网上也查过这个专业是在安省教育局注册过的.学校说EI读收8900,自费4500,年底报税时给你T2202A表,政府退还460*6=2700左右.
请问1.这个政府退费的事是真的吗?是否有附加条件?
2.Seneca里也有相似的专业(一年制),读哪个找工的希望更大?

Alex:

1. 你学的什么专业内没有告诉我
2. 关于退税,哪个学校有误导的嫌疑,培训是可以抵税的,比如你年收入50000,交了15000的税,但是你付了4000块学费,政府可以从税里面减掉学费。但是这个事情很复杂。如果你那一年没有收入,也就没有税可交,也就没有可以抵的,因为政府已经给你最大福利了。如果你低收入,你可抵的税很少,政府已经给了你GST退税了,学费抵得就很少了。他给你列的情况估计是最理想的了。这个东西需要会计师帮你算。

Cindy:
谢谢你及时答复,我刚从网上查到T2202A表就是给读完书有工作的人抵税用的.所以他们的说法而是有点问题.
我选的专业是药物分析Diploma班,Seneca的chemical lab tech-pharmaceutical(CLP)要读3年,所以不想读了.
我不知他们找工的比率是否也是吹的,如果化4000块钱能找到专业工作那倒也算了.有没有其他好的建议?

非常感谢你的答复,让我少走弯路.谢谢了!

Alex:
You are welcome. I also learn new information from you. I don't know this college so I can't give you suggestion, however, I can give you some advice about searching a good school:
1. Know about the history of school, basically, long history means good reputation. But there are also some new schools with good reputation
2. ask for one to two free class, talking with teachers there, observing their facillityies, discussing with students
3. See if most of students are Chinese, if it is, it is not a good college. Good college should have different students.
4. Don't choose a college using Chinese as a teaching language.
5. Choose at least two private college, let them compare each other. You also can compare teachers and facillities.

popjoy : 2008-08-19#148
回复: toronto college介绍,选择,相关问题交流

很多,请看我的第一份帖子的最前面的链接,你可以在那里查到。我所在学校有Community and Social Service 专业,但没有law clerk专业。基本上我觉得,大部分西人学校都还是各有所长的。你可以选择性的考察。


LZ在那个学校啊?

alexwei : 2008-08-19#149
回复: toronto college介绍,选择,相关问题交流

PM

zhanzhan7 : 2008-08-19#150
回复: toronto college介绍,选择,相关问题交流

介绍一下社会工作者行业

个人奋斗方向:如果成为专业social worker, 工资可以达到70000,但是一般需要有social science master degree. 而 bachelor 毕业实际上和college毕业薪资差别不是太大,我个人觉得可以在拿到diploma后工作两年,直接申请master(用国内的学历)+diploma+工作经验,这样不错。


这个帖子对我们新移民有极大的帮助啊!:wdb10:

请问alexwei, "(用国内的学历)+diploma+工作经验"申请master的这个工作经验是指拿到本地diploma之后的相关工作经验吗?
另外,我在国内是读化学的,国内经验与化学和会计无关,是否可以申请加拿大的会计的diploma呢?毕业之后能否申请master?
谢谢!


不行吧
这里上说是都要荣誉学士毕业的,diploma肯定不行

szfrost : 2008-08-19#151
回复: toronto college介绍,选择,相关问题交流

LZ真是热心人啊。

有个问题请教请教。我LP原来在国内一直在银行工作,现在正在打工,同时计划学习。她想学会计,以后不想做银行工作;但背景也没有干过专门的会计,希望能找个短期的课程培训一下北美的会计基础知识和软件操作,也就是出来后能做BOOKKEEPING或会计入门工作就行,所以不知道该学什么专业。
LZ能否指点一下
短期培训(6个月-1年的)最好学那类PROGRAM?
有那些可选择的学校?
她打工已满900小时,具备EI条件,什么时候开始学比较好?

望抽空回复为盼

popjoy : 2008-08-20#152
回复: toronto college介绍,选择,相关问题交流

LZ, 我大致看了一下学校网站.有几个问题:

CSW: 毕业后只有DIPLOMA, 就业怎样? 是否属于这个行业的最初级?

CSW: 需要多长时间完成学习? 费用多少, 学校提供实习吗?

我在华人开的LAW OFFICE工作, 但9月结束(AS CONTRACT JOB FOR MATERNITY LEAVE) 想请您帮忙给些建议: LAW CLECK 和 OFFICE ADMINISTRATION FOR LEGAL 在实际就业时的区别怎样?OFFICE ADMINISTRATION 有必要学DIPLOMA 吗?



THANKS IN ADVANCE

alexwei : 2008-08-20#153
回复: toronto college介绍,选择,相关问题交流

LZ, 我大致看了一下学校网站.有几个问题:

CSW: 毕业后只有DIPLOMA, 就业怎样? 是否属于这个行业的最初级?

CSW: 需要多长时间完成学习? 费用多少, 学校提供实习吗?

我在华人开的LAW OFFICE工作, 但9月结束(AS CONTRACT JOB FOR MATERNITY LEAVE) 想请您帮忙给些建议: LAW CLECK 和 OFFICE ADMINISTRATION FOR LEGAL 在实际就业时的区别怎样?OFFICE ADMINISTRATION 有必要学DIPLOMA 吗?



THANKS IN ADVANCE


Law Clerk 是一个正式的职业,office Administration for legal 恐怕是个夹生产品,不学也罢。Office Administration 有没有diploma 差别不大。但是如果能够进入这个行业,还是至少需要几个月时间学习,主要是熟悉这里使用的软件,掌握高级的MS office的使用技巧,提高打字速度,掌握电话技巧,办公室一般工作流程,商业写作技巧,行政管理基本知识等。
Law Clerk, 就业前景一般,不过可以号称法律顾问(做一些简单的咨询工作),开办自己的咨询公司。薪资在21块左右,最高31,最低13。
CSW diploma 课程,25weeks. 就业前景向好,平均薪资20元左右,最低13。75,最高28。29。不属于初级岗位,属于专业岗位。薪资参考:(Service Canada)
http://www.labourmarketinformation....occ=4212&job=&search_key=3&bkmark=1&details=1
提供实习。

alexwei : 2008-08-20#154
回复: toronto college介绍,选择,相关问题交流

不行吧
这里上说是都要荣誉学士毕业的,diploma肯定不行
本地diploma申请肯定不行,如果国内有承认的人文方面本科学历,可以通过靠托福,申请在这里读social worker的研究生,本地的diploma和工作经验作为你申请研究生的资历。

alexwei : 2008-08-20#155
回复: toronto college介绍,选择,相关问题交流

LZ真是热心人啊。

有个问题请教请教。我LP原来在国内一直在银行工作,现在正在打工,同时计划学习。她想学会计,以后不想做银行工作;但背景也没有干过专门的会计,希望能找个短期的课程培训一下北美的会计基础知识和软件操作,也就是出来后能做BOOKKEEPING或会计入门工作就行,所以不知道该学什么专业。
LZ能否指点一下
短期培训(6个月-1年的)最好学那类PROGRAM?
有那些可选择的学校?
她打工已满900小时,具备EI条件,什么时候开始学比较好?

望抽空回复为盼
1、银行薪资很好,多伦多作为加国金融中心,工作机会多,如果英语不错,建议通过考CSC等进入银行工作,比学会计容易。
2、打工满900个小时,如果laid off, 可以拿到EI, 在EI领取2个月左右可以开始申请政府funding.申请大约需要3-4个月时间。
3、6个月可以拿一个会计的大专文凭了,可以申请EI学费读书。
4、学会计的学校遍地都是,注意教学资质的考察,和建议最好选择由本地有经验的会计师授课的课程。
如果需要咨询进一步问题,发个PM给我,电话沟通。

greensleeves : 2008-08-20#156
回复: toronto college介绍,选择,相关问题交流

看了楼主的帖子,真是受益匪浅哦。崇拜!!
有个问题请教楼主,我打算申请centennial 和georgebrown的practical nursing,看了他们的assessment 的要求,好像是要考生物,化学,数学之类的,因为我学这个专业纯粹是门外汉,要从头开始,要好好准备估计才能过关被录取(希望),请问这种学校的录取考试有没有指定书目呢?还有相关题型和范围能否知道呢?我看他们网站上好像没有过多描述了,有那种sample也不是这个专业的。
谢谢楼主!纷纷表示了..

appleSZ : 2008-08-20#157
回复: toronto college介绍,选择,相关问题交流

留个记号!

wexliu : 2008-08-20#158
回复: toronto college介绍,选择,相关问题交流

楼主辛苦了!
楼主对防火工程有了解吗?我报了SENECA COLLEGE的防火工程技术专业,正在犹豫是否要去上。从学校网页看,就业率似乎很高,但我一直不太相信。到招工网站上搜寻,招聘广告不多。马上要开学了,心理还没有一点底。望楼主指点指点。多谢!

szfrost : 2008-08-20#159
回复: toronto college介绍,选择,相关问题交流

1、银行薪资很好,多伦多作为加国金融中心,工作机会多,如果英语不错,建议通过考CSC等进入银行工作,比学会计容易。
2、打工满900个小时,如果laid off, 可以拿到EI, 在EI领取2个月左右可以开始申请政府funding.申请大约需要3-4个月时间。
3、6个月可以拿一个会计的大专文凭了,可以申请EI学费读书。
4、学会计的学校遍地都是,注意教学资质的考察,和建议最好选择由本地有经验的会计师授课的课程。
如果需要咨询进一步问题,发个PM给我,电话沟通。


多谢多谢,马上就PM你。费心了

beryl.ca : 2008-08-20#160
回复: toronto college介绍,选择,相关问题交流

好人:wdb10:

osaka : 2008-08-21#161
回复: toronto college介绍,选择,相关问题交流

2、打工满900个小时,如果laid off, 可以拿到EI, 在EI领取2个月左右可以开始申请政府funding.申请大约需要3-4个月时间。
请问打工满900小时是必须固定在一个地方,还是几个地方累积起来也可以?谢谢.

alexwei : 2008-08-21#162
回复: toronto college介绍,选择,相关问题交流

上次有TZ问到关于税收抵减的问题,为了尽可能让大家明白加拿大在学生学费方面的税收政策,特地找到了一个注册会计师李金萍女士在网上的文章,非常专业。

税款抵减额与所得扣除额不同之处是在於后者是造成所得的减少,而前者是直接抵减税款。


可以申报的税款抵减额包括:基本个人免税额、高龄免税额、配偶免税额、合格扶养人免税额、CPP供款、EI保费、本身的学费和教育费、医疗费用、慈善费用等。


李金萍解释,本身的学费(tuition fees for self)作为抵减额专案,要求付给加拿大的大学或专科以上的教育机构或就业移民部(The Minister of Employment and Immigration)认证的职业训练机构的学费,每个教育机构学费金额在100元以上者,可享受免税额。


可抵减的学费包括入学申请费、图书馆及实验室费、测验费、强制性的电脑服务费、函授课程?的书籍讲义费、证书及文凭费。另外一些附加费,例如体育及保健服务费(但不包含学生协会活动费)也可以抵减,但一般的膳宿费则不可以抵扣。


另外,李金萍也提醒,国外的大学若所暂册的课程最少连续13周的时间,?为全时间课程,?取得学位者,学费也可能可以抵扣,但要视该大学是否为加拿大税务局所认可的。若某国外大学?不在税务局认可的名单?,需要一定的程式申请。


除了学费,教育费(education amount for self)也可申报。李金萍表示,作为全时间学生(full time student),每月可申报400元的教育费免税额。部分时间学生(part time student)至少连续3星期的课程,每月至少12个小时,每月的免税额是120元。2006年又新增的一项教科书减免额(textbook tax credit)规定:凡符合领取全时间(full time)教育款项的学生,有每月65元的减免额,部分时间(part time)学生减免额为每月20元。


若学生本身不需要使用全部的学费、教育费用和教科书抵减时,剩余的部分可先转给配偶、父母或祖父母,但一个子女可转移的学费、教育费用和教科书费用不能超过5,000


李金萍也指出,子女请家教或补习班的费用是不可以抵扣的;学习钢琴、才艺等非运动型的学费也不可以抵扣,但若才艺音乐学校开具T2202 T2202A的学费收据则可以申报。

问:我在2003年没有收入,不需要交税。所以,我当年的学费不用报税,等以后有收入时再报税可以吗?

答:不妥当。正常的方法是:将学费的税表T2202T2202(A)及时报税,税务局会给你一份Notice of Assessment,列出你还有多少金额没有使用,这些金额可以在自己有收入的时候抵税。也可以转给配偶、父母、祖父母抵税使用。有的纳税人仅用收据或支票存根报税,而没有拿到T2202T2202(A),吃亏很多。因为,按规定,每上一个月的Full-Time 课程,增加$400抵税额;每上一个月的Part-Time,增加$120抵税额。
举例说明:王先生在2003年读Full-Time课程五个月,学费$4,000。同年,又上Part-Time课程两个月,学费$1,000。如果仅以收据,即实际支付金额计算,可以抵税的金额为$5,000正确的抵税金额计算为:$4,000 + 1,000 + 400 x 5 + 120 x 2 = 7,240
若以最低税率22%计算,损失的税款为:(7,240-5000) X 22% = 493

greensleeves : 2008-08-21#163
回复: toronto college介绍,选择,相关问题交流

看了楼主的帖子,真是受益匪浅哦。崇拜!!
有个问题请教楼主,我打算申请centennial 和georgebrown的practical nursing,看了他们的assessment 的要求,好像是要考生物,化学,数学之类的,因为我学这个专业纯粹是门外汉,要从头开始,要好好准备估计才能过关被录取(希望),请问这种学校的录取考试有没有指定书目呢?还有相关题型和范围能否知道呢?我看他们网站上好像没有过多描述了,有那种sample也不是这个专业的。
谢谢楼主!纷纷表示了..
抱歉,再发一遍,麻烦楼主帮忙解答一下。谢谢!!

alexwei : 2008-08-21#164
回复: toronto college介绍,选择,相关问题交流

看了楼主的帖子,真是受益匪浅哦。崇拜!!
有个问题请教楼主,我打算申请centennial 和georgebrown的practical nursing,看了他们的assessment 的要求,好像是要考生物,化学,数学之类的,因为我学这个专业纯粹是门外汉,要从头开始,要好好准备估计才能过关被录取(希望),请问这种学校的录取考试有没有指定书目呢?还有相关题型和范围能否知道呢?我看他们网站上好像没有过多描述了,有那种sample也不是这个专业的。
谢谢楼主!纷纷表示了..
抱歉,再发一遍,麻烦楼主帮忙解答一下。谢谢!!
没有这方面经验,找正在学的学生问一下吧。

alexwei : 2008-08-21#165
回复: toronto college介绍,选择,相关问题交流

2、打工满900个小时,如果laid off, 可以拿到EI, 在EI领取2个月左右可以开始申请政府funding.申请大约需要3-4个月时间。
请问打工满900小时是必须固定在一个地方,还是几个地方累积起来也可以?谢谢.
一年之内,打工满900个小时,几个地方都可以。

greensleeves : 2008-08-21#166
回复: toronto college介绍,选择,相关问题交流

没有这方面经验,找正在学的学生问一下吧。
谢谢你!

alexwei : 2008-08-26#167
回复: toronto college介绍,选择,相关问题交流

一年之内,打工满900个小时,几个地方都可以。

一般情况下,你需要被laid off, 不过也有例外,具体可以去www.servicecanada.ca上面看

qipeng : 2008-08-28#168
回复: toronto college介绍,选择,相关问题交流

一直为专业事情困挠,今天第二次读这个帖子,收获很多。楼主真诚热心解答,值得赞叹!真心表示感谢!

nmffs : 2008-08-28#169
回复: toronto college介绍,选择,相关问题交流

好贴当收藏!

laodudu : 2008-09-01#170
回复: toronto college介绍,选择,相关问题交流

您好,Alexwei,
仔细读了(两遍)您的帖子及各位的交流问题,非常受启发:wdb10:
我的大致情况是:现在国内等DM,年龄40+,工科硕士,正操持自己的一个小公司。
我想若抵加经过一段安顿期后,能选择一个自己感兴趣,与当前所做的事(销售,市场)相关联,而且短平快的BUSINESS方面的专业学习。另外,最好是PART-TIME的课程(有时间可以打打短工贴补家用:wdb6:),还可以有目的的强化英语:wdb1:
我想所学课程越实用越好,所谓“实用”当然指的是就业率高一些(职位再低也无所谓),或者干一段时间后有能力自雇也不错。
这些凭空的想法不知是否现实,请您多给一些指教。
我初步选择的方向是MARKETING OR INTERNATIONAL TRADE。目的就是做一个适应加国环境的SALES。不知这两个方向是否适合于我?若较适合,哪些学校和专业比较好?
请ALEXWEI多费心解答。谢谢您:wdb17:

Thumbelina : 2008-09-01#171
回复: toronto college介绍,选择,相关问题交流

帖子很好,很强大!:wdb20:
谢谢楼主,关于报税的部分对我很有用。:wdb10:

alexwei : 2008-09-02#172
回复: toronto college介绍,选择,相关问题交流

一般来说,移民一开始会选择操作性的岗位,如会计,payroll, 采购,物流管理等,sales和marketing的岗位由于语言和文化的弱势,一开始较少人做,除非是做中国人自己生意的。如果确实对这个感兴趣,去Seneca或者GerogeBrown,Humber等学习个两年-三年的课程,等你出来的时候,你对这里也熟悉了,语言也差不多了,networking也有了。



您好,Alexwei,
仔细读了(两遍)您的帖子及各位的交流问题,非常受启发:wdb10:
我的大致情况是:现在国内等DM,年龄40+,工科硕士,正操持自己的一个小公司。
我想若抵加经过一段安顿期后,能选择一个自己感兴趣,与当前所做的事(销售,市场)相关联,而且短平快的BUSINESS方面的专业学习。另外,最好是PART-TIME的课程(有时间可以打打短工贴补家用:wdb6:),还可以有目的的强化英语:wdb1:
我想所学课程越实用越好,所谓“实用”当然指的是就业率高一些(职位再低也无所谓),或者干一段时间后有能力自雇也不错。
这些凭空的想法不知是否现实,请您多给一些指教。
我初步选择的方向是MARKETING OR INTERNATIONAL TRADE。目的就是做一个适应加国环境的SALES。不知这两个方向是否适合于我?若较适合,哪些学校和专业比较好?
请ALEXWEI多费心解答。谢谢您:wdb17:

popjoy : 2008-09-02#173
回复: toronto college介绍,选择,相关问题交流

请问LZ, PRIVATE COLLEGE 和 COMMUINTY COLLEGE 的DIPLOMA 有任何区别吗?

alexwei : 2008-09-02#174
回复: toronto college介绍,选择,相关问题交流

都是大专文凭,都归安省Ministry of Training, colleges and Universities 管。
A private career college is a school that prepares students for a specific job, or gives them specific skills, such as computer skills. There are more than 500 private career colleges in Ontario.
Community College offers:
Certificate programs, which take 1 year or less
Diploma programs, which take 2 or 3 years
Apprenticeship and certification programs for skilled trades such as a carpenter, chef, or welder
Programs that lead to a bachelor degree.
Programs offered with universities that can give you a degree and a diploma.

梦里雪飞扬 : 2008-09-02#175
回复: toronto college介绍,选择,相关问题交流

很详细有用的资料.谢谢!

laodudu : 2008-09-03#176
回复: toronto college介绍,选择,相关问题交流

一般来说,移民一开始会选择操作性的岗位,如会计,payroll, 采购,物流管理等,sales和marketing的岗位由于语言和文化的弱势,一开始较少人做,除非是做中国人自己生意的。如果确实对这个感兴趣,去Seneca或者GerogeBrown,Humber等学习个两年-三年的课程,等你出来的时候,你对这里也熟悉了,语言也差不多了,networking也有了。
谢谢您的答复。
对于我这样四张多的人,再上2-3年的学,感觉太长了些,能否推荐一些一年以内的课程和学校呢?多谢!

alexwei : 2008-09-03#177
回复: toronto college介绍,选择,相关问题交流

如果上marketing, 或者Trade, 不要diploma, 又是part time 学习,一年左右,建议到seneca读一个 相关certificate专业,你可以去他们网站查一下.如果想要diploma, 一年在community college里面做不到,你可以到私立大专,Canadian Business School. 不过费用肯定很高。

laodudu : 2008-09-03#178
回复: toronto college介绍,选择,相关问题交流

如果上marketing, 或者Trade, 不要diploma, 又是part time 学习,一年左右,建议到seneca读一个 相关certificate专业,你可以去他们网站查一下.如果想要diploma, 一年在community college里面做不到,你可以到私立大专,Canadian Business School. 不过费用肯定很高。
非常感谢,Alex,
您所说的这个Canadian Business School,是私立大专的通称,还是个具体学校?他的学费大致多少啊:wdb26:

alexwei : 2008-09-03#179
回复: toronto college介绍,选择,相关问题交流

是私立学校,Canadian business college, 就在我们学校不远地方,你可以Google一下,具体课程不是太清楚,费用估计向Marketing 类的,52-60周的课程,应该在13000以上,20000以下的区间(Total)

alexwei : 2008-09-03#180
回复: toronto college介绍,选择,相关问题交流

Labour Market Bulletin Greater Toronto Region Service Canada

An Analysis of the Greater Toronto Region Labour Market - First Quarter 2008

An overview of the Greater Toronto Region (GTR) Statistics Canada Economic Region 530 - It consists of the City of Toronto plus the surrounding regions of Durham, York, Peel and Halton (excluding Burlington). This area includes the municipalities of Ajax, Pickering, Whitby, Oshawa, Clarington, Uxbridge, Scugog, Brock, Georgina, East Gwillimbury, Whitchurch-Stouffville, Newmarket, Aurora, King, Vaughan, Richmond Hill, Markham, Mississauga, Brampton, Caledon, Halton Hills, Milton and Oakville.
There were 2,700 fewer jobs in the first three months of 2008 compared to the last quarter of 2007. The first quarter of 2008 saw a slight increase in the labour force for the Greater Toronto Region (GTR) with the addition of more than 10,000 people. While part-time employment increased by 12,100 positions, this could not offset the loss of 14,800 full-time jobs. The number of unemployed rose by almost 13,000 increasing the unemployment rate to 6.5% this quarter compared to 6.1% in the last quarter of 2007.
Compared to the same time last year the labour market appears stronger. There were 3,044,500 people employed in this quarter, an increase of 68,800 from the same period last year, with noteworthy growth in both full and part-time employment. The most significant improvement in employment over the past year has been found in the services-producing sector, with the largest increase in public administration. Professional, scientific and technical services and health care and social services also contributed to this sector’s growth. However, due to difficulties experienced by manufacturing, the goods-producing sector continues to lose ground. Since this time last year the GTR has lost about 10,000 manufacturing jobs.

alexwei : 2008-09-03#181
回复: toronto college介绍,选择,相关问题交流

Work value 是你在选择职业的时候要考虑的事情,选择职业后才能选择培训,因此,对于下面的问题,我们可以自己按照Very Important,Somewhat Important,Not Important 标注问题,对于特别在意的方面做好标注。
WORK VALUES EXERCISE

Work near your home
________

Work within 1 hour of home
________

Work for a small company
________

Work for a large company
________

Work outdoors
________

Work indoors
________

Be physically active
________

Work at a desk
________

Work with others
________

Work alone
________

Be part of a team
________

Compete with others


Be given work and directions
________

Work independently
________

Do same things each day
________

Have a variety of tasks
________

Be recognized for
achievements
________

Help others
________

Make a good income
________


________

helenshu : 2008-09-03#182
回复: toronto college介绍,选择,相关问题交流

楼主的帖子非常有用,我的情况是现在正在申请政府的EI培训,选的是accounting,不知道楼主对学这个专业有没有好的学校推荐?

alexwei : 2008-09-04#183
回复: toronto college介绍,选择,相关问题交流

楼主的帖子非常有用,我的情况是现在正在申请政府的EI培训,选的是accounting,不知道楼主对学这个专业有没有好的学校推荐?
关于学校选择的建议,好像我在帖子里有提到。我希望你能到我们这里看看我们的accounting课程。

alexwei : 2008-09-04#184
回复: toronto college介绍,选择,相关问题交流

近来收到了不少朋友发来的PM, 统计一下,总共有57条之多,我也尽量回答了。不过57个中,很多问题是重复的。因此,我也重复了多次。为了更有效率,同时让大家都能提供有价值信息,我建议,尽量放在帖子里问,都是匿名的,我想是没有关系的。除非发联系方式,问联系方式什么的。请支持。

kevin6825 : 2008-09-05#185
回复: toronto college介绍,选择,相关问题交流

问: toronto college of technology 是以IT为主的私校, 它的口碑和就业前景如何? 哪位了解?请帮忙答复.

alexwei : 2008-09-05#186
回复: toronto college介绍,选择,相关问题交流

不是很了解。
Diploma课程基本情况和费用如下:
http://www.riccpcc.serviceontario.c...STRING&parm_value_3=102189&parm_type_3=STRING

实地观察:
1、看广告,凡是广告言过其实的学校必然有点猫腻。诚信为立校之本。比如使用100%字样,煽动性的话语,一般不符合西人学校的风格
2、参观校园,如果是以IT为主,主要观察几点,第一,电脑设备是否完备充足,是否在下课后仍有条件使用电脑,很多地方把课安排的很满,一下课就赶你走,一般正规学校都有一些电脑可以下可使用的,第二,软件使用,是否都是正版,敢于买盗版的学校没有好学校,你想想,软件的钱都省,请老师的钱不也会省?
3、听课,听听老师讲课,首选本地老师,为什么,一举两得,既学IT,又学英语,别选外国老师,把口音带跑了,得不偿失,还不如讲中国话呢。看看学生使用的教材,如果是短期培训,一般只有讲义。但如果是diploma培训,学校应该提供教材,如果学生手里拿的都是复印件,一定有问题。
4、学生数量,去私立学校,不要以为学生多就好,应该合理,这样你才能有更多机会和老师交流,学生应该多元化。
5、老师和接待人员的谈话对照,一般先和接待人员谈,然后和老师问一下,验证几个问题,老师的行业经验和资历,如果有co-op,问一下co-op的事情,课程的设置,在校学生数量,就业情况,老师一般都会告诉你比较真实的情况,如果老师叫你问学校,就有点提前被授意的感觉,老师应该比谁都了解这些情况。

如果有疑问,就货比三家。


希望帮到你。

kevin6825 : 2008-09-07#187
回复: toronto college介绍,选择,相关问题交流

非常感激alexwei的建议.可能的话,可留下您的e-mail?以后可方便联系.

星星的新家 : 2008-09-07#188
回复: toronto college介绍,选择,相关问题交流

好帖!:wdb17::wdb17:
有个问题请教一下lz,偶也想去多伦多读个college,现在还在等待dm,如果利用等待的时间考个托福或雅斯之类的,是不是就可以凭这个成绩进college呢?通常哪种成绩会得到认可呢?大约要多少分啊?
先谢过了!:wdb10::wdb10:

alexwei : 2008-09-07#189
回复: toronto college介绍,选择,相关问题交流

College 不需要toefl成绩,有自己的考试,如果考过,分数也可以做参考。门槛很低,如果你Toefl成绩好,不如去university.

czsurgeon : 2008-09-07#190
回复: toronto college介绍,选择,相关问题交流

你的很多说法是一种误导:

1 公立学院肯定好与私立学院,而且学费也便宜不少。
2 OSAP贷款肯定合算,如果9月入学,一般有3000千禧奖,有小孩的话还有1200的补助,这些都不用还。
3 一般不管你贷多少,最后2学期只还7000
4 学生贷款利息是PRIME+1左右, 但这是可以抵税的,算下来也和房贷差不多了,为什么要过早还?
5 PSW只是考个证,还好说一点,不过最近一个国人办的私立学院的文凭不被政府承认,可是给学的人带来不少麻烦哦!
5 如果有公立学院可以上,当然不选择私立的啦。

1、不知道政府是否已经批给你12000了。
2、是否确认可以减免5000。减免计划中是要给学生债务减免到7000的计划,一般是指针对一年的学习,并且和你的收入相关,根据的报税纪录。如果可以保证,应该也是不错。
3、建议你不要10年还完,那样即使你只还7000,你需要支付的利息已经5000了。
4、为了避免付利息,尽量在一年内还清。因为这个贷款的利率很高。

LINI : 2008-09-08#191
回复: toronto college介绍,选择,相关问题交流

我现在还在国内,准备是明年登陆。在国内是药剂师,过去想先读个PT,后争取考pharmacist。想问下LZ,我是先打工900h拿EI才上college,还是直接上college好?两样有什么区别?有人说拿EI不能上学,到底怎么回事?
还有,听说在多伦多读书比蒙城贵得多,且福利不如蒙城,这方面你有了解吗?
另外,我对那些什么退税、报税、抵税等财会专业的名词特头痛,但那方面一壳不通将来肯定要吃亏,请问在报税时会不会有人辅导?
问题较多,盼一一答复,万分感谢。

LINI : 2008-09-08#192
回复: toronto college介绍,选择,相关问题交流

忘了一个重要问题:如果我想不上university而直接考pharmacist,可以吗?是不是要申请同等学历认证?这个认证有什么要求,怎么做(国内准备)?
谢了.

alexwei : 2008-09-08#193
回复: toronto college介绍,选择,相关问题交流

蒙城福利不清楚,去蒙城坛口看看,多伦多毕竟是第一大城市,自有其特点。据说可以通过认证考试将国内的药剂师变成加国承认的,叫做bRIDGE PROGRAM. 听说有这种培训班,学10个月还是?记不清了,但是听一个医生朋友讲,即使通过了,用人单位还是很难接受,他们要这边上UNIVERSITY的,未经求证。能直接上学就不要打工,早点进入专业工作。

WWW.OCPINFO.COM 安省药剂师网
WWW.PEBC.CA 管药剂师考试的


我现在还在国内,准备是明年登陆。在国内是药剂师,过去想先读个PT,后争取考pharmacist。想问下LZ,我是先打工900h拿EI才上college,还是直接上college好?两样有什么区别?有人说拿EI不能上学,到底怎么回事?
还有,听说在多伦多读书比蒙城贵得多,且福利不如蒙城,这方面你有了解吗?
另外,我对那些什么退税、报税、抵税等财会专业的名词特头痛,但那方面一壳不通将来肯定要吃亏,请问在报税时会不会有人辅导?
问题较多,盼一一答复,万分感谢。

alexwei : 2008-09-08#194
回复: toronto college介绍,选择,相关问题交流

5 如果有公立学院可以上,当然不选择私立的啦 ?
1 公立学院肯定好与私立学院,而且学费也便宜不少?

5 PSW只是考个证,还好说一点,不过最近一个国人办的私立学院的文凭不被政府承认,可是给学的人带来不少麻烦哦!为国人悲哀。

2 OSAP贷款肯定合算,如果9月入学,一般有3000千禧奖,有小孩的话还有1200的补助,这些都不用还.YES
3 一般不管你贷多少,最后2学期只还7000, YES
4 学生贷款利息是PRIME+1左右, 但这是可以抵税的,算下来也和房贷差不多了,为什么要过早还?(YES)

LINI : 2008-09-09#195
回复: toronto college介绍,选择,相关问题交流

非常感谢!看来要做回本行都是要痛苦的了。听说去做美容师或是按摩师之类的可能只接受几个月的简单培训就可以了。每小时11刀起呢,好象也不比这药理助理差。而且语言要求不高。不知是不是?

laodudu : 2008-09-10#196
回复: toronto college介绍,选择,相关问题交流

是私立学校,Canadian business college, 就在我们学校不远地方,你可以Google一下,具体课程不是太清楚,费用估计向Marketing 类的,52-60周的课程,应该在13000以上,20000以下的区间(Total)
这样的课程会不会带CO-OP呢?就业前景如何?谢谢!

alexwei : 2008-09-10#197
回复: toronto college介绍,选择,相关问题交流

不清楚了,直接问吧

lindazhu2006 : 2008-09-10#198
回复: toronto college介绍,选择,相关问题交流

ALEX, 我对社工这个专业很感兴趣,你觉得这个行业的就业前景如何?
其实选择读书,我最在乎的是就业前景,及是否适合自己。我个人认为,社工这个专业还是比较适合我的,现在我关心的就是就业问题了。

我听人说,这个专业的就业方向大都是些非盈利机构。我们平时在网上,报纸上都很少看到这些机构招聘。那毕业后如何找工作,你知道一二吗?谢谢!

alexwei : 2008-09-10#199
回复: toronto college介绍,选择,相关问题交流

就业前景, 很好,我应该讲过多次了, 薪资, toronto, 2007劳工部数据,平均薪资20.6/h, 最高28, 最低13。
非盈利机构招收社工有自己独立方法。最常用的是资源共享,有个网站,www.charityvillage.com 你可以看一下。
社工只是大方向,你还要知道你具体想要从事什么工作,比如,青年工作者,安居服务,就业辅导,无家可归人士服务,教育服务,孩子服务,老人,社会福利,康体,妇女等等,每个方向的要求都有一定差别,学习中需要重点研究,才能找到满意工作。
如果你还没有决定学习,建议你先作义工,积累经验。如果你准备学习,也可以边学习边作义工。

lindazhu2006 : 2008-09-10#200
回复: toronto college介绍,选择,相关问题交流

谢谢!ALEX, 你们学校有社工这个专业吗?
你们除了做OSAP,做不做EI呢?
学生毕业后,你们又是如何帮助学生就业的呢?
可能问题较多。如果方便的话,我们可以QQ交谈吗?你可以在悄悄话里告诉我你的QQ。我加你

alexwei : 2008-09-10#201
回复: toronto college介绍,选择,相关问题交流

我没有QQ, 我PM一个电话,你愿意打也可以。不愿意PM也可以。

alexwei : 2008-09-12#202
回复: toronto college介绍,选择,相关问题交流

又改版了?

wanderinrain : 2008-09-22#203
回复: toronto college介绍,选择,相关问题交流

senica的时间长,中间有假期,实际学习时间我估计在9个月左右,而且开学时间固定,不好选择,批给你的生活费不负责暑假部分,另外,你只看到了学费,公立大学的其他费用较高,一般还有一个统一的杂费,大概在6、7百块钱,还有书本和材料费,估计也要不少,而且这个课程上公立大学也没有多大意思。半年5000块的时间成本较少(早上一个月班工资也有2000块呢),应该每个月都能上课,不过能否批多少贷款未知,因为半年5000+生活费6个月约7200=12000。这个可以去osap网站上去模拟看看自己能拿到多少。对于半年的职业培训,如果经济允许,建议不要申请贷款学习,因为1、半年后就要开始还款,2、凡是申请OSAP贷款学习的学费都贵很多。比如这个课程,如果自费的话包括所有费用3000块左右,现在你申请OSAP,岂不是要多掏2000块! 而且有的学校可以分期付款
3、职业培训的学习一般都比较灵活,你可以选择白天班,晚上班,或者周末班,通过打些工来补贴日常费用。

所以,我觉得贷款上学是不得已的做法。

我即将登陆,看到这个帖真是时候。谢谢,顶起

ketaoxq : 2008-09-24#204
回复: toronto college介绍,选择,相关问题交流

看了楼主的帖子,真是受益匪浅哦。崇拜!!
有个问题请教楼主,我打算申请centennial 和georgebrown的practical nursing,看了他们的assessment 的要求,好像是要考生物,化学,数学之类的,因为我学这个专业纯粹是门外汉,要从头开始,要好好准备估计才能过关被录取(希望),请问这种学校的录取考试有没有指定书目呢?还有相关题型和范围能否知道呢?我看他们网站上好像没有过多描述了,有那种sample也不是这个专业的。
谢谢楼主!纷纷表示了..
抱歉,再发一遍,麻烦楼主帮忙解答一下。谢谢!!

同问哦,也想知道,也想过去读护士

stonehaven : 2008-09-25#205
回复: toronto college介绍,选择,相关问题交流

Alex,需要课程介绍,给个e-mail吧,万分感谢!!!:wdb17::wdb17::wdb17:
e-mail:luckydejinjin@gmail.com


应你要求,先讲这个:
PT: Pharmacy Technician
就业前景:Very Good, 还是和老年社会有关。
薪资水平:如果在零售,10-15块,如果在社区community health care. 10-20块,医院:19-26块起,有些人成为了药剂师,工资约7万/年起,但是要再继续学习获得degree.
性别:男女均衡
语言:要求较高,学习辛苦,因为有大量东西要记,中国人毕业率高,老外经常吃不消。
自雇:自己做生意的占15%。 年收入较高。
学位:diploma, 同时还有行业的几个certificate
学习时间:40周,要学得抓紧,政府马上要把它改成两年的课程了
课程介绍:太长了,需要的给mail.

popjoy : 2008-09-25#206
回复: toronto college介绍,选择,相关问题交流

请问LZ对成为NUTRITIONIST所需要的学历要求及程序等的了解吗?或者可否给以有相关专业的学校介绍?
谢谢

SallySun2007 : 2008-09-26#207
回复: toronto college介绍,选择,相关问题交流

好贴,顶!

告诉我萱萱 : 2008-09-28#208
回复: toronto college介绍,选择,相关问题交流

楼主我也需要pt的课程介绍
多谢你啦
tellmesharon@gmail.com
辛苦你啦

alexwei : 2008-09-30#209
回复: toronto college介绍,选择,相关问题交流

楼主我也需要pt的课程介绍
多谢你啦
tellmesharon@gmail.com
辛苦你啦

我觉得你的背景学PT没有太大必要,PT适合医学和药学背景的人在加拿大从新走向专业岗位的一种选择,本身PT的工资并不高。你可以继续考虑你这个方向。

nickaihua : 2008-09-30#210
回复: toronto college介绍,选择,相关问题交流

好人好贴!

osaka : 2008-10-01#211
回复: toronto college介绍,选择,相关问题交流

我觉得你的背景学PT没有太大必要,PT适合医学和药学背景的人在加拿大从新走向专业岗位的一种选择,本身PT的工资并不高。你可以继续考虑你这个方向。
请问旅游方面的专业毕业好找工作吗?谢谢.

告诉我萱萱 : 2008-10-01#212
回复: toronto college介绍,选择,相关问题交流

旅游方向有没有比较好的专业啊?
可不可以推荐一个。。。。。。
我觉得你的背景学PT没有太大必要,PT适合医学和药学背景的人在加拿大从新走向专业岗位的一种选择,本身PT的工资并不高。你可以继续考虑你这个方向。

alexwei : 2008-10-03#213
回复: toronto college介绍,选择,相关问题交流

旅游方向有没有比较好的专业啊?
可不可以推荐一个。。。。。。
酒店管理专业。

告诉我萱萱 : 2008-10-03#214
回复: toronto college介绍,选择,相关问题交流

酒店管理专业。
酒店管理出来工作怎么样啊?
我以前遇见的一个女孩子也是学酒店管理
多大毕业的也只是做前台

alexwei : 2008-10-06#215
回复: toronto college介绍,选择,相关问题交流

It is very natrual for a graduate from UOT working from entry level. Think about those graduates in universities in China. In Canada, work experience is the most important thing. If you have many years experience combined with related local education background, you will be easy to find a professional job. But if you are a new graducates from universities(if you don't have related work experiences before), you still need find a very entry level job. Many local graduates work in service industry as cashiers, waiters, date clerk, recepionist and labor worker for one or two years after they graduate from universities. Do you really want to spend 3 years + 2 years to finish career change? Hotel management is always not so good, not so bad. But if you work for hotels for some time, your experiences can help you work in any places in the world. No one can say they will stay in Canada forever. So think the further futures.

sundy529721 : 2008-10-06#216
回复: toronto college介绍,选择,相关问题交流

谢谢楼主提供的资料信息,我打算请教楼主,我在国内是学的工业分析,现在想转行学PSW或者PAYROLL,因为都没有接触过,不知道哪个就业更容易些。

alexwei : 2008-10-08#217
回复: toronto college介绍,选择,相关问题交流

PSW is much easy to find a job. Payroll is hot, but actually if you don't have any background in HR or accounting, it is very hard to find a job. Only big companies need specific payroll clerks or outsourse their payroll to some professional companies.

告诉我萱萱 : 2008-10-11#218
回复: toronto college介绍,选择,相关问题交流

看到回帖已经晚了好几天
还是非常感谢你的建议和这么多的帮助
我已经决定继续我的专业
谢谢&感恩节快乐

It is very natrual for a graduate from UOT working from entry level. Think about those graduates in universities in China. In Canada, work experience is the most important thing. If you have many years experience combined with related local education background, you will be easy to find a professional job. But if you are a new graducates from universities(if you don't have related work experiences before), you still need find a very entry level job. Many local graduates work in service industry as cashiers, waiters, date clerk, recepionist and labor worker for one or two years after they graduate from universities. Do you really want to spend 3 years + 2 years to finish career change? Hotel management is always not so good, not so bad. But if you work for hotels for some time, your experiences can help you work in any places in the world. No one can say they will stay in Canada forever. So think the further futures.

给茶杯续水 : 2008-10-12#219
回复: toronto college介绍,选择,相关问题交流

目前在国内已递交申请,但是还没有拿到visa,是91新政后的case,可不可以现在就申请college,先过去呢?这样可以节省时间呀,看到目前申请遥遥无期,真的很急呀!先谢谢啦!

philip : 2008-10-12#220
回复: toronto college介绍,选择,相关问题交流

谢谢,我现在国内供电公司工作,电气工程师,明年准备长登多伦多,请楼主给个建议我选择何种专业比较好。另,本人已38岁,继续学习是为了找工作。

alexwei : 2008-10-17#221
回复: toronto college介绍,选择,相关问题交流

目前在国内已递交申请,但是还没有拿到visa,是91新政后的case,可不可以现在就申请college,先过去呢?这样可以节省时间呀,看到目前申请遥遥无期,真的很急呀!先谢谢啦!

If you apply for college right now, you belong to international students. That means you need pay four times tuition than immigrants.

alexwei : 2008-10-17#222
回复: toronto college介绍,选择,相关问题交流

谢谢,我现在国内供电公司工作,电气工程师,明年准备长登多伦多,请楼主给个建议我选择何种专业比较好。另,本人已38岁,继续学习是为了找工作。

Too many choice for you. don't worry. when you land toronto, I can give you some suggestion. 电工 is a basic choice.

tittat : 2008-10-18#223
回复: toronto college介绍,选择,相关问题交流

1、想读计算机方面的,多伦多读哪个COLLEGE好?CERTIFICATE会不会比DIPLOMA更容易就业?就业后薪资会因为持的证书是CERTIFICATE而不是DIPLOMA就少了?
2、现在还在国内,还没有去还没有租房子,没有地址,能在国内通过网申请吗?
3、现在还来得及申请1月份的吗?
4、多伦多计算机方面的COLLEGE好找工作吗?

alexwei : 2008-10-19#224
回复: toronto college介绍,选择,相关问题交流

如果本身是学计算机的,干计算机的,再读计算机college完全没有必要,找个热门的考个证就行了。比如QA. 如果本身不是干计算机的,劝你也别学,大把专业人才,你现在才开始学有点晚了。你要根据你的特长来。
2.你登录了才能申请
4.计算机方面能力比文凭重要多了

1、想读计算机方面的,多伦多读哪个COLLEGE好?CERTIFICATE会不会比DIPLOMA更容易就业?就业后薪资会因为持的证书是CERTIFICATE而不是DIPLOMA就少了?
2、现在还在国内,还没有去还没有租房子,没有地址,能在国内通过网申请吗?
3、现在还来得及申请1月份的吗?
4、多伦多计算机方面的COLLEGE好找工作吗?

就是过日子 : 2008-10-20#225
回复: toronto college介绍,选择,相关问题交流

敬爱的alexwei,麻烦你也帮我出出主意。
先说说我的情况,比较特殊,尽量写清楚。
1.我,男性,投资移民,登陆的时候估计35岁。
2.在国内是职高学历,买了一个大学文凭。职高学的是计算机,也已经落后了。
3.和朋友合伙开公司,这些年全靠嘴忽悠了,没啥手艺。
4.现在在复习英语,我保证至少到六级,托福争取80分。
5.不想也没条件坐吃山空
6.上了学将来找个工作,不求发展,但求平静生活。
7.毕竟是投资移民,暂时没有经济和家庭负担。
8.不怕辛苦。
9.我自己曾经想学旅游(不是酒店管理的那种)和社工。
麻烦你啊,帮我看看这样的情况学什么好些呢?辛苦了。

alexwei : 2008-10-20#226
回复: toronto college介绍,选择,相关问题交流

要看你的爱好,有很多工作可以做。社工是个不错的方向,不过工作起来并不轻松。任何工作开始几年都是比较辛苦的。你可以来了先读college,等两年后出来,生活和语言都适应了,工作机会就大了

alexwei : 2008-10-20#227
回复: toronto college介绍,选择,相关问题交流

如果不想读两年书,边打labor边学语言也行。只要语言学好了,找个好点工作就容易了,学习也容易了

mm01 : 2008-10-21#228
回复: toronto college介绍,选择,相关问题交流

http://www.georgebrown.ca/Marketing/FTCal/fashion/F102.aspx
请楼主邦看看这个专业的找工前景如何?这个专业和我先生以前在国内的工作有些关联,他想上这个专业,我担心这个毕业了找工前景不是很好,谢谢了

就是过日子 : 2008-10-21#229
回复: toronto college介绍,选择,相关问题交流

多谢啊,我就准备干这个了。呵呵

alexwei : 2008-10-21#230
回复: toronto college介绍,选择,相关问题交流

首先是你先生喜欢,并且擅长,没有这两点,学什么都找不到好工作,工作机会一直有,但是并不是热门。但也总是很多人找到满意工作。这个专业不是中国人擅长的。不过如果你先生有足够的兴趣和爱好,可以尝试



http://www.georgebrown.ca/Marketing/FTCal/fashion/F102.aspx
请楼主邦看看这个专业的找工前景如何?这个专业和我先生以前在国内的工作有些关联,他想上这个专业,我担心这个毕业了找工前景不是很好,谢谢了

就是过日子 : 2008-10-21#231
回复: toronto college介绍,选择,相关问题交流

给lz添加声望啦,呵呵,刚学会的。

嘴上不说心里却不安 : 2008-10-22#232
回复: toronto college介绍,选择,相关问题交流

非常感谢LZ。

明年登陆后打算申请COLLEGE DIPLOMA或GRADUATE CERTIFICATE,有些问题请教:
1。两年多前考的G类IELTS成绩可以作为英语水平证明吗?
2。1不行的话,登陆后参加的ESL/ELT等培训可以被接受吗?
3。1和2都不行的话,有无可能学校先录取再要求参加学校提供的相关语言培训吗?

先谢谢了。

mm01 : 2008-10-22#233
回复: toronto college介绍,选择,相关问题交流

谢谢楼主的答复,楼主真是好人,感谢!

alexwei : 2008-10-24#234
回复: toronto college介绍,选择,相关问题交流

考diploma不用这些成绩,参加他们自己的英语测试就行。行就上课,不行也会让你上课,加学一门英语而已。

非常感谢LZ。

明年登陆后打算申请COLLEGE DIPLOMA或GRADUATE CERTIFICATE,有些问题请教:
1。两年多前考的G类IELTS成绩可以作为英语水平证明吗?
2。1不行的话,登陆后参加的ESL/ELT等培训可以被接受吗?
3。1和2都不行的话,有无可能学校先录取再要求参加学校提供的相关语言培训吗?

先谢谢了。

alexwei : 2008-10-24#235
回复: toronto college介绍,选择,相关问题交流

最近回信慢,因为工作忙,但是放心,我还是会尽我所能的,其实大家帮助我的更多,我学到了很多很多东西,在这里感谢所有问问题的LZ.

rain123456 : 2008-10-26#236
回复: toronto college介绍,选择,相关问题交流

关于College的信息和专业选择找了好久,希望得到alexwei的建议:

1.我在国内学的是临床医学,但本科毕业后没有到医院,直接去了公司做客服,技术支持和数据管理.做的面比较杂,在加拿大找工作有很大的局限性.而且看了jobfuture上的资料,做customer service, office clerk的薪水和发展空间非常有限
2.我不太想选择nurse, PT, PSW的专业, medical sonographer/ultrasound technician的职业如何?现在看工资不错,不知以后几年的市场需求量怎么样?
3.加拿大的sonographer证书在别的国家/地区是否会被认可? 以后回国的话是否可以用上?
4.我看了michener的Ultrasound 课程需要5个学期, 学费和杂费加起来大概1完3的样子,这个时间和费用与将来的工作前景是否匹配?

问题比较多,麻烦您了,谢谢

samuelchina : 2008-10-26#237
回复: toronto college介绍,选择,相关问题交流

Too many choice for you. don't worry. when you land toronto, I can give you some suggestion. 电工 is a basic choice.

楼主,你好,我在国内是会计,原想过去参加会计短期培训继续做会计的,可是好象工资很低,还不如LABOR,而要想往上走就要再考那些证书,要花好几年时间,年龄也不小了,所以想找个技术工的培训算了,因为在无忧网上经常看到电焊工的招聘,而且工资不比货车司机低,真的那么好就业吗?还是他们是做培训广告吹的?还是说其实很辛苦所以工资高?请您给我个建议好吗?最坏就老老实实再做会计了,只是在国内考证都考怕了:wdb1:

alexwei : 2008-10-26#238
回复: toronto college介绍,选择,相关问题交流

你的跨度好大,如果你离开学校久了,又没有从医,估计你那些医学知识也丢的差不多了。首先问你个问题:你对医学相关专业有没有兴趣。我一直坚持的是首先你的职业要符合你的兴趣和长处。不然你学什么都没有很好前途。
1、customer service是个非常不错的入门职业,特别在银行和电讯等行业,这个纠正你。不仅工资还可以,时间灵活,而且工作机会多。并且你能够了解到很多业务知识,如果你肯学习,借着这个职位慢慢申请其他职位是完全可能的。另外这个职位锻炼语言,锻炼你的即时问题处理能力,又培训你。难点反而在于你能否申请到。因为要求很好的语言表达能力,还有基本上你要参加一些专业的培训。office Clerk就更不容易,如果在西人公司工作的话,行政人员基本上是本地人或者CBC. 中国人适合的职业并不是所谓entry level的职业,而是一些技术性岗位,因为技术的要求,对于其他的要求就会低些。
2。总体上医疗行业相关都不错,如果有兴趣就去学。未来6-7年都应该保持一个需求旺的形式。再以后谁知道。
3。考虑回流,这些工作没有什么意思,国内医院人才济济,人际关系复杂,凭一个国外的证书恐怕没什么用。
4。1.3万对于以后的收入来说是个小数字。

关于College的信息和专业选择找了好久,希望得到alexwei的建议:

1.我在国内学的是临床医学,但本科毕业后没有到医院,直接去了公司做客服,技术支持和数据管理.做的面比较杂,在加拿大找工作有很大的局限性.而且看了jobfuture上的资料,做customer service, office clerk的薪水和发展空间非常有限
2.我不太想选择nurse, PT, PSW的专业, medical sonographer/ultrasound technician的职业如何?现在看工资不错,不知以后几年的市场需求量怎么样?
3.加拿大的sonographer证书在别的国家/地区是否会被认可? 以后回国的话是否可以用上?
4.我看了michener的Ultrasound 课程需要5个学期, 学费和杂费加起来大概1完3的样子,这个时间和费用与将来的工作前景是否匹配?

问题比较多,麻烦您了,谢谢

alexwei : 2008-10-26#239
回复: toronto college介绍,选择,相关问题交流

告诉你一个事实,如果你以前没有做过,即使考了证,也很难找到电焊工作,因为这个工作操作性很强,老外从小动手能力强,考证不行,做起来行,国内大把技术背景的人在这里,你半路出家哪里有机会。最后只是拿个证而已。建议你还是选择非机工类的证书进行学习。首先,把这边的会计熟悉一下,一定用的上的。然后再看看你想做什么,

楼主,你好,我在国内是会计,原想过去参加会计短期培训继续做会计的,可是好象工资很低,还不如LABOR,而要想往上走就要再考那些证书,要花好几年时间,年龄也不小了,所以想找个技术工的培训算了,因为在无忧网上经常看到电焊工的招聘,而且工资不比货车司机低,真的那么好就业吗?还是他们是做培训广告吹的?还是说其实很辛苦所以工资高?请您给我个建议好吗?最坏就老老实实再做会计了,只是在国内考证都考怕了:wdb1:

嘴上不说心里却不安 : 2008-10-26#240
回复: toronto college介绍,选择,相关问题交流

考diploma不用这些成绩,参加他们自己的英语测试就行。行就上课,不行也会让你上课,加学一门英语而已。

万分感谢!

雅思7分水平能通过学校的测试吧?

alexwei : 2008-10-29#241
回复: toronto college介绍,选择,相关问题交流

一点问题都没有

samuelchina : 2008-10-30#242
回复: toronto college介绍,选择,相关问题交流

谢谢alexwei

selwynwan : 2008-10-30#243
回复: toronto college介绍,选择,相关问题交流

楼主,请教GAS TECHNICIAN的前景如何?

alexwei : 2008-10-30#244
回复: toronto college介绍,选择,相关问题交流

Gas Tech 应该属于服务类技术工作吧,一般这种工作在加拿大都还不错。

千里马 : 2008-10-30#245
回复: toronto college介绍,选择,相关问题交流

ALEX热心,帮顶

alexwei : 2008-10-31#246
回复: toronto college介绍,选择,相关问题交流

Thanks a lot.

zhouliyuan : 2008-11-03#247
回复: toronto college介绍,选择,相关问题交流

请问:我大学学的计算机应用,一直在信息中心从事计算机维护管理工作,但是多年不编程了,想在加拿大上个COLLEGE学习软件测试专业或是电工,LZ觉得前景如何?学费、学期多久?另外本人年近40,英语不好,不知能否通过英语测试,因为在阿省申请NAIT要求英语听说读写都要75分以上,怕通不过,所以想到多伦多上学,听说英语无论什么水平均可入学,只不过参加相应的英语班ESL,是吗?是先上英语,通过后再上专业课,还是一起上,算一门成绩?需要增加多少学费?如行,推荐个学校。非常感谢!

tittat : 2008-11-06#248
回复: toronto college介绍,选择,相关问题交流

我也想知道

alexwei : 2008-11-07#249
回复: toronto college介绍,选择,相关问题交流

软件测试不错,你的背景学起来不难,不过不需要上长期的课程,有很多软件测试工程师的培训,大概2-3个月part time学习,学费也不高。 就可以考取证书,年薪在4万-10万,视乎您的经验,而且对语言要求不是太高。如果你在多伦多,我倒是可以推荐,阿省不了解。电工专业 看你了,对于经验要求比较高,将来自己干,没有公司福利。
如果感兴趣QA, 可以PM我联系方式,我要给你推销:wdb23:,电工可以去常青藤。
请问:我大学学的计算机应用,一直在信息中心从事计算机维护管理工作,但是多年不编程了,想在加拿大上个COLLEGE学习软件测试专业或是电工,LZ觉得前景如何?学费、学期多久?另外本人年近40,英语不好,不知能否通过英语测试,因为在阿省申请NAIT要求英语听说读写都要75分以上,怕通不过,所以想到多伦多上学,听说英语无论什么水平均可入学,只不过参加相应的英语班ESL,是吗?是先上英语,通过后再上专业课,还是一起上,算一门成绩?需要增加多少学费?如行,推荐个学校。非常感谢!

osaka : 2008-11-08#250
回复: toronto college介绍,选择,相关问题交流

请教楼主:做过文科老师的人,除了幼教,还有没有和教育相关的职业可以选择的?看到过一个education assistant certificate ,具体做什么工作?谢谢.

千里马 : 2008-11-08#251
回复: toronto college介绍,选择,相关问题交流

ALEX:您帮我看看我哥哥的这个规划是否符合加国实情,如果可行,我将来也照本宣科
1 到加后读COLLEGE的PT,取得资格后先做PT,同时提交国内学历证明申请,获准后参加药剂师考试。
2 如国内学历证明不被认可,则读个本科,再申请参加药剂师考试。
你看如何?不求快,但求稳。BTW,PT课程改为两年的了么?他想登陆埃德蒙顿。
谢谢!

千里马 : 2008-11-08#252
回复: toronto college介绍,选择,相关问题交流

需要PT课程及入学条件资料 tj139@sohu.com 谢谢!

alexwei : 2008-11-08#253
回复: toronto college介绍,选择,相关问题交流

这个专业可以去做教育助理,就是做一些教师的辅助工作,有一个朋友考过,在TDSB工作,还不错,不过英语要求高。

请教楼主:做过文科老师的人,除了幼教,还有没有和教育相关的职业可以选择的?看到过一个education assistant certificate ,具体做什么工作?谢谢.

alexwei : 2008-11-08#254
回复: toronto college介绍,选择,相关问题交流

我记得北美技术学院有个药剂师的嫁接培训课程,可以通过直接考试获得药剂师的证书,你可以试一下。
ALEX:您帮我看看我哥哥的这个规划是否符合加国实情,如果可行,我将来也照本宣科
1 到加后读COLLEGE的PT,取得资格后先做PT,同时提交国内学历证明申请,获准后参加药剂师考试。
2 如国内学历证明不被认可,则读个本科,再申请参加药剂师考试。
你看如何?不求快,但求稳。BTW,PT课程改为两年的了么?他想登陆埃德蒙顿。
谢谢!

千里马 : 2008-11-08#255
回复: toronto college介绍,选择,相关问题交流

我记得北美技术学院有个药剂师的嫁接培训课程,可以通过直接考试获得药剂师的证书,你可以试一下。
据谣传要取消了,原来看过一个帖子,谢谢ALEX啦!

smilegirl : 2008-11-08#256
回复: toronto college介绍,选择,相关问题交流

能否介绍一下药物分析这个专业

alexwei : 2008-11-10#257
回复: toronto college介绍,选择,相关问题交流

不懂,但是凡是和健康有关的都是好专业,比较有名的是加拿大药学院,私立,课程都贼贵。不要去公立college, 这些药的东西,他们不会交。

popjoy : 2008-11-11#258
回复: toronto college介绍,选择,相关问题交流

请问楼主, 对YORKVILLE 和 TRIOS 这两所私校有什么建议和看法?另外怎样才能了解到某个私校的某专业在相关行业里的口碑呢?直接询问雇主好像不太实际。

alexwei : 2008-11-11#259
回复: toronto college介绍,选择,相关问题交流

私立学校大部分是正规的学校,如何选择一所私立学校呢,我觉得学校有没有名气不重要,因为名气再大也不过是个college. 我记得我说过,即便如公立college 也不过是培养技工和初级工作人员的地方,别指望学出来之后就可以横行天下,大把人出来才知道college是个什么分量。
搞清楚这一点,就是告诉你不要把学校的名气放在选择的范围内,要看什么呢?又要分两部分,大专文凭课程和短期培训课程(大部分是考证的培训)。
大专文凭课程又贵,时间又长,当然要慎重,有几点一定要考虑:
1、试听课程,和老师沟通。学校都允许试听。首先看看老师的教学水平,这些老师都是有行业工作经验的,而且拿小时工资,或者分成,大部分老师教的要好于公立college大锅饭里的老师,问问老师,就业率阿,薪资阿,就业方向阿,老师一般不会夸大。和sales讲的对照一下。要听就要听一天的课。学生就没啥可问的。
2、老师的经历一定要了解,私立学校,学校只是提供了教学实施,老师才是帮助你学习,就业的人,老师在行业内资历深,当然对你将来就业有好处。
3、有没有实习,怎样实习,多少时间,有没有明确写在course outline里面。这个更重要,有最好,其实学校里学的东西如果不去实习一下,工作的时候肯定抓瞎。实习单独收费吗
4、诚信,如果你通过和sales,老师聊天,发现有不一致的地方,或者感觉很多夸大的地方,就要小心,如果有的sales搬出所谓校长或者经理,给你很多承诺,更要小心,没有诚信的学校决不能选。天知道未来会怎么搞你。
5、课程的时间,看什么课程,一般老外学校喜欢把课程搞得很长,其实没有必要,还是短点好。
6、设备。如果用到设备,一定去看看。
7、至于什么流程阿,学校好不好看啊,大不大阿,统统不在考虑之列,做广告多了,自然学生就多,我们就把学校看成landlord, 关键还是深入进入看课堂和课程。有的学校教课就挤在一个小房间里,但是照样教学,就业高。因为好的老师一般轻易不会换地方。

如果有具体你想学的专业,或许我还能提供点意见。

关于短期培训,一定要看的是:
1、老师是否是行业内的高手,简单说,老师有没有工作,干什么样的工作,和文凭培训刚好相反,没工作的老师咱还不敢碰,为啥,短期培训的重点在于考一些专业证书,进入专业领域工作,老师的实际经验越丰富,讲的东西越对你今后进入这个领域有帮助。至于考证,没有谁搞不定。
2、价格,一般我们都希望价格越低越好,但是在这个领域要小心,有经验,职位高的老师,本身工资就高,利用业余时间搞搞培训,当然要价更高。没有工作的,或者根本找不到专业的工作的老师很多,13-20块钱可以请一大堆,这些老师能把你教到年薪8万的工作吗,痴人说梦吧。所以一定要记住一分钱,一分货的道理,把价格和老师的资历对应起来,做一个比较。千万别乱花钱,少花3-400块钱小事情,耽误几个月的时间可是大事情。

总之,考察考察考察,用自己的眼睛,自己的耳朵,自己的嘴巴,去看,去判断,无论公立,私立,无论中国人的还是西人的。理性思考,不要被名气两个毫无用处的字蒙蔽,不要道听途说。还有一点,考虑到自己的实际情况,交通啦,时间拉等等

wingwinggg : 2008-11-13#260
回复: toronto college介绍,选择,相关问题交流

请问一下各位了解护士入学要求前辈;

如果从头开始学习护士,申请入学除了托福雅思成绩还需要什么资料? 我了解到多伦多有college和U联合办学的4年护士学位课程,看到它们的网站要求内容比较头痛,模糊,所以特意上来请教各位,不知道它们要求的12年级的数学化学生物的什么意思,我们国内的高中成绩单可以吗?还是要到这里的成人高中修读这些课程,等成高毕业后才能申请?

有经验和了解的前辈,请多多指教.感谢.^_^

心如止水 : 2008-11-13#261
回复: toronto college介绍,选择,相关问题交流

请问:Ontario College of Art and Design 是university 还是College?这个学校怎么样?

米妮宝宝 : 2008-11-13#262
回复: toronto college介绍,选择,相关问题交流

Alex: 麻烦给介绍一下Dental Hygiene 和 PT哪个就业前景好?哪个好学些?哪个学校好?多谢啦

alexwei : 2008-11-17#263
回复: toronto college介绍,选择,相关问题交流

看名字当然是college. 查看college 的资质和所开的课程可以去
Ministry of training,colleges and university 上面,安省的都在上面
全国的如果是公立college和university
可以去 Association of colleges and universities
私立college可以去
Association of Private career collegies

对于学校我不敢推荐,请自己去判断,或者有在那里的介绍一下?
请问:Ontario College of Art and Design 是university 还是College?这个学校怎么样?

alexwei : 2008-11-17#264
回复: toronto college介绍,选择,相关问题交流

Dy 学习出来工资高些,但是full time 工作不多,自己干得多些。PT工资低,但是未来在药物健康领域发展是个起步。都不好学。学校不推荐,我原来在的学校的PT的老师不错,如果需要联系信息可以PM.
Alex: 麻烦给介绍一下Dental Hygiene 和 PT哪个就业前景好?哪个好学些?哪个学校好?多谢啦

alexwei : 2008-11-22#265
回复: toronto college介绍,选择,相关问题交流

最近心情不太好,因为发现华人学校的声誉不太好,为什么我们不能把学校办成名校,却总是把学校变成一个卖大白菜的地方。联想到Made in China的不良名声,不由得长叹一声,何时,何时这一切能够改变。

千里马 : 2008-11-23#266
回复: toronto college介绍,选择,相关问题交流

最近心情不太好,因为发现华人学校的声誉不太好,为什么我们不能把学校办成名校,却总是把学校变成一个卖大白菜的地方。联想到Made in China的不良名声,不由得长叹一声,何时,何时这一切能够改变。
陪你一起叹息:wdb7:

小雪 : 2008-11-23#267
回复: toronto college介绍,选择,相关问题交流

最近心情不太好,因为发现华人学校的声誉不太好,为什么我们不能把学校办成名校,却总是把学校变成一个卖大白菜的地方。联想到Made in China的不良名声,不由得长叹一声,何时,何时这一切能够改变。
ALEX.
我想请问你知道兽医的就业和读书情况吗?还有营养学的就业薪水都怎么样?好像加拿大人不是很重视营养是不是找工作不是很好找的啊?、、谢谢

alexwei : 2008-11-23#268
回复: toronto college介绍,选择,相关问题交流

Sorry, I don't understand your profession. You can visit www.jobfutures.com or www.labourmarketinformation.ca for answer.

小雪 : 2008-11-23#269
回复: toronto college介绍,选择,相关问题交流

Sorry, I don't understand your profession. You can visit www.jobfutures.com or www.labourmarketinformation.ca for answer.


我是护理专业大专但是我不太想过去做这个专业了,想去重新学一个专业,医学相关的,比如营养学还有兽医,不知到你有没有这方面的信息关于未来职业的发展就业情况还有薪水还有读书的相关信息!谢谢你的关注!

alexwei : 2008-11-24#270
回复: toronto college介绍,选择,相关问题交流

可以告诉我这两个专业的标准英文吗?或许我可以帮你search一下

jessieliu0807 : 2008-11-25#271
回复: toronto college介绍,选择,相关问题交流

好贴,感谢alexwei!
我在国内学的临床医学,也有几年临床经验,过去后想学与健康相关专业,希望不要超过2年的学时。这方面有好的推荐吗?
自己是比较喜欢康复师,ultrasound技师和洗牙师,不知道这些专业是否需要长时间的等待才能入学?学费大概要多少?

alexwei : 2008-11-25#272
回复: toronto college介绍,选择,相关问题交流

有很多学校教这些课程,不需要等,都还不错啦,大部分是超过一年的课程
好贴,感谢alexwei!
我在国内学的临床医学,也有几年临床经验,过去后想学与健康相关专业,希望不要超过2年的学时。这方面有好的推荐吗?
自己是比较喜欢康复师,ultrasound技师和洗牙师,不知道这些专业是否需要长时间的等待才能入学?学费大概要多少?
洗牙师应该在20000-30000加币左右吧,学习时间1年半左右
超声技师似乎便宜一点,主要是要考个证吧,康复师不错的行业,和洗牙师的费用差不多,16000-30000之间

joney : 2008-11-26#273
回复: toronto college介绍,选择,相关问题交流

有很多学校教这些课程,不需要等,都还不错啦,大部分是超过一年的课程

洗牙师应该在20000-30000加币左右吧,学习时间1年半左右
超声技师似乎便宜一点,主要是要考个证吧,康复师不错的行业,和洗牙师的费用差不多,16000-30000之间
能详细说明一下洗牙师的情况吗?听说证很难考。

sheaya : 2008-11-27#274
回复: toronto college介绍,选择,相关问题交流

最近心情不太好,因为发现华人学校的声誉不太好,为什么我们不能把学校办成名校,却总是把学校变成一个卖大白菜的地方。联想到Made in China的不良名声,不由得长叹一声,何时,何时这一切能够改变。


前段时间,朋友带我去了一间韩国超市,虽然面积很小,但东西摆放很整齐,也显得干净很多,同样的东西,那里就可以标价比华人超市略高。当时内心也在叹息,为什么同样是超市,同样一件事情,华人的东西,除了以价格来进行竞争外,就似乎没有别的出路了?!为什么,华人,总和“便宜”、“质劣”划上等号?!

alexwei : 2008-11-28#275
回复: toronto college介绍,选择,相关问题交流

真巧,我前几天也去了一趟韩国超市,确实不错。东西超正宗,在Young ST上,过了Steel 往北。中国人的民族性决定了我们的价格竞争策略。我们一向是:攘外必先安内。所以先要通过竞争把自己人灭了。结果大家都奄奄一息。

pingh : 2008-12-01#276
回复: toronto college介绍,选择,相关问题交流

请教了,我儿子在渥太华学语言,会上Algonquin College,原申请的是国际贸易,现觉得不行,想问问,上市场营销呢还是上企业管理呢,谢谢啦

pingh : 2008-12-01#277
回复: toronto college介绍,选择,相关问题交流

想问问哪个专业容易找工作或移民

芹菜 : 2008-12-02#278
回复: toronto college介绍,选择,相关问题交流

真巧,我前几天也去了一趟韩国超市,确实不错。东西超正宗,在Young ST上,过了Steel 往北。中国人的民族性决定了我们的价格竞争策略。我们一向是:攘外必先安内。所以先要通过竞争把自己人灭了。结果大家都奄奄一息。


我经常去那家吃,在超市里的便餐非常的好吃,建议试一下大骨头汤套餐和炒面,很地道!!!:wdb17:

芹菜 : 2008-12-02#279
回复: toronto college介绍,选择,相关问题交流

最近心情不太好,因为发现华人学校的声誉不太好,为什么我们不能把学校办成名校,却总是把学校变成一个卖大白菜的地方。联想到Made in China的不良名声,不由得长叹一声,何时,何时这一切能够改变。

来了多伦多一个月了,你的叹息我感同身受。都是中国人,我却不敢住在中国人聚集的地方,因为明显的脏乱差,真不想这么说。但是。。。。。。朋友做生意,货品都不敢说是中国来的,撒谎的时候我和她心里都挺难受的。

alexwei : 2008-12-02#280
回复: toronto college介绍,选择,相关问题交流

想问问哪个专业容易找工作或移民
如果是留学生,看看能不能找个本科的Business Admin上一下,上college要选择实用的科目,比如Accounting等。这个college很有名吗,我倒是不怎么听说过?

愚工 : 2008-12-02#281
回复: toronto college介绍,选择,相关问题交流

我是学工业电气自动化的,一直从事相关工作,主要是电气图设计,PLC, HMI和Visual Basic编程,包括现场调试,维护。计划明年长登多伦多,上网搜了加拿大的相关情况,发现做工程师没资格,做电工不受欢迎,不知道是不是要先读书才行?请楼主指点

pingh : 2008-12-02#282
回复: toronto college介绍,选择,相关问题交流

谢谢啦,我儿子在加上了二年高中,平均成绩70分左右,好象申请本科比较难吧

alexwei : 2008-12-03#283
回复: toronto college介绍,选择,相关问题交流

我不太了解这个专业,对不起
我是学工业电气自动化的,一直从事相关工作,主要是电气图设计,PLC, HMI和Visual Basic编程,包括现场调试,维护。计划明年长登多伦多,上网搜了加拿大的相关情况,发现做工程师没资格,做电工不受欢迎,不知道是不是要先读书才行?请楼主指点

alexwei : 2008-12-03#284
回复: toronto college介绍,选择,相关问题交流

本科是难搞定,但也不是没有,可以选择College+本科的项目,就是本科承认College的学分,这样college毕业后可以直接读大学的大三大四的课程,完成本科。不过这个本科不太可能是最著名的几所学校。
谢谢啦,我儿子在加上了二年高中,平均成绩70分左右,好象申请本科比较难吧

alexwei : 2008-12-03#285
回复: toronto college介绍,选择,相关问题交流

[FONT=宋体][/FONT]
我这里有一个关于就业机会分析的讲座12月6日
[FONT=宋体]经济衰退,金融危机,笼罩世界,人人自危。然而,危就是机,危机之中的机会永远留给那些有准备的人。肯尼迪邀请多名行业专家齐聚,畅谈不同行业的就业就会,提供专业建议,为您[/FONT]2009[FONT=宋体]年的职业规划带来金点子,让你为[/FONT]09[FONT=宋体]年机会做好最充分准备![/FONT]
[FONT=宋体]本次研讨会的目的:就业机会分析,工作职位介绍,薪金分析,找工技巧,升职策略,全部行业专家指引,经验分享,[/FONT]
[FONT=宋体]本次研讨会的主题:[/FONT]
SAS [FONT=宋体]的就业机会,工作职位,薪金分析,如何拿到[/FONT]SAS[FONT=宋体]工作,什么背景的人士适合从事[/FONT]SAS [FONT=宋体]工作[/FONT]
[FONT=宋体]主讲:[/FONT] Ms. Jenney Du[FONT=宋体],本市大医院任[/FONT]Senior Data Analyst[FONT=宋体];[/FONT]
Mr. Wei Lu, CIBC[FONT=宋体]高级数据分析师。[/FONT]
[FONT=宋体]时间:[/FONT]12[FONT=宋体]:[/FONT]30-1[FONT=宋体]:[/FONT]30

PMP[FONT=宋体]工作职位分析,在哪些领域就业,[/FONT]PMP[FONT=宋体]证书的价值[/FONT]
[FONT=宋体]主讲:[/FONT]Mr. Paul Li[FONT=宋体],[/FONT]10[FONT=宋体]年以上项目管理经验,长期担任跨国公司[/FONT] IT manager[FONT=宋体],现就职于某大型[/FONT]IT[FONT=宋体]跨国公司[/FONT]Senior BA[FONT=宋体]。[/FONT]
[FONT=宋体]时间:[/FONT]1[FONT=宋体]:[/FONT]30-2[FONT=宋体]:[/FONT]30

[FONT=宋体]质量管理在各行业的应用,[/FONT]CQE [FONT=宋体]的价值,工作机会,未来的发展空间[/FONT]
[FONT=宋体]主讲:[/FONT]Mr. Tony Li, [FONT=宋体]曾就职于医药,建筑,重型机械领域担任质量工程师和项目经理,现就职于某大型汽车厂质量管理岗位[/FONT]
[FONT=宋体]时间:[/FONT]2[FONT=宋体]:[/FONT]30-3[FONT=宋体]:[/FONT]20

Six Sigma+Lean - [FONT=宋体]生产管理顶尖技术在北美企业的应用,为您带来的就业机会,[/FONT]Six Sigma[FONT=宋体]证书的价值,未来的职业发展空间[/FONT]
[FONT=宋体]主讲:[/FONT]Mr Carl Fan, 17[FONT=宋体]年质量管理和[/FONT]Six Sigma[FONT=宋体]管理经验,持有[/FONT]CSSBB[FONT=宋体],[/FONT] CQE, CQA, CQM and CQIA [FONT=宋体]等高级国际认证,同时也是多伦多六西格玛先驱之一。[/FONT]
[FONT=宋体]时间:[/FONT]3[FONT=宋体]:[/FONT]20-4[FONT=宋体]:[/FONT]20

SAP [FONT=宋体]行业工作机会分析及介绍[/FONT]
1. SAP [FONT=宋体]是什么?[/FONT]
2. [FONT=宋体]为什么[/FONT]Fortune 500[FONT=宋体]排名前[/FONT]500[FONT=宋体]强的公司中有[/FONT]80%[FONT=宋体]都在用[/FONT]SAP?
3. [FONT=宋体]什么背景的人适合学[/FONT]SAP?
4. SAP[FONT=宋体]就业的前景如何?[/FONT]
5. SAP[FONT=宋体]工作的[/FONT]hidden market[FONT=宋体]在哪[/FONT]?
6. SAP[FONT=宋体]工作的职位都有哪些,[/FONT]title[FONT=宋体]都是什么,日常工作是什么?[/FONT]
7. SAP[FONT=宋体]工作的薪金是在什么[/FONT]level?
8. [FONT=宋体]从事[/FONT]SAS[FONT=宋体]工作是不是一定要有相关的专业知识或背景?[/FONT]
9. SAP[FONT=宋体]证书在找工作时有什么用?[/FONT]
10. [FONT=宋体]那些模块最热门[/FONT]
[FONT=宋体]主讲:咨询公司高级顾问[/FONT]
[FONT=宋体]时间:[/FONT]3[FONT=宋体]:[/FONT]20-5[FONT=宋体]:[/FONT]20
[FONT=宋体]金融行业就业形势分析及行业机会[/FONT]
[FONT=宋体]主讲:[/FONT]Jim Zhang, [FONT=宋体]特许金融分析师,金融学硕士,[/FONT]10[FONT=宋体]余年金融投资经验[/FONT]
[FONT=宋体]时间:[/FONT]2[FONT=宋体]:[/FONT]00- 3[FONT=宋体]:[/FONT]00
[FONT=宋体]理财讲座:如何保护自己的投资[/FONT]
[FONT=宋体]•[/FONT] [FONT=宋体]在加拿大如何让储蓄存款得到全额保护[/FONT]
[FONT=宋体]•[/FONT] [FONT=宋体]如何将市场下跌转变成盈利时机[/FONT]
[FONT=宋体]•[/FONT] [FONT=宋体]如何善用[/FONT]09[FONT=宋体]年新型省税及免税投资工具[/FONT]---[FONT=宋体]免税储蓄帐户[/FONT]
[FONT=宋体]•[/FONT] [FONT=宋体]经济萧条如何缓解财务压力,实现家庭经济的良性循环[/FONT]
[FONT=宋体]主讲:[/FONT]Jim Zhang[FONT=宋体],多伦多华人投儋顾问中屈指可数的拥有金融投儋行业最高资质证书特许金融分析师([/FONT]CFA[FONT=宋体])头衔的投资理财专家,曾担任大型投资公司基金经理,财务总监等职务,有十余年金融投儋行业的丰富经验,擅长管理投儋组合和分析金融动态。[/FONT]

[FONT=宋体]电话[/FONT]: [FONT=宋体]([/FONT]416[FONT=宋体])[/FONT]412-0165 / (416) 412 - 3088 Rain[FONT=宋体](请提前预约!)[/FONT]

[FONT=宋体]网址:[/FONT]www.kcollege.com/prom

[FONT=宋体]地址:[/FONT]Suite 502, 1200 Eglinton Ave. E, Toronto [FONT=宋体]([/FONT]Dom Mills [FONT=宋体]和[/FONT] Eglinton[FONT=宋体]交叉口东北角[/FONT] , Eglinton /D.V.P[FONT=宋体]高速公路西北角)有大量免费停车位提供[/FONT]



huluzhu : 2008-12-15#286
回复: toronto college介绍,选择,相关问题交流

好贴,留爪

samuelchina : 2008-12-18#287
回复: toronto college介绍,选择,相关问题交流

再次请教下ALEXWEI:Dental Hygiene是不是很难申请?有人说申请了好几年都没成功。还有G11的化学、生物、物理相当于什么程度啊?还有私立学校会比较好入学吗?
有人在念这个专业的吗?请赐教下,万分感谢!

alexwei : 2008-12-18#288
回复: toronto college介绍,选择,相关问题交流

您的背景挺好,可是赶上经济不景气,找专业工作目前有点困难。建议您可以先做读书打算,一开始工程师是不太现实的,可以去一些学校,比如常青藤(私立)学院,他们有相关课程。他们有工业自动化,工业设计的相关课程。如果你想转行做管理,其实质量管理工程师也很不错CQE,这个证书很硬的,很多华人做这个,老外不喜欢这个。不过有一点,什么工作都需要英语,所以别忘了提高英语。
我是学工业电气自动化的,一直从事相关工作,主要是电气图设计,PLC, HMI和Visual Basic编程,包括现场调试,维护。计划明年长登多伦多,上网搜了加拿大的相关情况,发现做工程师没资格,做电工不受欢迎,不知道是不是要先读书才行?请楼主指点

alexwei : 2008-12-18#289
回复: toronto college介绍,选择,相关问题交流

G11就是我们高二的水平,我们国内的高中学历,在这里只承认为G10,所以如果你需要G11,你需要到成人高中去学习。但是如果你大学学的化学,生物等专业也没有问题。如果你的工作和Dental一点关系都没有,申请会困难,因为现在很热门。私立学校不要背景,只要你的英文还过得去,都可以入学。你可以尝试一下,我觉得下决心学习,还是一个不错的方向,加油。

再次请教下ALEXWEI:Dental Hygiene是不是很难申请?有人说申请了好几年都没成功。还有G11的化学、生物、物理相当于什么程度啊?还有私立学校会比较好入学吗?
有人在念这个专业的吗?请赐教下,万分感谢!

alexwei : 2008-12-18#290
回复: toronto college介绍,选择,相关问题交流

33

alexwei : 2008-12-22#291
回复: toronto college介绍,选择,相关问题交流

最近看到加拿大的新移民政策,感到加拿大健康医疗,教育和技工仍然紧缺。建议大家紧紧扣紧这几个方向,可以考虑护士,药剂师助理,社会工作者(健康领域),各种技工
另外政府加大对于基建的投资,因此基建技工,特种机械操作工,以及相关高级一点的岗位都可以考虑
__________________
鸟人

Toronto College 介绍,专业选择

huluzhu : 2008-12-26#292
回复: toronto college介绍,选择,相关问题交流

留爪,要好好学习,天天向上

amymanhua : 2008-12-28#293
回复: toronto college介绍,选择,相关问题交流

请教ALEX,如果想在加当小学或中学教师的话,应该做些什么准备?比如,读什么学校、什么专业、考什么资格证书?
谢谢。

alexwei : 2008-12-28#294
回复: toronto college介绍,选择,相关问题交流

当老师啊,不是很清楚,至少要本科毕业,还要有执照吧。多伦多好像有个教师协会 Ontario College of Teachers. 网址:www.oct.ca

flychange : 2008-12-29#295
回复: toronto college介绍,选择,相关问题交流

介绍一下社会工作者行业
国内没有社工这个行业,不过大家应该清楚这个是干什么的,我们遇到的settlement Counselor, Employment Counselor都是属于这个行业。这个行业对于我们移民来说有一个优势,我们会1门外语,而且是目前最大的移民群体的外语,各个机构都在扩展对于中国移民的服务。这样子就需要我们了。社工为什么要拓展服务呢,大家知道,no profit机构的钱来自于政府,企业或个人捐助,自我经营收入,其中政府最多,而政府的钱是要根据你做了多少事情来得,比如政府今年有7个亿是帮助新移民的,会分配到各个非营利机构,这些机构就要做一些针对新移民的服务来申请经费,同时还要有效果,如果没有政府的funding, 这个机构就要关门了。所以从事这个行业的人往往有marketing、工商管理,教师的背景,然后获得了本地学历。那么这个行业有多大呢,这个行业非常大,和商业基本上快差不多了。那些机构属于no-profit, 医院,学校,就业、安居服务机构,慈善机构,老人服务机构,绿色和平组织,环境和动物保护组织等。他的行业自成体系,你在一般的招聘网站或者agency那里很少看到招聘的信息。需要什么样的人呢,其实和商业一样,需要sales, accounting, administration, customer service, technician.不一样的地方,核心的服务人员是社工,教师,或者医务人员。大家可以去www.charityvillige.com 上去看,这是加拿大最大的非营利机构的招聘网站。
就业前景:不错
薪资水平:16-20左右。
性别:女性居多
个人奋斗方向:如果成为专业social worker, 工资可以达到70000,但是一般需要有social science master degree. 而 bachelor 毕业实际上和college毕业薪资差别不是太大,我个人觉得可以在拿到diploma后工作两年,直接申请master(用国内的学历)+diploma+工作经验,这样不错。
课程介绍
• Introduction to Community Service Worker
• Social Welfare, Diversity and Awareness
• The Family
• Couples, Family and Interpersonal Communication
• Oriented Case Management
• Environmental Issues and Crisis Interventions and
Prevention
• Bereavement and the Elderly
• Respond to Abuse
• Understanding Human Behavior
• Communication Skills
• Counseling Skills
• Report Writing for the CSW
• Field placement

By the end of the course, students will be able to:
• Demonstrate knowledge of the range of social and community service organizations in their own community as well as detailed knowledge of an agency.
• Practice the role of a team member within a social and community service organization.
• Demonstrate beginning level professional skills
• Apply theory and skills learned in the independent study components of this course.
我对这个感兴趣
谢谢指点

flychange : 2008-12-29#296
回复: toronto college介绍,选择,相关问题交流

6月10日续,如何选择college呢,论坛上这类东西可多了,我谈谈自己看法。我觉得核心就一个,快,时间越短越好。为什么呢,第一点,college 就是一个大专,你不管怎么学,拿什么文凭,到社会上也是一个具备初级技能的毕业生。虽说,seneca等在我们圈子名气响亮,可是并不入本地人眼。我有几个银行和政府的朋友,有本地的也有移民。我问他们关于college的问题,他们一致认为,college是非常初级的,只能从事初级工作。而且他们其实不是很清楚或者很在意college之间有什么差别。而到了大学这一级别,才开始注意学校之间的区别,这和国内差不多。第二点,既然college文凭不怎么样,我为什么还需要呢,因为我要本地教育背景阿。移民都是高学历,被这个卡住了,只好搞个本地文凭,其实大家都在学校里呆了不少年,知道学校里学的东西工作中能用得很少,一旦工作了,还得吃自己的工作经验。那这样,我们上college的目的就明确了。搞个本地文凭,既然如此,同样是diploma, 越快拿到得越好。千万不要去学那些什么2年,三年的课程。1年的本地工作经验足以媲美三年的学校教育。其实还有一点,这里的人并不是不认你在国内的工作和教育经历,只是他们认为如果你能够通过本地的学习获得技能,这样你的经验加上本地获得的技能才能把你的才干发挥出来。
其实短的课程未必不好,紧凑,只教实用的。大家如果仔细看各community大学的课程(我那里三所大学的全套),头三个学期都是基础课,一点用没有。那是给这里的高中生用的。有人说了,不对,我确实感觉学到了很多东西。是的,你主要在头三个学期学到的是英语。有些知识,翻译成中文就是普普通通的东西,一看就懂。但是你的口语恐怕提高不大。工作环境是提高口语的最好地方。所以如果集中学一个快的课,省下的时间去工作,是一个好的选择。
总而言之,时间最重要,同样都是一样的文凭,从哪拿都一样,越快越好。但是还是要找质量好的课,好的专业,这个问题下回讨论。
MS有理
谢谢 加SW!

alexwei : 2008-12-29#297
回复: toronto college介绍,选择,相关问题交流

Thanks a lot.
分享一个网站:
www.labourmarketinformation.ca

ameko0908 : 2008-12-31#298
回复: college介绍及选择个人谈,幼教,私人护理,社工,医师行政助理等热门行业介绍

再说说多伦多的college, 多伦多的college分为两种,community college和private college,Community colleg就是俗称的公立大专,像Seneca, Geoge Brown,特点是学科非常全,基本上涵盖各个行业,学分可以被各个大学接受。有1年到3年的certifate, diploma和Degree的课程,也有时间更短的培训。越好。千万不要去学那些什么2年,三年的课程。
请问:1. 多伦多的college哪些是比较有名的? 2. 无论读多长时间,是不是都有coop? 3. certifate, , diploma和Degree有什么区别呀?按照楼主的意思,是不是读1年的有个certifate就足够了呢? :wdb2:

alexwei : 2008-12-31#299
回复: toronto college介绍,选择,相关问题交流

需要什么指点?
我对这个感兴趣
谢谢指点

alexwei : 2008-12-31#300
回复: college介绍及选择个人谈,幼教,私人护理,社工,医师行政助理等热门行业介绍

1、Seneca,Greogebrown
2、不一定有co-op
3、diploma 是大专,degree 是本科及以上,cerificate 代表某种资质
请问:1. 多伦多的college哪些是比较有名的? 2. 无论读多长时间,是不是都有coop? 3. certifate, , diploma和Degree有什么区别呀?按照楼主的意思,是不是读1年的有个certifate就足够了呢? :wdb2:

ameko0908 : 2009-01-04#301
回复: college介绍及选择个人谈,幼教,私人护理,社工,医师行政助理等热门行业介绍

1、Seneca,Greogebrown
2、不一定有co-op
3、diploma 是大专,degree 是本科及以上,cerificate 代表某种资质
请问:college里面也有本科和研究生的学位?那和正规大学里的本科和研究生有什么区别呀?(我对此了解太少了~~谢谢)

clgclyj : 2009-01-08#302
回复: toronto college介绍,选择,相关问题交流

感谢anlexwei, 非常实用的帖子。有问题请教,给你留悄悄话了。谢谢先。

大饼 : 2009-01-10#303
回复: college介绍及选择个人谈,幼教,私人护理,社工,医师行政助理等热门行业介绍

关于公立和私人职业学校的课程和资质,可以到www.edu.gov.on.ca上去查, 所有经过政府审批的学校,以及私立职业学校的经过政府审批的diploma的课程在上面都可以查到。

首先一个概念,加拿大大专(College)跟我们国内的大专概念大不相同,更偏重于职业培训,注重的是技能和动手实践能力。大专的课程设置比较全面,合理,由浅入深易于接受。除了理论基础课程,更多的是职业培训课程、大学转学分课程和热门职业证书培训。同时,许多大专都与企业合作,甚至为大型企业设置特定专业,而且学院还经常调整专业,开设新兴的专业,尽量为学生就业创造机会,因此,大专生普遍比本科生好找工作,但如果想要做管理工作和科研工作,还得再读大学。
再说说多伦多的college, 多伦多的college分为两种,community college和private college,Community colleg就是俗称的公立大专,像Seneca, Geoge Brown,特点是学科非常全,基本上涵盖各个行业,学分可以被各个大学接受。有1年到3年的certifate, diploma和Degree的课程,也有时间更短的培训。可以在http://www.edu.gov.on.ca/eng/general/list/college.html上找到On省所有Community 大专,一共24所。学生可以向政府申请OSAP贷款上学,但是大部分课程不能够申请EI学费上学,因为学制比较长,其实有的专业实际课时不多,但是由于Community College 假期和大学类似,时间较长,所以拖得比较长。政府可不愿意你拿着EI学费还休假,不过由于竞争的关系,他们也推出一些适合申请EI学费的专业。学生毕业后的就业率在70%到80%之间。
还有一类叫private college, 也可以叫private career college, 故名思义,是私人办的以职业培训为主的college, 一般规模不大,没有专门的校园。他们不可能开设像community college那么全的课程,所以都选择市场上的热门专业,并且可以较短的时间让学生莆眨玫iploma和Certificate(如果这个行业需要专门的certificate).由于规模不大,因此各个学校都有不同的侧重点,有比较强的专业,因此选择起来要困难一些。课程一般不超过一年,提供diploma,还有很多的课程是专门帮你通过一些证书考试的,有的学校学生可以申请OSAP,但是大部分学校不行。他们主要面向自费学习和EI学费学生。由于没有假期,上课时间灵活(一般每个专业都是滚动开课,所以学生在一年当中的任何一个月都可以入学)。这些学校,就业率是命根子,如果没有很高的就业率,就难招到学生,政府也很难批准EI学生入学。因此他们的专业一定是就业趋势很好的,老师一般是专业知识强并在行业有多年工作经验的,不仅负责教授学生技能,还要帮助学生实习和找工作的指导,因此把老师叫instructor,这是和community College教学非常不同的一点,Community教学还是正规的学校式教育,一个专业,7、8门课,每个老师各教一摊。大部分private college的一个班一个老师带到底,像师傅带徒弟,老师是老板雇的,因此压力更大些。学校也积极和各种社会资源雇主联系给学生提供co-op 和就业机会,因此就业率比community college高,好的能够达到90%以上。现在比较热门的,比如IT的一些专业,社工CSW,牙医助理,幼教助理ECA,药剂师PT,Computer Accounting,护工PSW,医生行政助理MOA等。但是private college 良莠不齐,首先大家可以去政府的网站
http://www.gov.on.ca/ont/portal/!ut...s/7_0_252/_s.7_0_A/7_0_252/_l/en?docid=053263 去搜索,确保这个college是政府审批过的正在经营的college. 然后,看看这个学校的网站,看看有没有感兴趣的课程。

关于如何选择学校的问题,我想留到下次再写,希望能够逐渐补充些资料,给大家一个好的参考。也希望大家和我充分交流,提供不同意见。

6月10日续,如何选择college呢,论坛上这类东西可多了,我谈谈自己看法。我觉得核心就一个,快,时间越短越好。为什么呢,第一点,college 就是一个大专,你不管怎么学,拿什么文凭,到社会上也是一个具备初级技能的毕业生。虽说,seneca等在我们圈子名气响亮,可是并不入本地人眼。我有几个银行和政府的朋友,有本地的也有移民。我问他们关于college的问题,他们一致认为,college是非常初级的,只能从事初级工作。而且他们其实不是很清楚或者很在意college之间有什么差别。而到了大学这一级别,才开始注意学校之间的区别,这和国内差不多。第二点,既然college文凭不怎么样,我为什么还需要呢,因为我要本地教育背景阿。移民都是高学历,被这个卡住了,只好搞个本地文凭,其实大家都在学校里呆了不少年,知道学校里学的东西工作中能用得很少,一旦工作了,还得吃自己的工作经验。那这样,我们上college的目的就明确了。搞个本地文凭,既然如此,同样是diploma, 越快拿到得越好。千万不要去学那些什么2年,三年的课程。1年的本地工作经验足以媲美三年的学校教育。其实还有一点,这里的人并不是不认你在国内的工作和教育经历,只是他们认为如果你能够通过本地的学习获得技能,这样你的经验加上本地获得的技能才能把你的才干发挥出来。
其实短的课程未必不好,紧凑,只教实用的。大家如果仔细看各community大学的课程(我那里三所大学的全套),头三个学期都是基础课,一点用没有。那是给这里的高中生用的。有人说了,不对,我确实感觉学到了很多东西。是的,你主要在头三个学期学到的是英语。有些知识,翻译成中文就是普普通通的东西,一看就懂。但是你的口语恐怕提高不大。工作环境是提高口语的最好地方。所以如果集中学一个快的课,省下的时间去工作,是一个好的选择。
总而言之,时间最重要,同样都是一样的文凭,从哪拿都一样,越快越好。但是还是要找质量好的课,好的专业,这个问题下回讨论。

第三回,关于专业的选择,找热门的技能,两个办法,一个是去www.jobfutures.com上去看,那里可以查到各个专业工作的薪资水平,需要的学历,相关的专业,以及就业趋势。另外一个方法更简单,看看报纸上的各个 carrer college和培训学校的广告,那上面列的专业绝对都是比较好就业的专业,因为就业率是各个学校的生命线。没有前景,华而不实的专业绝对不会落入他们的眼睛。不过也不是绝对,career college主要关注初级技能培训和证书考试,对于有些比如医生,律师之类的可能前景很好,但是需要去大学读master. 我对于技工这块不太了解,不过大家可以去www.serviceontario.ca上面看看,在school & work里面,apprenticeship 是政府大力支持的,有很多的服务和培训。有些学校是专门教授这些的,比如常青藤。community college里面也有相关课程。不过这一块更要当心,有些技能淘汰的很快。其他方面,IT方面硬件是没有什么前途的,软件现在的课程可多了,软件测试,数据分析SAS, 网络安全都算热门。不过如果在国内不是IT人士,学这个恐怕已经晚了。还有一些课程,适合不同背景的人学习,就业前景基本可以,在这里可以介绍一下。
首先是电算化会计,其实学出来bookkeeper,不管你学两年的课程,还是6个月的速成,找工作都差不多,因为会计最需要经验,所以没有经验的只有从基层老老实实干起。在国内干过的,还是好找,那些从来没干过的,学了两年,还是难找。不过现在很多小公司找行政员工,如果同时能做些会计工作竞争力会大很多。
平均薪资:会计$15.78左右,bookkeeper 14.8左右,一开始也就拿个10-12块
就业情况:一直都还可以,比平均情况好。但是由于中国移民学这个的太多,所以现在的竞争也非常激烈
性别:女性占了91%,看来男性没什么机会,但是CFA男性多一些。不过要拿时间熬。光想办法考证是不行的,还要有机会进入事务所或大些的公司混些经验。所以如果希望将来能够发展空间大,那就要宁为牛后,不为鸡首。到大的事务所做做义工,合同工可能是必要的。
建议:学这个行业选择一个6-8个月的速成就可以了。
下一部分:幼教助理 Early Childcare Assistant
ECE o和ECA 介绍,
幼儿教育老师和幼儿教育助理,ECE需要两年学习,需要到community College里面学,ECA有很多6个月的短期课程,其实工资差不多,差1、2块钱。这个行业除了diploma外还要有certificate,基本上你在学习期间就可以考过这个证,你就可以工作了。考试费很便宜:50元左右
英语要求:ECA的英语要求不是太高
行业前景:不错,而且自雇比例颇高,达到41%。所以比较适合需要照顾家庭的女性。
平均薪资:12.87.这个工资实在没有吸引力,但是适合自雇,自己当老板赚多少,这里没有统计,但是自在啊。
性别:还是有2%的男性的!,而且男性的竞争力特强!
课程的情况看第二页的帖子
三:私人护理(personal support worker)
其实如果是需要找个挣钱多一些的职业,这个职业比ECA强,首先工资高,其次有了这个专业的diploma一样可以做ECA的工作,因为课程里面有专们如何照顾孩子的。而且学到的知识其实对自己对家庭都很有好处。只是我们中国人有的时候有偏见,觉得去照顾病人,老人,收拾床铺不太好对家中父老交待。说在国外干什么呢,私人护理,都干些啥,收拾屋子,陪老人聊天,照顾病人。人家会觉得你好好的,跑国外干这个。但是我觉得这个职业不错,听我分析一下
就业前景:很不错。整个世界都进入老年社会了。失业率只有1%!。未来中国也很需要,另外,发达社会越来越重视对于老人,病人,disable包括孩子的照顾和护理。其实不仅是照顾吃穿活动那么简单,还有精神上的照顾,属于体验经济的一类,阳光产业。而且这个行业对于自己的性格磨练很有好处,你会变得很有耐心,细心和爱心,听说对于生老病死的看法也会大大改变。自雇和part time的机会也不错,照顾照顾邻里,有40%的人做part time. 平时做义工的也是这个行业比较多。
薪资:16-18也有到20块的,而且,由于从事一些和病人打交道的工作,一般健康福利会比较不错。
工作环境:其实我去过老人院和老人公寓,环境真得很不错。由于工作中动体力的时间较多,所以一般都不会让你连续工作时间太长的。
语言要求:这个工作一般对语言要求较高,不过有一点,会一门外语绝对是一个财富。
发展空间:这个专业实际上是Nurse Aid,发展当然是朝Rigester Nurse了。热门阿。
性别:女性占91%。 但是:听说急需男性阿,虽说有器械帮助,但是还是有些体力活的,所以经常在老人院里和医院里见到五大三粗的女性。还有,男性也有隐私阿,男同志有的时候也想让同性来照顾一下。顺便说一句,男护士也是非常非常受欢迎。
课程介绍,上这个课的教室很有意思,大家有兴趣可以参观。这个课对于实际操作要求很强,经常要演练急救,比如,突然有人晕倒了,怎么处理,学生们七手八脚的上。
这个课必须要西人教师上,因为西人和我们在处理人的问题上首先观念大大不同。西人把安全放在第一位,首先必须安全。所以什么民间土法,没有经过验证的不用。比如脖子不舒服,趁他不注意给他嘎吱一下。所以同样操作,观念不同,处理的方法不同。
课程介绍太长了,需要的留下邮件。包括,心理学的知识,人际沟通,营养学,老人护理,临终关怀,危机处理,家庭护理,收拾房屋,以及相关的法律法规,当然,实习是非常重要的一环
PT: Pharmacy Technician
就业前景:Very Good, 还是和老年社会有关。
薪资水平:如果在零售,10-15块,如果在社区community health care. 10-20块,医院:19-26块起,有些人成为了药剂师,工资约7万/年起,但是要再继续学习获得degree.
性别:男女均衡
语言:要求较高,学习辛苦,因为有大量东西要记,中国人毕业率高,老外经常吃不消。
自雇:自己做生意的占15%。 年收入较高。
学位:diploma, 同时还有行业的几个certificate
学习时间:40周,要学得抓紧,政府马上要把它改成两年的课程了
课程介绍:太长了,需要的给mail.
谢谢,好贴:wdb17:
Community Service Worker 介绍,请点,就不用费时找了

大饼 : 2009-01-10#304
回复: toronto college介绍,选择,相关问题交流

专业有两个:国际商务管理 国际物流与供应链管理
毕业之后可以拿研究生毕业证和相应的专业的职业资格证:
加拿大国际贸易协会(FITT)颁发的执业资格证书CITP (Certified International Trade Professional)
加拿大采购管理协会(PMAC)颁发的执业资格证书 CPP (Certified Professional Purchaser)
想毕业后靠着证书留在加拿大,并找到一份工作
女儿学的是商务管理,好贴:wdb10:

大饼 : 2009-01-10#305
回复: toronto college介绍,选择,相关问题交流

介绍一下社会工作者行业
国内没有社工这个行业,不过大家应该清楚这个是干什么的,我们遇到的settlement Counselor, Employment Counselor都是属于这个行业。这个行业对于我们移民来说有一个优势,我们会1门外语,而且是目前最大的移民群体的外语,各个机构都在扩展对于中国移民的服务。这样子就需要我们了。社工为什么要拓展服务呢,大家知道,no profit机构的钱来自于政府,企业或个人捐助,自我经营收入,其中政府最多,而政府的钱是要根据你做了多少事情来得,比如政府今年有7个亿是帮助新移民的,会分配到各个非营利机构,这些机构就要做一些针对新移民的服务来申请经费,同时还要有效果,如果没有政府的funding, 这个机构就要关门了。所以从事这个行业的人往往有marketing、工商管理,教师的背景,然后获得了本地学历。那么这个行业有多大呢,这个行业非常大,和商业基本上快差不多了。那些机构属于no-profit, 医院,学校,就业、安居服务机构,慈善机构,老人服务机构,绿色和平组织,环境和动物保护组织等。他的行业自成体系,你在一般的招聘网站或者agency那里很少看到招聘的信息。需要什么样的人呢,其实和商业一样,需要sales, accounting, administration, customer service, technician.不一样的地方,核心的服务人员是社工,教师,或者医务人员。大家可以去www.charityvillige.com 上去看,这是加拿大最大的非营利机构的招聘网站。
就业前景:不错
薪资水平:16-20左右。
性别:女性居多
个人奋斗方向:如果成为专业social worker, 工资可以达到70000,但是一般需要有social science master degree. 而 bachelor 毕业实际上和college毕业薪资差别不是太大,我个人觉得可以在拿到diploma后工作两年,直接申请master(用国内的学历)+diploma+工作经验,这样不错。
课程介绍
• Introduction to Community Service Worker
• Social Welfare, Diversity and Awareness
• The Family
• Couples, Family and Interpersonal Communication
• Oriented Case Management
• Environmental Issues and Crisis Interventions and
Prevention
• Bereavement and the Elderly
• Respond to Abuse
• Understanding Human Behavior
• Communication Skills
• Counseling Skills
• Report Writing for the CSW
• Field placement

By the end of the course, students will be able to:
• Demonstrate knowledge of the range of social and community service organizations in their own community as well as detailed knowledge of an agency.
• Practice the role of a team member within a social and community service organization.
• Demonstrate beginning level professional skills
• Apply theory and skills learned in the independent study components of this course.
留脚印

大饼 : 2009-01-10#306
回复: toronto college介绍,选择,相关问题交流

网上找到一个文章,供参考,是个学校的宣传来着。个人说两点:
关于英语,很多专业需要的工作语言要求都不高。但是我们不能光听学校的宣传。首先,只要是服务人的工作,相对于对着机器,电脑的工作,语言的要求要高很多。你无法设定你的客户(患者)会如何和你交流。你需要向客户解释很多事情。你或你的老板需要你通过交流保持和客户的良好关系。另外一方面,你的同事和老板需要和你沟通,能够自在交流的部下和同事可以使气氛活跃,工作顺畅。所以,就这两个工作或者还有其他工作,我觉得对语言要求不高恐怕要打折扣。当然会讲国语是个优势,不过在这里会几种语言的人多的是。你可以选择为本国移民工作,但是语言不好的人都挤在这里,结果就是工资大打折扣。而且一般诊所即使医生讲国语,也希望助理的英文好。有一个调查,说移民的工资大约平只有加国平均的一半,我看和我们的英语差距有些对应。
如果要学这个课,建议一定找西人教学的,最好的本地人,一年学下来,不会唱歌,至少也会哼哼。一开始听不懂,没关系,还可以看书。不过不要把学习中提高英语期望太高,因为这个课主要是听和动手,交流不多,如果要提高交流水平,学习中间还要建议参加提高口语的班。

工作职责:
顶,收藏
虽然牙医助理和洗牙师都在在普通牙医诊所,专科牙医诊所 (齿髓病诊所,牙周病诊所,牙齿矫形诊所和牙医儿科等),医院的口腔科,牙科实验室以及公共健康部门等领域工作,但具体工作职责却完全不同。牙医助理的工作职责主要是辅助牙医Dentist对病人进行诊断和治疗,其工作包括准备相关器械和材料,消毒处理,拍牙片,取牙模等,同时指导病人如何保持口腔卫生和口腔护理。洗牙师的工作除了cleaning洗牙, fluoride抛光和加氟等洗牙具体工作之外,还包括拍摄X光照片,了解病人个人信息,病史,和提供牙齿健康教育。

就业前景:

牙医助理和洗牙师两种工作在加拿大一直以来都保持非常高的就业率。加拿大统计局根据各种统计数字做出对加拿大各类工作的综合评价;其中能够被评为“就业前景好”的行业屈指可数,例如大陆移民朋友非常青睐的会计员和文员等工作,统计局的评价均为“一般”。而牙医助理和洗牙师都被评为就业前景非常好的行业。

工资待遇:

牙医助理和洗牙师这两种工作在工薪和待遇方面差别较大。加拿大统计局提供的数据显示,牙医助理的平均工资在$16.51加元每小时,工作一般都是全职工作 Full Time,工作环境好,工作时间有规律,福利待遇好。另一方面,洗牙师工资就高很多,初入行的洗牙师都可以拿到$35加元每小时,在温哥华甚至可以拿到 $40每小时。洗牙师工作一般都从part time开始,三年以后找到全职带福利的工作的可能性很高。当然,洗牙师一般都回同时选择两份左右的part time工作,保证一周工作在35-40小时左右,并有选择工作时间的自由。

入门要求:

加拿大牙医助理分为几个等级,初级牙医助理只要有承认的专业毕业文凭Diploma就可以上岗;而二级牙医助理就需要持毕业文凭在相关部门进行考试,持专业执照上岗。一般来说,初级牙医助理的课程大概在8-10个月左右,而二级牙医助理的学习时间要再加上4-6个月。而洗牙师必须持有专业洗牙师协会颁发的专业执照才可以上岗工作,如果打算考取洗牙师牌照,必须首先完成洗牙师专业培训,洗牙师专业课程一般在18-36个月左右。

英语高求:

牙科技术工作对英语的要求都是整个医疗健康体系中较低的。工作性质决定了牙医助理和洗牙师都不需要与病人进行大量的英语沟通,简单的解答和服务英语就足够了。但是,因为从业人员必须拥有专业毕业证书,所以在学习当中必须要涉及一些基础的医疗健康知识。具备基本的英语能力的华人移民朋友,即使没有医疗背景,只要肯下功夫,都能在这两个职业方向有所发展。

alexwei : 2009-01-20#307
回复: toronto college介绍,选择,相关问题交流

政府的Second Career Training,资助最长两年,最多28000元的学习,只要失业就可以申请。

ccx123 : 2009-01-22#308
回复: toronto college介绍,选择,相关问题交流

楼主真是好人啊,不厌其烦回答问题。先谢谢了,已经受益匪浅了。
我想请教一下,我是大学教师(金融保险),有博士学位,马上就四十岁了,估计DM快下来了。但一直很迷茫,不知道自己过去后能干点什么。看了楼主的帖子,很受启发,自我感觉很有爱心,所以感觉做社工挺不错的,但估计也要先读个加国文凭才行。
请问楼主,直接去读COLLEGE的社工专业合适,还是申请University 呢?像我这种情况做社工合适吗?有什么短平快的途径没有?我的博士学位对社工工作会不会有帮助?年近四十了还要从头学起,嗬嗬,呵呵呵。

dreaming : 2009-01-28#309
回复: toronto college介绍,选择,相关问题交流

全部学习,很有用!谢谢LZ加声望了!

alexwei : 2009-01-28#310
回复: toronto college介绍,选择,相关问题交流

楼主真是好人啊,不厌其烦回答问题。先谢谢了,已经受益匪浅了。
我想请教一下,我是大学教师(金融保险),有博士学位,马上就四十岁了,估计DM快下来了。但一直很迷茫,不知道自己过去后能干点什么。看了楼主的帖子,很受启发,自我感觉很有爱心,所以感觉做社工挺不错的,但估计也要先读个加国文凭才行。
请问楼主,直接去读COLLEGE的社工专业合适,还是申请University 呢?像我这种情况做社工合适吗?有什么短平快的途径没有?我的博士学位对社工工作会不会有帮助?年近四十了还要从头学起,嗬嗬,呵呵呵。

alexwei : 2009-01-28#311
回复: toronto college介绍,选择,相关问题交流

私立college有6-8个月文凭课程,有实习,实用性强,不过入行工作要从初级做起。公立college一般2年课程,不同方向,youth worker, settlement worker, employment counsellor. 都不错,不过这些都是social service worker 不是social worker, 上university 可以做social worker. 工资高很多。都是一个选择,很难说优劣

alexwei : 2009-02-01#312
回复: toronto college介绍,选择,相关问题交流

Happy new year

mikefeng : 2009-02-01#313
回复: toronto college介绍,选择,相关问题交流

私立college有6-8个月文凭课程,有实习,实用性强,不过入行工作要从初级做起。公立college一般2年课程,不同方向,youth worker, settlement worker, employment counsellor. 都不错,不过这些都是social service worker 不是social worker, 上university 可以做social worker. 工资高很多。都是一个选择,很难说优劣

先谢过LZ。 LZ很专业,想根据个人情况请教LZ。
本人正在申请EI培训,给份表格要求选择3个college,其中一个公立两个私立,多数人选私立college, 但我发现有些私立college 很糟糕,在那学不到东西,这是不是只是个别现象?能选公立college吗?在网上看不到合适的课程(都是1年以上的非training课)。

hpp6676 : 2009-02-01#314
回复: toronto college介绍,选择,相关问题交流

非常感谢这些有用的信息。
我也面临一些困惑,想请教一下:
本人国内本科毕业于水利水电建筑工程专业,有七年的设计与CAD经验,也能从事工程造价工作。 来加以后,发现搞设计没资格,CAD及ESTIMATOR就业暗淡, 于是想念个COLLEGE,转行到社工或牙医助理,不知LZ有何建议?方向是否可以?抑或是否有转行必要? 不胜感谢!

alexwei : 2009-02-06#315
回复: toronto college介绍,选择,相关问题交流

EI training 很难选择公立学院,私立也有很不错的,你学什么呢
\
先谢过LZ。 LZ很专业,想根据个人情况请教LZ。
本人正在申请EI培训,给份表格要求选择3个college,其中一个公立两个私立,多数人选私立college, 但我发现有些私立college 很糟糕,在那学不到东西,这是不是只是个别现象?能选公立college吗?在网上看不到合适的课程(都是1年以上的非training课)。

yezhijing : 2009-02-08#316
回复: toronto college介绍,选择,相关问题交流

很好的帖子!
想请问楼主:
国内英教本科申请留学转专业除了人力资源管理能转还有其他的postgraduate program certificate能读吗?早期教育如何?会展管理如何?
有何不错的私人college 能转成功的一年或8个月的专业推荐?
目的还是为了就业!
多谢!

bearzheng : 2009-02-10#317
回复: toronto college介绍,选择,相关问题交流

很强的帖子,一直从第一页看到最后一页。选专业还是要慎重,还是要好好考虑再做。有个问题想问一下,我想做customer service,还需要上college学吗,如果直接可以找工作,有没有网站可以介绍,如果需要学的话,能不能介绍就业高的学校。

谢谢!

Vivian521 : 2009-02-10#318
回复: toronto college介绍,选择,相关问题交流

你好Alex,一口气看了你大部分的帖子,很受用。你是个很nice,很热心的人。我现在面临一个很难抉择的困境,不知如何是好。先简单把我的情况告诉你,希望你能认真阅读,谢谢。
我和我的男朋友决定今年下半年去多伦多,他6月份能拿到工作签证,我拿学生签证去college学习一年,这样在他工作两年后就可以申请移民了。但是目前我面临选择学校和课程的问题,我不知道什么样的课程适合我同时又能找到相对不错的工作,看了你的介绍,我对牙医助理和社工的专业很感兴趣。但是我有几个问题很疑惑,
1.如果没有移民身份,只有学生签证的话,可以学牙医助理或者社工课程吗?对于我的情况,学费大概是多少?
2.我在国内大学学的是园艺专业(果树,蔬菜方面),毕业后在外企从事Admin assistant有两年,我的专业能申请college的牙医助理或社工课程吗?
3.牙医助理或社工与office adminstration相比,找工作哪个会比较容易?有人曾经推荐我学office adminstration.
4. 我目前的英语水平大概是LINK5-6吧,没有测过,和一个加拿大的朋友聊天时他评估的。因为在外企工作,英语相对来说还可以。如果能够申请牙医助理或社工的课程,可以在登陆后直接学习吗?还是要先学习一段时间的语言?
5.您认为牙医助理和社工相比,哪个更好,能容易找到工作?

一口气问了你这么多问题,一定会占用您很多宝贵的时间,很抱歉。真的很希望您能仔细帮我解答我的问题,谢谢!我的msn是 maggice521@hotmail.com 如果方便的话,希望能与您在msn上讨论,万分感谢。:wdb17:

焦急的等待着......

clgclyj : 2009-02-11#319
回复: toronto college介绍,选择,相关问题交流

多谢LZ, 非常热心的人

大饼 : 2009-02-12#320
回复: toronto college介绍,选择,相关问题交流

留个人脚印

lugordon : 2009-02-27#321
回复: toronto college介绍,选择,相关问题交流

软件测试不错,你的背景学起来不难,不过不需要上长期的课程,有很多软件测试工程师的培训,大概2-3个月part time学习,学费也不高。 就可以考取证书,年薪在4万-10万,视乎您的经验,而且对语言要求不是太高。如果你在多伦多,我倒是可以推荐,阿省不了解。电工专业 看你了,对于经验要求比较高,将来自己干,没有公司福利。
如果感兴趣QA, 可以PM我联系方式,我要给你推销:wdb23:,电工可以去常青藤。

偶对这个也感兴趣了,改天也向ALEXWEI老师请教。

BTW,你是鸟人兄?

快比 : 2009-02-27#322
回复: toronto college介绍,选择,相关问题交流

赴加后继续学习不得不看的帖,好!

alexwei : 2009-03-06#323
回复: toronto college介绍,选择,相关问题交流

我回来了

alexwei : 2009-03-06#324
回复: toronto college介绍,选择,相关问题交流

偶对这个也感兴趣了,改天也向ALEXWEI老师请教。

BTW,你是鸟人兄?
谢谢了。

alexwei : 2009-03-06#325
回复: toronto college介绍,选择,相关问题交流

学生签证的话也可以学,不过最好是公立学院2年及以上的课程。因为牵涉到一个未来工作签证的问题。上什么专业都可以,价钱贵,估计一年12000以上,另外你可能还要学一些语言课程,估计也要10000万以上。
你好Alex,一口气看了你大部分的帖子,很受用。你是个很nice,很热心的人。我现在面临一个很难抉择的困境,不知如何是好。先简单把我的情况告诉你,希望你能认真阅读,谢谢。
我和我的男朋友决定今年下半年去多伦多,他6月份能拿到工作签证,我拿学生签证去college学习一年,这样在他工作两年后就可以申请移民了。但是目前我面临选择学校和课程的问题,我不知道什么样的课程适合我同时又能找到相对不错的工作,看了你的介绍,我对牙医助理和社工的专业很感兴趣。但是我有几个问题很疑惑,
1.如果没有移民身份,只有学生签证的话,可以学牙医助理或者社工课程吗?对于我的情况,学费大概是多少?
2.我在国内大学学的是园艺专业(果树,蔬菜方面),毕业后在外企从事Admin assistant有两年,我的专业能申请college的牙医助理或社工课程吗?
3.牙医助理或社工与office adminstration相比,找工作哪个会比较容易?有人曾经推荐我学office adminstration.
4. 我目前的英语水平大概是LINK5-6吧,没有测过,和一个加拿大的朋友聊天时他评估的。因为在外企工作,英语相对来说还可以。如果能够申请牙医助理或社工的课程,可以在登陆后直接学习吗?还是要先学习一段时间的语言?
5.您认为牙医助理和社工相比,哪个更好,能容易找到工作?

一口气问了你这么多问题,一定会占用您很多宝贵的时间,很抱歉。真的很希望您能仔细帮我解答我的问题,谢谢!我的msn是 maggice521@hotmail.com 如果方便的话,希望能与您在msn上讨论,万分感谢。:wdb17:

焦急的等待着......

helene_st_w : 2009-03-08#326
回复: college介绍及选择个人谈,幼教,私人护理,社工,医师行政助理等热门行业介绍

关于公立和私人职业学校的课程和资质,可以到www.edu.gov.on.ca上去查, 所有经过政府审批的学校,以及私立职业学校的经过政府审批的diploma的课程在上面都可以查到。

首先一个概念,加拿大大专(College)跟我们国内的大专概念大不相同,更偏重于职业培训,注重的是技能和动手实践能力。大专的课程设置比较全面,合理,由浅入深易于接受。除了理论基础课程,更多的是职业培训课程、大学转学分课程和热门职业证书培训。同时,许多大专都与企业合作,甚至为大型企业设置特定专业,而且学院还经常调整专业,开设新兴的专业,尽量为学生就业创造机会,因此,大专生普遍比本科生好找工作,但如果想要做管理工作和科研工作,还得再读大学。
再说说多伦多的college, 多伦多的college分为两种,community college和private college,Community colleg就是俗称的公立大专,像Seneca, Geoge Brown,特点是学科非常全,基本上涵盖各个行业,学分可以被各个大学接受。有1年到3年的certifate, diploma和Degree的课程,也有时间更短的培训。可以在http://www.edu.gov.on.ca/eng/general/list/college.html上找到On省所有Community 大专,一共24所。学生可以向政府申请OSAP贷款上学,但是大部分课程不能够申请EI学费上学,因为学制比较长,其实有的专业实际课时不多,但是由于Community College 假期和大学类似,时间较长,所以拖得比较长。政府可不愿意你拿着EI学费还休假,不过由于竞争的关系,他们也推出一些适合申请EI学费的专业。学生毕业后的就业率在70%到80%之间。
还有一类叫private college, 也可以叫private career college, 故名思义,是私人办的以职业培训为主的college, 一般规模不大,没有专门的校园。他们不可能开设像community college那么全的课程,所以都选择市场上的热门专业,并且可以较短的时间让学生莆眨玫iploma和Certificate(如果这个行业需要专门的certificate).由于规模不大,因此各个学校都有不同的侧重点,有比较强的专业,因此选择起来要困难一些。课程一般不超过一年,提供diploma,还有很多的课程是专门帮你通过一些证书考试的,有的学校学生可以申请OSAP,但是大部分学校不行。他们主要面向自费学习和EI学费学生。由于没有假期,上课时间灵活(一般每个专业都是滚动开课,所以学生在一年当中的任何一个月都可以入学)。这些学校,就业率是命根子,如果没有很高的就业率,就难招到学生,政府也很难批准EI学生入学。因此他们的专业一定是就业趋势很好的,老师一般是专业知识强并在行业有多年工作经验的,不仅负责教授学生技能,还要帮助学生实习和找工作的指导,因此把老师叫instructor,这是和community College教学非常不同的一点,Community教学还是正规的学校式教育,一个专业,7、8门课,每个老师各教一摊。大部分private college的一个班一个老师带到底,像师傅带徒弟,老师是老板雇的,因此压力更大些。学校也积极和各种社会资源雇主联系给学生提供co-op 和就业机会,因此就业率比community college高,好的能够达到90%以上。现在比较热门的,比如IT的一些专业,社工CSW,牙医助理,幼教助理ECA,药剂师PT,Computer Accounting,护工PSW,医生行政助理MOA等。但是private college 良莠不齐,首先大家可以去政府的网站
http://www.gov.on.ca/ont/portal/!ut...s/7_0_252/_s.7_0_A/7_0_252/_l/en?docid=053263 去搜索,确保这个college是政府审批过的正在经营的college. 然后,看看这个学校的网站,看看有没有感兴趣的课程。

关于如何选择学校的问题,我想留到下次再写,希望能够逐渐补充些资料,给大家一个好的参考。也希望大家和我充分交流,提供不同意见。

6月10日续,如何选择college呢,论坛上这类东西可多了,我谈谈自己看法。我觉得核心就一个,快,时间越短越好。为什么呢,第一点,college 就是一个大专,你不管怎么学,拿什么文凭,到社会上也是一个具备初级技能的毕业生。虽说,seneca等在我们圈子名气响亮,可是并不入本地人眼。我有几个银行和政府的朋友,有本地的也有移民。我问他们关于college的问题,他们一致认为,college是非常初级的,只能从事初级工作。而且他们其实不是很清楚或者很在意college之间有什么差别。而到了大学这一级别,才开始注意学校之间的区别,这和国内差不多。第二点,既然college文凭不怎么样,我为什么还需要呢,因为我要本地教育背景阿。移民都是高学历,被这个卡住了,只好搞个本地文凭,其实大家都在学校里呆了不少年,知道学校里学的东西工作中能用得很少,一旦工作了,还得吃自己的工作经验。那这样,我们上college的目的就明确了。搞个本地文凭,既然如此,同样是diploma, 越快拿到得越好。千万不要去学那些什么2年,三年的课程。1年的本地工作经验足以媲美三年的学校教育。其实还有一点,这里的人并不是不认你在国内的工作和教育经历,只是他们认为如果你能够通过本地的学习获得技能,这样你的经验加上本地获得的技能才能把你的才干发挥出来。
其实短的课程未必不好,紧凑,只教实用的。大家如果仔细看各community大学的课程(我那里三所大学的全套),头三个学期都是基础课,一点用没有。那是给这里的高中生用的。有人说了,不对,我确实感觉学到了很多东西。是的,你主要在头三个学期学到的是英语。有些知识,翻译成中文就是普普通通的东西,一看就懂。但是你的口语恐怕提高不大。工作环境是提高口语的最好地方。所以如果集中学一个快的课,省下的时间去工作,是一个好的选择。
总而言之,时间最重要,同样都是一样的文凭,从哪拿都一样,越快越好。但是还是要找质量好的课,好的专业,这个问题下回讨论。

第三回,关于专业的选择,找热门的技能,两个办法,一个是去www.jobfutures.com上去看,那里可以查到各个专业工作的薪资水平,需要的学历,相关的专业,以及就业趋势。另外一个方法更简单,看看报纸上的各个 carrer college和培训学校的广告,那上面列的专业绝对都是比较好就业的专业,因为就业率是各个学校的生命线。没有前景,华而不实的专业绝对不会落入他们的眼睛。不过也不是绝对,career college主要关注初级技能培训和证书考试,对于有些比如医生,律师之类的可能前景很好,但是需要去大学读master. 我对于技工这块不太了解,不过大家可以去www.serviceontario.ca上面看看,在school & work里面,apprenticeship 是政府大力支持的,有很多的服务和培训。有些学校是专门教授这些的,比如常青藤。community college里面也有相关课程。不过这一块更要当心,有些技能淘汰的很快。其他方面,IT方面硬件是没有什么前途的,软件现在的课程可多了,软件测试,数据分析SAS, 网络安全都算热门。不过如果在国内不是IT人士,学这个恐怕已经晚了。还有一些课程,适合不同背景的人学习,就业前景基本可以,在这里可以介绍一下。
首先是电算化会计,其实学出来bookkeeper,不管你学两年的课程,还是6个月的速成,找工作都差不多,因为会计最需要经验,所以没有经验的只有从基层老老实实干起。在国内干过的,还是好找,那些从来没干过的,学了两年,还是难找。不过现在很多小公司找行政员工,如果同时能做些会计工作竞争力会大很多。
平均薪资:会计$15.78左右,bookkeeper 14.8左右,一开始也就拿个10-12块
就业情况:一直都还可以,比平均情况好。但是由于中国移民学这个的太多,所以现在的竞争也非常激烈
性别:女性占了91%,看来男性没什么机会,但是CFA男性多一些。不过要拿时间熬。光想办法考证是不行的,还要有机会进入事务所或大些的公司混些经验。所以如果希望将来能够发展空间大,那就要宁为牛后,不为鸡首。到大的事务所做做义工,合同工可能是必要的。
建议:学这个行业选择一个6-8个月的速成就可以了。
下一部分:幼教助理 Early Childcare Assistant
ECE o和ECA 介绍,
幼儿教育老师和幼儿教育助理,ECE需要两年学习,需要到community College里面学,ECA有很多6个月的短期课程,其实工资差不多,差1、2块钱。这个行业除了diploma外还要有certificate,基本上你在学习期间就可以考过这个证,你就可以工作了。考试费很便宜:50元左右
英语要求:ECA的英语要求不是太高
行业前景:不错,而且自雇比例颇高,达到41%。所以比较适合需要照顾家庭的女性。
平均薪资:12.87.这个工资实在没有吸引力,但是适合自雇,自己当老板赚多少,这里没有统计,但是自在啊。
性别:还是有2%的男性的!,而且男性的竞争力特强!
课程的情况看第二页的帖子
三:私人护理(personal support worker)
其实如果是需要找个挣钱多一些的职业,这个职业比ECA强,首先工资高,其次有了这个专业的diploma一样可以做ECA的工作,因为课程里面有专们如何照顾孩子的。而且学到的知识其实对自己对家庭都很有好处。只是我们中国人有的时候有偏见,觉得去照顾病人,老人,收拾床铺不太好对家中父老交待。说在国外干什么呢,私人护理,都干些啥,收拾屋子,陪老人聊天,照顾病人。人家会觉得你好好的,跑国外干这个。但是我觉得这个职业不错,听我分析一下
就业前景:很不错。整个世界都进入老年社会了。失业率只有1%!。未来中国也很需要,另外,发达社会越来越重视对于老人,病人,disable包括孩子的照顾和护理。其实不仅是照顾吃穿活动那么简单,还有精神上的照顾,属于体验经济的一类,阳光产业。而且这个行业对于自己的性格磨练很有好处,你会变得很有耐心,细心和爱心,听说对于生老病死的看法也会大大改变。自雇和part time的机会也不错,照顾照顾邻里,有40%的人做part time. 平时做义工的也是这个行业比较多。
薪资:16-18也有到20块的,而且,由于从事一些和病人打交道的工作,一般健康福利会比较不错。
工作环境:其实我去过老人院和老人公寓,环境真得很不错。由于工作中动体力的时间较多,所以一般都不会让你连续工作时间太长的。
语言要求:这个工作一般对语言要求较高,不过有一点,会一门外语绝对是一个财富。
发展空间:这个专业实际上是Nurse Aid,发展当然是朝Rigester Nurse了。热门阿。
性别:女性占91%。 但是:听说急需男性阿,虽说有器械帮助,但是还是有些体力活的,所以经常在老人院里和医院里见到五大三粗的女性。还有,男性也有隐私阿,男同志有的时候也想让同性来照顾一下。顺便说一句,男护士也是非常非常受欢迎。
课程介绍,上这个课的教室很有意思,大家有兴趣可以参观。这个课对于实际操作要求很强,经常要演练急救,比如,突然有人晕倒了,怎么处理,学生们七手八脚的上。
这个课必须要西人教师上,因为西人和我们在处理人的问题上首先观念大大不同。西人把安全放在第一位,首先必须安全。所以什么民间土法,没有经过验证的不用。比如脖子不舒服,趁他不注意给他嘎吱一下。所以同样操作,观念不同,处理的方法不同。
课程介绍太长了,需要的留下邮件。包括,心理学的知识,人际沟通,营养学,老人护理,临终关怀,危机处理,家庭护理,收拾房屋,以及相关的法律法规,当然,实习是非常重要的一环
PT: Pharmacy Technician
就业前景:Very Good, 还是和老年社会有关。
薪资水平:如果在零售,10-15块,如果在社区community health care. 10-20块,医院:19-26块起,有些人成为了药剂师,工资约7万/年起,但是要再继续学习获得degree.
性别:男女均衡
语言:要求较高,学习辛苦,因为有大量东西要记,中国人毕业率高,老外经常吃不消。
自雇:自己做生意的占15%。 年收入较高。
学位:diploma, 同时还有行业的几个certificate
学习时间:40周,要学得抓紧,政府马上要把它改成两年的课程了
课程介绍:太长了,需要的给mail.

Community Service Worker 介绍,请点,就不用费时找了
太有才了!!十分有帮助,特别是在此风雨飘摇之际,希望得到详细的email. 谢了!!!加个分!!:wdb17:zhu_ct@yahoo.com.cn

sherry0820 : 2009-03-08#327
回复: toronto college介绍,选择,相关问题交流

好详细啊,谢谢分享!!

helene_st_w : 2009-03-08#328
回复: toronto college介绍,选择,相关问题交流

应你要求,先讲这个:
PT: Pharmacy Technician
就业前景:Very Good, 还是和老年社会有关。
薪资水平:如果在零售,10-15块,如果在社区community health care. 10-20块,医院:19-26块起,有些人成为了药剂师,工资约7万/年起,但是要再继续学习获得degree.
性别:男女均衡
语言:要求较高,学习辛苦,因为有大量东西要记,中国人毕业率高,老外经常吃不消。
自雇:自己做生意的占15%。 年收入较高。
学位:diploma, 同时还有行业的几个certificate
学习时间:40周,要学得抓紧,政府马上要把它改成两年的课程了
课程介绍:太长了,需要的给mail.
特别关注:现在有没有变化,还是40周么?我预计快的话5月登陆。是否需要提前攒足小时数?vancouver是多少小时?还有EI是否是想贷款一样要定期还款的?希望得到详情。zhu_ct@yahoo.com.cn

helene_st_w : 2009-03-09#329
回复: toronto college介绍,选择,相关问题交流

PSW is much easy to find a job. Payroll is hot, but actually if you don't have any background in HR or accounting, it is very hard to find a job. Only big companies need specific payroll clerks or outsourse their payroll to some professional companies.
麻烦解释一下psw 是什么意思?全称是?

helene_st_w : 2009-03-09#330
回复: toronto college介绍,选择,相关问题交流

我回来了
很高兴看到了你的归来。总之,受益匪浅!!加分!!
现说说我的情况,烦请Alexwei给点suggestion.我是学的专业很偏同时也没有工作经历,主要工作经历是外贸,工作中很少用上语言。现在的水平自问ielts 平均6.0-6.5 听说读写中写差点在5.5-6 听说读在6.5-7的样子。(也可能高估了)但还可以吧。技术移民,现在dm。到加后一切从头开始。年龄42。 拿到visa就会很快安排登陆。作贸易恐怕有一定的困难。社工、洗牙师、customer service等等我多可以考虑。第一步是survival,and then upgrade myself. 但是男怕选错行呀。现在对于我来说可能没有太多的在选择机会了。正如你所述,时间是最大的挑战。谢谢!连续几个帖子,辛苦了,再次感谢!!同时望早回复。:wdb17::wdb17::wdb32::wdb32:

helene_st_w : 2009-03-09#331
回复: toronto college介绍,选择,相关问题交流

alexwei how can I give you another 声望?

铁汉 : 2009-03-09#332
回复: toronto college介绍,选择,相关问题交流

Personal Support Worker 私人护理

加拿大的天空 : 2009-03-09#333
回复: toronto college介绍,选择,相关问题交流

好好学习,天天向上。

alexwei : 2009-03-13#334
回复: toronto college介绍,选择,相关问题交流

Personal Support Worker 私人护理,或叫护工
麻烦解释一下psw 是什么意思?全称是?

alexwei : 2009-03-13#335
回复: toronto college介绍,选择,相关问题交流

语言如果是弱项的话还是抓紧刚来的时间提高一下语言,刚来的时候提高语言最快,什么专业工作都需要语言,参加一些政府机构提供的就业和找工辅导的workshop.搞清楚加拿大的就业市场,掌握一些基本工具。这时候再选择方向。我觉得比较好。
很高兴看到了你的归来。总之,受益匪浅!!加分!!
现说说我的情况,烦请Alexwei给点suggestion.我是学的专业很偏同时也没有工作经历,主要工作经历是外贸,工作中很少用上语言。现在的水平自问ielts 平均6.0-6.5 听说读写中写差点在5.5-6 听说读在6.5-7的样子。(也可能高估了)但还可以吧。技术移民,现在dm。到加后一切从头开始。年龄42。 拿到visa就会很快安排登陆。作贸易恐怕有一定的困难。社工、洗牙师、customer service等等我多可以考虑。第一步是survival,and then upgrade myself. 但是男怕选错行呀。现在对于我来说可能没有太多的在选择机会了。正如你所述,时间是最大的挑战。谢谢!连续几个帖子,辛苦了,再次感谢!!同时望早回复。:wdb17::wdb17::wdb32::wdb32:

alexwei : 2009-03-13#336
回复: toronto college介绍,选择,相关问题交流

Yes, 我有时候称自己鸟人,自娱自乐
偶对这个也感兴趣了,改天也向ALEXWEI老师请教。

BTW,你是鸟人兄?

kathy66 : 2009-04-01#337
回复: toronto college介绍,选择,相关问题交流

AlexWei老师,我可遇到明白人了,我想上college,目前正痛苦的选择专业,我是软件编程行业,在加拿大也有一年的工作经验,可是考虑到做这个比较累,尤其对女的来说,还要照顾家庭,所以想找一个轻松点的工作。所以想到做软件测试,但不知去哪个college比较好,就业前景怎么样。还有就是想到做社工,如果作社工,学什么能最大限度的用上我的计算机知识。非常非常感谢!:wdb17:

fenggubaby : 2009-04-02#338
回复: toronto college介绍,选择,相关问题交流

鸟人兄:您好~~~

真心向您请教logistics(物流)专业~~~

如果是在加读完这个专业的学院课程,以后的工作前景怎么样啊?

另外:恳请推荐几个学校吧,谢谢!!!

litkitty : 2009-04-04#339
回复: toronto college介绍,选择,相关问题交流

PT: Pharmacy Technician
就业前景:Very Good, 还是和老年社会有关。
薪资水平:如果在零售,10-15块,如果在社区community health care. 10-20块,医院:19-26块起,有些人成为了药剂师,工资约7万/年起,但是要再继续学习获得degree.
性别:男女均衡
语言:要求较高,学习辛苦,因为有大量东西要记,中国人毕业率高,老外经常吃不消。
自雇:自己做生意的占15%。 年收入较高。
学位:diploma, 同时还有行业的几个certificate
学习时间:40周,要学得抓紧,政府马上要把它改成两年的课程了
课程介绍:太长了,需要的给mail.


这个就是我的专业啊,已经改成两年的了,我要努力,争取以后能考下药师执照

OAKVILLE08 : 2009-04-05#340
回复: toronto college介绍,选择,相关问题交流

本人来加拿大几年了,只想告诉大家,千万别相信这个LZ所写的。她的目的就是想大家掏钱念她所在的私立学校,她学校有开的课程。想不到这么多成年人这么轻信于人!

allenwang : 2009-04-06#341
回复: toronto college介绍,选择,相关问题交流

本人来加拿大几年了,只想告诉大家,千万别相信这个LZ所写的。她的目的就是想大家掏钱念她所在的私立学校,她学校有开的课程。想不到这么多成年人这么轻信于人!

我是觉得ALEX写的很好,给出了很多有用的信息,不管他的目的是什么,我从中得到了很多有用的东西,想不相信我们自有判断。你如果愿意帮助人的话,也可以为大家解答一下问题或者提供有关方面的信息。

小鬼旸 : 2009-04-07#342
回复: college介绍及选择个人谈,幼教,私人护理,社工,医师行政助理等热门行业介绍

关于公立和私人职业学校的课程和资质,可以到www.edu.gov.on.ca上去查, 所有经过政府审批的学校,以及私立职业学校的经过政府审批的diploma的课程在上面都可以查到。

首先一个概念,加拿大大专(College)跟我们国内的大专概念大不相同,更偏重于职业培训,注重的是技能和动手实践能力。大专的课程设置比较全面,合理,由浅入深易于接受。除了理论基础课程,更多的是职业培训课程、大学转学分课程和热门职业证书培训。同时,许多大专都与企业合作,甚至为大型企业设置特定专业,而且学院还经常调整专业,开设新兴的专业,尽量为学生就业创造机会,因此,大专生普遍比本科生好找工作,但如果想要做管理工作和科研工作,还得再读大学。
再说说多伦多的college, 多伦多的college分为两种,community college和private college,Community colleg就是俗称的公立大专,像Seneca, Geoge Brown,特点是学科非常全,基本上涵盖各个行业,学分可以被各个大学接受。有1年到3年的certifate, diploma和Degree的课程,也有时间更短的培训。可以在http://www.edu.gov.on.ca/eng/general/list/college.html上找到On省所有Community 大专,一共24所。学生可以向政府申请OSAP贷款上学,但是大部分课程不能够申请EI学费上学,因为学制比较长,其实有的专业实际课时不多,但是由于Community College 假期和大学类似,时间较长,所以拖得比较长。政府可不愿意你拿着EI学费还休假,不过由于竞争的关系,他们也推出一些适合申请EI学费的专业。学生毕业后的就业率在70%到80%之间。
还有一类叫private college, 也可以叫private career college, 故名思义,是私人办的以职业培训为主的college, 一般规模不大,没有专门的校园。他们不可能开设像community college那么全的课程,所以都选择市场上的热门专业,并且可以较短的时间让学生莆眨玫iploma和Certificate(如果这个行业需要专门的certificate).由于规模不大,因此各个学校都有不同的侧重点,有比较强的专业,因此选择起来要困难一些。课程一般不超过一年,提供diploma,还有很多的课程是专门帮你通过一些证书考试的,有的学校学生可以申请OSAP,但是大部分学校不行。他们主要面向自费学习和EI学费学生。由于没有假期,上课时间灵活(一般每个专业都是滚动开课,所以学生在一年当中的任何一个月都可以入学)。这些学校,就业率是命根子,如果没有很高的就业率,就难招到学生,政府也很难批准EI学生入学。因此他们的专业一定是就业趋势很好的,老师一般是专业知识强并在行业有多年工作经验的,不仅负责教授学生技能,还要帮助学生实习和找工作的指导,因此把老师叫instructor,这是和community College教学非常不同的一点,Community教学还是正规的学校式教育,一个专业,7、8门课,每个老师各教一摊。大部分private college的一个班一个老师带到底,像师傅带徒弟,老师是老板雇的,因此压力更大些。学校也积极和各种社会资源雇主联系给学生提供co-op 和就业机会,因此就业率比community college高,好的能够达到90%以上。现在比较热门的,比如IT的一些专业,社工CSW,牙医助理,幼教助理ECA,药剂师PT,Computer Accounting,护工PSW,医生行政助理MOA等。但是private college 良莠不齐,首先大家可以去政府的网站
http://www.gov.on.ca/ont/portal/!ut...s/7_0_252/_s.7_0_A/7_0_252/_l/en?docid=053263 去搜索,确保这个college是政府审批过的正在经营的college. 然后,看看这个学校的网站,看看有没有感兴趣的课程。

关于如何选择学校的问题,我想留到下次再写,希望能够逐渐补充些资料,给大家一个好的参考。也希望大家和我充分交流,提供不同意见。

6月10日续,如何选择college呢,论坛上这类东西可多了,我谈谈自己看法。我觉得核心就一个,快,时间越短越好。为什么呢,第一点,college 就是一个大专,你不管怎么学,拿什么文凭,到社会上也是一个具备初级技能的毕业生。虽说,seneca等在我们圈子名气响亮,可是并不入本地人眼。我有几个银行和政府的朋友,有本地的也有移民。我问他们关于college的问题,他们一致认为,college是非常初级的,只能从事初级工作。而且他们其实不是很清楚或者很在意college之间有什么差别。而到了大学这一级别,才开始注意学校之间的区别,这和国内差不多。第二点,既然college文凭不怎么样,我为什么还需要呢,因为我要本地教育背景阿。移民都是高学历,被这个卡住了,只好搞个本地文凭,其实大家都在学校里呆了不少年,知道学校里学的东西工作中能用得很少,一旦工作了,还得吃自己的工作经验。那这样,我们上college的目的就明确了。搞个本地文凭,既然如此,同样是diploma, 越快拿到得越好。千万不要去学那些什么2年,三年的课程。1年的本地工作经验足以媲美三年的学校教育。其实还有一点,这里的人并不是不认你在国内的工作和教育经历,只是他们认为如果你能够通过本地的学习获得技能,这样你的经验加上本地获得的技能才能把你的才干发挥出来。
其实短的课程未必不好,紧凑,只教实用的。大家如果仔细看各community大学的课程(我那里三所大学的全套),头三个学期都是基础课,一点用没有。那是给这里的高中生用的。有人说了,不对,我确实感觉学到了很多东西。是的,你主要在头三个学期学到的是英语。有些知识,翻译成中文就是普普通通的东西,一看就懂。但是你的口语恐怕提高不大。工作环境是提高口语的最好地方。所以如果集中学一个快的课,省下的时间去工作,是一个好的选择。
总而言之,时间最重要,同样都是一样的文凭,从哪拿都一样,越快越好。但是还是要找质量好的课,好的专业,这个问题下回讨论。

第三回,关于专业的选择,找热门的技能,两个办法,一个是去www.jobfutures.com上去看,那里可以查到各个专业工作的薪资水平,需要的学历,相关的专业,以及就业趋势。另外一个方法更简单,看看报纸上的各个 carrer college和培训学校的广告,那上面列的专业绝对都是比较好就业的专业,因为就业率是各个学校的生命线。没有前景,华而不实的专业绝对不会落入他们的眼睛。不过也不是绝对,career college主要关注初级技能培训和证书考试,对于有些比如医生,律师之类的可能前景很好,但是需要去大学读master. 我对于技工这块不太了解,不过大家可以去www.serviceontario.ca上面看看,在school & work里面,apprenticeship 是政府大力支持的,有很多的服务和培训。有些学校是专门教授这些的,比如常青藤。community college里面也有相关课程。不过这一块更要当心,有些技能淘汰的很快。其他方面,IT方面硬件是没有什么前途的,软件现在的课程可多了,软件测试,数据分析SAS, 网络安全都算热门。不过如果在国内不是IT人士,学这个恐怕已经晚了。还有一些课程,适合不同背景的人学习,就业前景基本可以,在这里可以介绍一下。
首先是电算化会计,其实学出来bookkeeper,不管你学两年的课程,还是6个月的速成,找工作都差不多,因为会计最需要经验,所以没有经验的只有从基层老老实实干起。在国内干过的,还是好找,那些从来没干过的,学了两年,还是难找。不过现在很多小公司找行政员工,如果同时能做些会计工作竞争力会大很多。
平均薪资:会计$15.78左右,bookkeeper 14.8左右,一开始也就拿个10-12块
就业情况:一直都还可以,比平均情况好。但是由于中国移民学这个的太多,所以现在的竞争也非常激烈
性别:女性占了91%,看来男性没什么机会,但是CFA男性多一些。不过要拿时间熬。光想办法考证是不行的,还要有机会进入事务所或大些的公司混些经验。所以如果希望将来能够发展空间大,那就要宁为牛后,不为鸡首。到大的事务所做做义工,合同工可能是必要的。
建议:学这个行业选择一个6-8个月的速成就可以了。
下一部分:幼教助理 Early Childcare Assistant
ECE o和ECA 介绍,
幼儿教育老师和幼儿教育助理,ECE需要两年学习,需要到community College里面学,ECA有很多6个月的短期课程,其实工资差不多,差1、2块钱。这个行业除了diploma外还要有certificate,基本上你在学习期间就可以考过这个证,你就可以工作了。考试费很便宜:50元左右
英语要求:ECA的英语要求不是太高
行业前景:不错,而且自雇比例颇高,达到41%。所以比较适合需要照顾家庭的女性。
平均薪资:12.87.这个工资实在没有吸引力,但是适合自雇,自己当老板赚多少,这里没有统计,但是自在啊。
性别:还是有2%的男性的!,而且男性的竞争力特强!
课程的情况看第二页的帖子
三:私人护理(personal support worker)
其实如果是需要找个挣钱多一些的职业,这个职业比ECA强,首先工资高,其次有了这个专业的diploma一样可以做ECA的工作,因为课程里面有专们如何照顾孩子的。而且学到的知识其实对自己对家庭都很有好处。只是我们中国人有的时候有偏见,觉得去照顾病人,老人,收拾床铺不太好对家中父老交待。说在国外干什么呢,私人护理,都干些啥,收拾屋子,陪老人聊天,照顾病人。人家会觉得你好好的,跑国外干这个。但是我觉得这个职业不错,听我分析一下
就业前景:很不错。整个世界都进入老年社会了。失业率只有1%!。未来中国也很需要,另外,发达社会越来越重视对于老人,病人,disable包括孩子的照顾和护理。其实不仅是照顾吃穿活动那么简单,还有精神上的照顾,属于体验经济的一类,阳光产业。而且这个行业对于自己的性格磨练很有好处,你会变得很有耐心,细心和爱心,听说对于生老病死的看法也会大大改变。自雇和part time的机会也不错,照顾照顾邻里,有40%的人做part time. 平时做义工的也是这个行业比较多。
薪资:16-18也有到20块的,而且,由于从事一些和病人打交道的工作,一般健康福利会比较不错。
工作环境:其实我去过老人院和老人公寓,环境真得很不错。由于工作中动体力的时间较多,所以一般都不会让你连续工作时间太长的。
语言要求:这个工作一般对语言要求较高,不过有一点,会一门外语绝对是一个财富。
发展空间:这个专业实际上是Nurse Aid,发展当然是朝Rigester Nurse了。热门阿。
性别:女性占91%。 但是:听说急需男性阿,虽说有器械帮助,但是还是有些体力活的,所以经常在老人院里和医院里见到五大三粗的女性。还有,男性也有隐私阿,男同志有的时候也想让同性来照顾一下。顺便说一句,男护士也是非常非常受欢迎。
课程介绍,上这个课的教室很有意思,大家有兴趣可以参观。这个课对于实际操作要求很强,经常要演练急救,比如,突然有人晕倒了,怎么处理,学生们七手八脚的上。
这个课必须要西人教师上,因为西人和我们在处理人的问题上首先观念大大不同。西人把安全放在第一位,首先必须安全。所以什么民间土法,没有经过验证的不用。比如脖子不舒服,趁他不注意给他嘎吱一下。所以同样操作,观念不同,处理的方法不同。
课程介绍太长了,需要的留下邮件。包括,心理学的知识,人际沟通,营养学,老人护理,临终关怀,危机处理,家庭护理,收拾房屋,以及相关的法律法规,当然,实习是非常重要的一环
PT: Pharmacy Technician
就业前景:Very Good, 还是和老年社会有关。
薪资水平:如果在零售,10-15块,如果在社区community health care. 10-20块,医院:19-26块起,有些人成为了药剂师,工资约7万/年起,但是要再继续学习获得degree.
性别:男女均衡
语言:要求较高,学习辛苦,因为有大量东西要记,中国人毕业率高,老外经常吃不消。
自雇:自己做生意的占15%。 年收入较高。
学位:diploma, 同时还有行业的几个certificate
学习时间:40周,要学得抓紧,政府马上要把它改成两年的课程了
课程介绍:太长了,需要的给mail.

Community Service Worker 介绍,请点,就不用费时找了
收藏:wdb6:

litkitty : 2009-04-07#343
回复: toronto college介绍,选择,相关问题交流

收藏一下

回青橙 : 2009-04-08#344
回复: toronto college介绍,选择,相关问题交流

ALEX你好。我是生物化工硕士毕业,有3年放射医学研究背景(不是临床,是科研),能直接找工作么?能找哪方面的?如果再读个学位,选哪个比较好?我是女的,28岁,英语还OK,托福和雅思都是接近满分。多谢帮忙。

神龙摆尾 : 2009-04-08#345
回复: toronto college介绍,选择,相关问题交流

收藏了,谢谢。

lily_cheng : 2009-04-09#346
回复: toronto college介绍,选择,相关问题交流

你好,我已经得到了college的accounting的offer,准备9月份开始上学,我只是有一个问题想问一下,毕业之后怎样才能考CGA呢? 报考程序是怎样的呢?

谢谢!

alexwei : 2009-04-14#347
回复: toronto college介绍,选择,相关问题交流

你好,我觉得你的背景很好,可惜在国内的医学研究背景这里不太承认。朋友有些学医的找到了一些医学方面Research的工作,你可以直接试一下,如果到了加拿大,我可以介绍几个学医的朋友。他们都挺乐与助人的。我看学生物的,有医学背景的,基本这几个方向
一些实验室的实验员工作
Clinic Research
放射技师(考证)
理疗师(考证)
医学数据分析师(进修)
洗牙师(进修考证)
药物分析(进修)
药厂的初级工作
你来了之后可以先去做一些volunteer工作
ALEX你好。我是生物化工硕士毕业,有3年放射医学研究背景(不是临床,是科研),能直接找工作么?能找哪方面的?如果再读个学位,选哪个比较好?我是女的,28岁,英语还OK,托福和雅思都是接近满分。多谢帮忙。

alexwei : 2009-04-14#348
回复: toronto college介绍,选择,相关问题交流

不懂,Thanks.如果你了解到了麻烦告诉我,我也学习一下
你好,我已经得到了college的accounting的offer,准备9月份开始上学,我只是有一个问题想问一下,毕业之后怎样才能考CGA呢? 报考程序是怎样的呢?

谢谢!

alexwei : 2009-04-14#349
回复: toronto college介绍,选择,相关问题交流

社工我觉得你就别学了,跨度太大,而且工资不高,软件测试不错,对你来说学起来应该很轻松。很多college都有这个专业,我觉得你可以都去听听。几点建议:
1、老师最好是华人,华人老师比较会教IT,你也不是去学英语,犯不着找个老外。
2、老师一定要是正在做这个,而且level比较高的,因为,只有高Level的测试工程师才有充分的经验,老师所在的公司应该是业界数的着的。这个要问到。
3、不要听学校的,主要听老师介绍。
4、一定要有项目作,没有几个项目拿来练习,学了一点用没有。
学校信息请看华人报纸,加拿大都市报上面最全了
AlexWei老师,我可遇到明白人了,我想上college,目前正痛苦的选择专业,我是软件编程行业,在加拿大也有一年的工作经验,可是考虑到做这个比较累,尤其对女的来说,还要照顾家庭,所以想找一个轻松点的工作。所以想到做软件测试,但不知去哪个college比较好,就业前景怎么样。还有就是想到做社工,如果作社工,学什么能最大限度的用上我的计算机知识。非常非常感谢!:wdb17:

yutou1995 : 2009-04-17#350
回复: toronto college介绍,选择,相关问题交流

AlexWei,我想知道“软件测试”与“SAP”目前哪个更好就业?

alexwei : 2009-04-29#351
回复: toronto college介绍,选择,相关问题交流

个人观点,软件测试的市场需求更大一些,包括软件企业,金融系统,政府公共服务机构,电子制造企业等都有需求,也就是说需求比较广泛。另外入门比较容易。
SAP作为一个大型的生产资源管理,业务资源管理系统,在很多大企业有不错的应用,但是学习SAP并不是去做最终用户,因为那只需要内部培训就行了,而且每个职位只要知道自己部分的功能就行了。如果做SAP独立顾问,在北美这个行业已经很成熟了。市场需求是有,但是总的容量不大,另外对于经验的要求很高,不光是SAP的经验还有北美相关行业的经验。
仅供参考

seven1023 : 2009-05-01#352
回复: toronto college介绍,选择,相关问题交流

想了解下fashion buyer或fashion designer的就业情况,和薪酬!!!

我本想申请fashion design的,但是因为seneca要读三年所以改成了fashion business
想出来后从事和fashion 相关的行业,希望楼主可以帮我问问,谢拉,这累信息实在太少了

lindally : 2009-05-07#353
回复: toronto college介绍,选择,相关问题交流

麻烦LZ,我想问下多伦多COLLOGE里是否有质量管理方面的专业?是否好就业?我在国内做了十年的质量管理。

alexwei : 2009-05-08#354
回复: toronto college介绍,选择,相关问题交流

想了解下fashion buyer或fashion designer的就业情况,和薪酬!!!不太懂这个行业


我本想申请fashion design的,但是因为seneca要读三年所以改成了fashion business
想出来后从事和fashion 相关的行业,希望楼主可以帮我问问,谢拉,这累信息实在太少了

alexwei : 2009-05-08#355
回复: toronto college介绍,选择,相关问题交流

我建议不要去上什么质量管理的两年制学习了,太基础了。好的办法是参加CQE的培训,考一个CQE的证书,这个证书有的人不太多,在质量管理领域内含金量挺高,然后可以考虑再考个六sigma的黑带。这个在制造业也非常有用的证书。而且这两个证及时将来回流,也非常有用。College的文凭对有些行业有用,有些行业就用处不大
麻烦LZ,我想问下多伦多COLLOGE里
是否有质量管理方面的专业?是否好就业?我在国内做了十年的质量管理。

alexwei : 2009-05-08#356
回复: toronto college介绍,选择,相关问题交流

可以直接找工作,这里健康领域有很多Research的工作,你可以试试,英语一定要好,最好通过Volunteer的机会认识行业里面的人。如果想做放射技师,还要学习和烤执照
ALEX你好。我是生物化工硕士毕业,有3年放射医学研究背景(不是临床,是科研),能直接找工作么?能找哪方面的?如果再读个学位,选哪个比较好?我是女的,28岁,英语还OK,托福和雅思都是接近满分。多谢帮忙。

lindally : 2009-05-09#357
回复: toronto college介绍,选择,相关问题交流

我建议不要去上什么质量管理的两年制学习了,太基础了。好的办法是参加CQE的培训,考一个CQE的证书,这个证书有的人不太多,在质量管理领域内含金量挺高,然后可以考虑再考个六sigma的黑带。这个在制造业也非常有用的证书。而且这两个证及时将来回流,也非常有用。College的文凭对有些行业有用,有些行业就用处不大
谢谢LZ答复!我在国内有注册的ISO9000证书,我们公司现在也有绿带培训。虽然我曾经接待过欧美很多大公司的质量监察,但我的英语不好,到那边考证书一定很费劲,可能还是会选择基础的工作。

lindally : 2009-05-11#358
回复: toronto college介绍,选择,相关问题交流

LZ,今天我查询了一下ASQ的网站,目前在北京有分支机构,可以考CQE和黑带,考试费用大概在4KRMB,CQE的培训费在4KRMB,黑带的要贵些。这两个考试题是中英文对照,但据说中文翻译的很烂。我想请教一下,在国内考ASQ的证书到加是否承认吧?在加考大概多少费用?也许我在国内先考一个CEQ,有利于尽早找到专业工作。

jasunber : 2009-05-11#359
回复: toronto college介绍,选择,相关问题交流

谢谢LZ的热心介绍和解答, 希望继续!!

alexwei : 2009-05-11#360
回复: toronto college介绍,选择,相关问题交流

如果要来加拿大,就不要在国内考证,同样的证,都打了折扣,来这里考吧,还算是加拿大教育背景呢!
LZ,今天我查询了一下ASQ的网站,目前在北京有分支机构,可以考CQE和黑带,考试费用大概在4KRMB,CQE的培训费在4KRMB,黑带的要贵些。这两个考试题是中英文对照,但据说中文翻译的很烂。我想请教一下,在国内考ASQ的证书到加是否承认吧?在加考大概多少费用?也许我在国内先考一个CEQ,有利于尽早找到专业工作。

jasur : 2009-05-12#361
回复: toronto college介绍,选择,相关问题交流

alex写的很详细喔~~~~大饱眼福,不过能给我点建议么~!
~~~我本科今年在国内完成,上的生物,很讨厌哦(以为以后会感兴趣选的),我现已拿到visa,8月底就到GUELPH了,走的研究生预科项目,雅思够了,因为gpa不高(家里又说不读研又可惜)怕不录取所以就按那个申请啦,,再说了,听说研究生转别的专业非常的难,我也不想天天在那儿research来research去的,所以在这考虑,在读完那个项目后想选择college读,读完能掌握些实际的东西就找工作~因为老妈是老个会计了,我也挺喜欢processing data之类的那种活,似乎有点想选读会计,反正想读那种出来后就能上手干点基础活慢慢往上爬的东西啦~~~~~我英语还不错,听力和口语,大学生物没学出来,倒是学了英语了,写作一般不过~看LZ对我有什么好建议慷慨的分享下拉,目前看了mohawk college的accounting有赚钱实习的阶段,像这样的好不~

tinyhuhu : 2009-05-12#362
回复: toronto college介绍,选择,相关问题交流

谢谢分享

microline : 2009-05-12#363
回复: toronto college介绍,选择,相关问题交流

好帖,收藏了,我目前在国内联通公司做网络维护管理,12年通信行业工作经验,大多数时间在做数据通信网络的规划和维护,5年通信工程项目管理经历,pmp,国内注册通信项目工程师,监理工程师,98年拿过一个CISCO的CCNA,就目前加拿大的就业环境,数据网络维护分析和通信工程项目管理那个方面更有优势,马上就要长登了,想有所准备,谢谢了!

lindally : 2009-05-12#364
回复: toronto college介绍,选择,相关问题交流

如果要来加拿大,就不要在国内考证,同样的证,都打了折扣,来这里考吧,还算是加拿大教育背景呢!
再次感谢!

jasunber : 2009-05-12#365
回复: toronto college介绍,选择,相关问题交流

LZ好啊, 从头到尾的看了所有的帖子,发现lz你很热心,也很了解college方面的消息, 昨天发的pm估计你没看到, 所以还是在这里发吧,
是有关读CMA课程的问题:
本人现在在爱尔兰学Master of Management Accounting和Business Information System的, 一直比较偏好Accounting, 一开始对CIMA认证比较感兴趣, 最近发现加拿大CMA与CIMA有很多学科类似, 但是由于CMA认证的流程有所不同, 必须持有Canadian大学Accounting Related的Bachelor Degree才可以参加考试, 但是加拿大各省都有各自的系统, 对于加拿大外甚至省外的大学都要重新evaluate, 根据学过的科目来给出exemptions, 并告知还要修哪些学科才能合格, 并提供一些大学课程和一些program进行下一步学习, 不过问题来了, 一些学校只提供part-time的课程, 还有些学校提供full-time, 几乎每个大学都有Continuing Education, 有一些College也提供CMA认证的课程学习, 让我从Bachelor开始学是不现实的, 学MBA是不划算的, 再学个两年的Master估计都吃不消, 学一些你提到的帮助考证的课程还比较靠谱, 但是我有几个问题不太明白, 希望你能解答一下:
1.去加拿大留学必须得是full-time学习吗?(这一点官方网站上并没有列出);
2.什么样得课程才算为Full-time?(比如说一周学习多长时间), 如果一周学习时间达到要求算不算full-time学生?
3.如果是full-time的, 申请colloege的non-degree的课程, 比如certificate, 还有认证的program, 能否拿到study-permit和TRV?

先谢谢了!

alexwei : 2009-05-12#366
回复: toronto college介绍,选择,相关问题交流

1.去加拿大留学必须得是full-time学习吗?(这一点官方网站上并没有列出);
必须full time学习
2.什么样得课程才算为Full-time?(比如说一周学习多长时间), 如果一周学习时间达到要求算不算full-time学生?
一周20个小时以上的full time课程,有的15-20小时也可以,part time或许可以达到,但是Full time的课程都是白天上课。
3.如果是full-time的, 申请colloege的non-degree的课程, 比如certificate, 还有认证的program, 能否拿到study-permit和TRV?
你要上diploma或者Degree的课程,不能选择certificate的课程。

先谢谢了![/quote]

alexwei : 2009-05-12#367
回复: toronto college介绍,选择,相关问题交流

CO-OP program都是要多交钱的,赚钱就不要想了,搞点加拿大经验吧

alex写的很详细喔~~~~大饱眼福,不过能给我点建议么~!
~~~我本科今年在国内完成,上的生物,很讨厌哦(以为以后会感兴趣选的),我现已拿到visa,8月底就到GUELPH了,走的研究生预科项目,雅思够了,因为gpa不高(家里又说不读研又可惜)怕不录取所以就按那个申请啦,,再说了,听说研究生转别的专业非常的难,我也不想天天在那儿research来research去的,所以在这考虑,在读完那个项目后想选择college读,读完能掌握些实际的东西就找工作~因为老妈是老个会计了,我也挺喜欢processing data之类的那种活,似乎有点想选读会计,反正想读那种出来后就能上手干点基础活慢慢往上爬的东西啦~~~~~我英语还不错,听力和口语,大学生物没学出来,倒是学了英语了,写作一般不过~看LZ对我有什么好建议慷慨的分享下拉,目前看了mohawk college的accounting有赚钱实习的阶段,像这样的好不~

jasunber : 2009-05-13#368
回复: toronto college介绍,选择,相关问题交流

1.去加拿大留学必须得是full-time学习吗?(这一点官方网站上并没有列出);
必须full time学习
2.什么样得课程才算为Full-time?(比如说一周学习多长时间), 如果一周学习时间达到要求算不算full-time学生?
一周20个小时以上的full time课程,有的15-20小时也可以,part time或许可以达到,但是Full time的课程都是白天上课。
3.如果是full-time的, 申请colloege的non-degree的课程, 比如certificate, 还有认证的program, 能否拿到study-permit和TRV?
你要上diploma或者Degree的课程,不能选择certificate的课程。

先谢谢了!
[/quote]


OK, 谢谢LZ了, 几个很重要的问题已经很明确了, 一周20小时不算多, 白天上课也无所谓, diploma或者Degree,这个有点难办啊...

alexwei : 2009-05-15#369
回复: toronto college介绍,选择,相关问题交流

感觉筒子背景好强,不如现在就在网站上申请一下职位,Rogers, Telus, Bell Canada 都可以试一下。你的专业方向我不懂。
好帖,收藏了,我目前在国内联通公司做网络维护管理,12年通信行业工作经验,大多数时间在做数据通信网络的规划和维护,5年通信工程项目管理经历,pmp,国内注册通信项目工程师,监理工程师,98年拿过一个CISCO的CCNA,就目前加拿大的就业环境,数据网络维护分析和通信工程项目管理那个方面更有优势,马上就要长登了,想有所准备,谢谢了!

jasur : 2009-05-16#370
回复: toronto college介绍,选择,相关问题交流

CO-OP program都是要多交钱的,赚钱就不要想了,搞点加拿大经验吧
~~~是,我看了,要多交900$,他们说不保证有钱赚,但是有8个月work term,最后一学期有个work experience~~这些能算经验么??唉,留学生出来找工作也不知道怎么样啊!

manmanma : 2009-05-16#371
回复: toronto college介绍,选择,相关问题交流

It's extremly useful, We are struggling for the career change of my husband right now. Would you mind leave your contact number to me, please?I am PMing you.:wdb6:

水玻莉 : 2009-05-30#372
回复: toronto college介绍,选择,相关问题交流

LZ真是热心的人,,看完了全部的帖子,,学了好多东西,
我也想说说我的情况,,因为在国内读的是文科类的,所以英语基础不是太好,,趁着拿ei期间,上了这的成人高中,现在也为选专业发愁,事实上我已经选的social service worker和internetion transportation and customs,但是分级考试的时候才考了130,,分数很低,不知道能不能拿到offer,,,,我现在也不知道该选什么?还是我可以申请EI学费学个短期的专业?似乎让我再在学校呆上两年,时间太长了,,我现在有点六神无主了,,lz给点见意我吧,,谢谢了

孙悟空000 : 2009-05-30#373
回复: toronto college介绍,选择,相关问题交流

收藏

alexwei : 2009-06-01#374
回复: toronto college介绍,选择,相关问题交流

建议上学,上学也可以找工作嘛,上学期间可以申请贷款,或者Second career training的补助,可以支持两年的
LZ真是热心的人,,看完了全部的帖子,,学了好多东西,
我也想说说我的情况,,因为在国内读的是文科类的,所以英语基础不是太好,,趁着拿ei期间,上了这的成人高中,现在也为选专业发愁,事实上我已经选的social service worker和internetion transportation and customs,但是分级考试的时候才考了130,,分数很低,不知道能不能拿到offer,,,,我现在也不知道该选什么?还是我可以申请EI学费学个短期的专业?似乎让我再在学校呆上两年,时间太长了,,我现在有点六神无主了,,lz给点见意我吧,,谢谢了

故乡的云 : 2009-06-01#375
回复: toronto college介绍,选择,相关问题交流

好贴需要顶。收藏

wudelin : 2009-06-03#376
回复: toronto college介绍,选择,相关问题交流

楼主你好,我搞数控编程的,手工编程和自动编程,能否推荐一个学校或如何做,谢谢。

linda060824 : 2009-06-03#377
回复: toronto college介绍,选择,相关问题交流

楼主可以介绍一些比较好的私立学校吗?常青藤,维多利亚等这些私立学院如何?还有更好的私立学校可以推荐吗,本人想读一些电算化会计方面的短期课程,并希望学校的就业率比较高。

jaycn90430 : 2009-06-06#378
回复: toronto college介绍,选择,相关问题交流

lz可否介绍一下COLLEGE中MANAGEMENT专业如何选修?到底修读几年较为合适?(我19岁,刚到这边,高中读到高二)

hascoming : 2009-06-09#379
回复: toronto college介绍,选择,相关问题交流

谢谢热心滴LZ 好贴~

hascoming : 2009-06-09#380
回复: toronto college介绍,选择,相关问题交流

鲜花送上~

hascoming : 2009-06-09#381
回复: toronto college介绍,选择,相关问题交流

RR

vivian7624 : 2009-06-09#382
回复: toronto college介绍,选择,相关问题交流

学习了

gracelht : 2009-06-13#383
回复: toronto college介绍,选择,相关问题交流

请问楼主,我在国内的健身房教中国民族舞蹈,不知道加拿大有这个行业吗?收入怎样,需要来加后再拿什么diploma吗?
谢谢!

annawujjmkkm : 2009-06-24#384
回复: toronto college介绍,选择,相关问题交流

介绍一下社会工作者行业
国内没有社工这个行业,不过大家应该清楚这个是干什么的,我们遇到的settlement Counselor, Employment Counselor都是属于这个行业。这个行业对于我们移民来说有一个优势,我们会1门外语,而且是目前最大的移民群体的外语,各个机构都在扩展对于中国移民的服务。这样子就需要我们了。社工为什么要拓展服务呢,大家知道,no profit机构的钱来自于政府,企业或个人捐助,自我经营收入,其中政府最多,而政府的钱是要根据你做了多少事情来得,比如政府今年有7个亿是帮助新移民的,会分配到各个非营利机构,这些机构就要做一些针对新移民的服务来申请经费,同时还要有效果,如果没有政府的funding, 这个机构就要关门了。所以从事这个行业的人往往有marketing、工商管理,教师的背景,然后获得了本地学历。那么这个行业有多大呢,这个行业非常大,和商业基本上快差不多了。那些机构属于no-profit, 医院,学校,就业、安居服务机构,慈善机构,老人服务机构,绿色和平组织,环境和动物保护组织等。他的行业自成体系,你在一般的招聘网站或者agency那里很少看到招聘的信息。需要什么样的人呢,其实和商业一样,需要sales, accounting, administration, customer service, technician.不一样的地方,核心的服务人员是社工,教师,或者医务人员。大家可以去www.charityvillige.com 上去看,这是加拿大最大的非营利机构的招聘网站。
就业前景:不错
薪资水平:16-20左右。
性别:女性居多
个人奋斗方向:如果成为专业social worker, 工资可以达到70000,但是一般需要有social science master degree. 而 bachelor 毕业实际上和college毕业薪资差别不是太大,我个人觉得可以在拿到diploma后工作两年,直接申请master(用国内的学历)+diploma+工作经验,这样不错。
课程介绍
• Introduction to Community Service Worker
• Social Welfare, Diversity and Awareness
• The Family
• Couples, Family and Interpersonal Communication
• Oriented Case Management
• Environmental Issues and Crisis Interventions and
Prevention
• Bereavement and the Elderly
• Respond to Abuse
• Understanding Human Behavior
• Communication Skills
• Counseling Skills
• Report Writing for the CSW
• Field placement

By the end of the course, students will be able to:
• Demonstrate knowledge of the range of social and community service organizations in their own community as well as detailed knowledge of an agency.
• Practice the role of a team member within a social and community service organization.
• Demonstrate beginning level professional skills
• Apply theory and skills learned in the independent study components of this course.
多谢谢提供这样的数据和信息.但不知道中国人现在这个行业就业的实际情况是怎么样子的,学习这个专业的中国毕业生的就业情况是怎么样子的呢??

xrl : 2009-07-01#385
回复: toronto college介绍,选择,相关问题交流

楼主,你好,看到你的贴子,我觉得太有道理了,我在国内做了十几年的会计,到这来以后,有人劝我去学GEOGE BROW 2年或3年的大专,我觉得时间太长,不学吧,没有本地教育背景又很难找工作,你说的那种短平快的1年以内的政府承认的会计学习有吗?能否告知,谢谢了。

janezhu : 2009-07-01#386
回复: toronto college介绍,选择,相关问题交流

这贴能翻到20页,足见楼主的功夫了。我也想请楼主帮忙给我出主意。我不断翻看不同贴已经有段日子了,但是还是没有头绪,请楼主帮我分析我的情况。不胜感谢!
说来话长,我高中毕业后上过中专:工程概预算;后来又上2年的成人大专,房地产经济与贸易;然后留在会计事务所里做资产评估,主攻房地产评估。之后又上了北大远程教育本科,也拿了学位:国际经济贸易。现在移民来到了加拿大,正在上LINC6.打算下半年考托福。
我喜欢经济类的课程,老公积极推荐我上university 的会计。我不知道我的情况能否申请第二学位(本科)?现在在第二学位(本科),MBA(Brock university 同意我可以申请),国际经济与贸易(加拿大有该专业的大学有2个,是本科,还不在安省,我家在安省的Hamilton),还是经济学研究生(经济学研究生要求的课程我都上过,平均分83)?到底哪个在加拿大有前途值得一拼啊?老公是大力支持我花3到4年全心学习,但我自己摸不着头绪......
请问楼主,象我的情况,适合申请什么呢?
谢谢!并期待您的答复。

alexwei : 2009-07-20#387
回复: toronto college介绍,选择,相关问题交流

我想如果为了就业考虑,似乎不要选择经济学之类的研究生课程。会计不错,不过要准备CGA,会计需要的更多的是证书。或者你考虑精算
这贴能翻到20页,足见楼主的功夫了。我也想请楼主帮忙给我出主意。我不断翻看不同贴已经有段日子了,但是还是没有头绪,请楼主帮我分析我的情况。不胜感谢!
说来话长,我高中毕业后上过中专:工程概预算;后来又上2年的成人大专,房地产经济与贸易;然后留在会计事务所里做资产评估,主攻房地产评估。之后又上了北大远程教育本科,也拿了学位:国际经济贸易。现在移民来到了加拿大,正在上LINC6.打算下半年考托福。
我喜欢经济类的课程,老公积极推荐我上university 的会计。我不知道我的情况能否申请第二学位(本科)?现在在第二学位(本科),MBA(Brock university 同意我可以申请),国际经济与贸易(加拿大有该专业的大学有2个,是本科,还不在安省,我家在安省的Hamilton),还是经济学研究生(经济学研究生要求的课程我都上过,平均分83)?到底哪个在加拿大有前途值得一拼啊?老公是大力支持我花3到4年全心学习,但我自己摸不着头绪......
请问楼主,象我的情况,适合申请什么呢?
谢谢!并期待您的答复。

alexwei : 2009-07-20#388
回复: toronto college介绍,选择,相关问题交流

会计只要把常用的会计软件搞清楚就行了,Geoge应该有短期的Certificate课程,你可以试一下,不需要Diploma的
楼主,你好,看到你的贴子,我觉得太有道理了,我在国内做了十几年的会计,到这来以后,有人劝我去学GEOGE BROW 2年或3年的大专,我觉得时间太长,不学吧,没有本地教育背景又很难找工作,你说的那种短平快的1年以内的政府承认的会计学习有吗?能否告知,谢谢了。

alexwei : 2009-07-20#389
回复: toronto college介绍,选择,相关问题交流

多谢谢提供这样的数据和信息.但不知道中国人现在这个行业就业的实际情况是怎么样子的,学习这个专业的中国毕业生的就业情况是怎么样子的呢??

当前就业情况-良好,
未来就业情况-难说
主要做服务大陆移民的社工,受大陆新移民增加速度的影响。
反周期行业,经济形势好转的就业机会反而少‘

chongdai : 2009-07-22#390
回复: toronto college介绍,选择,相关问题交流

首先是电算化会计,其实学出来bookkeeper,不管你学两年的课程,还是6个月的速成,找工作都差不多,因为会计最需要经验,所以没有经验的只有从基层老老实实干起。在国内干过的,还是好找,那些从来没干过的,学了两年,还是难找。不过现在很多小公司找行政员工,如果同时能做些会计工作竞争力会大很多。

yanyantze : 2009-07-22#391
回复: toronto college介绍,选择,相关问题交流

认真学习:wdb23:

amandazhu : 2009-07-23#392
回复: toronto college介绍,选择,相关问题交流

留爪!

gd1421 : 2009-07-30#393
回复: toronto college介绍,选择,相关问题交流

我来顶一下

janezhu : 2009-07-30#394
回复: toronto college介绍,选择,相关问题交流

谢谢楼主!

pwjnd : 2009-08-08#395
回复: toronto college介绍,选择,相关问题交流

受用,谢谢LZ!

bobo.lin1@163.com : 2009-08-27#396
回复: toronto college介绍,选择,相关问题交流

alex真是即专业又热心。
麻烦请教你几个问题,我已经短登过,打算后年长登(到时宝宝大概4岁),我的情况是,现在是国内大学保险专业的教师(金融也可以),硕士,去了那边想找一个时间短的,对找工作有帮助的课程,增加一下本地经验,英语还可以(上过双语的专业课,一直给外教做助教)。我现在有几个选择,想请你参谋一下。
1.最想做的是直接进入本地保险公司工作,不知是否有这样短期的培训课程,有助于应聘的,还是直接去应聘(毕竟是自己的专业和兴趣所在,自信会多一些)?
2.或者通过csc考试进入银行系统工作,但听说csc主要是sales方面的工作,不大想从事这方面的工作
3.看到你提到的社工,也很感兴趣,好像和教师关联很大,哪个college这方面的课程比较好,直接应聘有可能吗?
4,不知对加拿大的公务员有了解吗,有什么应聘的渠道,听说这个稳定,福利不错。
呵呵,问题有点多啊!麻烦你了

zyj1129 : 2009-08-28#397
回复: toronto college介绍,选择,相关问题交流

谢谢,楼主为大家提供这么多建议.
我也想请教几个问题,

注册日期: 2009-08
帖子: 5
送出鲜花次数: 0
在0帖子里收到0次鲜花
声望: 0
声望改变能力: 0 请教医学博士毕业申请加拿大博士后

--------------------------------------------------------------------------------

现在在日本读医学博士,主要是搞基础研究和临床没关系,国内是本科医学院毕业,不过来日本以后很难继续搞临床所以转向基础研究.毕业后想移民加拿大,不过看了很多资料好象条件不够,所以想先读一年博士后然后申请移民会不会容易些呢,如果直接找工作的话,比较倾向于制药公司或研究所,但是不知道加拿大这类的工作好不好找.
看了很多贴子,改行学PT或者PSW好像也不错,而且时间短,不知道可不可行呢,
还有英语不是很好,最近几年只学日语英语都扔了,是不是也需要找个学校先补补英语呢.一下问了这么多问题,不好意思,.

AAAF : 2009-08-31#398
回复: toronto college介绍,选择,相关问题交流

:wdb10:好帖要顶

clgclyj : 2009-08-31#399
回复: toronto college介绍,选择,相关问题交流

谢谢lz

clgclyj : 2009-08-31#400
回复: toronto college介绍,选择,相关问题交流

谢谢楼主,我今天打电话给你了,感谢热心解答和建议,非常中肯。以后有事情还要多请教啊:wdb17::wdb17::wdb6::wdb6:

Amanda0001 : 2009-09-10#401
回复: toronto college介绍,选择,相关问题交流

楼主能否告诉我多伦多有那些学校有社工,或者贸易物流方面的2年的COLLEGE以后可以转学分的啊,都有什么专业去?以后我想工作以后再一边工作一边进修UNIVERSITY啊。毕竟年龄不小了,耗不起4年时间啊。
谢谢啊!!!

susiewxw : 2009-09-22#402
回复: toronto college介绍,选择,相关问题交流

真是个热心的人!看到现在收益匪浅啊!
我老公也想去那里继续读书,他在国内有硕士学位(MPA),英语也是专业的。
想问问:
有什么专业可以推荐?
加拿大公务员队伍容易加入吗?
是读master还是college?
先谢谢了!

小鱼儿8848 : 2009-09-22#403
回复: toronto college介绍,选择,相关问题交流

谢谢!

罐罐碰瓢瓢 : 2009-09-23#404
回复: toronto college介绍,选择,相关问题交流

LZ,你好,我现在国内,想申请加拿大的研究生文凭课程,主要是想毕业能留加工作。因为我本身是学牙医的,中介说直接申请难度很大,建议我先申请一个文凭课程过去后再转洗牙师,但洗牙师一般都属于大专或本科水平,而且好像多对移民或加国公民开设,我这样倒转回去申请转专业能批吗?时间紧迫啊,真是太谢谢你了

schuman : 2009-09-25#405
回复: toronto college介绍,选择,相关问题交流

好贴

快快乐乐+无忧生活 : 2009-10-10#406
回复: toronto college介绍,选择,相关问题交流

你好 我想问下 我今年大二了 明年初去多伦多 我现在不知道该怎么读书了 可以college练1-2年 转学分去大学 还是有什么别的方法呢!?
直接申请大学恐怕我的各项指标都是达不到的,也想有个过渡阶段,这样子可以么?那读什么样的专业好就业呢!谢谢

lawyerwong : 2009-10-10#407
回复: toronto college介绍,选择,相关问题交流

我想问一下
1、安省的培训是不是都集中在college里面 有没有向montreal那里有很多的 vocational school 提供1-2年的职业培训的 而且还可以给贷款和补贴
2、安省上学申请贷款有没有在安省居住期间的限制 比如像魁省要求必须 第一次登陆注满3个月 从其它省转来的要住满1年
3、电工 或是 plumber 的培训这里有吗
4、在college 的短期培训 拿个certificate什么的以便上岗 这样的课程对以后再读college或是university 可以赚学分吗
5、interior decorating and display( interior designer) 的就业前景怎样

willfly : 2009-10-10#408
回复: toronto college介绍,选择,相关问题交流

强贴留爪!

redwhisk : 2009-10-13#409
回复: toronto college介绍,选择,相关问题交流

收藏。谢谢楼主

猪都笑了 : 2009-10-18#410
回复: toronto college介绍,选择,相关问题交流

up

syg103 : 2009-10-22#411
回复: college介绍及选择个人谈,幼教,私人护理,社工,医师行政助理等热门行业介绍

关于公立和私人职业学校的课程和资质,可以到www.edu.gov.on.ca上去查, 所有经过政府审批的学校,以及私立职业学校的经过政府审批的diploma的课程在上面都可以查到。

首先一个概念,加拿大大专(College)跟我们国内的大专概念大不相同,更偏重于职业培训,注重的是技能和动手实践能力。大专的课程设置比较全面,合理,由浅入深易于接受。除了理论基础课程,更多的是职业培训课程、大学转学分课程和热门职业证书培训。同时,许多大专都与企业合作,甚至为大型企业设置特定专业,而且学院还经常调整专业,开设新兴的专业,尽量为学生就业创造机会,因此,大专生普遍比本科生好找工作,但如果想要做管理工作和科研工作,还得再读大学。
再说说多伦多的college, 多伦多的college分为两种,community college和private college,Community colleg就是俗称的公立大专,像Seneca, Geoge Brown,特点是学科非常全,基本上涵盖各个行业,学分可以被各个大学接受。有1年到3年的certifate, diploma和Degree的课程,也有时间更短的培训。可以在http://www.edu.gov.on.ca/eng/general/list/college.html上找到On省所有Community 大专,一共24所。学生可以向政府申请OSAP贷款上学,但是大部分课程不能够申请EI学费上学,因为学制比较长,其实有的专业实际课时不多,但是由于Community College 假期和大学类似,时间较长,所以拖得比较长。政府可不愿意你拿着EI学费还休假,不过由于竞争的关系,他们也推出一些适合申请EI学费的专业。学生毕业后的就业率在70%到80%之间。
还有一类叫private college, 也可以叫private career college, 故名思义,是私人办的以职业培训为主的college, 一般规模不大,没有专门的校园。他们不可能开设像community college那么全的课程,所以都选择市场上的热门专业,并且可以较短的时间让学生莆眨玫iploma和Certificate(如果这个行业需要专门的certificate).由于规模不大,因此各个学校都有不同的侧重点,有比较强的专业,因此选择起来要困难一些。课程一般不超过一年,提供diploma,还有很多的课程是专门帮你通过一些证书考试的,有的学校学生可以申请OSAP,但是大部分学校不行。他们主要面向自费学习和EI学费学生。由于没有假期,上课时间灵活(一般每个专业都是滚动开课,所以学生在一年当中的任何一个月都可以入学)。这些学校,就业率是命根子,如果没有很高的就业率,就难招到学生,政府也很难批准EI学生入学。因此他们的专业一定是就业趋势很好的,老师一般是专业知识强并在行业有多年工作经验的,不仅负责教授学生技能,还要帮助学生实习和找工作的指导,因此把老师叫instructor,这是和community College教学非常不同的一点,Community教学还是正规的学校式教育,一个专业,7、8门课,每个老师各教一摊。大部分private college的一个班一个老师带到底,像师傅带徒弟,老师是老板雇的,因此压力更大些。学校也积极和各种社会资源雇主联系给学生提供co-op 和就业机会,因此就业率比community college高,好的能够达到90%以上。现在比较热门的,比如IT的一些专业,社工CSW,牙医助理,幼教助理ECA,药剂师PT,Computer Accounting,护工PSW,医生行政助理MOA等。但是private college 良莠不齐,首先大家可以去政府的网站
http://www.gov.on.ca/ont/portal/!ut...s/7_0_252/_s.7_0_A/7_0_252/_l/en?docid=053263 去搜索,确保这个college是政府审批过的正在经营的college. 然后,看看这个学校的网站,看看有没有感兴趣的课程。

关于如何选择学校的问题,我想留到下次再写,希望能够逐渐补充些资料,给大家一个好的参考。也希望大家和我充分交流,提供不同意见。

6月10日续,如何选择college呢,论坛上这类东西可多了,我谈谈自己看法。我觉得核心就一个,快,时间越短越好。为什么呢,第一点,college 就是一个大专,你不管怎么学,拿什么文凭,到社会上也是一个具备初级技能的毕业生。虽说,seneca等在我们圈子名气响亮,可是并不入本地人眼。我有几个银行和政府的朋友,有本地的也有移民。我问他们关于college的问题,他们一致认为,college是非常初级的,只能从事初级工作。而且他们其实不是很清楚或者很在意college之间有什么差别。而到了大学这一级别,才开始注意学校之间的区别,这和国内差不多。第二点,既然college文凭不怎么样,我为什么还需要呢,因为我要本地教育背景阿。移民都是高学历,被这个卡住了,只好搞个本地文凭,其实大家都在学校里呆了不少年,知道学校里学的东西工作中能用得很少,一旦工作了,还得吃自己的工作经验。那这样,我们上college的目的就明确了。搞个本地文凭,既然如此,同样是diploma, 越快拿到得越好。千万不要去学那些什么2年,三年的课程。1年的本地工作经验足以媲美三年的学校教育。其实还有一点,这里的人并不是不认你在国内的工作和教育经历,只是他们认为如果你能够通过本地的学习获得技能,这样你的经验加上本地获得的技能才能把你的才干发挥出来。
其实短的课程未必不好,紧凑,只教实用的。大家如果仔细看各community大学的课程(我那里三所大学的全套),头三个学期都是基础课,一点用没有。那是给这里的高中生用的。有人说了,不对,我确实感觉学到了很多东西。是的,你主要在头三个学期学到的是英语。有些知识,翻译成中文就是普普通通的东西,一看就懂。但是你的口语恐怕提高不大。工作环境是提高口语的最好地方。所以如果集中学一个快的课,省下的时间去工作,是一个好的选择。
总而言之,时间最重要,同样都是一样的文凭,从哪拿都一样,越快越好。但是还是要找质量好的课,好的专业,这个问题下回讨论。

第三回,关于专业的选择,找热门的技能,两个办法,一个是去www.jobfutures.com上去看,那里可以查到各个专业工作的薪资水平,需要的学历,相关的专业,以及就业趋势。另外一个方法更简单,看看报纸上的各个 carrer college和培训学校的广告,那上面列的专业绝对都是比较好就业的专业,因为就业率是各个学校的生命线。没有前景,华而不实的专业绝对不会落入他们的眼睛。不过也不是绝对,career college主要关注初级技能培训和证书考试,对于有些比如医生,律师之类的可能前景很好,但是需要去大学读master. 我对于技工这块不太了解,不过大家可以去www.serviceontario.ca上面看看,在school & work里面,apprenticeship 是政府大力支持的,有很多的服务和培训。有些学校是专门教授这些的,比如常青藤。community college里面也有相关课程。不过这一块更要当心,有些技能淘汰的很快。其他方面,IT方面硬件是没有什么前途的,软件现在的课程可多了,软件测试,数据分析SAS, 网络安全都算热门。不过如果在国内不是IT人士,学这个恐怕已经晚了。还有一些课程,适合不同背景的人学习,就业前景基本可以,在这里可以介绍一下。
首先是电算化会计,其实学出来bookkeeper,不管你学两年的课程,还是6个月的速成,找工作都差不多,因为会计最需要经验,所以没有经验的只有从基层老老实实干起。在国内干过的,还是好找,那些从来没干过的,学了两年,还是难找。不过现在很多小公司找行政员工,如果同时能做些会计工作竞争力会大很多。
平均薪资:会计$15.78左右,bookkeeper 14.8左右,一开始也就拿个10-12块
就业情况:一直都还可以,比平均情况好。但是由于中国移民学这个的太多,所以现在的竞争也非常激烈
性别:女性占了91%,看来男性没什么机会,但是CFA男性多一些。不过要拿时间熬。光想办法考证是不行的,还要有机会进入事务所或大些的公司混些经验。所以如果希望将来能够发展空间大,那就要宁为牛后,不为鸡首。到大的事务所做做义工,合同工可能是必要的。
建议:学这个行业选择一个6-8个月的速成就可以了。
下一部分:幼教助理 Early Childcare Assistant
ECE o和ECA 介绍,
幼儿教育老师和幼儿教育助理,ECE需要两年学习,需要到community College里面学,ECA有很多6个月的短期课程,其实工资差不多,差1、2块钱。这个行业除了diploma外还要有certificate,基本上你在学习期间就可以考过这个证,你就可以工作了。考试费很便宜:50元左右
英语要求:ECA的英语要求不是太高
行业前景:不错,而且自雇比例颇高,达到41%。所以比较适合需要照顾家庭的女性。
平均薪资:12.87.这个工资实在没有吸引力,但是适合自雇,自己当老板赚多少,这里没有统计,但是自在啊。
性别:还是有2%的男性的!,而且男性的竞争力特强!
课程的情况看第二页的帖子
三:私人护理(personal support worker)
其实如果是需要找个挣钱多一些的职业,这个职业比ECA强,首先工资高,其次有了这个专业的diploma一样可以做ECA的工作,因为课程里面有专们如何照顾孩子的。而且学到的知识其实对自己对家庭都很有好处。只是我们中国人有的时候有偏见,觉得去照顾病人,老人,收拾床铺不太好对家中父老交待。说在国外干什么呢,私人护理,都干些啥,收拾屋子,陪老人聊天,照顾病人。人家会觉得你好好的,跑国外干这个。但是我觉得这个职业不错,听我分析一下
就业前景:很不错。整个世界都进入老年社会了。失业率只有1%!。未来中国也很需要,另外,发达社会越来越重视对于老人,病人,disable包括孩子的照顾和护理。其实不仅是照顾吃穿活动那么简单,还有精神上的照顾,属于体验经济的一类,阳光产业。而且这个行业对于自己的性格磨练很有好处,你会变得很有耐心,细心和爱心,听说对于生老病死的看法也会大大改变。自雇和part time的机会也不错,照顾照顾邻里,有40%的人做part time. 平时做义工的也是这个行业比较多。
薪资:16-18也有到20块的,而且,由于从事一些和病人打交道的工作,一般健康福利会比较不错。
工作环境:其实我去过老人院和老人公寓,环境真得很不错。由于工作中动体力的时间较多,所以一般都不会让你连续工作时间太长的。
语言要求:这个工作一般对语言要求较高,不过有一点,会一门外语绝对是一个财富。
发展空间:这个专业实际上是Nurse Aid,发展当然是朝Rigester Nurse了。热门阿。
性别:女性占91%。 但是:听说急需男性阿,虽说有器械帮助,但是还是有些体力活的,所以经常在老人院里和医院里见到五大三粗的女性。还有,男性也有隐私阿,男同志有的时候也想让同性来照顾一下。顺便说一句,男护士也是非常非常受欢迎。
课程介绍,上这个课的教室很有意思,大家有兴趣可以参观。这个课对于实际操作要求很强,经常要演练急救,比如,突然有人晕倒了,怎么处理,学生们七手八脚的上。
这个课必须要西人教师上,因为西人和我们在处理人的问题上首先观念大大不同。西人把安全放在第一位,首先必须安全。所以什么民间土法,没有经过验证的不用。比如脖子不舒服,趁他不注意给他嘎吱一下。所以同样操作,观念不同,处理的方法不同。
课程介绍太长了,需要的留下邮件。包括,心理学的知识,人际沟通,营养学,老人护理,临终关怀,危机处理,家庭护理,收拾房屋,以及相关的法律法规,当然,实习是非常重要的一环
PT: Pharmacy Technician
就业前景:Very Good, 还是和老年社会有关。
薪资水平:如果在零售,10-15块,如果在社区community health care. 10-20块,医院:19-26块起,有些人成为了药剂师,工资约7万/年起,但是要再继续学习获得degree.
性别:男女均衡
语言:要求较高,学习辛苦,因为有大量东西要记,中国人毕业率高,老外经常吃不消。
自雇:自己做生意的占15%。 年收入较高。
学位:diploma, 同时还有行业的几个certificate
学习时间:40周,要学得抓紧,政府马上要把它改成两年的课程了
课程介绍:太长了,需要的给mail.

Community Service Worker 介绍,请点,就不用费时找了
不错
加花了

Mike-ch : 2009-10-27#412
回复: toronto college介绍,选择,相关问题交流

收藏啦

hzhg : 2009-10-28#413
回复: toronto college介绍,选择,相关问题交流

俺怎么就没有早发现这个这么有价值的帖子呢:wdb24:

那位知道楼主的联系方式,麻烦请告知,俺想请教楼主读书的一些事情。

PM俺就行,谢谢了:wdb19:

猫咪来啦 : 2009-10-29#414
回复: toronto college介绍,选择,相关问题交流

很好,收藏起来慢慢读,谢谢分享

冰水's : 2009-11-05#415
回复: toronto college介绍,选择,相关问题交流

三:私人护理(personal support worker)
其实如果是需要找个挣钱多一些的职业,这个职业比ECA强,首先工资高,其次有了这个专业的diploma一样可以做ECA的工作,因为课程里面有专们如何照顾孩子的。而且学到的知识其实对自己对家庭都很有好处。只是我们中国人有的时候有偏见,觉得去照顾病人,老人,收拾床铺不太好对家中父老交待。说在国外干什么呢,私人护理,都干些啥,收拾屋子,陪老人聊天,照顾病人。人家会觉得你好好的,跑国外干这个。但是我觉得这个职业不错,听我分析一下
就业前景:很不错。整个世界都进入老年社会了。失业率只有1%!。未来中国也很需要,另外,发达社会越来越重视对于老人,病人,disable包括孩子的照顾和护理。其实不仅是照顾吃穿活动那么简单,还有精神上的照顾,属于体验经济的一类,阳光产业。而且这个行业对于自己的性格磨练很有好处,你会变得很有耐心,细心和爱心,听说对于生老病死的看法也会大大改变。自雇和part time的机会也不错,照顾照顾邻里,有40%的人做part time. 平时做义工的也是这个行业比较多。
薪资:16-18也有到20块的,而且,由于从事一些和病人打交道的工作,一般健康福利会比较不错。
工作环境:其实我去过老人院和老人公寓,环境真得很不错。由于工作中动体力的时间较多,所以一般都不会让你连续工作时间太长的。
语言要求:这个工作一般对语言要求较高,不过有一点,会一门外语绝对是一个财富。
发展空间:这个专业实际上是Nurse Aid,发展当然是朝Rigester Nurse了。热门阿。
性别:女性占91%。 但是:听说急需男性阿,虽说有器械帮助,但是还是有些体力活的,所以经常在老人院里和医院里见到五大三粗的女性。还有,男性也有隐私阿,男同志有的时候也想让同性来照顾一下。顺便说一句,男护士也是非常非常受欢迎。
课程介绍,上这个课的教室很有意思,大家有兴趣可以参观。这个课对于实际操作要求很强,经常要演练急救,比如,突然有人晕倒了,怎么处理,学生们七手八脚的上。
这个课必须要西人教师上,因为西人和我们在处理人的问题上首先观念大大不同。西人把安全放在第一位,首先必须安全。所以什么民间土法,没有经过验证的不用。比如脖子不舒服,趁他不注意给他嘎吱一下。所以同样操作,观念不同,处理的方法不同。
课程介绍太长了,需要的留下邮件。包括,心理学的知识,人际沟通,营养学,老人护理,临终关怀,危机处理,家庭护理,收拾房屋,以及相关的法律法规,当然,实习是非常重要的一环

你好楼主,有好几个问题想问 (我是移民,landing后呆了一年回国了。准备明年再回加国,想申请这个专业)

1) 多伦多有哪些COLLEGE有这个专业,比较好的?
2)读书的周期多久的?
3)开始申请的时间是什么时候?

需要课程介绍。请发到viviannecheng@gmail.com

谢谢你了。

alexwei : 2009-11-06#416
回复: toronto college介绍,选择,相关问题交流

公立学校:Centennial, Seneca, Geroge Brown, Humber
私立:大约10来个,
国内的学分可以转一点,不过不要期望转太多。大部分大学课程3年就可以学完了。
楼主能否告诉我多伦多有那些学校有社工,或者贸易物流方面的2年的COLLEGE以后可以转学分的啊,都有什么专业去?以后我想工作以后再一边工作一边进修UNIVERSITY啊。毕竟年龄不小了,耗不起4年时间啊。
谢谢啊!!!

alexwei : 2009-11-06#417
回复: toronto college介绍,选择,相关问题交流

你好楼主,有好几个问题想问 (我是移民,landing后呆了一年回国了。准备明年再回加国,想申请这个专业)

1) 多伦多有哪些COLLEGE有这个专业,比较好的?
公立Centennial, Seneca, Geroge Brown和Humber,都没有这个专业,Geroge Brown 有护士专业。基本上只有私立College教这个,可以到www.edu.gov.gc.ca 上面查。

2)读书的周期多久的?
一般3,4个月就行了,还有一段时间实习。
3)开始申请的时间是什么时候?
私立学校一般随时都可以入学
我没有详细的课程介绍在手边。各个学校的课程设置都是不一样的,具体最好查各学校的网站。
注意这个专业由于门槛太低,已经快饱和了。

需要课程介绍。请发到viviannecheng@gmail.com

谢谢你了。

alexwei : 2009-11-06#418
回复: toronto college介绍,选择,相关问题交流

安省和Montreal差不多,Voational School就是Private Career College
贷款安省部分好像有要求
电工培训当然有
Certificate的课程也可以转学分不过要具体情况具体分析
不懂那个专业
我想问一下
1、安省的培训是不是都集中在college里面 有没有向montreal那里有很多的 vocational school 提供1-2年的职业培训的 而且还可以给贷款和补贴
2、安省上学申请贷款有没有在安省居住期间的限制 比如像魁省要求必须 第一次登陆注满3个月 从其它省转来的要住满1年
3、电工 或是 plumber 的培训这里有吗
4、在college 的短期培训 拿个certificate什么的以便上岗 这样的课程对以后再读college或是university 可以赚学分吗
5、interior decorating and display( interior designer) 的就业前景怎样

linda1668 : 2009-11-07#419
回复: toronto college介绍,选择,相关问题交流

楼主,你好,看到你是专业人士,所以想向你请教下。我目前在国内,正在和老公一起申请加技术移民,他在那边是会计。我在国内本科读的是社会工作,研究生读的是社会学,今年刚毕业,国内的社会学不吃香,我们同学毕业很少有干相关工作的,除了进机关和学校当辅导员。看到你说那边的社工还是很有前途,很鼓舞人心的。之前因为担心到加不好找工作,就一直考虑换专业。看网上都说学会计比较保险,所以他父母也一直建议我在国内有时间可以先学学,到时候不太吃力。刚看到你写的社会工作在那边就业前景不错,也属于朝阳行业,那么我到那边是否可以从事以这方面的工作为生?之前和老公也探讨过,他说这个专业需要英语水平非常高,需要到那边再好好学英语才行。而且这个行业一般华人是非常少的。
我现在困惑的是,究竟我到那边该以什么来做为我的发展方向?会计的话,学上一年,就可以找个工作。而且我的数学也不差,高中我们不分文理,其实我那时候理科比较好些,但后来阴差阳错进了社会工作这个专业。

学社会工作的话,我是否要那边再读个master,(国内的本科文凭他们应该承认吧)而且毕业了不知道人家那边接受不接受,作为华人在那边好不好找到工作?另外我是否需要在国内考托?(雅思可以么)

冰水's : 2009-11-11#420
回复: toronto college介绍,选择,相关问题交流

楼主你好,我怎么看不到你的回复呢?:wdb2:

冰水's : 2009-11-11#421
回复: toronto college介绍,选择,相关问题交流

sry, 仔细得看到了。谢谢。

最主要想做个比较。会去认真做做功课的。

beary-rene : 2009-11-12#422
回复: toronto college介绍,选择,相关问题交流

留个爪印儿慢慢看

mentexcathy : 2009-11-13#423
回复: toronto college介绍,选择,相关问题交流

一口气看了这里的大部分贴,这里好人真多,好贴也多。
我快landing了,发愁过去后干啥?
我大学毕业在国内做过十余年儿科医生,近8年在做医疗设备公司工作。----一看就知道年龄过四张了。
如果加拿大landing后可以做啥工作?
看网上说药剂师助理、医疗技师、儿童护理、护工等等工作挺热闹的,难道这些工作新移民申请很多?都是有医学背景的人适合做的?工作好找吗?
有关护工,在国内都是农村刚来的人干的,医生出去干这个心里落差是不是特别大,而且这工作在加要求语言能力非常高?
各位还有什么好的建议?
非常感谢!

alexwei : 2009-11-16#424
回复: toronto college介绍,选择,相关问题交流

护工不是一个移民可以长期干的行业,建议可以考虑放射技师,药物分析,检验员,洗牙师等行业

hzhg : 2009-11-16#425
回复: toronto college介绍,选择,相关问题交流

看到楼主又出现了,不容易呀。

俺大学学的工商管理,后来一直在高校从事教学,主要侧重于战略管理,后又从事了几年企业管理咨询工作,侧重于业务流程重组和人力资源管理咨询。由于所学和所做均是依赖文化背景的软科学,所以来加后自然就失去了自己的优势,从事回原来的工作是不可能的了,所以想通过读书来达到提高英语和获得本地文凭的目的,俺不想在读书上花费太多时间,读个college或者post-graduate的课程;只是一直很困惑,不知该选择什么样的专业;

希望能听到楼主的建议,先谢谢了!

alexwei : 2009-11-19#426
回复: toronto college介绍,选择,相关问题交流

一般来说,移民由于文化和语言的劣势,应该选择技术工作,学习一些专业技术,所以很多IT人士工作好找,就是如此。软的方面要想出头就要有四、五年卧薪尝胆的打算。
1、下决心学好语言,语言要达到精通需要专门的学习,一天花8个小时学语言不过分
2、选择一些名牌学校的管理方向的硕士课程,相对来说硕士课程对于我们更好些,本科课程其实都是些基础课,没什么用。和国内一样,加拿大对于学历也很重视。
3、看看有没有一些专业技术需要掌握来增强竞争力,比如什么行业证书等,或者某种分析软件。
4、认准行业,哪怕做volunteer,也坚决不回头。
这样的话,能成。我见过非常多在西人领域内工作的软技能的移民。

hzhg : 2009-11-19#427
回复: toronto college介绍,选择,相关问题交流

谢谢楼主。

只是感觉年龄已大,所以才难以决定。

mersara : 2009-11-24#428
回复: toronto college介绍,选择,相关问题交流

你的帖子太好了~ 竟然持续了一年都没沉~~~牛!
所以也想借个地方问下,我其实在澳洲已经是酒店管理diploma毕业的了,但是好像过来加拿大在这个方面不太好找工作,所以想读个好找工作的课程。
看了你的讲解有个问题:

洗牙师和法律顾问读完要考取牌照的吧,但好像是要就读的学校有相关的license才能参加相关的行业考试,是这样的么? 能分别介绍个在这两个方面好的学校么?
此外,如果读公立学校是否可以申请coop ,以便更好找工作?(我是已经移民的)


此外,lz能给留个联系方式么?万分感谢~~!!

littlehappybird : 2009-11-25#429
回复: toronto college介绍,选择,相关问题交流

学习

小鬼旸 : 2009-12-03#430
回复: 求学选择个人谈及幼教,私人护理,社工,医师行政助理等热门行业介绍

ding

alexwei : 2009-12-03#431
回复: toronto college介绍,选择,相关问题交流

中国开放对加拿大的旅游了,你的机会不错,不过澳洲的学历不太好使,不如再在加拿大读一个相关专业

你的帖子太好了~ 竟然持续了一年都没沉~~~牛!
所以也想借个地方问下,我其实在澳洲已经是酒店管理diploma毕业的了,但是好像过来加拿大在这个方面不太好找工作,所以想读个好找工作的课程。
看了你的讲解有个问题:

洗牙师和法律顾问读完要考取牌照的吧,但好像是要就读的学校有相关的license才能参加相关的行业考试,是这样的么? 能分别介绍个在这两个方面好的学校么?
此外,如果读公立学校是否可以申请coop ,以便更好找工作?(我是已经移民的)


此外,lz能给留个联系方式么?万分感谢~~!!

mersara : 2009-12-07#432
回复: toronto college介绍,选择,相关问题交流

中国开放对加拿大的旅游了,你的机会不错,不过澳洲的学历不太好使,不如再在加拿大读一个相关专业

嗯,著著。想?下你迂得牙狒助理跟洗牙?哪?更好?呢?:wdb19:

著著!!

stone_T : 2009-12-09#433
回复: toronto college介绍,选择,相关问题交流

应你要求,先讲这个:
PT: Pharmacy Technician
就业前景:Very Good, 还是和老年社会有关。
薪资水平:如果在零售,10-15块,如果在社区community health care. 10-20块,医院:19-26块起,有些人成为了药剂师,工资约7万/年起,但是要再继续学习获得degree.
性别:男女均衡
语言:要求较高,学习辛苦,因为有大量东西要记,中国人毕业率高,老外经常吃不消。
自雇:自己做生意的占15%。 年收入较高。
学位:diploma, 同时还有行业的几个certificate
学习时间:40周,要学得抓紧,政府马上要把它改成两年的课程了
课程介绍:太长了,需要的给mail.


入门要求如何?
英文几级?
化学?

还有洗牙师如何?

不胜感激!

Tangjie : 2009-12-09#434
回复: 求学选择个人谈及幼教,私人护理,社工,医师行政助理等热门行业介绍

有用,收藏了!

xue_yc_65 : 2009-12-10#435
回复: 求学选择个人谈及幼教,私人护理,社工,医师行政助理等热门行业介绍

收藏!谢谢!

ljyl99 : 2009-12-11#436
回复: 求学选择个人谈及幼教,私人护理,社工,医师行政助理等热门行业介绍

谢谢楼主的分享!收获良多啊。

teqquila : 2009-12-16#437
回复: toronto college介绍,选择,相关问题交流

一般来说,移民由于文化和语言的劣势,应该选择技术工作,学习一些专业技术,所以很多IT人士工作好找,就是如此。软的方面要想出头就要有四、五年卧薪尝胆的打算。
1、下决心学好语言,语言要达到精通需要专门的学习,一天花8个小时学语言不过分
2、选择一些名牌学校的管理方向的硕士课程,相对来说硕士课程对于我们更好些,本科课程其实都是些基础课,没什么用。和国内一样,加拿大对于学历也很重视。
3、看看有没有一些专业技术需要掌握来增强竞争力,比如什么行业证书等,或者某种分析软件。
4、认准行业,哪怕做volunteer,也坚决不回头。
这样的话,能成。我见过非常多在西人领域内工作的软技能的移民。



楼主您好!有一些问题想请教下:

本人国内大学是修国际贸易的,英语专八,毕业后在上海一直就职于英美国家500强公司在国内的分公司或办事处,有三四年的财务分析经验, 一年多国际采购,一年多总裁助理. 之后有六年经营美容会所的经验.

象我这样,是在国内读个CGA证书出来, 还是到加拿大COLLEGE读个短期的会计班?在薪资上有多大差异?

或是干脆先干美容这一行? 将来自行开业的话行业前景好吗?

本人英语还不错,如果刚来时直接找个办公室文员之类的工作有希望吗?
我其实并不很喜欢会计这行,如果改其他专业的话,有没有比较有前途,薪酬较好的专业推荐??

问题比较多,麻烦您了!!

achillean : 2009-12-17#438
回复: toronto college介绍,选择,相关问题交流

如果上marketing, 或者Trade, 不要diploma, 又是part time 学习,一年左右,建议到seneca读一个 相关certificate专业,你可以去他们网站查一下.如果想要diploma, 一年在community college里面做不到,你可以到私立大专,Canadian Business School. 不过费用肯定很高。

那么这么certificate 和diploma找起工作来有什么区别吗??
我看到很多学校的certificate是part-time的,也就说没有入学要求,只要你可以读完十四门功课,就可以拿到这个certificate,没有什么英文十二级要求,数学十一级要求,或者托福,雅思之类的要求.
但是,diploma却有很多的要求.
从这个角度来说,是不是diploma 比certificate更加证明个人有能力呀(我是说在加拿大生活的能力,包括英文,当地文化之类的)
不知道, 这两种东西拿出来的功效是一样的吗??当然,就是为了找份工作, 不知道哪一种更好找工作呀??
现在真的迷惑了,什么是加拿大学习背景?? 一个没有入门要求的certificate是否也是可以的呀??

westwolf11 : 2009-12-22#439
回复: toronto college介绍,选择,相关问题交流

楼主,你好,看到你是专业人士,所以想向你请教下。我目前在国内,正在和老公一起申请加技术移民,他在那边是会计。我在国内本科读的是社会工作,研究生读的是社会学,今年刚毕业,国内的社会学不吃香,我们同学毕业很少有干相关工作的,除了进机关和学校当辅导员。看到你说那边的社工还是很有前途,很鼓舞人心的。之前因为担心到加不好找工作,就一直考虑换专业。看网上都说学会计比较保险,所以他父母也一直建议我在国内有时间可以先学学,到时候不太吃力。刚看到你写的社会工作在那边就业前景不错,也属于朝阳行业,那么我到那边是否可以从事以这方面的工作为生?之前和老公也探讨过,他说这个专业需要英语水平非常高,需要到那边再好好学英语才行。而且这个行业一般华人是非常少的。
我现在困惑的是,究竟我到那边该以什么来做为我的发展方向?会计的话,学上一年,就可以找个工作。而且我的数学也不差,高中我们不分文理,其实我那时候理科比较好些,但后来阴差阳错进了社会工作这个专业。

学社会工作的话,我是否要那边再读个master,(国内的本科文凭他们应该承认吧)而且毕业了不知道人家那边接受不接受,作为华人在那边好不好找到工作?另外我是否需要在国内考托?(雅思可以么)

社会学和社工是两个完全不同的概念。不要因为中文里他们都姓社就以为他们有共同之处。

社工是为有需要的个人、家庭、机构和社区提供专业社会服务,关键是能干活。

社工与社会学不相关!

westwolf11 : 2009-12-22#440
回复: toronto college介绍,选择,相关问题交流

看到楼主又出现了,不容易呀。

俺大学学的工商管理,后来一直在高校从事教学,主要侧重于战略管理,后又从事了几年企业管理咨询工作,侧重于业务流程重组和人力资源管理咨询。由于所学和所做均是依赖文化背景的软科学,所以来加后自然就失去了自己的优势,从事回原来的工作是不可能的了,所以想通过读书来达到提高英语和获得本地文凭的目的,俺不想在读书上花费太多时间,读个college或者post-graduate的课程;只是一直很困惑,不知该选择什么样的专业;

希望能听到楼主的建议,先谢谢了!

确实,你的学历和经历都得彻底归零了。专业选择1、看自己得喜好及入行难度;2、看自己年龄其实你移民牺牲很大,真得有必要移吗?

tina1966 : 2009-12-22#441
回复: 求学选择个人谈及幼教,私人护理,社工,医师行政助理等热门行业介绍

楼主,你好!我今年43岁,来加拿大6个月了,以前在国内学的是机械专业,做过机械采购和管理,也做过档案管理,现在加拿大一小城市,因老公以前工作和生活在许多大城市,现在又重新回到这个小城市,找到一个稳定的工作,所以现在我们再移到其它城市的可能性几乎没有,本想来加拿大后做一些本专业的工作,可这个小城市也没有什么工业和商贸,比较适合养老,所以老年人比较多,所以我只有立足我们这个城市,重新去学一个专业,寻找一些其他的工作机会,有朋友推荐去学PSW,也有推荐学财会,幼师,等等,不过我比较倾向去学PSW,不知你有什么好的建议?另外,你们学校有些什么专业?和这些专业都什么时候开学?学习多长时间及费用?一下问这么多问题,麻烦你了!
谢谢你!
祝圣诞快乐及新年快乐!

粉红猪 : 2009-12-23#442
回复: 求学选择个人谈及幼教,私人护理,社工,医师行政助理等热门行业介绍

你好,我是刚登陆不久的新移民。在国内 我是高中的英语老师。过来后LINC 测试是7级。想到COLLEGE 里读书。有两个专业是想考虑的,一个是社工专业一个是配镜师(初级的)。请您给些指点。哪个的就业前景更好点。据说 现在社工的读得人太多了,不好找工作是吗?我老公是国内公务员。大学的专业是文秘。在国内单位里属于个人兴趣电脑弄的不错 。他们局里的电脑信息都是他在做的。整个局域网也是他在搞的和维护的。但是他英语不行。LINC测试只有3级。我觉得好像是还没有。请问像他这样读一阵语言后,学软件测试可能吗?(国内只有经验,没有文凭)。另如果想学个技术工的活。请问有什么建议?(他手巧)

we25888 : 2009-12-28#443
回复: 求学选择个人谈及幼教,私人护理,社工,医师行政助理等热门行业介绍

楼主对各行业都很熟悉啊,
能讲一下房地产评估吗,
新移民如果是RICS(皇家测量师学会会员),想在加拿大工作的话,还得有什么条件(本地教育、本地工作、语言)或是走怎样的过程比较好呢?
薪水工作时间如何?
看了好多地方,没有找到具体的介绍,可能是太专业了吧:)

danni : 2009-12-30#444
回复: 求学选择个人谈及幼教,私人护理,社工,医师行政助理等热门行业介绍

好帖子,一直看到最后,但是后来很少看到LZ的回复啦。是PM了,还是LZ太忙呢???

sinoxixi : 2009-12-30#445
回复: 求学选择个人谈及幼教,私人护理,社工,医师行政助理等热门行业介绍

一口气看完了所有的帖子,谢谢楼主热心和wlh的解答,你俩的答复让我获益良多。
还有个关于成绩单的信息想咨询下:
一、看了一些国外学校的网页,对于成绩单的要求部分没有做明确的说明。要求国内学校出具成绩单时密封、加盖骑缝章这是通常的要求,可是否一定是要国内的地址往外发呢(Mcgill大学的入学要求是明确要求国内的学校往外寄)还是说,将成绩单自行带至加国,到了申请学校时候,向国外的学校递交材料就可?
二、另外,国内已经本科毕业的话,不需要再提供高中时期的英文成绩单了,对不?(我非常怀疑我读的高中学校可能压根就没有开具成绩单的地方)
三、个人陈述和推荐信是必须的么(Mcgill明确有这方面的要求,可我在多市的collge上网页好像没有发现)

大老爷们 : 2010-01-03#446
回复: 求学选择个人谈及幼教,私人护理,社工,医师行政助理等热门行业介绍

楼主写的好!谢谢了,支持顶一下!!

LLY-02 : 2010-01-07#447
回复: 求学选择个人谈及幼教,私人护理,社工,医师行政助理等热门行业介绍

楼主写的好!谢谢了,支持顶一下!!

Anier : 2010-01-07#448
回复: 求学选择个人谈及幼教,私人护理,社工,医师行政助理等热门行业介绍

请问不是加拿大公民可以申请社工的工作吗

huxiaojing0187 : 2010-01-21#449
回复: 求学选择个人谈及幼教,私人护理,社工,医师行政助理等热门行业介绍

这个帖子非常有用,非常感谢!

墨菲 : 2010-01-27#450
回复: 求学选择个人谈及幼教,私人护理,社工,医师行政助理等热门行业介绍

:wdb17::wdb17::wdb19::wdb10:

terror79 : 2010-02-07#451
回复: 求学选择个人谈及幼教,私人护理,社工,医师行政助理等热门行业介绍

谢谢

styxpatron : 2010-02-09#452
回复: 求学选择个人谈及幼教,私人护理,社工,医师行政助理等热门行业介绍

谢谢分享!

一样的星空 : 2010-02-14#453
回复: 求学选择个人谈及幼教,私人护理,社工,医师行政助理等热门行业介绍

楼主,你好!我对Interior Design这个专业比较感兴趣,请问能否对它作一点介绍,这个专业学出来是不是很难找工作,对语言的要求是不是很高,请帮忙指点一下.谢谢.

alexwei : 2010-02-16#454
回复: 求学选择个人谈及幼教,私人护理,社工,医师行政助理等热门行业介绍

这个专业对于语言要求较高,如果要到设计公司工作的话。
楼主,你好!我对Interior Design这个专业比较感兴趣,请问能否对它作一点介绍,这个专业学出来是不是很难找工作,对语言的要求是不是很高,请帮忙指点一下.谢谢.

alexwei : 2010-02-16#455
回复: 求学选择个人谈及幼教,私人护理,社工,医师行政助理等热门行业介绍

根据各个学校的要求来。
一口气看完了所有的帖子,谢谢楼主热心和wlh的解答,你俩的答复让我获益良多。
还有个关于成绩单的信息想咨询下:
一、看了一些国外学校的网页,对于成绩单的要求部分没有做明确的说明。要求国内学校出具成绩单时密封、加盖骑缝章这是通常的要求,可是否一定是要国内的地址往外发呢(Mcgill大学的入学要求是明确要求国内的学校往外寄)还是说,将成绩单自行带至加国,到了申请学校时候,向国外的学校递交材料就可?
二、另外,国内已经本科毕业的话,不需要再提供高中时期的英文成绩单了,对不?(我非常怀疑我读的高中学校可能压根就没有开具成绩单的地方)
三、个人陈述和推荐信是必须的么(Mcgill明确有这方面的要求,可我在多市的collge上网页好像没有发现)

alexwei : 2010-02-16#456
回复: 求学选择个人谈及幼教,私人护理,社工,医师行政助理等热门行业介绍

英语不错,社工稳定但是工资不高,门槛低,应该需求还是不错的。配镜师我不了解。先生如果英语不太好,学习测试之类应该不错。IT方向永远是个好选择,但是要有刻苦学习的准备。
你好,我是刚登陆不久的新移民。在国内 我是高中的英语老师。过来后LINC 测试是7级。想到COLLEGE 里读书。有两个专业是想考虑的,一个是社工专业一个是配镜师(初级的)。请您给些指点。哪个的就业前景更好点。据说 现在社工的读得人太多了,不好找工作是吗?我老公是国内公务员。大学的专业是文秘。在国内单位里属于个人兴趣电脑弄的不错 。他们局里的电脑信息都是他在做的。整个局域网也是他在搞的和维护的。但是他英语不行。LINC测试只有3级。我觉得好像是还没有。请问像他这样读一阵语言后,学软件测试可能吗?(国内只有经验,没有文凭)。另如果想学个技术工的活。请问有什么建议?(他手巧)

janey : 2010-02-19#457
回复: 求学选择个人谈及幼教,私人护理,社工,医师行政助理等热门行业介绍

关于公立和私人职业学校的课程和资质,可以到www.edu.gov.on.ca上去查, 所有经过政府审批的学校,以及私立职业学校的经过政府审批的diploma的课程在上面都可以查到。

首先一个概念,加拿大大专(College)跟我们国内的大专概念大不相同,更偏重于职业培训,注重的是技能和动手实践能力。大专的课程设置比较全面,合理,由浅入深易于接受。除了理论基础课程,更多的是职业培训课程、大学转学分课程和热门职业证书培训。同时,许多大专都与企业合作,甚至为大型企业设置特定专业,而且学院还经常调整专业,开设新兴的专业,尽量为学生就业创造机会,因此,大专生普遍比本科生好找工作,但如果想要做管理工作和科研工作,还得再读大学。

再说说多伦多的college, 多伦多的college分为两种,community college和private college,Community colleg就是俗称的公立大专,像Seneca, Geoge Brown,特点是学科非常全,基本上涵盖各个行业,学分可以被各个大学接受。有1年到3年的certifate, diploma和Degree的课程,也有时间更短的培训。可以在[URL http://www.edu.gov.on.ca/eng/general/list/college.html[/url]上找到On省所有Community 大专,一共24所。学生可以向政府申请OSAP贷款上学,但是大部分课程不能够申请EI学费上学,因为学制比较长,其实有的专业实际课时不多,但是由于Community College 假期和大学类似,时间较长,所以拖得比较长。政府可不愿意你拿着EI学费还休假,不过由于竞争的关系,他们也推出一些适合申请EI学费的专业。学生毕业后的就业率在70%到80%之间。

还有一类叫private college, 也可以叫private career college, 故名思义,是私人办的以职业培训为主的college, 一般规模不大,没有专门的校园。他们不可能开设像community college那么全的课程,所以都选择市场上的热门专业,并且可以较短的时间让学生莆眨玫iploma和Certificate(如果这个行业需要专门的certificate).由于规模不大,因此各个学校都有不同的侧重点,有比较强的专业,因此选择起来要困难一些。课程一般不超过一年,提供diploma,还有很多的课程是专门帮你通过一些证书考试的,有的学校学生可以申请OSAP,但是大部分学校不行。他们主要面向自费学习和EI学费学生。由于没有假期,上课时间灵活(一般每个专业都是滚动开课,所以学生在一年当中的任何一个月都可以入学)。这些学校,就业率是命根子,如果没有很高的就业率,就难招到学生,政府也很难批准EI学生入学。因此他们的专业一定是就业趋势很好的,老师一般是专业知识强并在行业有多年工作经验的,不仅负责教授学生技能,还要帮助学生实习和找工作的指导,因此把老师叫instructor,这是和community College教学非常不同的一点,Community教学还是正规的学校式教育,一个专业,7、8门课,每个老师各教一摊。大部分private college的一个班一个老师带到底,像师傅带徒弟,老师是老板雇的,因此压力更大些。学校也积极和各种社会资源雇主联系给学生提供co-op 和就业机会,因此就业率比community college高,好的能够达到90%以上。现在比较热门的,比如IT的一些专业,社工CSW,牙医助理,幼教助理ECA,药剂师PT,Computer Accounting,护工PSW,医生行政助理MOA等。但是private college 良莠不齐,首先大家可以去政府的网站
http://www.gov.on.ca/ont/portal/!ut...s/7_0_252/_s.7_0_A/7_0_252/_l/en?docid=053263 去搜索,确保这个college是政府审批过的正在经营的college. 然后,看看这个学校的网站,看看有没有感兴趣的课程。

关于如何选择学校的问题,我想留到下次再写,希望能够逐渐补充些资料,给大家一个好的参考。也希望大家和我充分交流,提供不同意见。

6月10日续,如何选择college呢,论坛上这类东西可多了,我谈谈自己看法。我觉得核心就一个,快,时间越短越好。为什么呢,第一点,college 就是一个大专,你不管怎么学,拿什么文凭,到社会上也是一个具备初级技能的毕业生。虽说,seneca等在我们圈子名气响亮,可是并不入本地人眼。我有几个银行和政府的朋友,有本地的也有移民。我问他们关于college的问题,他们一致认为,college是非常初级的,只能从事初级工作。而且他们其实不是很清楚或者很在意college之间有什么差别。而到了大学这一级别,才开始注意学校之间的区别,这和国内差不多。第二点,既然college文凭不怎么样,我为什么还需要呢,因为我要本地教育背景阿。移民都是高学历,被这个卡住了,只好搞个本地文凭,其实大家都在学校里呆了不少年,知道学校里学的东西工作中能用得很少,一旦工作了,还得吃自己的工作经验。那这样,我们上college的目的就明确了。搞个本地文凭,既然如此,同样是diploma, 越快拿到得越好。千万不要去学那些什么2年,三年的课程。1年的本地工作经验足以媲美三年的学校教育。其实还有一点,这里的人并不是不认你在国内的工作和教育经历,只是他们认为如果你能够通过本地的学习获得技能,这样你的经验加上本地获得的技能才能把你的才干发挥出来。

其实短的课程未必不好,紧凑,只教实用的。大家如果仔细看各community大学的课程(我那里三所大学的全套),头三个学期都是基础课,一点用没有。那是给这里的高中生用的。有人说了,不对,我确实感觉学到了很多东西。是的,你主要在头三个学期学到的是英语。有些知识,翻译成中文就是普普通通的东西,一看就懂。但是你的口语恐怕提高不大。工作环境是提高口语的最好地方。所以如果集中学一个快的课,省下的时间去工作,是一个好的选择。
总而言之,时间最重要,同样都是一样的文凭,从哪拿都一样,越快越好。但是还是要找质量好的课,好的专业,这个问题下回讨论。

第三回,关于专业的选择,找热门的技能,两个办法,一个是去www.jobfutures.com上去看,那里可以查到各个专业工作的薪资水平,需要的学历,相关的专业,以及就业趋势。另外一个方法更简单,看看报纸上的各个 carrer college和培训学校的广告,那上面列的专业绝对都是比较好就业的专业,因为就业率是各个学校的生命线。没有前景,华而不实的专业绝对不会落入他们的眼睛。不过也不是绝对,career college主要关注初级技能培训和证书考试,对于有些比如医生,律师之类的可能前景很好,但是需要去大学读master. 我对于技工这块不太了解,不过大家可以去www.serviceontario.ca上面看看,在school & work里面,apprenticeship 是政府大力支持的,有很多的服务和培训。有些学校是专门教授这些的,比如常青藤。community college里面也有相关课程。不过这一块更要当心,有些技能淘汰的很快。其他方面,IT方面硬件是没有什么前途的,软件现在的课程可多了,软件测试,数据分析SAS, 网络安全都算热门。不过如果在国内不是IT人士,学这个恐怕已经晚了。还有一些课程,适合不同背景的人学习,就业前景基本可以,在这里可以介绍一下。

首先是电算化会计,其实学出来bookkeeper,不管你学两年的课程,还是6个月的速成,找工作都差不多,因为会计最需要经验,所以没有经验的只有从基层老老实实干起。在国内干过的,还是好找,那些从来没干过的,学了两年,还是难找。不过现在很多小公司找行政员工,如果同时能做些会计工作竞争力会大很多。

平均薪资:会计$15.78左右,bookkeeper 14.8左右,一开始也就拿个10-12块
就业情况:一直都还可以,比平均情况好。但是由于中国移民学这个的太多,所以现在的竞争也非常激烈

性别:女性占了91%,看来男性没什么机会,但是CFA男性多一些。不过要拿时间熬。光想办法考证是不行的,还要有机会进入事务所或大些的公司混些经验。所以如果希望将来能够发展空间大,那就要宁为牛后,不为鸡首。到大的事务所做做义工,合同工可能是必要的。
建议:学这个行业选择一个6-8个月的速成就可以了。

下一部分:幼教助理 Early Childcare Assistant
ECE o和ECA 介绍,
幼儿教育老师和幼儿教育助理,ECE需要两年学习,需要到community College里面学,ECA有很多6个月的短期课程,其实工资差不多,差1、2块钱。这个行业除了diploma外还要有certificate,基本上你在学习期间就可以考过这个证,你就可以工作了。考试费很便宜:50元左右
英语要求:ECA的英语要求不是太高
行业前景:不错,而且自雇比例颇高,达到41%。所以比较适合需要照顾家庭的女性。
平均薪资:12.87.这个工资实在没有吸引力,但是适合自雇,自己当老板赚多少,这里没有统计,但是自在啊。
性别:还是有2%的男性的!,而且男性的竞争力特强!
课程的情况看第二页的帖子

三:私人护理(personal support worker)
其实如果是需要找个挣钱多一些的职业,这个职业比ECA强,首先工资高,其次有了这个专业的diploma一样可以做ECA的工作,因为课程里面有专们如何照顾孩子的。而且学到的知识其实对自己对家庭都很有好处。只是我们中国人有的时候有偏见,觉得去照顾病人,老人,收拾床铺不太好对家中父老交待。说在国外干什么呢,私人护理,都干些啥,收拾屋子,陪老人聊天,照顾病人。人家会觉得你好好的,跑国外干这个。但是我觉得这个职业不错,听我分析一下
就业前景:很不错。整个世界都进入老年社会了。失业率只有1%!。未来中国也很需要,另外,发达社会越来越重视对于老人,病人,disable包括孩子的照顾和护理。其实不仅是照顾吃穿活动那么简单,还有精神上的照顾,属于体验经济的一类,阳光产业。而且这个行业对于自己的性格磨练很有好处,你会变得很有耐心,细心和爱心,听说对于生老病死的看法也会大大改变。自雇和part time的机会也不错,照顾照顾邻里,有40%的人做part time. 平时做义工的也是这个行业比较多。
薪资:16-18也有到20块的,而且,由于从事一些和病人打交道的工作,一般健康福利会比较不错。
工作环境:其实我去过老人院和老人公寓,环境真得很不错。由于工作中动体力的时间较多,所以一般都不会让你连续工作时间太长的。
语言要求:这个工作一般对语言要求较高,不过有一点,会一门外语绝对是一个财富。
发展空间:这个专业实际上是Nurse Aid,发展当然是朝Rigester Nurse了。热门阿。
性别:女性占91%。 但是:听说急需男性阿,虽说有器械帮助,但是还是有些体力活的,所以经常在老人院里和医院里见到五大三粗的女性。还有,男性也有隐私阿,男同志有的时候也想让同性来照顾一下。顺便说一句,男护士也是非常非常受欢迎。
课程介绍,上这个课的教室很有意思,大家有兴趣可以参观。这个课对于实际操作要求很强,经常要演练急救,比如,突然有人晕倒了,怎么处理,学生们七手八脚的上。
这个课必须要西人教师上,因为西人和我们在处理人的问题上首先观念大大不同。西人把安全放在第一位,首先必须安全。所以什么民间土法,没有经过验证的不用。比如脖子不舒服,趁他不注意给他嘎吱一下。所以同样操作,观念不同,处理的方法不同。
课程介绍太长了,需要的留下邮件。包括,心理学的知识,人际沟通,营养学,老人护理,临终关怀,危机处理,家庭护理,收拾房屋,以及相关的法律法规,当然,实习是非常重要的一环
PT: Pharmacy Technician
就业前景:Very Good, 还是和老年社会有关。
薪资水平:如果在零售,10-15块,如果在社区community health care. 10-20块,医院:19-26块起,有些人成为了药剂师,工资约7万/年起,但是要再继续学习获得degree.
性别:男女均衡
语言:要求较高,学习辛苦,因为有大量东西要记,中国人毕业率高,老外经常吃不消。
自雇:自己做生意的占15%。 年收入较高。
学位:diploma, 同时还有行业的几个certificate
学习时间:40周,要学得抓紧,政府马上要把它改成两年的课程了
课程介绍:太长了,需要的给mail.

Community Service Worker 介绍,请点,就不用费时找了

我想了解PT的课程介绍,请发到jjzly@126.com,谢谢!

PaulineLin : 2010-02-19#458
回复: 求学选择个人谈及幼教,私人护理,社工,医师行政助理等热门行业介绍

:wdb10:

Smily : 2010-02-24#459
回复: 求学选择个人谈及幼教,私人护理,社工,医师行政助理等热门行业介绍

PT: Pharmacy Technician
就业前景:Very Good, 还是和老年社会有关。
薪资水平:如果在零售,10-15块,如果在社区community health care. 10-20块,医院:19-26块起,有些人成为了药剂师,工资约7万/年起,但是要再继续学习获得degree.
性别:男女均衡
语言:要求较高,学习辛苦,因为有大量东西要记,中国人毕业率高,老外经常吃不消。
自雇:自己做生意的占15%。 年收入较高。
学位:diploma, 同时还有行业的几个certificate
学习时间:40周,要学得抓紧,政府马上要把它改成两年的课程了
课程介绍:太长了,需要的给mail.

Community Service Worker 介绍,请点,就不用费时找了[/quote]


我因为药师在这边评不过,没有本科证,但我想WATERLOO的药学专业,但是要六年,你说,像我在国内有8年正规药学学习,十年药房工作,有没有办法直接就读四年,而不是先大学两年再PHARMACY 4年呢??帮帮我呀。。。我都不知怎么办了。。6年太长了,我都三十了。

Smily : 2010-02-24#460
回复: toronto college介绍,选择,相关问题交流

如果我在国内是药学本科毕业,在这边可不可以靠托福上研究生呢?

本地diploma申请肯定不行,如果国内有承认的人文方面本科学历,可以通过靠托福,申请在这里读social worker的研究生,本地的diploma和工作经验作为你申请研究生的资历。

lucky-xin : 2010-02-25#461
回复: 求学选择个人谈及幼教,私人护理,社工,医师行政助理等热门行业介绍

LZ您好,谢谢您提供这非常有用的信息,我想读ECA的课程,但不知在温哥华哪个学校有这个6个月的课程,我在VCC 里只看到有两年有关于幼儿教育方面的课程,请问在温哥华有哪些college有六个月的ECA课程,再次感谢!

anna75 : 2010-02-25#462
回复: 求学选择个人谈及幼教,私人护理,社工,医师行政助理等热门行业介绍

有用的关于COLLEGE的文章,学习中。

palace : 2010-02-28#463
回复: 求学选择个人谈及幼教,私人护理,社工,医师行政助理等热门行业介绍

感谢LZ的无私奉献,我想这对很多同我一般(正一筹莫展准备登陆的)新移民而言,无疑是一盏指路明灯。再次感谢楼主。

打算登陆温哥华,请问那边有比较好的有关paralegal或者interior design专业的college么? 另外,自觉英语水平还不够,是先full-time在成人高中或ESL学英语呢,还是直接报名上college?

mimisoso : 2010-03-02#464
回复: 求学选择个人谈及幼教,私人护理,社工,医师行政助理等热门行业介绍

楼主,你好:
我现在有点担心去读了certificate还是找不到工作会么?
我介绍下我的情况:我申请的是乔治布朗学院的食品营养管理(学些营养学啊,老年学啊,社会学,临床等等吧)这样我在大学本科里不是很对应 的学科,学的东西是不是会多点,虽然可能是理论的,但是对于我来说,对以后的工作还是有基础作用呢?还有我这个专业,以后会从事什么工作呢,就业几率怎么样?我如果想多考几个相关的certificate,您觉得学哪些对以后工作有帮助呢?


先谢过了~~呵呵:wdb17::wdb17:

佳加life : 2010-03-04#465
回复: 求学选择个人谈及幼教,私人护理,社工,医师行政助理等热门行业介绍

我的移民手续还在进行中,我办的是家庭团聚类移民,估计8月份会过到渥太华:wdb9:。我也是这几天苦于自己对那边的不了解:wdb2:,意外看了你发的帖,给我很大启示,我对那个PSW和PT专业挺感兴趣的,但我在国内是教少儿英语的。希望前辈给点建议:wdb6:

mxy101 : 2010-03-05#466
回复: 求学选择个人谈及幼教,私人护理,社工,医师行政助理等热门行业介绍

lz能否比较一下labour工和college的特点?
想请教一下:为何许多移民选择打labour工,而不是上个1-2年的college拿个diploma后,找个相对轻松的工作?:wdb2:

luhaolin : 2010-03-13#467
回复: 求学选择个人谈及幼教,私人护理,社工,医师行政助理等热门行业介绍

楼主 我想问下 学建筑技术员 这个专业有前途没。。感觉百年理工这专业怎么样 这个专业对英语要求是不是不用太好 。。。谢谢了

luhaolin : 2010-03-13#468
回复: 求学选择个人谈及幼教,私人护理,社工,医师行政助理等热门行业介绍

楼主能介绍几个 读建筑比较可以的college吗 多伦多的 乔治和Seneca那个好点呢

zhou8p : 2010-03-13#469
回复: 求学选择个人谈及幼教,私人护理,社工,医师行政助理等热门行业介绍

再次感谢楼主提供的非常实用的信息! 能否再多提供些温哥华college的信息? 谢谢。

jasmine mimi : 2010-03-14#470
回复: 求学选择个人谈及幼教,私人护理,社工,医师行政助理等热门行业介绍

谢谢楼主的信息!在温哥华,幼教(ECE/ECA),护工(RCA),医疗行政助理都很难找到工作了,前几年是热门,正因为这样,很多人都去读了,现在变了供大于求!本人也在发愁不知道选择什么专业呢。要强调的是,如果读幼教,自雇还是不错的,护工如果是上门服务的需求也很大,但要进医院或老人院就比较难!

cris : 2010-03-15#471
回复: 求学选择个人谈及幼教,私人护理,社工,医师行政助理等热门行业介绍

也请给我一份08liao@163.com
谢谢

qjy : 2010-03-19#472
回复: 求学选择个人谈及幼教,私人护理,社工,医师行政助理等热门行业介绍

good

qjy : 2010-03-19#473
回复: 求学选择个人谈及幼教,私人护理,社工,医师行政助理等热门行业介绍

楼主真是好人呀!请问楼主我想去学社工,在英语方面有什么要求/?另外,我个子不高,是典型的南方人,社工工作应该不要求身强力壮吧?我们想去卡尔加里,有什么学校可以修社工这个专业?另外我老公是学医学检验的,在那边工作好找不?在卡城有什么学校可以在最短的时间内修一下医学检验,谢谢了,给你加声望了

shouwang : 2010-03-22#474
回复: 求学选择个人谈及幼教,私人护理,社工,医师行政助理等热门行业介绍

问一下楼主了解environment protection这个专业吗?
我本科毕业想去读centennial college的环保专业 留学移民
因为本科也是学这个的 所以感觉学习难度不会太大 雅思6.5分 还是去了熟悉语言和环境
lz知道环保这个专业好就业吗?还有一个比较迷茫的是不知道是读一年的fast-track还是2年的post-secondary program
麻烦lz指点一下

rainydaysunday : 2010-03-24#475
回复: 求学选择个人谈及幼教,私人护理,社工,医师行政助理等热门行业介绍

麻烦楼主及各位tx帮我瞧瞧我的个人情况及未来规划是否可行,以便我及时修正!!!!!!!!!!!!谢谢!

本人是一名中学英语教师,79年出生,老公是公务员,78年的,有一女,2005年出生,07年12月以我为主审递的香港,08年8月收到fn,算是赶上91新政的末班车(当时对移民事务也是一无所知,委托的是北京的中介,他们给递的香港,压根就不懂原来还可以DIY,而且香港的速度是如此之慢!!!!!)
本人虽然毕业有10年左右了(2001至今),而且只是教中学,但一直还是有在学英语(只是没有打算移民前目标不明确,没有现在认真)雅思08年4月已考,根据新规定能拿到16分的满分,另外09年开始自学法语,(完全有信心在我登陆时把它学好),日语是我大学时的第二外语,平时通过看日剧及一段段的心血来潮突击看书,大概二级左右吧(说和写弱一些),粤语会一点(这也是我接下来努力的目标之一),西班牙语初级(感觉和英语及法语还是有很多相似之处,平时有空就看点当调剂)。因为我女儿现在是在幼儿园念中班,寄午,所以比较有空,而且传说香港平均要等7.69年,我想着技多不压身,于是就乱七八糟的学一堆。(经常被我老公取笑说老女人了,却比我学生还勤奋)
对于在加的职业选择方面,我一直还是很明确,基本是定为幼儿教育方面,ECA 或ECE(本来更想选择小学,但根据论坛上各位tx的说法,小学老师得教数学等,而我的理科很烂,于是就放弃了),为此,平常除了语言的准备之外,我还想自学点手工,简笔画,面点制作之类的。
每天的学习很充实,但经常也有质疑自己的时候,老是问自己有必要这么折腾吗,毕竟年龄在一天一天的变大,香港的等待期又那么长,而且能不能出的去还另说!!!!!!!!老想着自己是不是太天真了????

rainydaysunday : 2010-03-24#476
回复: 求学选择个人谈及幼教,私人护理,社工,医师行政助理等热门行业介绍

另外,我还想了解一下有没有那种可以全程在中国国内远程学习,并能拿到能被市场认可的证书????因为我是递香港的91新政尾巴,等待期无限长,想在此期间除了学语言,同时也做点专业上的准备。

妙恋 : 2010-03-25#477
回复: 求学选择个人谈及幼教,私人护理,社工,医师行政助理等热门行业介绍

吃了晚饭再过来学习。谢谢楼主!

hope梦想 : 2010-03-27#478
回复: 求学选择个人谈及幼教,私人护理,社工,医师行政助理等热门行业介绍

关注

hope梦想 : 2010-03-27#479
回复: 求学选择个人谈及幼教,私人护理,社工,医师行政助理等热门行业介绍

楼主俺是学护士出身的,像你说的私人护理倒是应该可以尝试去做一下,也是为了小孩也有机会能够出去,俺现在恶补英语,以前毕业那时候差点到澳洲去了,后来实习就没去,现在想着怎样能够出去,前年又学了段美容,呵呵,总想人生应该走出国门去看看什么的!想通过中介出去,但看到很多人都说找不好的就麻烦呢!

hope梦想 : 2010-03-27#480
回复: 求学选择个人谈及幼教,私人护理,社工,医师行政助理等热门行业介绍

望楼主能够给俺指点一些信息,谢谢

hope梦想 : 2010-03-28#481
回复: 求学选择个人谈及幼教,私人护理,社工,医师行政助理等热门行业介绍

楼主咋不来了呢

老海 : 2010-04-16#482
回复: 求学选择个人谈及幼教,私人护理,社工,医师行政助理等热门行业介绍

向楼主表示感谢。给我发私人护理和药剂技师的材料好吗?我信箱:jldxly@yahoo.com

hlnxi : 2010-04-28#483
回复: 求学选择个人谈及幼教,私人护理,社工,医师行政助理等热门行业介绍

一口气看到这里.
想请教LZ,我在国内电信公司工作,本科自动控制,研究生计算机信息控制,来加一直在家带孩子,现在孩子大了,想去找份轻松工作,做回本职好象不太可能.
进入电信公司做customer service还是软件测试,有没有好的短期培训?

ziyunxuan : 2010-04-29#484
回复: 求学选择个人谈及幼教,私人护理,社工,医师行政助理等热门行业介绍

请把有关的信息给我发一份谢谢!amwaydani@yahoo.ca

向往未来 : 2010-04-29#485
回复: 求学选择个人谈及幼教,私人护理,社工,医师行政助理等热门行业介绍

很好的帖子。真的谢谢您。为各位受益匪浅的读者谢谢你。你的电话是4163650505-226吧,希望能联系到你。

向往未来 : 2010-04-29#486
回复: 求学选择个人谈及幼教,私人护理,社工,医师行政助理等热门行业介绍

我LP已经登陆了一段时间,她想学护士或洗牙护理。希望能得到你的帮助。我登陆想去赌场做派牌员,不知道你有没有什么好建议,或者有哪里好的培训机构?希望您能看到。我的E-MAIL:xiaolinbing@sina.com

alexwei : 2010-04-29#487
回复: 求学选择个人谈及幼教,私人护理,社工,医师行政助理等热门行业介绍

软件测试应该比较适合有IT基础的女性,薪资也还可以,估计在4万到7万之间Customer Service工作其实挺累。你可以翻翻都市报,几个华人学校都有这方面的短期培训,费用应该也不高。不过课上老师只是带你进门,自己要学的东西还是很多。不要指望学个把月就能找到工作,一般来所学习的时间应该在3-6个月,改简历找工作再花个3-6个月,还是要有一年的打算。
培训方面,不要关注价格,主要还是要和老师多聊,因为老师是最能帮到你的人,有的学校老师就是教书,下课就跑。有的老师帮你改简历,回答你很多问题,甚至介绍工作机会,所以选择一个有经验,能够真心帮助你的老师是最重要的。
一口气看到这里.
想请教LZ,我在国内电信公司工作,本科自动控制,研究生计算机信息控制,来加一直在家带孩子,现在孩子大了,想去找份轻松工作,做回本职好象不太可能.
进入电信公司做customer service还是软件测试,有没有好的短期培训?

alexwei : 2010-04-29#488
回复: 求学选择个人谈及幼教,私人护理,社工,医师行政助理等热门行业介绍

很好的帖子。真的谢谢您。为各位受益匪浅的读者谢谢你。你的电话是4163650505-226吧,希望能联系到你。

不是我现在的电话,谢谢

alexwei : 2010-04-29#489
回复: 求学选择个人谈及幼教,私人护理,社工,医师行政助理等热门行业介绍

护士挺好,只有公立College和大学有这个专业,可以选择先上College 2年的护士专业(应该叫RPN吧),以后再深造大学RN。或者直接上大学。不过护士专业很难进。洗牙师不错,不过投资挺大,最好找一家大的机构学习。
quote=向往未来;5406985]我LP已经登陆了一段时间,她想学护士或洗牙护理。希望能得到你的帮助。我登陆想去赌场做派牌员,不知道你有没有什么好建议,或者有哪里好的培训机构?希望您能看到。我的E-MAIL:xiaolinbing@sina.com[/quote]

alexwei : 2010-04-29#490
回复: 求学选择个人谈及幼教,私人护理,社工,医师行政助理等热门行业介绍

ECA现在已经很难找工作了,建议直接读ECE.
LZ您好,谢谢您提供这非常有用的信息,我想读ECA的课程,但不知在温哥华哪个学校有这个6个月的课程,我在VCC 里只看到有两年有关于幼儿教育方面的课程,请问在温哥华有哪些college有六个月的ECA课程,再次感谢!

alexwei : 2010-04-29#491
回复: 求学选择个人谈及幼教,私人护理,社工,医师行政助理等热门行业介绍

以我的观点,Labour没有前途,其实移民来到这里,绝大部分人都还是接受了再培训,包括上College, 短期培训,考取证书等。培训是躲不过去的,打labour 是为了解决生活问题,但是打labour能够有发展空间吗,在北美制造业衰退的背景下,你的生活如何保障? 惟有不断学习,才有不断上升的空间,才有未来美好的生活。当然,富有的投资移民除外。
另外说一句,我觉得在北美,其实很少有悠闲的工作,只不过有些人劳心,有些人劳力。

lz能否比较一下labour工和college的特点?
想请教一下:为何许多移民选择打labour工,而不是上个1-2年的college拿个diploma后,找个相对轻松的工作?:wdb2:

alexwei : 2010-04-29#492
回复: 求学选择个人谈及幼教,私人护理,社工,医师行政助理等热门行业介绍

读两年容易移民
问一下楼主了解environment protection这个专业吗?
我本科毕业想去读centennial college的环保专业 留学移民
因为本科也是学这个的 所以感觉学习难度不会太大 雅思6.5分 还是去了熟悉语言和环境
lz知道环保这个专业好就业吗?还有一个比较迷茫的是不知道是读一年的fast-track还是2年的post-secondary program
麻烦lz指点一下

alexwei : 2010-04-29#493
回复: 求学选择个人谈及幼教,私人护理,社工,医师行政助理等热门行业介绍

小城市比较适合经营便利店
楼主,你好!我今年43岁,来加拿大6个月了,以前在国内学的是机械专业,做过机械采购和管理,也做过档案管理,现在加拿大一小城市,因老公以前工作和生活在许多大城市,现在又重新回到这个小城市,找到一个稳定的工作,所以现在我们再移到其它城市的可能性几乎没有,本想来加拿大后做一些本专业的工作,可这个小城市也没有什么工业和商贸,比较适合养老,所以老年人比较多,所以我只有立足我们这个城市,重新去学一个专业,寻找一些其他的工作机会,有朋友推荐去学PSW,也有推荐学财会,幼师,等等,不过我比较倾向去学PSW,不知你有什么好的建议?另外,你们学校有些什么专业?和这些专业都什么时候开学?学习多长时间及费用?一下问这么多问题,麻烦你了!
谢谢你!
祝圣诞快乐及新年快乐!

dandelion_foo : 2010-04-29#494
回复: 求学选择个人谈及幼教,私人护理,社工,医师行政助理等热门行业介绍

支持。
我刚登陆一个月,在国内就是软件测试,近两年写自动测试脚本。现在对当地文化背景各方面都信心不足,没信心,不敢找工作。刚申请了几个COLLEGE的1年的PROGRAM,想弥补一下没有开发经验的缺憾,好继续做自动测试。


软件测试应该比较适合有IT基础的女性,薪资也还可以,估计在4万到7万之间Customer Service工作其实挺累。你可以翻翻都市报,几个华人学校都有这方面的短期培训,费用应该也不高。不过课上老师只是带你进门,自己要学的东西还是很多。不要指望学个把月就能找到工作,一般来所学习的时间应该在3-6个月,改简历找工作再花个3-6个月,还是要有一年的打算。
培训方面,不要关注价格,主要还是要和老师多聊,因为老师是最能帮到你的人,有的学校老师就是教书,下课就跑。有的老师帮你改简历,回答你很多问题,甚至介绍工作机会,所以选择一个有经验,能够真心帮助你的老师是最重要的。

hlnxi : 2010-04-30#495
回复: 求学选择个人谈及幼教,私人护理,社工,医师行政助理等热门行业介绍

软件测试应该比较适合有IT基础的女性,薪资也还可以,估计在4万到7万之间Customer Service工作其实挺累。你可以翻翻都市报,几个华人学校都有这方面的短期培训,费用应该也不高。不过课上老师只是带你进门,自己要学的东西还是很多。不要指望学个把月就能找到工作,一般来所学习的时间应该在3-6个月,改简历找工作再花个3-6个月,还是要有一年的打算。
培训方面,不要关注价格,主要还是要和老师多聊,因为老师是最能帮到你的人,有的学校老师就是教书,下课就跑。有的老师帮你改简历,回答你很多问题,甚至介绍工作机会,所以选择一个有经验,能够真心帮助你的老师是最重要的。
不错,我去找找.感谢:wdb6:

向往未来 : 2010-05-01#496
回复: 求学选择个人谈及幼教,私人护理,社工,医师行政助理等热门行业介绍

不是我现在的电话,谢谢
可以提供你现在的电话吗?我的e-mail:xiaolinbing@sina.com.

siemen95 : 2010-05-02#497
回复: toronto college介绍,选择,相关问题交流

介绍一下社会工作者行业
国内没有社工这个行业,不过大家应该清楚这个是干什么的,我们遇到的settlement Counselor, Employment Counselor都是属于这个行业。这个行业对于我们移民来说有一个优势,我们会1门外语,而且是目前最大的移民群体的外语,各个机构都在扩展对于中国移民的服务。这样子就需要我们了。社工为什么要拓展服务呢,大家知道,no profit机构的钱来自于政府,企业或个人捐助,自我经营收入,其中政府最多,而政府的钱是要根据你做了多少事情来得,比如政府今年有7个亿是帮助新移民的,会分配到各个非营利机构,这些机构就要做一些针对新移民的服务来申请经费,同时还要有效果,如果没有政府的funding, 这个机构就要关门了。所以从事这个行业的人往往有marketing、工商管理,教师的背景,然后获得了本地学历。那么这个行业有多大呢,这个行业非常大,和商业基本上快差不多了。那些机构属于no-profit, 医院,学校,就业、安居服务机构,慈善机构,老人服务机构,绿色和平组织,环境和动物保护组织等。他的行业自成体系,你在一般的招聘网站或者agency那里很少看到招聘的信息。需要什么样的人呢,其实和商业一样,需要sales, accounting, administration, customer service, technician.不一样的地方,核心的服务人员是社工,教师,或者医务人员。大家可以去www.charityvillige.com 上去看,这是加拿大最大的非营利机构的招聘网站。
就业前景:不错
薪资水平:16-20左右。
性别:女性居多
个人奋斗方向:如果成为专业social worker, 工资可以达到70000,但是一般需要有social science master degree. 而 bachelor 毕业实际上和college毕业薪资差别不是太大,我个人觉得可以在拿到diploma后工作两年,直接申请master(用国内的学历)+diploma+工作经验,这样不错。
课程介绍
• Introduction to Community Service Worker
• Social Welfare, Diversity and Awareness
• The Family
• Couples, Family and Interpersonal Communication
• Oriented Case Management
• Environmental Issues and Crisis Interventions and
Prevention
• Bereavement and the Elderly
• Respond to Abuse
• Understanding Human Behavior
• Communication Skills
• Counseling Skills
• Report Writing for the CSW
• Field placement

By the end of the course, students will be able to:
• Demonstrate knowledge of the range of social and community service organizations in their own community as well as detailed knowledge of an agency.
• Practice the role of a team member within a social and community service organization.
• Demonstrate beginning level professional skills
• Apply theory and skills learned in the independent study components of this course.

做个记号,慢慢研究

keer_coois : 2010-05-02#498
回复: 求学选择个人谈及幼教,私人护理,社工,医师行政助理等热门行业介绍

非常荣幸能够在这里遇到楼主,您真的是给我们这些即将移民和已经登陆的移民提供了很多实用的信息。我想咨询一下楼主,我本科是电子信息工程专业,研究生是通信专业,现在在大学当老师,准备办理移民。到加后还是想继续从事相关的专业,不太像把时间浪费在读博士上面(都反映即使读完博士,也不一定能在大学任教,工作反倒不好找),所以还是继续从事技术方面的工作,麻烦楼主给提供一些建议,到那边是读college,u,还是master?

alexwei : 2010-05-02#499
回复: 求学选择个人谈及幼教,私人护理,社工,医师行政助理等热门行业介绍

Although Many people say it is very easy to get jobs with college diploma, however, from my point of view, Canadian workplace need more workers with higher education background, so if well prepared in China, I strongly suggest you apply for a master course after you land Canada as soon as possible. Master will have more opportunities to get high salary jobs.
非常荣幸能够在这里遇到楼主,您真的是给我们这些即将移民和已经登陆的移民提供了很多实用的信息。我想咨询一下楼主,我本科是电子信息工程专业,研究生是通信专业,现在在大学当老师,准备办理移民。到加后还是想继续从事相关的专业,不太像把时间浪费在读博士上面(都反映即使读完博士,也不一定能在大学任教,工作反倒不好找),所以还是继续从事技术方面的工作,麻烦楼主给提供一些建议,到那边是读college,u,还是master?

alexwei : 2010-05-02#500
回复: 求学选择个人谈及幼教,私人护理,社工,医师行政助理等热门行业介绍

May I know what do you want to know? we can share opinions here.

可以提供你现在的电话吗?我的e-mail:xiaolinbing@sina.com.

alexwei : 2010-05-02#501
回复: 求学选择个人谈及幼教,私人护理,社工,医师行政助理等热门行业介绍

Be patient, keep learning, you will find your job. It is not so hard for IT guys.
支持。
我刚登陆一个月,在国内就是软件测试,近两年写自动测试脚本。现在对当地文化背景各方面都信心不足,没信心,不敢找工作。刚申请了几个COLLEGE的1年的PROGRAM,想弥补一下没有开发经验的缺憾,好继续做自动测试。

dandelion_foo : 2010-05-03#502
回复: 求学选择个人谈及幼教,私人护理,社工,医师行政助理等热门行业介绍

thank you so very much.:wdb17:
Be patient, keep learning, you will find your job. It is not so hard for IT guys.

keer_coois : 2010-05-04#503
回复: 求学选择个人谈及幼教,私人护理,社工,医师行政助理等热门行业介绍

Although Many people say it is very easy to get jobs with college diploma, however, from my point of view, Canadian workplace need more workers with higher education background, so if well prepared in China, I strongly suggest you apply for a master course after you land Canada as soon as possible. Master will have more opportunities to get high salary jobs.


thank u!!!:wdb23::wdb23::wdb23::wdb23:

登高远望669 : 2010-05-04#504
回复: 求学选择个人谈及幼教,私人护理,社工,医师行政助理等热门行业介绍

真高兴遇到LZ这么明白的人。想咨询一下,我女儿今年秋天要去加留学上大学,想毕业后移民,已经收到阿尔伯塔的录取了,但是对选什么专业好找工作、有利于移民非常不了解,她自己也没有非常喜欢的专业。原先想学会计,一了解很多留学生都是学会计的,她有些想学师范,将来毕业当老师,也不知道加拿大学师范的好找工作吗?什么专业好找工作、利于移民呢?希望得到您的帮助,谢谢了!

alexwei : 2010-05-05#505
回复: 求学选择个人谈及幼教,私人护理,社工,医师行政助理等热门行业介绍

Both are good. Accounting is easy for Chinese. Education is one of the best career in Canada, however, Chinese are much less competitive in this field because of language and culture.
真高兴遇到LZ这么明白的人。想咨询一下,我女儿今年秋天要去加留学上大学,想毕业后移民,已经收到阿尔伯塔的录取了,但是对选什么专业好找工作、有利于移民非常不了解,她自己也没有非常喜欢的专业。原先想学会计,一了解很多留学生都是学会计的,她有些想学师范,将来毕业当老师,也不知道加拿大学师范的好找工作吗?什么专业好找工作、利于移民呢?希望得到您的帮助,谢谢了!

登高远望669 : 2010-05-06#506
回复: 求学选择个人谈及幼教,私人护理,社工,医师行政助理等热门行业介绍

Both are good. Accounting is easy for Chinese. Education is one of the best career in Canada, however, Chinese are much less competitive in this field because of language and culture.
谢谢LZ这么快就答复了!

lake 501E : 2010-05-10#507
回复: 求学选择个人谈及幼教,私人护理,社工,医师行政助理等热门行业介绍

楼主,你好,看到您这么热心的,不厌其烦的回答问题,真让人感动!我也有问题想问问您。我在国内学的是生物专业,在职业学校里做老师18年了,同时也做些管理工作。另外前几年在美国取得了公共管理的硕士学位。我于半个月前登陆温哥华,前两天到这边的一个college咨询了一下牙医助理的培训,并通过了英文测试。她们告诉我说以我这种情况可以申请贷款。学费大概是一万三左右,48周的培训项目。今天又去了一个叫Skill connect的部门咨询了一下,这是一个为新移民办的职业培训项目,自己负担学费的3分之一,但是也要通过一个英文考试,并且还要与自己职业相关的才能允许参与进来。我看了一下,我有资格参加的培训只有教育方面的和管理方面的,比如教育助理,幼儿教师等。她们说管理方面的很局限,不建议读。但是我看到有个帖子说现在温哥华像幼儿教师都已经找不到工作了,就怕是学完了没有用了。现在正在犹豫当中,不知道自己该如何选择,楼主有没有什么好的建议,麻烦你在百忙之中帮我分析一下,万分感谢!(我的英文水平雅思6.5.,年龄39,女性)。

zhou8p : 2010-05-11#508
回复: 求学选择个人谈及幼教,私人护理,社工,医师行政助理等热门行业介绍

真高兴LZ终于回来了!我的问题是:国内高中毕业(没考大学),雅思5.5,估计年底前登陆(移民身份)。这种条件能直接报考COLLEGE的验光师、洗牙师之类的专业吗?盼复!谢谢。

alexwei : 2010-05-11#509
回复: 求学选择个人谈及幼教,私人护理,社工,医师行政助理等热门行业介绍

5.5 is not enough. 6(a) maybe OK
真高兴LZ终于回来了!我的问题是:国内高中毕业(没考大学),雅思5.5,估计年底前登陆(移民身份)。这种条件能直接报考COLLEGE的验光师、洗牙师之类的专业吗?盼复!谢谢。

alexwei : 2010-05-11#510
回复: 求学选择个人谈及幼教,私人护理,社工,医师行政助理等热门行业介绍

You English is not bad. If you want know what kind of jobs have good opportunities in the future, you can go to www.jobfutures.ca
楼主,你好,看到您这么热心的,不厌其烦的回答问题,真让人感动!我也有问题想问问您。我在国内学的是生物专业,在职业学校里做老师18年了,同时也做些管理工作。另外前几年在美国取得了公共管理的硕士学位。我于半个月前登陆温哥华,前两天到这边的一个college咨询了一下牙医助理的培训,并通过了英文测试。她们告诉我说以我这种情况可以申请贷款。学费大概是一万三左右,48周的培训项目。今天又去了一个叫Skill connect的部门咨询了一下,这是一个为新移民办的职业培训项目,自己负担学费的3分之一,但是也要通过一个英文考试,并且还要与自己职业相关的才能允许参与进来。我看了一下,我有资格参加的培训只有教育方面的和管理方面的,比如教育助理,幼儿教师等。她们说管理方面的很局限,不建议读。但是我看到有个帖子说现在温哥华像幼儿教师都已经找不到工作了,就怕是学完了没有用了。现在正在犹豫当中,不知道自己该如何选择,楼主有没有什么好的建议,麻烦你在百忙之中帮我分析一下,万分感谢!(我的英文水平雅思6.5.,年龄39,女性)。

zhou8p : 2010-05-17#511
回复: 求学选择个人谈及幼教,私人护理,社工,医师行政助理等热门行业介绍

请问LZ,在加拿大学个六个月的Certificate,学费大约是多少?谢谢

baobaoning : 2010-05-19#512
回复: 求学选择个人谈及幼教,私人护理,社工,医师行政助理等热门行业介绍

:wdb6:好贴,值得学习

TeddyAviva : 2010-05-21#513
回复: 求学选择个人谈及幼教,私人护理,社工,医师行政助理等热门行业介绍

很使用,非常感谢!

stekey : 2010-05-22#514
回复: 求学选择个人谈及幼教,私人护理,社工,医师行政助理等热门行业介绍

好贴!强烈支持~
我是今年的刚毕业的,学的是经济类的税务,GPA较高。还在国内,现在想通过留学移民加国,在温哥华有亲戚。现在有两条路选择,一是转学分去魁省的BISHOP读second drgree,令一个是先申请的Thompson river的MBA,再转学去温哥华BCIT。移民是主要目的,但也希望通过学习提升自己。中介推荐我去读drgree,说不要浪费了GPA;但亲戚说来加主要是找工作,叫我不要在意读什么,在北美经验最重要,来了再说。我对管理金融类不感兴趣,所以希望转专业。我对多媒体制作,即视频音频软件的应用比较感兴趣,但都是业余爱好,也不知道这些读出来能干什么。
想请教一下,我由文科转去读计算机或者环境工程等课程可不可取,我对那些方面是零基础。另外,我是去魁省读学位好还是去温哥华读个certificate或者diploma好呢?
Thanks
P.S:现在努力准备雅思中

richardbingo : 2010-05-27#515
回复: 求学选择个人谈及幼教,私人护理,社工,医师行政助理等热门行业介绍

好好学习。

大恶花 : 2010-05-27#516
回复: 求学选择个人谈及幼教,私人护理,社工,医师行政助理等热门行业介绍

不知道那个bishop univ的second degree怎样啊。。。中介也推荐我读那个。我大学刚毕业,这算是拿个双学位。我觉得能转两年学分听起来不错,但是国外大学都是宽进严出,能不能毕业又是另外一回事了,对吧。还有,中国人读会计真的很容易吗~~会计这个学科其实也蛮精深的哈。不像小公司做账那么容易上手。

另外一个中介推荐我读一个很新的college专业,叫the children's entertainment 课程介绍是这么写的
[FONT=宋体]the program offers:[/FONT]

[FONT=宋体]Hands-on practical learning [/FONT]
[FONT=宋体]Career-ready skills [/FONT]
[FONT=宋体]High-level industry professionals teaching [/FONT]
[FONT=宋体]and helping you connect with the industry [/FONT]
[FONT=宋体]On-site professional software [/FONT]
[FONT=宋体]Extensive career and global job opportunities [/FONT]

[FONT=宋体]You will develop new skills in:[/FONT]
[FONT=宋体]Writing for Children [/FONT]
[FONT=宋体]Children’s Entertainment Marketplace [/FONT]
[FONT=宋体]Marketing of Children's learning and entertainment products [/FONT]
[FONT=宋体]Project Management [/FONT]
[FONT=宋体]Pitching [/FONT]
[FONT=宋体]TV and multi-platform production [/FONT]
[FONT=宋体]Team building for creative production [/FONT]
[FONT=宋体]Child Development [/FONT]

[FONT=宋体]这是偏商科吗?有用武之地么请楼主坚定一下啊~~大谢!![/FONT]

alexwei : 2010-05-27#517
回复: 求学选择个人谈及幼教,私人护理,社工,医师行政助理等热门行业介绍

请问LZ,在加拿大学个六个月的Certificate,学费大约是多少?谢谢
It depends. very hard to give you a number.

alexwei : 2010-05-27#518
回复: 求学选择个人谈及幼教,私人护理,社工,医师行政助理等热门行业介绍

Ask agent if he/she has ever been to Canada. Your relative is right. Choosing a way helping your immigrants. I think business Admin, Accounting are easy.
不知道那个bishop univ的second degree怎样啊。。。中介也推荐我读那个。我大学刚毕业,这算是拿个双学位。我觉得能转两年学分听起来不错,但是国外大学都是宽进严出,能不能毕业又是另外一回事了,对吧。还有,中国人读会计真的很容易吗~~会计这个学科其实也蛮精深的哈。不像小公司做账那么容易上手。

另外一个中介推荐我读一个很新的college专业,叫the children's entertainment 课程介绍是这么写的
[FONT=宋体]the program offers:[/FONT]

[FONT=宋体]Hands-on practical learning [/FONT]
[FONT=宋体]Career-ready skills [/FONT]
[FONT=宋体]High-level industry professionals teaching [/FONT]
[FONT=宋体]and helping you connect with the industry [/FONT]
[FONT=宋体]On-site professional software [/FONT]
[FONT=宋体]Extensive career and global job opportunities [/FONT]

[FONT=宋体]You will develop new skills in:[/FONT]
[FONT=宋体]Writing for Children [/FONT]
[FONT=宋体]Children’s Entertainment Marketplace [/FONT]
[FONT=宋体]Marketing of Children's learning and entertainment products [/FONT]
[FONT=宋体]Project Management [/FONT]
[FONT=宋体]Pitching [/FONT]
[FONT=宋体]TV and multi-platform production [/FONT]
[FONT=宋体]Team building for creative production [/FONT]
[FONT=宋体]Child Development [/FONT]

[FONT=宋体]这是偏商科吗?有用武之地么请楼主坚定一下啊~~大谢!![/FONT]

大傻子 : 2010-05-28#519
回复: 求学选择个人谈及幼教,私人护理,社工,医师行政助理等热门行业介绍

楼主,你好!仔细拜读过你的所有帖子,对你的为人、知识钦佩。有一事相求:我一朋友,原大学教英语,因此语言凑合,已来加半年,无合适工作,苦恼!打算学习社工,但又无奈年纪太大(55岁)。据我所知加国学习很苦,年纪大的人难以吃得消,且不易毕业。朋友现在徘徊之中,请您能给予指导,非常感谢!

xyf_sg : 2010-05-28#520
回复: 求学选择个人谈及幼教,私人护理,社工,医师行政助理等热门行业介绍

献花了!作个记号,常来看看。

喃喃<一只会飞的鱼> : 2010-05-28#521
回复: 求学选择个人谈及幼教,私人护理,社工,医师行政助理等热门行业介绍

楼主你好!先介绍一下我的情况,来加已经四个月了,ESL水平三级,想报考college 学习dental assistant 这个专业。以我现在的英语情况上college 是否吃力,就业前景怎么样,请楼主给我一些建议!感谢!

alexwei : 2010-05-28#522
回复: 求学选择个人谈及幼教,私人护理,社工,医师行政助理等热门行业介绍

Take colleges' English Test first. I think they may need other exam.
楼主你好!先介绍一下我的情况,来加已经四个月了,ESL水平三级,想报考college 学习dental assistant 这个专业。以我现在的英语情况上college 是否吃力,就业前景怎么样,请楼主给我一些建议!感谢!

alexwei : 2010-05-28#523
回复: 求学选择个人谈及幼教,私人护理,社工,医师行政助理等热门行业介绍

Community Service worker maybe fit for him/her.
楼主,你好!仔细拜读过你的所有帖子,对你的为人、知识钦佩。有一事相求:我一朋友,原大学教英语,因此语言凑合,已来加半年,无合适工作,苦恼!打算学习社工,但又无奈年纪太大(55岁)。据我所知加国学习很苦,年纪大的人难以吃得消,且不易毕业。朋友现在徘徊之中,请您能给予指导,非常感谢!

xiaoxiaoai : 2010-05-31#524
回复: 求学选择个人谈及幼教,私人护理,社工,医师行政助理等热门行业介绍

先收藏了~~~强烈感谢LZ!

xuchaofan : 2010-06-02#525
回复: 求学选择个人谈及幼教,私人护理,社工,医师行政助理等热门行业介绍

我是新手,咋向您致敬啊?强烈致敬

聆文 : 2010-06-07#526
回复: toronto college介绍,选择,相关问题交流

顶,就是需要alexwei这样系统介绍的,介绍的很专业,也很系统!希望工作学习里继续多出一些这样的帖子!

qjy : 2010-06-08#527
回复: 求学选择个人谈及幼教,私人护理,社工,医师行政助理等热门行业介绍

楼主真是好人呀!请问楼主我想去学社工,在英语方面有什么要求/?另外我老公是医学检验的,这个专业在加国好找工作吗?我想给你加声望,或是送花什么的,可是不会操作,先谢谢你了,

sea71 : 2010-06-10#528
回复: 求学选择个人谈及幼教,私人护理,社工,医师行政助理等热门行业介绍

41岁的国内大学的英语老师,到加国后学什么专业好呢?正在纠结中。恳请lz和tx们帮忙分析一下。

alexwei : 2010-06-10#529
回复: 求学选择个人谈及幼教,私人护理,社工,医师行政助理等热门行业介绍

Social Service worker need high English skills, I think Linc 6+ is a must. Every college has English Test. There will tell you if you can take it or not. Medical or dental related are easy to find a job, however, you must get specific certificate in Canada.
楼主真是好人呀!请问楼主我想去学社工,在英语方面有什么要求/?另外我老公是医学检验的,这个专业在加国好找工作吗?我想给你加声望,或是送花什么的,可是不会操作,先谢谢你了,

alexwei : 2010-06-10#530
回复: 求学选择个人谈及幼教,私人护理,社工,医师行政助理等热门行业介绍

Social Service Worker
41岁的国内大学的英语老师,到加国后学什么专业好呢?正在纠结中。恳请lz和tx们帮忙分析一下。

christina_din : 2010-06-13#531
回复: 求学选择个人谈及幼教,私人护理,社工,医师行政助理等热门行业介绍

请问楼主啊,目前读ECE的就业市场还好吗?
如果去私校读,好吗

qjy : 2010-06-15#532
回复: 求学选择个人谈及幼教,私人护理,社工,医师行政助理等热门行业介绍

thanks

happyshark : 2010-06-16#533
回复: 求学选择个人谈及幼教,私人护理,社工,医师行政助理等热门行业介绍

lz你好,拜读了你的帖子感觉心里明朗多了。我想请教一下,我和爱人今年6月拿到了大信封,准备暑假短登一次,我爱人是在国内大学教英语的,我想请问一下,在国内学英语,出去之后等于没有了专业,依您的经验来看应该如何就业?如果学习的话,改学什么样的专业比较好,谢谢。期待您的回答。

wangduoyangyang : 2010-06-16#534
回复: 求学选择个人谈及幼教,私人护理,社工,医师行政助理等热门行业介绍

一口气看完全部30页,太有用了。 感谢楼主啊。
想麻烦问问Alex,想学社工,想去private college,但是不知道怎样才能了解哪些学校的口碑好,就业率高。好像没有官方的数据之类啊?
政府官方网站也只能告诉大家哪些学校有资质。
在网站上乱搜一气好像也得不到什么有用信息,愁人啊。。。。。
请楼主帮忙吧。谢谢先。

go888 : 2010-06-19#535
回复: 求学选择个人谈及幼教,私人护理,社工,医师行政助理等热门行业介绍

162贴 , 抵税指导

冷恋一生 : 2010-06-20#536
回复: 求学选择个人谈及幼教,私人护理,社工,医师行政助理等热门行业介绍

hello,Thank you so much for any input.
I am a 28-year old mom.My English is fine,Linc 7-8.I am really interested in Dental assistant and lab technician or even PRN.
Please give me some advice ,I would be really appricated.Thank you.

wowos : 2010-06-21#537
回复: toronto college介绍,选择,相关问题交流

三:私人护理(personal support worker)
其实如果是需要找个挣钱多一些的职业,这个职业比ECA强,首先工资高,其次有了这个专业的diploma一样可以做ECA的工作,因为课程里面有专们如何照顾孩子的。而且学到的知识其实对自己对家庭都很有好处。只是我们中国人有的时候有偏见,觉得去照顾病人,老人,收拾床铺不太好对家中父老交待。说在国外干什么呢,私人护理,都干些啥,收拾屋子,陪老人聊天,照顾病人。人家会觉得你好好的,跑国外干这个。但是我觉得这个职业不错,听我分析一下
就业前景:很不错。整个世界都进入老年社会了。失业率只有1%!。未来中国也很需要,另外,发达社会越来越重视对于老人,病人,disable包括孩子的照顾和护理。其实不仅是照顾吃穿活动那么简单,还有精神上的照顾,属于体验经济的一类,阳光产业。而且这个行业对于自己的性格磨练很有好处,你会变得很有耐心,细心和爱心,听说对于生老病死的看法也会大大改变。自雇和part time的机会也不错,照顾照顾邻里,有40%的人做part time. 平时做义工的也是这个行业比较多。
薪资:16-18也有到20块的,而且,由于从事一些和病人打交道的工作,一般健康福利会比较不错。
工作环境:其实我去过老人院和老人公寓,环境真得很不错。由于工作中动体力的时间较多,所以一般都不会让你连续工作时间太长的。
语言要求:这个工作一般对语言要求较高,不过有一点,会一门外语绝对是一个财富。
发展空间:这个专业实际上是Nurse Aid,发展当然是朝Rigester Nurse了。热门阿。
性别:女性占91%。 但是:听说急需男性阿,虽说有器械帮助,但是还是有些体力活的,所以经常在老人院里和医院里见到五大三粗的女性。还有,男性也有隐私阿,男同志有的时候也想让同性来照顾一下。顺便说一句,男护士也是非常非常受欢迎。
课程介绍,上这个课的教室很有意思,大家有兴趣可以参观。这个课对于实际操作要求很强,经常要演练急救,比如,突然有人晕倒了,怎么处理,学生们七手八脚的上。
这个课必须要西人教师上,因为西人和我们在处理人的问题上首先观念大大不同。西人把安全放在第一位,首先必须安全。所以什么民间土法,没有经过验证的不用。比如脖子不舒服,趁他不注意给他嘎吱一下。所以同样操作,观念不同,处理的方法不同。
课程介绍太长了,需要的留下邮件。包括,心理学的知识,人际沟通,营养学,老人护理,临终关怀,危机处理,家庭护理,收拾房屋,以及相关的法律法规,当然,实习是非常重要的一环

请问楼主,这个课程只适合已移民加拿大的人申请吗?还在FN阶段的准移民也能以留学的方式申请?因为我们夹在91新政和227之间,不知还要等到哪一天才有希望,所以考虑是不是能以留学的方式先去,一方面学一门技能,另一方面将为也好为移民增加一些职业优势,不知有没有可能性?

alexwei : 2010-06-27#538
回复: toronto college介绍,选择,相关问题交流

现在技术移民越来越难,留学可以申请经验类移民,是个移民曲线救国之路。不过需要注意两点:
1,需要至少2年Fulltime的学习,语言学习不算。
2。毕业后在加拿大工作一年以上。
当然还有更细的,不过对于那些着急出国的人,可以试一下
请问楼主,这个课程只适合已移民加拿大的人申请吗?还在FN阶段的准移民也能以留学的方式申请?因为我们夹在91新政和227之间,不知还要等到哪一天才有希望,所以考虑是不是能以留学的方式先去,一方面学一门技能,另一方面将为也好为移民增加一些职业优势,不知有没有可能性?

alexwei : 2010-06-27#539
回复: 求学选择个人谈及幼教,私人护理,社工,医师行政助理等热门行业介绍

you can try Lab technician.
hello,Thank you so much for any input.
I am a 28-year old mom.My English is fine,Linc 7-8.I am really interested in Dental assistant and lab technician or even PRN.
Please give me some advice ,I would be really appricated.Thank you.

alexwei : 2010-06-27#540
回复: 求学选择个人谈及幼教,私人护理,社工,医师行政助理等热门行业介绍

没有数据,只有自己去学校了解了,一般Private College 都可以试听,和老师聊一下,感觉一下。如果连这个要求都做不到,这个学校也不要去了,太冒险了。
一口气看完全部30页,太有用了。 感谢楼主啊。
想麻烦问问Alex,想学社工,想去private college,但是不知道怎样才能了解哪些学校的口碑好,就业率高。好像没有官方的数据之类啊?
政府官方网站也只能告诉大家哪些学校有资质。
在网站上乱搜一气好像也得不到什么有用信息,愁人啊。。。。。
请楼主帮忙吧。谢谢先。

alexwei : 2010-06-27#541
回复: 求学选择个人谈及幼教,私人护理,社工,医师行政助理等热门行业介绍

英语好多好,可以上大学阿,不要着急找工作。喜欢什么学什么。
lz你好,拜读了你的帖子感觉心里明朗多了。我想请教一下,我和爱人今年6月拿到了大信封,准备暑假短登一次,我爱人是在国内大学教英语的,我想请问一下,在国内学英语,出去之后等于没有了专业,依您的经验来看应该如何就业?如果学习的话,改学什么样的专业比较好,谢谢。期待您的回答。

wowos : 2010-06-28#542
回复: toronto college介绍,选择,相关问题交流

现在技术移民越来越难,留学可以申请经验类移民,是个移民曲线救国之路。不过需要注意两点:
1,需要至少2年Fulltime的学习,语言学习不算。
2。毕业后在加拿大工作一年以上。
当然还有更细的,不过对于那些着急出国的人,可以试一下

非常感谢!!

dlc18909867963 : 2010-06-28#543
回复: 求学选择个人谈及幼教,私人护理,社工,医师行政助理等热门行业介绍

在国内当记者,将近10年,去加拿大的话,读什么专业比较能迅速移民??谢谢

jishiduo : 2010-06-29#544
回复: 求学选择个人谈及幼教,私人护理,社工,医师行政助理等热门行业介绍

收藏

dlc18909867963 : 2010-06-29#545
回复: 求学选择个人谈及幼教,私人护理,社工,医师行政助理等热门行业介绍

在国内当记者,将近10年,去加拿大的话,读什么专业比较能迅速移民??谢谢


再顶一下!!谢谢!!

ooyahaha : 2010-06-30#546
回复: toronto college介绍,选择,相关问题交流

介绍一下社会工作者行业
国内没有社工这个行业,不过大家应该清楚这个是干什么的,我们遇到的settlement Counselor, Employment Counselor都是属于这个行业。这个行业对于我们移民来说有一个优势,我们会1门外语,而且是目前最大的移民群体的外语,各个机构都在扩展对于中国移民的服务。这样子就需要我们了。社工为什么要拓展服务呢,大家知道,no profit机构的钱来自于政府,企业或个人捐助,自我经营收入,其中政府最多,而政府的钱是要根据你做了多少事情来得,比如政府今年有7个亿是帮助新移民的,会分配到各个非营利机构,这些机构就要做一些针对新移民的服务来申请经费,同时还要有效果,如果没有政府的funding, 这个机构就要关门了。所以从事这个行业的人往往有marketing、工商管理,教师的背景,然后获得了本地学历。那么这个行业有多大呢,这个行业非常大,和商业基本上快差不多了。那些机构属于no-profit, 医院,学校,就业、安居服务机构,慈善机构,老人服务机构,绿色和平组织,环境和动物保护组织等。他的行业自成体系,你在一般的招聘网站或者agency那里很少看到招聘的信息。需要什么样的人呢,其实和商业一样,需要sales, accounting, administration, customer service, technician.不一样的地方,核心的服务人员是社工,教师,或者医务人员。大家可以去www.charityvillige.com 上去看,这是加拿大最大的非营利机构的招聘网站。
就业前景:不错
薪资水平:16-20左右。
性别:女性居多
个人奋斗方向:如果成为专业social worker, 工资可以达到70000,但是一般需要有social science master degree. 而 bachelor 毕业实际上和college毕业薪资差别不是太大,我个人觉得可以在拿到diploma后工作两年,直接申请master(用国内的学历)+diploma+工作经验,这样不错。
课程介绍
• Introduction to Community Service Worker
• Social Welfare, Diversity and Awareness
• The Family
• Couples, Family and Interpersonal Communication
• Oriented Case Management
• Environmental Issues and Crisis Interventions and
Prevention
• Bereavement and the Elderly
• Respond to Abuse
• Understanding Human Behavior
• Communication Skills
• Counseling Skills
• Report Writing for the CSW
• Field placement

By the end of the course, students will be able to:
• Demonstrate knowledge of the range of social and community service organizations in their own community as well as detailed knowledge of an agency.
• Practice the role of a team member within a social and community service organization.
• Demonstrate beginning level professional skills
• Apply theory and skills learned in the independent study components of this course.
学习:wdb10:

ooyahaha : 2010-06-30#547
回复: toronto college介绍,选择,相关问题交流

其他经济类的我觉得还是MBA可能好一些。至于accounting 和social worker.不好回答,会计是一个永远不会太差不会太好的行业,需求稳定。social worker是一个正在发展的行业,机会多些,但是压力大些,要看你是否适合这个工作。
首先你要先评估自己,你是适合和人打交道的工作,还是适合和物打交道的工作。也就是你的职业倾向。www.jobfutures.ca 上有简单的测试,或者去COSTI或者YMCA看有没有职业测试。免费的。
第二,你的过去的工作经验是否对你在相关行业发展有用,比如你以前做教师,当然对于你做Social service worker非常好.
第三, 你自己的兴趣。工作都是boring,找一个不太boring的工作多好。
写下这些,找出交集,确定目标,坚定不移。我觉得选择的时候如果确实结合了自己的长处,兴趣,不会错的,失败的人是不能坚持按照目标前进,遇到困难就在出发点找原因,结果朝三暮四。
时间对于我们最宝贵,早点定目标吧。
谢谢你的话:wdb17:

悲回锾 : 2010-07-02#548
回复: 求学选择个人谈及幼教,私人护理,社工,医师行政助理等热门行业介绍

想?下有什麽好的?科?校和??推溯???要求是什麽?我雅思六月拿的5.5(listening5 reading5.5 writing5.5 speaking6.0)最好是今年檫?的。多著!

wangduoyangyang : 2010-07-02#549
回复: 求学选择个人谈及幼教,私人护理,社工,医师行政助理等热门行业介绍

谢谢 Alex。准备下个月找几个Private college 去看看。

bellafeng : 2010-07-02#550
回复: 求学选择个人谈及幼教,私人护理,社工,医师行政助理等热门行业介绍

谢谢楼主分享,请问如果本人在国内做国际贸易到加国哪个专业比较好?

悲回锾 : 2010-07-02#551
回复: 求学选择个人谈及幼教,私人护理,社工,医师行政助理等热门行业介绍

雅思分5.5,力5.0,作都是5.5,口6.0能什麽的科校?我是高中,加高考,成各科平均大是80分(分分100),我的目的是移民,忙分析下什麽程合我,!

alexwei : 2010-07-04#552
回复: 求学选择个人谈及幼教,私人护理,社工,医师行政助理等热门行业介绍

专科学校不需要雅思成绩。多伦多就那么几个,都差不多。
想?下有什麽好的?科?校和??推溯???要求是什麽?我雅思六月拿的5.5(listening5 reading5.5 writing5.5 speaking6.0)最好是今年檫?的。多著!

alexwei : 2010-07-04#553
回复: 求学选择个人谈及幼教,私人护理,社工,医师行政助理等热门行业介绍

看你喜欢,贸易就不要再读了
谢谢楼主分享,请问如果本人在国内做国际贸易到加国哪个专业比较好?

Hill3 : 2010-07-08#554
回复: 求学选择个人谈及幼教,私人护理,社工,医师行政助理等热门行业介绍

楼主能介绍一下农业技术方面的学院吗?谢谢!

ghwnn2005 : 2010-07-08#555
回复: 求学选择个人谈及幼教,私人护理,社工,医师行政助理等热门行业介绍

谢谢分享!

lovetojourney : 2010-07-17#556
回复: 求学选择个人谈及幼教,私人护理,社工,医师行政助理等热门行业介绍

旅游管理呢

zhy2001 : 2010-07-18#557
回复: 求学选择个人谈及幼教,私人护理,社工,医师行政助理等热门行业介绍

想理解psw的课程介绍,请楼主发到我的邮箱zhy2001@pku.org.cn

lei tong : 2010-07-20#558
回复: 求学选择个人谈及幼教,私人护理,社工,医师行政助理等热门行业介绍

我是新来的,我住在蒙特利尔,看到这么实用的帖子和楼主这么多耐心的回复真是万分感动:wdb10::wdb17:。在这里,我也想请教楼主现在学习牙医助理专业的前景如何,虽然我住在法语区,也有学法语,但我还是比较习惯说英语,请问如果学习这个专业是用英文学还是用法文学?请给我一些建议,谢谢!

eva08 : 2010-07-20#559
回复: 求学选择个人谈及幼教,私人护理,社工,医师行政助理等热门行业介绍

谢谢,有用

xiangxiu : 2010-07-20#560
回复: 求学选择个人谈及幼教,私人护理,社工,医师行政助理等热门行业介绍

私人护理(personal support worker)哪里可以学习啊?文凭是否加拿大各省通用?

christina_din : 2010-07-21#561
回复: 求学选择个人谈及幼教,私人护理,社工,医师行政助理等热门行业介绍

alexwei,请问下pharmacy technician和medical adminstrative assistant哪个专业读出来比较容易就业呢?收入高不?
谢谢

jiang184jing : 2010-07-22#562
回复: 求学选择个人谈及幼教,私人护理,社工,医师行政助理等热门行业介绍

请教楼住, 我刚来多伦多不久, 移民, 前几天和几个韩国人吃饭,听他说seneca的航空飞行员是非常有名的,我对这个专业感兴趣,而且他说这个专业有很多中国人啊,但怎么也Google不到有关这个专业的讨论??

emilyoffcie : 2010-07-23#563
回复: 求学选择个人谈及幼教,私人护理,社工,医师行政助理等热门行业介绍

学习了,不知道,楼主的意见是否适用于到加留学的学生(没有申请的移民的),但是想毕业后留在加国的,不知道可否适用,尤其是专业的选择。

我在国内成人大学,读英语,后来到酒店工作,一直做总经理秘书,现在31了,想出国,移民条件不够。
只有走留学再移民的路。
刚开始,选了酒店管理这个专业,至少觉得这个沾点儿边,一年或两年的college课程(humber or niagara),主要是从签证的角度出发,觉得这样可以有理由说明来加的目的,是否毕业后能在加找到工作?
但是无意中看到了LZ的贴子,觉得帮助良多,对于专业的选择,自己也不知道应该选择那个,毕业后在加找到工作的可能性高?PSW不知道我这样背景的人可不可学习。
国内的中介,提供的信息不太负责任,所以,想听听已经登陆的好心人的意见。

happyshark : 2010-07-23#564
回复: 求学选择个人谈及幼教,私人护理,社工,医师行政助理等热门行业介绍

楼主你好,其实来加拿大之前就拜读了你的文章,现在我正在加拿大的图书馆里查信息,再次看到你的帖子,更加对你对加拿大就业,学习方面的认识感到佩服了。
我有几个问题想请教:先简单介绍一下我们的情况。我和我的爱人在国内是大学教师,分别交工业设计和英语。我们利用暑假短登一周了,首先在YMCA做了语言测试,我是5级,我爱人是8级(因为她是交英语的),上了免费的linc课程。总体来说班上的同学都觉得我们的速度是挺快的。
我们现在感到困惑的还是专业的选择问题:首先说我的爱人,她在linc的老师说她可以去seneca 读一个教英语教师相关的course,然后到linc这样的机构里应聘当英语老师。不知道当地这样的教师工资收入怎样,是不是part-time的比较多。她英语不错,但是想既然出来了,是不是能再学些另外的专业,更好就业,赚的钱能比教师多一点。比如你说的药剂师等等。另外学护士的话是不是前景比较好。还有人建议她去物流,或者牙科助理,洗牙师之类的。所以现在我们也拿不准那些是比较好的专业,那些不好就业。希望借你的经验和知识补己之短。

再说我自己,我想问一下会不会有机械制图之类的专业可学,我本身是从事设计的,画图能力不错。另外,如果不从事本专业的话,我曾经听人说过RMT(注册按摩师)比较好就业,不知道是不是这样。我还甚至听人介绍Casino dealer 的活不错。我不知道这些情况是不是属实,或者这些工作是不是有发展的潜力。一想到一切要从新来过,真的觉得挺不容易的。
最后我还想问一下,蒙城是不是比多伦多要好就业,生活压力和日常开支会少一些?
?嗦了这么多,希望别介意,再次表示感谢。

五月石榴 : 2010-07-23#565
回复: 求学选择个人谈及幼教,私人护理,社工,医师行政助理等热门行业介绍

好贴,必须顶一下。

不懂就问 : 2010-07-24#566
回复: toronto college介绍,选择,相关问题交流

现在技术移民越来越难,留学可以申请经验类移民,是个移民曲线救国之路。不过需要注意两点:
1,需要至少2年Fulltime的学习,语言学习不算。
2。毕业后在加拿大工作一年以上。
当然还有更细的,不过对于那些着急出国的人,可以试一下

我读了曼省的要求,只需一学年(8个月即可),我现在就想以最快的速度出去。而且曼省也没说要毕业后在加工作一年以上。

难道我理解错了?

selina_cywang : 2010-08-05#567
回复: 求学选择个人谈及幼教,私人护理,社工,医师行政助理等热门行业介绍

想在COLLEGE读HR或MARKETING之类的专业, 不知道难度如何, 对英语要求很高吗?

younghusband : 2010-08-06#568
回复: 求学选择个人谈及幼教,私人护理,社工,医师行政助理等热门行业介绍

国内医学本科申请这里的college cegep universte之类的学校需要学历认证么?因为专业多半是不会认的,所以不知道是不是还是需要进行学历认证。想报 像牙医助理,洗牙师,医疗档案管理 之类的医疗相关专业。不知道是不是需要去认证?谢谢了

而且我看了一下洗牙师的课程,发现里面有些社会学科,比如哲学之类的课程,请问国内大学毕业的人报这样的college之后也要跟着一起学这些非专业类课程吗?

雪白 : 2010-08-20#569
回复: 求学选择个人谈及幼教,私人护理,社工,医师行政助理等热门行业介绍

楼主还在吗?
我也鼓起勇气谈谈我的情况吧。
1、我在国内的专业是经济学,世界经济,硕士读了两年,没毕业,移民了,就放弃了。
2、我英语不太好,没考过托福什么的。
2、我挺喜欢孩子的,想学ece。看过一些贴子,建议如果想从事这方面的工作,就先去做义工,但现在义工工作很难找,很多地方招义工,首先要求你有ece或eca证书。
3、有朋友建议我,既然学经济出身的,应该还是往这方面努力,比如学会计或金融什么的。但是感觉这方面竞争也挺激烈的。
4、我也想过学社工,但又害怕自己并不是很擅长与人相处。另外觉得会不会工作很枯燥。

说得有点乱,不知楼主有什么建议。

superwoman : 2010-08-20#570
回复: toronto college介绍,选择,相关问题交流

介绍一下社会工作者行业
国内没有社工这个行业,不过大家应该清楚这个是干什么的,我们遇到的settlement Counselor, Employment Counselor都是属于这个行业。这个行业对于我们移民来说有一个优势,我们会1门外语,而且是目前最大的移民群体的外语,各个机构都在扩展对于中国移民的服务。这样子就需要我们了。社工为什么要拓展服务呢,大家知道,no profit机构的钱来自于政府,企业或个人捐助,自我经营收入,其中政府最多,而政府的钱是要根据你做了多少事情来得,比如政府今年有7个亿是帮助新移民的,会分配到各个非营利机构,这些机构就要做一些针对新移民的服务来申请经费,同时还要有效果,如果没有政府的funding, 这个机构就要关门了。所以从事这个行业的人往往有marketing、工商管理,教师的背景,然后获得了本地学历。那么这个行业有多大呢,这个行业非常大,和商业基本上快差不多了。那些机构属于no-profit, 医院,学校,就业、安居服务机构,慈善机构,老人服务机构,绿色和平组织,环境和动物保护组织等。他的行业自成体系,你在一般的招聘网站或者agency那里很少看到招聘的信息。需要什么样的人呢,其实和商业一样,需要sales, accounting, administration, customer service, technician.不一样的地方,核心的服务人员是社工,教师,或者医务人员。大家可以去www.charityvillige.com 上去看,这是加拿大最大的非营利机构的招聘网站。
就业前景:不错
薪资水平:16-20左右。
性别:女性居多
个人奋斗方向:如果成为专业social worker, 工资可以达到70000,但是一般需要有social science master degree. 而 bachelor 毕业实际上和college毕业薪资差别不是太大,我个人觉得可以在拿到diploma后工作两年,直接申请master(用国内的学历)+diploma+工作经验,这样不错。
课程介绍
• Introduction to Community Service Worker
• Social Welfare, Diversity and Awareness
• The Family
• Couples, Family and Interpersonal Communication
• Oriented Case Management
• Environmental Issues and Crisis Interventions and
Prevention
• Bereavement and the Elderly
• Respond to Abuse
• Understanding Human Behavior
• Communication Skills
• Counseling Skills
• Report Writing for the CSW
• Field placement

By the end of the course, students will be able to:
• Demonstrate knowledge of the range of social and community service organizations in their own community as well as detailed knowledge of an agency.
• Practice the role of a team member within a social and community service organization.
• Demonstrate beginning level professional skills
• Apply theory and skills learned in the independent study components of this course.

社工是不是学的人太多了?世界各国移民都看好。很多Nonprofit的组织没有国家拨款相继倒闭。中国人的竞争力到底如何?英文好有Marketing背景,就好找了吗?另外上面有人提到学fundraising是不是更实用?
多谢指导。

alexwei : 2010-08-20#571
回复: 求学选择个人谈及幼教,私人护理,社工,医师行政助理等热门行业介绍

就业还可以,不过语言要好,工资就soso, 也就12-15
alexwei,请问下pharmacy technician和medical adminstrative assistant哪个专业读出来比较容易就业呢?收入高不?
谢谢

alexwei : 2010-08-20#572
回复: toronto college介绍,选择,相关问题交流

加拿大的国情决定了社会服务是一个很大的就业市场,现在政府经费紧张,倒闭几个组织也很正常,他的就业缺口主要来源于未来一段时间Baby Boomer一代退休的人。移民在这个领域工作的人很多,对于语言是有一定的要求的。Marketing 背景很有帮助,但是专业知识是最重要的。Fundrasing 是为机构到处筹钱的,如果你喜欢压力,可以做,不过我个人觉得华人在这个领域没优势。
社工是不是学的人太多了?世界各国移民都看好。很多Nonprofit的组织没有国家拨款相继倒闭。中国人的竞争力到底如何?英文好有Marketing背景,就好找了吗?另外上面有人提到学fundraising是不是更实用?
多谢指导。

alexwei : 2010-08-20#573
回复: 求学选择个人谈及幼教,私人护理,社工,医师行政助理等热门行业介绍

不好意思,难建议,ECE就业还可以吧,不过对语言有要求的。会计也不错

楼主还在吗?
我也鼓起勇气谈谈我的情况吧。
1、我在国内的专业是经济学,世界经济,硕士读了两年,没毕业,移民了,就放弃了。
2、我英语不太好,没考过托福什么的。
2、我挺喜欢孩子的,想学ece。看过一些贴子,建议如果想从事这方面的工作,就先去做义工,但现在义工工作很难找,很多地方招义工,首先要求你有ece或eca证书。
3、有朋友建议我,既然学经济出身的,应该还是往这方面努力,比如学会计或金融什么的。但是感觉这方面竞争也挺激烈的。
4、我也想过学社工,但又害怕自己并不是很擅长与人相处。另外觉得会不会工作很枯燥。

说得有点乱,不知楼主有什么建议。

alexwei : 2010-08-20#574
回复: toronto college介绍,选择,相关问题交流

不清楚,是不是所谓省移民计划,是你自己看的官方网站还是哪里的中文信息。最好搞清楚。我这是移民局网站上的要求
我读了曼省的要求,只需一学年(8个月即可),我现在就想以最快的速度出去。而且曼省也没说要毕业后在加工作一年以上。

难道我理解错了?

alexwei : 2010-08-20#575
回复: 求学选择个人谈及幼教,私人护理,社工,医师行政助理等热门行业介绍

8级还要学LINC,5级都不用了,我记得我那时候才测了3级,但是我已经开始找工作了。
第一个建议,尽快参加一些机构关于招工方面的服务,比如ELT program, Job Searching Club, Mentor Program. 你可以找找COSTI, JVS Toronto, YMCA. 了解加拿大就业市场,提高沟通技巧,学会找工的方法。这可能花你三-五个月时间.Linc 没有用,真的,在Linc班里,就好像呆在加拿大的中国,与世隔绝。
第二个建议,考一个英语证可以,不过教师的竞争非常激烈,英文教师遍地都是,Linc这样的机构已经严重过剩。护士很不错,如果愿意拼搏,是个好方向。一个好工作最好是薪资高(至少4万以上),福利好,稳定,未来有上升空间,受人尊敬。护士符合这个条件,我个人坚决鼓励。
第三个建议:设计,绘图方面我记得还是有机会的,你可以通过参加就业服务来了解这方面的就业市场,该学习学习,该考证考证,机会还是有的!

赶紧跳出Linc,这是我的建议。
楼主你好,其实来加拿大之前就拜读了你的文章,现在我正在加拿大的图书馆里查信息,再次看到你的帖子,更加对你对加拿大就业,学习方面的认识感到佩服了。
我有几个问题想请教:先简单介绍一下我们的情况。我和我的爱人在国内是大学教师,分别交工业设计和英语。我们利用暑假短登一周了,首先在YMCA做了语言测试,我是5级,我爱人是8级(因为她是交英语的),上了免费的linc课程。总体来说班上的同学都觉得我们的速度是挺快的。
我们现在感到困惑的还是专业的选择问题:首先说我的爱人,她在linc的老师说她可以去seneca 读一个教英语教师相关的course,然后到linc这样的机构里应聘当英语老师。不知道当地这样的教师工资收入怎样,是不是part-time的比较多。她英语不错,但是想既然出来了,是不是能再学些另外的专业,更好就业,赚的钱能比教师多一点。比如你说的药剂师等等。另外学护士的话是不是前景比较好。还有人建议她去物流,或者牙科助理,洗牙师之类的。所以现在我们也拿不准那些是比较好的专业,那些不好就业。希望借你的经验和知识补己之短。

再说我自己,我想问一下会不会有机械制图之类的专业可学,我本身是从事设计的,画图能力不错。另外,如果不从事本专业的话,我曾经听人说过RMT(注册按摩师)比较好就业,不知道是不是这样。我还甚至听人介绍Casino dealer 的活不错。我不知道这些情况是不是属实,或者这些工作是不是有发展的潜力。一想到一切要从新来过,真的觉得挺不容易的。
最后我还想问一下,蒙城是不是比多伦多要好就业,生活压力和日常开支会少一些?
?嗦了这么多,希望别介意,再次表示感谢。

alexwei : 2010-08-20#576
回复: 求学选择个人谈及幼教,私人护理,社工,医师行政助理等热门行业介绍

酒店管理还行,对于留学生,属于商科,压力小,容易过,未来移民不难。不过你工作了这么久,申请过来读College, 估计签证难申请,有移民倾向。 如果将来万一拒签,建议可以换一个 Pre-MBA的方向申请,比较适合工作过,年龄大的人,学习计划容易通过。
移民学这个没有必要,移民的选择面广。
学习了,不知道,楼主的意见是否适用于到加留学的学生(没有申请的移民的),但是想毕业后留在加国的,不知道可否适用,尤其是专业的选择。

我在国内成人大学,读英语,后来到酒店工作,一直做总经理秘书,现在31了,想出国,移民条件不够。
只有走留学再移民的路。
刚开始,选了酒店管理这个专业,至少觉得这个沾点儿边,一年或两年的college课程(humber or niagara),主要是从签证的角度出发,觉得这样可以有理由说明来加的目的,是否毕业后能在加找到工作?
但是无意中看到了LZ的贴子,觉得帮助良多,对于专业的选择,自己也不知道应该选择那个,毕业后在加找到工作的可能性高?PSW不知道我这样背景的人可不可学习。
国内的中介,提供的信息不太负责任,所以,想听听已经登陆的好心人的意见。

superwoman : 2010-08-25#577
回复: toronto college介绍,选择,相关问题交流

加拿大的国情决定了社会服务是一个很大的就业市场,现在政府经费紧张,倒闭几个组织也很正常,他的就业缺口主要来源于未来一段时间Baby Boomer一代退休的人。移民在这个领域工作的人很多,对于语言是有一定的要求的。Marketing 背景很有帮助,但是专业知识是最重要的。Fundrasing 是为机构到处筹钱的,如果你喜欢压力,可以做,不过我个人觉得华人在这个领域没优势。

老大,这里说的专业知识指什么?这种服务组织通常我发现都是很General的,如果口语好,有Marketing背景,在这种组织里应该应聘些什么职位呢?是不是都要求先做义工才会考虑呢?或是至少学个短期的Community Service之类的课程?
麻烦啦!

superwoman : 2010-08-25#578
回复: 求学选择个人谈及幼教,私人护理,社工,医师行政助理等热门行业介绍

给您发了悄悄话!请查收!

sarefina : 2010-08-26#579
回复: 求学选择个人谈及幼教,私人护理,社工,医师行政助理等热门行业介绍

一口气从头看到尾,受益匪浅!!!很感谢各位同胞的分享呢!LZ人真好
Alex,有几个问题想请你帮忙分析一下
我国内本科刚毕业,但因为怕超龄所以申请了休学,要明年才拿毕业证。属于移民,这个visa刚拿到。读的是药事管理专业(主要学药学和管理方面的课程),大学期间属于活跃分子,做过行政和外贸方面的工作。雅思考了个6,有需要的会再考,6.5应该是没问题的。
现在问题主要是
1、因为不喜欢本科的专业,想转。打算申请Food nutriology(食品营养学)、会展(EVENT MANAGEMENT)、酒店管理(Hotel and Tourism Management)或国际商贸(International Business)的master,这几个专业的就业前景哪个会好些呢?容易找到工作嘛?
2、有个朋友刚从多伦多回国,他建议我如果不是非常想念书想早点工作的话,还是直接去college读个实用的专业。master有些专业需要读到Doctor才能有出路,否则工作会比较难找,是这样吗?:wdb2:上边说的那三四个专业,是读master 还是college比较实际呢?(我个人比较想快点工作)
3、父母长登温哥华,我自己还没决定要去温哥华还是多伦多。如果是上边边的四个专业,哪个城市工作会好找些呢?哪间学校会比较好点呢?
不好意思,一下子问了这么多问题。
先谢过alex和其他朋友们了~!!!

alexwei : 2010-08-26#580
回复: toronto college介绍,选择,相关问题交流

我也说不好,应该算是一种门槛吧,如果你没有社工相关专业的学历,恐怕根本就没有面试的机会。
老大,这里说的专业知识指什么?这种服务组织通常我发现都是很General的,如果口语好,有Marketing背景,在这种组织里应该应聘些什么职位呢?是不是都要求先做义工才会考虑呢?或是至少学个短期的Community Service之类的课程?
麻烦啦!

alexwei : 2010-08-26#581
回复: 求学选择个人谈及幼教,私人护理,社工,医师行政助理等热门行业介绍

我个人观点,College 确实比较好找一些初级工作,不过稳定之后还是要继续学习比较好,高学历到哪里都还是要的。你说的几个专业不太了解。
一口气从头看到尾,受益匪浅!!!很感谢各位同胞的分享呢!LZ人真好
Alex,有几个问题想请你帮忙分析一下
我国内本科刚毕业,但因为怕超龄所以申请了休学,要明年才拿毕业证。属于移民,这个visa刚拿到。读的是药事管理专业(主要学药学和管理方面的课程),大学期间属于活跃分子,做过行政和外贸方面的工作。雅思考了个6,有需要的会再考,6.5应该是没问题的。
现在问题主要是
1、因为不喜欢本科的专业,想转。打算申请Food nutriology(食品营养学)、会展(EVENT MANAGEMENT)、酒店管理(Hotel and Tourism Management)或国际商贸(International Business)的master,这几个专业的就业前景哪个会好些呢?容易找到工作嘛?
2、有个朋友刚从多伦多回国,他建议我如果不是非常想念书想早点工作的话,还是直接去college读个实用的专业。master有些专业需要读到Doctor才能有出路,否则工作会比较难找,是这样吗?:wdb2:上边说的那三四个专业,是读master 还是college比较实际呢?(我个人比较想快点工作)
3、父母长登温哥华,我自己还没决定要去温哥华还是多伦多。如果是上边边的四个专业,哪个城市工作会好找些呢?哪间学校会比较好点呢?
不好意思,一下子问了这么多问题。
先谢过alex和其他朋友们了~!!!

sarefina : 2010-08-26#582
回复: 求学选择个人谈及幼教,私人护理,社工,医师行政助理等热门行业介绍

那我能在哪儿查到这些专业的相关课程资料呢?还有它们的就业形势

天亮说晚安 : 2010-09-03#583
回复: 求学选择个人谈及幼教,私人护理,社工,医师行政助理等热门行业介绍

十分有用的贴。。。

青箬翠竹 : 2010-09-06#584
回复: 求学选择个人谈及幼教,私人护理,社工,医师行政助理等热门行业介绍

一口气读完楼主的帖子,很有用,谢谢你了。

hxl670723 : 2010-09-06#585
回复: 求学选择个人谈及幼教,私人护理,社工,医师行政助理等热门行业介绍


alexwei : 2010-09-06#586
回复: 求学选择个人谈及幼教,私人护理,社工,医师行政助理等热门行业介绍

www.ontariojobfutures.ca
www.labourmarketinformation.ca
那我能在哪儿查到这些专业的相关课程资料呢?还有它们的就业形势

飞燕 : 2010-09-06#587
回复: 求学选择个人谈及幼教,私人护理,社工,医师行政助理等热门行业介绍

ding

crazyfatcat : 2010-09-08#588
回复: 求学选择个人谈及幼教,私人护理,社工,医师行政助理等热门行业介绍

LZ你好。能介绍一下personal support worker课程介绍么?我看你说很长,如果可以的话。给发我邮箱么?jody.goertzen@gmail.com 谢谢,你的贴子真的很有用。

wing.happy : 2010-09-08#589
回复: 求学选择个人谈及幼教,私人护理,社工,医师行政助理等热门行业介绍

LZ你好。能介绍一下personal support worker课程介绍么?我看你说很长,如果可以的话。给发我邮箱么?jody.goertzen@gmail.com 谢谢,你的贴子真的很有用。

同问~~superfanqie1985@yahoo.com.cn 感谢・・・

newway3 : 2010-09-10#590
回复: 求学选择个人谈及幼教,私人护理,社工,医师行政助理等热门行业介绍

本人现在大温,雅思6.5,国内英语专业(考这点分有点抱歉呵)的妈妈,希望未来的工作能兼顾小孩(5岁),目前想先学点技术,肯请楼主和各位知情者指点下培训方向,最好能提供培训机构线索。这样的培训课程最佳:weekday上课,学习压力不太大(以便照顾小孩),培训时间长点也可。提前表示感谢!欢迎大家多提想法!

nancyjch : 2010-09-14#591
回复: 求学选择个人谈及幼教,私人护理,社工,医师行政助理等热门行业介绍

楼主,先谢谢你的无私帮助,看了你的贴获益匪浅!同时还希望你能为我指点迷津,谢谢!!!也希望大家能多提宝贵意见!
我目前想为接下来的加国生活提前做准备(尽管移民还没有批下来),本人不能接受过全职主妇的生活。主要现状情况如下:
1.受教育和工作情况:本科学的是英语教育专业,研究生学的是企业管理专业,现已经从事8年的高校英语教师兼学生管理及大学生就业指导工作;
2.、家庭情况,本人现已是两个小孩的妈妈,大宝3岁,小宝刚出生,想为以后的培训和工作寻找方向;
3、我是直接申请读什么级别,什么专业会比较好?

sarefina : 2010-09-15#592
回复: 求学选择个人谈及幼教,私人护理,社工,医师行政助理等热门行业介绍


谢谢alex!我会认真研究研究的~:wdb6:

yun_zhan : 2010-09-15#593
回复: 求学选择个人谈及幼教,私人护理,社工,医师行政助理等热门行业介绍

good!!!

生命=努力+谦让 : 2010-09-16#594
回复: 求学选择个人谈及幼教,私人护理,社工,医师行政助理等热门行业介绍

LZ人真好,谢谢!!!

LISALLA : 2010-09-16#595
回复: 求学选择个人谈及幼教,私人护理,社工,医师行政助理等热门行业介绍

谢谢楼主的好贴!:wdb17:

肯尼迪学院 : 2010-09-16#596
回复: 求学选择个人谈及幼教,私人护理,社工,医师行政助理等热门行业介绍

??

康乐0100 : 2010-09-25#597
回复: 求学选择个人谈及幼教,私人护理,社工,医师行政助理等热门行业介绍

哈偏好

mrs.bean : 2010-09-25#598
回复: 求学选择个人谈及幼教,私人护理,社工,医师行政助理等热门行业介绍

本人现在大温,雅思6.5,国内英语专业(考这点分有点抱歉呵)的妈妈,希望未来的工作能兼顾小孩(5岁),目前想先学点技术,肯请楼主和各位知情者指点下培训方向,最好能提供培训机构线索。这样的培训课程最佳:weekday上课,学习压力不太大(以便照顾小孩),培训时间长点也可。提前表示感谢!欢迎大家多提想法!

试下远程教育吧。我就是一名兼职Early Childhood Education的远程教育学生,全职daycare老师,半职house wife. 很充实但没太大压力!

newway3 : 2010-09-26#599
回复: 求学选择个人谈及幼教,私人护理,社工,医师行政助理等热门行业介绍

谢谢楼上回复!我是一人在这里带孩子,因为雅思为G类,所以目前无法申请Online的college课程。但在合适时候,远程教育的确是一个办法,就是不知道就业市场对这个的认可度如何?

mrs.bean : 2010-09-26#600
回复: 求学选择个人谈及幼教,私人护理,社工,医师行政助理等热门行业介绍

谢谢楼上回复!我是一人在这里带孩子,因为雅思为G类,所以目前无法申请Online的college课程。但在合适时候,远程教育的确是一个办法,就是不知道就业市场对这个的认可度如何?

学校有入学考试的。

我不清楚别的科目,就ECE来说,在BC ECE Registry的网页上有列有被认可设有ECE专业的院校。

我认为加拿大远程教育的认可度跟全职出来差不了多少。我身边有亲戚朋友都通过远程教育升值,有学会计bachelor degree的(Athabasca University),教育master的(SFU),高校管理 master的(U of Calgary)。

我们都是在职的,对写assignment很有帮助,因为都是理论与实践经验相结合的。

fanpeiyan : 2010-10-12#601
回复: 求学选择个人谈及幼教,私人护理,社工,医师行政助理等热门行业介绍

lz你好,我是国内临床医学本科毕业,想去加拿大留学后移民,gpa较低,学校成绩一般,中介建议我读百年理工的一年研究生文凭 职业健康管理专业或者综合健康咨询师,前者带实习,请问这两个专业工作好找吗?哪个好点?毕业后工作一年能申请移民吗?本人对加国护士也很感兴趣,长远想向这个发展,但家庭条件一般,不想花太多钱读书,想将来一边工作一边往上读,不知lz有什么高见可以给我呢?我有几个想法:1)先读百年理工的两年实践护士,找工作后申请移民几年后再读RN, 2)先读一年的职业健康管理,工作后申请移民再读part time的实践护士,或者贷款读full time的实践护士,也有人说可以直接用国内的医学毕业背景转学分读大学护理学的大三大四,本人女,27了,家庭条件一般,不知那条路比较合适?其实如果职业健康管理就业前景好,有深造机会,薪水高,也可以考虑。麻烦解答,不甚感激!!!

jamie003125 : 2010-10-13#602
回复: 求学选择个人谈及幼教,私人护理,社工,医师行政助理等热门行业介绍

必须收藏

yglwxl : 2010-10-18#603
回复: 求学选择个人谈及幼教,私人护理,社工,医师行政助理等热门行业介绍

mark
关于公立和私人职业学校的课程和资质,可以到www.edu.gov.on.ca上去查, 所有经过政府审批的学校,以及私立职业学校的经过政府审批的diploma的课程在上面都可以查到。

首先一个概念,加拿大大专(College)跟我们国内的大专概念大不相同,更偏重于职业培训,注重的是技能和动手实践能力。大专的课程设置比较全面,合理,由浅入深易于接受。除了理论基础课程,更多的是职业培训课程、大学转学分课程和热门职业证书培训。同时,许多大专都与企业合作,甚至为大型企业设置特定专业,而且学院还经常调整专业,开设新兴的专业,尽量为学生就业创造机会,因此,大专生普遍比本科生好找工作,但如果想要做管理工作和科研工作,还得再读大学。

再说说多伦多的college, 多伦多的college分为两种,community college和private college,Community colleg就是俗称的公立大专,像Seneca, Geoge Brown,特点是学科非常全,基本上涵盖各个行业,学分可以被各个大学接受。有1年到3年的certifate, diploma和Degree的课程,也有时间更短的培训。可以在[URL http://www.edu.gov.on.ca/eng/general/list/college.html[/url]上找到On省所有Community 大专,一共24所。学生可以向政府申请OSAP贷款上学,但是大部分课程不能够申请EI学费上学,因为学制比较长,其实有的专业实际课时不多,但是由于Community College 假期和大学类似,时间较长,所以拖得比较长。政府可不愿意你拿着EI学费还休假,不过由于竞争的关系,他们也推出一些适合申请EI学费的专业。学生毕业后的就业率在70%到80%之间。

还有一类叫private college, 也可以叫private career college, 故名思义,是私人办的以职业培训为主的college, 一般规模不大,没有专门的校园。他们不可能开设像community college那么全的课程,所以都选择市场上的热门专业,并且可以较短的时间让学生莆眨玫iploma和Certificate(如果这个行业需要专门的certificate).由于规模不大,因此各个学校都有不同的侧重点,有比较强的专业,因此选择起来要困难一些。课程一般不超过一年,提供diploma,还有很多的课程是专门帮你通过一些证书考试的,有的学校学生可以申请OSAP,但是大部分学校不行。他们主要面向自费学习和EI学费学生。由于没有假期,上课时间灵活(一般每个专业都是滚动开课,所以学生在一年当中的任何一个月都可以入学)。这些学校,就业率是命根子,如果没有很高的就业率,就难招到学生,政府也很难批准EI学生入学。因此他们的专业一定是就业趋势很好的,老师一般是专业知识强并在行业有多年工作经验的,不仅负责教授学生技能,还要帮助学生实习和找工作的指导,因此把老师叫instructor,这是和community College教学非常不同的一点,Community教学还是正规的学校式教育,一个专业,7、8门课,每个老师各教一摊。大部分private college的一个班一个老师带到底,像师傅带徒弟,老师是老板雇的,因此压力更大些。学校也积极和各种社会资源雇主联系给学生提供co-op 和就业机会,因此就业率比community college高,好的能够达到90%以上。现在比较热门的,比如IT的一些专业,社工CSW,牙医助理,幼教助理ECA,药剂师PT,Computer Accounting,护工PSW,医生行政助理MOA等。但是private college 良莠不齐,首先大家可以去政府的网站
http://www.gov.on.ca/ont/portal/!ut/p/.cmd/cs/.ce/7_0_A/.s/7_0_252/_s.7_0_A/7_0_252/_l/en?docid=053263 去搜索,确保这个college是政府审批过的正在经营的college. 然后,看看这个学校的网站,看看有没有感兴趣的课程。

关于如何选择学校的问题,我想留到下次再写,希望能够逐渐补充些资料,给大家一个好的参考。也希望大家和我充分交流,提供不同意见。

6月10日续,如何选择college呢,论坛上这类东西可多了,我谈谈自己看法。我觉得核心就一个,快,时间越短越好。为什么呢,第一点,college 就是一个大专,你不管怎么学,拿什么文凭,到社会上也是一个具备初级技能的毕业生。虽说,seneca等在我们圈子名气响亮,可是并不入本地人眼。我有几个银行和政府的朋友,有本地的也有移民。我问他们关于college的问题,他们一致认为,college是非常初级的,只能从事初级工作。而且他们其实不是很清楚或者很在意college之间有什么差别。而到了大学这一级别,才开始注意学校之间的区别,这和国内差不多。第二点,既然college文凭不怎么样,我为什么还需要呢,因为我要本地教育背景阿。移民都是高学历,被这个卡住了,只好搞个本地文凭,其实大家都在学校里呆了不少年,知道学校里学的东西工作中能用得很少,一旦工作了,还得吃自己的工作经验。那这样,我们上college的目的就明确了。搞个本地文凭,既然如此,同样是diploma, 越快拿到得越好。千万不要去学那些什么2年,三年的课程。1年的本地工作经验足以媲美三年的学校教育。其实还有一点,这里的人并不是不认你在国内的工作和教育经历,只是他们认为如果你能够通过本地的学习获得技能,这样你的经验加上本地获得的技能才能把你的才干发挥出来。

其实短的课程未必不好,紧凑,只教实用的。大家如果仔细看各community大学的课程(我那里三所大学的全套),头三个学期都是基础课,一点用没有。那是给这里的高中生用的。有人说了,不对,我确实感觉学到了很多东西。是的,你主要在头三个学期学到的是英语。有些知识,翻译成中文就是普普通通的东西,一看就懂。但是你的口语恐怕提高不大。工作环境是提高口语的最好地方。所以如果集中学一个快的课,省下的时间去工作,是一个好的选择。
总而言之,时间最重要,同样都是一样的文凭,从哪拿都一样,越快越好。但是还是要找质量好的课,好的专业,这个问题下回讨论。

第三回,关于专业的选择,找热门的技能,两个办法,一个是去www.jobfutures.com上去看,那里可以查到各个专业工作的薪资水平,需要的学历,相关的专业,以及就业趋势。另外一个方法更简单,看看报纸上的各个 carrer college和培训学校的广告,那上面列的专业绝对都是比较好就业的专业,因为就业率是各个学校的生命线。没有前景,华而不实的专业绝对不会落入他们的眼睛。不过也不是绝对,career college主要关注初级技能培训和证书考试,对于有些比如医生,律师之类的可能前景很好,但是需要去大学读master. 我对于技工这块不太了解,不过大家可以去www.serviceontario.ca上面看看,在school & work里面,apprenticeship 是政府大力支持的,有很多的服务和培训。有些学校是专门教授这些的,比如常青藤。community college里面也有相关课程。不过这一块更要当心,有些技能淘汰的很快。其他方面,IT方面硬件是没有什么前途的,软件现在的课程可多了,软件测试,数据分析SAS, 网络安全都算热门。不过如果在国内不是IT人士,学这个恐怕已经晚了。还有一些课程,适合不同背景的人学习,就业前景基本可以,在这里可以介绍一下。

首先是电算化会计,其实学出来bookkeeper,不管你学两年的课程,还是6个月的速成,找工作都差不多,因为会计最需要经验,所以没有经验的只有从基层老老实实干起。在国内干过的,还是好找,那些从来没干过的,学了两年,还是难找。不过现在很多小公司找行政员工,如果同时能做些会计工作竞争力会大很多。

平均薪资:会计$15.78左右,bookkeeper 14.8左右,一开始也就拿个10-12块
就业情况:一直都还可以,比平均情况好。但是由于中国移民学这个的太多,所以现在的竞争也非常激烈

性别:女性占了91%,看来男性没什么机会,但是CFA男性多一些。不过要拿时间熬。光想办法考证是不行的,还要有机会进入事务所或大些的公司混些经验。所以如果希望将来能够发展空间大,那就要宁为牛后,不为鸡首。到大的事务所做做义工,合同工可能是必要的。
建议:学这个行业选择一个6-8个月的速成就可以了。

下一部分:幼教助理 Early Childcare Assistant
ECE o和ECA 介绍,
幼儿教育老师和幼儿教育助理,ECE需要两年学习,需要到community College里面学,ECA有很多6个月的短期课程,其实工资差不多,差1、2块钱。这个行业除了diploma外还要有certificate,基本上你在学习期间就可以考过这个证,你就可以工作了。考试费很便宜:50元左右
英语要求:ECA的英语要求不是太高
行业前景:不错,而且自雇比例颇高,达到41%。所以比较适合需要照顾家庭的女性。
平均薪资:12.87.这个工资实在没有吸引力,但是适合自雇,自己当老板赚多少,这里没有统计,但是自在啊。
性别:还是有2%的男性的!,而且男性的竞争力特强!
课程的情况看第二页的帖子

三:私人护理(personal support worker)
其实如果是需要找个挣钱多一些的职业,这个职业比ECA强,首先工资高,其次有了这个专业的diploma一样可以做ECA的工作,因为课程里面有专们如何照顾孩子的。而且学到的知识其实对自己对家庭都很有好处。只是我们中国人有的时候有偏见,觉得去照顾病人,老人,收拾床铺不太好对家中父老交待。说在国外干什么呢,私人护理,都干些啥,收拾屋子,陪老人聊天,照顾病人。人家会觉得你好好的,跑国外干这个。但是我觉得这个职业不错,听我分析一下
就业前景:很不错。整个世界都进入老年社会了。失业率只有1%!。未来中国也很需要,另外,发达社会越来越重视对于老人,病人,disable包括孩子的照顾和护理。其实不仅是照顾吃穿活动那么简单,还有精神上的照顾,属于体验经济的一类,阳光产业。而且这个行业对于自己的性格磨练很有好处,你会变得很有耐心,细心和爱心,听说对于生老病死的看法也会大大改变。自雇和part time的机会也不错,照顾照顾邻里,有40%的人做part time. 平时做义工的也是这个行业比较多。
薪资:16-18也有到20块的,而且,由于从事一些和病人打交道的工作,一般健康福利会比较不错。
工作环境:其实我去过老人院和老人公寓,环境真得很不错。由于工作中动体力的时间较多,所以一般都不会让你连续工作时间太长的。
语言要求:这个工作一般对语言要求较高,不过有一点,会一门外语绝对是一个财富。
发展空间:这个专业实际上是Nurse Aid,发展当然是朝Rigester Nurse了。热门阿。
性别:女性占91%。 但是:听说急需男性阿,虽说有器械帮助,但是还是有些体力活的,所以经常在老人院里和医院里见到五大三粗的女性。还有,男性也有隐私阿,男同志有的时候也想让同性来照顾一下。顺便说一句,男护士也是非常非常受欢迎。
课程介绍,上这个课的教室很有意思,大家有兴趣可以参观。这个课对于实际操作要求很强,经常要演练急救,比如,突然有人晕倒了,怎么处理,学生们七手八脚的上。
这个课必须要西人教师上,因为西人和我们在处理人的问题上首先观念大大不同。西人把安全放在第一位,首先必须安全。所以什么民间土法,没有经过验证的不用。比如脖子不舒服,趁他不注意给他嘎吱一下。所以同样操作,观念不同,处理的方法不同。
课程介绍太长了,包括,心理学的知识,人际沟通,营养学,老人护理,临终关怀,危机处理,家庭护理,收拾房屋,以及相关的法律法规,当然,实习是非常重要的一环
PT: Pharmacy Technician
就业前景:Very Good, 还是和老年社会有关。
薪资水平:如果在零售,10-15块,如果在社区community health care. 10-20块,医院:19-26块起,有些人成为了药剂师,工资约7万/年起,但是要再继续学习获得degree.
性别:男女均衡
语言:要求较高,学习辛苦,因为有大量东西要记,中国人毕业率高,老外经常吃不消。
自雇:自己做生意的占15%。 年收入较高。
学位:diploma, 同时还有行业的几个certificate
学习时间:40周,要学得抓紧,政府马上要把它改成两年的课程了
课程介绍:太长了

Community Service Worker 介绍,请点,就不用费时找了

木尘子 : 2010-10-18#604
回复: 求学选择个人谈及幼教,私人护理,社工,医师行政助理等热门行业介绍

lz你好,我是国内临床医学本科毕业,想去加拿大留学后移民,gpa较低,学校成绩一般,中介建议我读百年理工的一年研究生文凭 职业健康管理专业或者综合健康咨询师,前者带实习,请问这两个专业工作好找吗?哪个好点?毕业后工作一年能申请移民吗?本人对加国护士也很感兴趣,长远想向这个发展,但家庭条件一般,不想花太多钱读书,想将来一边工作一边往上读,不知lz有什么高见可以给我呢?我有几个想法:1)先读百年理工的两年实践护士,找工作后申请移民几年后再读RN, 2)先读一年的职业健康管理,工作后申请移民再读part time的实践护士,或者贷款读full time的实践护士,也有人说可以直接用国内的医学毕业背景转学分读大学护理学的大三大四,本人女,27了,家庭条件一般,不知那条路比较合适?其实如果职业健康管理就业前景好,有深造机会,薪水高,也可以考虑。麻烦解答,不甚感激!!!
读护士应该比较好找工作,老年学之类专业也行,没钱可以申请贷款。2年college毕业也就护士比较高薪。分读大学护理3,4 不错。

Maryliu : 2010-10-19#605
回复: 求学选择个人谈及幼教,私人护理,社工,医师行政助理等热门行业介绍

:wdb17:

学习

synsia : 2010-10-19#606
回复: toronto college介绍,选择,相关问题交流

我读了曼省的要求,只需一学年(8个月即可),我现在就想以最快的速度出去。而且曼省也没说要毕业后在加工作一年以上。

难道我理解错了?
:wdb43:关注。。。

dengsg : 2010-10-19#607
回复: 求学选择个人谈及幼教,私人护理,社工,医师行政助理等热门行业介绍

学习,thanks

laopoxuxu : 2010-10-31#608
回复: 求学选择个人谈及幼教,私人护理,社工,医师行政助理等热门行业介绍

请问楼主,从事按摩工作的话到加国是不是需要再考取什么执照之类的,我在国内是有资格证书!l另外,之前一直做有氧操教练,现在困惑我的是我在网上基本看不到类似招聘信息也听说是需要考取执照的,不知道在什么地方可以看到相关的一些信息呢,非常感谢!

yyz : 2010-11-07#609
回复: toronto college介绍,选择,相关问题交流

有用,谢谢。


上次有TZ问到关于税收抵减的问题,为了尽可能让大家明白加拿大在学生学费方面的税收政策,特地找到了一个注册会计师李金萍女士在网上的文章,非常专业。

税款抵减额与所得扣除额不同之处是在於后者是造成所得的减少,而前者是直接抵减税款。

可以申报的税款抵减额包括:基本个人免税额、高龄免税额、配偶免税额、合格扶养人免税额、CPP供款、EI保费、本身的学费和教育费、医疗费用、慈善费用等。

李金萍解释,本身的学费(tuition fees for self)作为抵减额专案,要求付给加拿大的大学或专科以上的教育机构或就业移民部(The Minister of Employment and Immigration)认证的职业训练机构的学费,每个教育机构学费金额在100元以上者,可享受免税额。

可抵减的学费包括入学申请费、图书馆及实验室费、测验费、强制性的电脑服务费、函授课程?的书籍讲义费、证书及文凭费。另外一些附加费,例如体育及保健服务费(但不包含学生协会活动费)也可以抵减,但一般的膳宿费则不可以抵扣。

另外,李金萍也提醒,国外的大学若所暂册的课程最少连续13周的时间,?为全时间课程,?取得学位者,学费也可能可以抵扣,但要视该大学是否为加拿大税务局所认可的。若某国外大学?不在税务局认可的名单?,需要一定的程式申请。

除了学费,教育费(education amount for self)也可申报。李金萍表示,作为全时间学生(full time student),每月可申报400元的教育费免税额。部分时间学生(part time student)至少连续3星期的课程,每月至少12个小时,每月的免税额是120元。2006年又新增的一项教科书减免额(textbook tax credit)规定:凡符合领取全时间(full time)教育款项的学生,有每月65元的减免额,部分时间(part time)学生减免额为每月20元。

若学生本身不需要使用全部的学费、教育费用和教科书抵减时,剩余的部分可先转给配偶、父母或祖父母,但一个子女可转移的学费、教育费用和教科书费用不能超过5,000

李金萍也指出,子女请家教或补习班的费用是不可以抵扣的;学习钢琴、才艺等非运动型的学费也不可以抵扣,但若才艺音乐学校开具T2202 T2202A的学费收据则可以申报。
问:我在2003年没有收入,不需要交税。所以,我当年的学费不用报税,等以后有收入时再报税可以吗?

答:不妥当。正常的方法是:将学费的税表T2202T2202(A)及时报税,税务局会给你一份Notice of Assessment,列出你还有多少金额没有使用,这些金额可以在自己有收入的时候抵税。也可以转给配偶、父母、祖父母抵税使用。有的纳税人仅用收据或支票存根报税,而没有拿到T2202T2202(A),吃亏很多。因为,按规定,每上一个月的Full-Time 课程,增加$400抵税额;每上一个月的Part-Time,增加$120抵税额。
举例说明:王先生在2003年读Full-Time课程五个月,学费$4,000。同年,又上Part-Time课程两个月,学费$1,000。如果仅以收据,即实际支付金额计算,可以抵税的金额为$5,000正确的抵税金额计算为:$4,000 + 1,000 + 400 x 5 + 120 x 2 = 7,240
若以最低税率22%计算,损失的税款为:(7,240-5000) X 22% = 493

jr-canada : 2010-11-13#610
回复: 求学选择个人谈及幼教,私人护理,社工,医师行政助理等热门行业介绍

好贴留印,以后好好学习

weina5706 : 2010-11-22#611
回复: 求学选择个人谈及幼教,私人护理,社工,医师行政助理等热门行业介绍

有人在centennial 学psw, 我已经报了 9月2011 上 又一起的吗 我们可以讨论下 加我 qq 909099349

weina5706 : 2010-11-22#612
回复: 求学选择个人谈及幼教,私人护理,社工,医师行政助理等热门行业介绍

我打算学完psw 就上part time 继续学 RPN

havefun : 2010-11-23#613
回复: 求学选择个人谈及幼教,私人护理,社工,医师行政助理等热门行业介绍

Alex 还来这里吗?

要进银行做客户代表需要什麽敲门砖?

是csc 吗?

xun003 : 2010-11-28#614
回复: toronto college介绍,选择,相关问题交流

三:私人护理(personal support worker)
课程介绍太长了,需要的留下邮件。包括,心理学的知识,人际沟通,营养学,老人护理,临终关怀,危机处理,家庭护理,收拾房屋,以及相关的法律法规,当然,实习是非常重要的一环

先谢谢楼主。。麻烦发一下课程到cindy12_10@hotmail.com

tina1966 : 2010-11-30#615
回复: 求学选择个人谈及幼教,私人护理,社工,医师行政助理等热门行业介绍

请教各位,我想去学PSW,除了VICTORIA外,还有其他的学校么?

tina1966 : 2010-11-30#616
回复: 求学选择个人谈及幼教,私人护理,社工,医师行政助理等热门行业介绍

谢谢楼主。。麻烦发一下PSW课程到tinawtj@hotmail.com

我的快乐人生 : 2010-12-01#617
回复: 求学选择个人谈及幼教,私人护理,社工,医师行政助理等热门行业介绍

请教楼主,我准备去加拿大学习自动控制,哪个大学比较好

andrea000 : 2010-12-03#618
回复: 求学选择个人谈及幼教,私人护理,社工,医师行政助理等热门行业介绍

LZ好,
本人大学日语专业,毕业后做过一点时间翻译,后来考的会计初级。这两年在做会计,听朋友说,在加拿大要是干会计的话得一直考,考过CGA再过CA...貌似不是3或5年就能够挨过的。。。。所以,如果选别的专业有啥好的建议,我比较倾向于护理,但是据说也是个专业性很强的选择。。。请大虾指点!!

win1122ying : 2010-12-09#619
回复: 求学选择个人谈及幼教,私人护理,社工,医师行政助理等热门行业介绍

好帖、谢谢

rabbit775 : 2010-12-09#620
回复: 求学选择个人谈及幼教,私人护理,社工,医师行政助理等热门行业介绍

LZ这样分析的背景是不是针对新移民的?那些还没有成功移民的是否可以通过这种方式进入加国?他们能够申请到留学签证吗?

emptyfly : 2010-12-12#621
回复: 求学选择个人谈及幼教,私人护理,社工,医师行政助理等热门行业介绍

帖子写的很详尽,对于College以及就业等介绍的很全面~

看样子只能走IT民工的老路了

yzdy2009 : 2010-12-12#622
回复: 求学选择个人谈及幼教,私人护理,社工,医师行政助理等热门行业介绍

留脚印

至尊寶 : 2010-12-16#623
回复: toronto college介绍,选择,相关问题交流

自己先顶着,还要接着写呢
说真的写的真是太好太全(我觉得很全面了再写估计得写本类似指南的书了)了
能否讲下BCIT这个学校呢?:wdb17:

caroora : 2010-12-18#624
回复: toronto college介绍,选择,相关问题交流

本人来加拿大几年了,只想告诉大家,千万别相信这个LZ所写的。她的目的就是想大家掏钱念她所在的私立学校,她学校有开的课程。想不到这么多成年人这么轻信于人!
能给出证据吗?
似乎LZ 没有推荐某个具体的学校,而且还建议上网查,上正规定私立大专。能够详细解释下?

结束 : 2011-01-04#625
回复: toronto college介绍,选择,相关问题交流

不是很了解,不过感觉不是很大就业空间,如果是做delivery倒是可以,长途货运的薪水不错,只是很辛苦,而且要花一、两年时间考牌
请问楼主长途货运要考什么牌呢?

肯尼迪学院 : 2011-01-05#626
回复: toronto college介绍,选择,相关问题交流

货车司机牌,具体请查阅相关驾校。
请问楼主长途货运要考什么牌呢?

bettysu : 2011-01-06#627
回复: 求学选择个人谈及幼教,私人护理,社工,医师行政助理等热门行业介绍

mark了

synsia : 2011-01-09#628
回复: 求学选择个人谈及幼教,私人护理,社工,医师行政助理等热门行业介绍

请问LZ有关于特教方面的信息吗?谢过先。。。

WhiteDexter : 2011-01-10#629
回复: 求学选择个人谈及幼教,私人护理,社工,医师行政助理等热门行业介绍

楼主知不知道做了护工后,难道就没有上升空间了,做一辈子的护工,能不能接着读做护士什么的

olina1127 : 2011-01-18#630
回复: 求学选择个人谈及幼教,私人护理,社工,医师行政助理等热门行业介绍

多多学习

茜茜公主 : 2011-01-18#631
回复: 求学选择个人谈及幼教,私人护理,社工,医师行政助理等热门行业介绍

好贴,留个脚印

阿砍 : 2011-01-20#632
回复: 求学选择个人谈及幼教,私人护理,社工,医师行政助理等热门行业介绍

受益匪浅啊
有没有同学知道木匠专业读完了好不好找工作啊?:wdb2:

tomhwcheung : 2011-01-26#633
回复: 求学选择个人谈及幼教,私人护理,社工,医师行政助理等热门行业介绍

请问知道social worker / social service worker两者有什么不同吗?

我本科毕业,看到York university, toronto university有2年的社工硕士,值得报读吗? 是不是选择读硕士比较容易移民??

mogusaen : 2011-02-01#634
回复: 求学选择个人谈及幼教,私人护理,社工,医师行政助理等热门行业介绍

学习了解

niki208 : 2011-02-01#635
回复: 求学选择个人谈及幼教,私人护理,社工,医师行政助理等热门行业介绍

mark

jialumama : 2011-02-10#636
回复: 求学选择个人谈及幼教,私人护理,社工,医师行政助理等热门行业介绍

很好,谢谢!

jmouse : 2011-02-26#637
回复: 求学选择个人谈及幼教,私人护理,社工,医师行政助理等热门行业介绍

留印

西安男孩 : 2011-03-03#638
回复: 求学选择个人谈及幼教,私人护理,社工,医师行政助理等热门行业介绍

楼主真是个热心人,在些先谢谢了,我也有问题想请教:
1、加拿大的college里,都提供有哪些建筑方面的program呢?比如说,助理建筑师、顾问建筑师等;
2、学制一般都是多久的?有1年以下的吗?学费贵吗?
3、提供的文凭是certificate,diploma,还是degree?
4、以后要上univercityr攻读master的话,这些学分能获得承认吗?
5、请问由college提供的建筑方面的program的学生,就业率和薪金怎么样呢?
谢谢

lily2011 : 2011-03-04#639
回复: 求学选择个人谈及幼教,私人护理,社工,医师行政助理等热门行业介绍

看了全文,有所收获,有所改观!

Tianhai : 2011-03-06#640
回复: 求学选择个人谈及幼教,私人护理,社工,医师行政助理等热门行业介绍

学习

欣欣妈妈 : 2011-03-06#641
回复: 求学选择个人谈及幼教,私人护理,社工,医师行政助理等热门行业介绍

ddddddd

tomhwcheung : 2011-03-10#642
回复: 求学选择个人谈及幼教,私人护理,社工,医师行政助理等热门行业介绍

很老的帖子了,楼主还在吗??

请问楼主知道social worker / social service worker两个有什么不同吗?

我本科毕业,看到York university, toronto university有2年的社工硕士,值得报读吗? 另外,多伦多地区还有那里有社工读的??

无限感谢了。

jessicatan : 2011-03-22#643
回复: 求学选择个人谈及幼教,私人护理,社工,医师行政助理等热门行业介绍

同问!我花了三个小时拜读完这个帖子,受益匪浅。夏天就要登陆了,正处于迷茫中

cherry1029 : 2011-03-23#644
回复: 求学选择个人谈及幼教,私人护理,社工,医师行政助理等热门行业介绍

欣喜的发现了楼主的好帖,麻烦发一份私人护理的课程到haiying1029@yahoo.com谢谢

淡若轻风 : 2011-03-27#645
回复: 求学选择个人谈及幼教,私人护理,社工,医师行政助理等热门行业介绍

留印

xuniuu : 2011-04-16#646
回复: 求学选择个人谈及幼教,私人护理,社工,医师行政助理等热门行业介绍


呀,你也来了啊。

Saffi : 2011-04-19#647
回复: 求学选择个人谈及幼教,私人护理,社工,医师行政助理等热门行业介绍

有用

XIAOXIANGMA : 2011-04-22#648
回复: 求学选择个人谈及幼教,私人护理,社工,医师行政助理等热门行业介绍

不错!

luaisun : 2011-04-22#649
回复: 求学选择个人谈及幼教,私人护理,社工,医师行政助理等热门行业介绍

记号

枫叶梦梦 : 2011-04-22#650
回复: 求学选择个人谈及幼教,私人护理,社工,医师行政助理等热门行业介绍

back up

枫叶梦梦 : 2011-04-22#651
回复: 求学选择个人谈及幼教,私人护理,社工,医师行政助理等热门行业介绍

ths al ot

哈娃歌恰 : 2011-05-05#652
回复: toronto college介绍,选择,相关问题交流

应你要求,先讲这个:
PT: Pharmacy Technician
就业前景:Very Good, 还是和老年社会有关。
薪资水平:如果在零售,10-15块,如果在社区community health care. 10-20块,医院:19-26块起,有些人成为了药剂师,工资约7万/年起,但是要再继续学习获得degree.
性别:男女均衡
语言:要求较高,学习辛苦,因为有大量东西要记,中国人毕业率高,老外经常吃不消。
自雇:自己做生意的占15%。 年收入较高。
学位:diploma, 同时还有行业的几个certificate
学习时间:40周,要学得抓紧,政府马上要把它改成两年的课程了
课程介绍:太长了,需要的给mail.


你好,想了解一下课程 musicandcolour@yahoo.com.cn
谢谢!!:wdb37:

海在那边 : 2011-06-05#653
回复: 求学选择个人谈及幼教,私人护理,社工,医师行政助理等热门行业介绍

想了解蒙特利尔哪里有学早教助理ECA的?哪位知道请给些信息,不胜感激!

花哥奶油茶 : 2011-06-09#654
回复: 求学选择个人谈及幼教,私人护理,社工,医师行政助理等热门行业介绍

back up...

christing : 2011-06-23#655
回复: 求学选择个人谈及幼教,私人护理,社工,医师行政助理等热门行业介绍

想知道当钢琴老师都需要什么资格。

wangwei520 : 2011-06-26#656
回复: 求学选择个人谈及幼教,私人护理,社工,医师行政助理等热门行业介绍

首先顶下。拜谢楼主,楼主你好请教,我还在国内,是一名护士,我想去那做住家护理的工作,可不知道能行?谢谢

FANNY.LING : 2011-07-01#657
回复: 求学选择个人谈及幼教,私人护理,社工,医师行政助理等热门行业介绍

:wdb10:好贴,顶起,收藏,向LZ致谢

平凡的生活不平淡 : 2011-07-15#658
回复: 求学选择个人谈及幼教,私人护理,社工,医师行政助理等热门行业介绍

一口气看到这儿,真是受益匪浅。我是市场营销专业和旅游管理专业的大学老师,请问加国的旅游市场如何?谢谢!祝移友们一切顺利!

微风经过 : 2011-07-20#659
回复: 求学选择个人谈及幼教,私人护理,社工,医师行政助理等热门行业介绍

好贴,关注

醒之外 : 2011-07-27#660
回复: 求学选择个人谈及幼教,私人护理,社工,医师行政助理等热门行业介绍

好贴

jingnan0229 : 2011-08-06#661
回复: 求学选择个人谈及幼教,私人护理,社工,医师行政助理等热门行业介绍

顶楼主!

请问楼主private career college 里面几个月的diploma课程,国际学生来读的话,读完的话有办法申请移民吗?

trueak : 2011-08-20#662
回复: 求学选择个人谈及幼教,私人护理,社工,医师行政助理等热门行业介绍

非常感谢楼主的分享,受益了

siemen95 : 2011-08-25#663
回复: toronto college介绍,选择,相关问题交流

上次有TZ问到关于税收抵减的问题,为了尽可能让大家明白加拿大在学生学费方面的税收政策,特地找到了一个注册会计师李金萍女士在网上的文章,非常专业。

税款抵减额与所得扣除额不同之处是在於后者是造成所得的减少,而前者是直接抵减税款。

可以申报的税款抵减额包括:基本个人免税额、高龄免税额、配偶免税额、合格扶养人免税额、CPP供款、EI保费、本身的学费和教育费、医疗费用、慈善费用等。

李金萍解释,本身的学费(tuition fees for self)作为抵减额专案,要求付给加拿大的大学或专科以上的教育机构或就业移民部(The Minister of Employment and Immigration)认证的职业训练机构的学费,每个教育机构学费金额在100元以上者,可享受免税额。

可抵减的学费包括入学申请费、图书馆及实验室费、测验费、强制性的电脑服务费、函授课程?的书籍讲义费、证书及文凭费。另外一些附加费,例如体育及保健服务费(但不包含学生协会活动费)也可以抵减,但一般的膳宿费则不可以抵扣。

另外,李金萍也提醒,国外的大学若所暂册的课程最少连续13周的时间,?为全时间课程,?取得学位者,学费也可能可以抵扣,但要视该大学是否为加拿大税务局所认可的。若某国外大学?不在税务局认可的名单?,需要一定的程式申请。

除了学费,教育费(education amount for self)也可申报。李金萍表示,作为全时间学生(full time student),每月可申报400元的教育费免税额。部分时间学生(part time student)至少连续3星期的课程,每月至少12个小时,每月的免税额是120元。2006年又新增的一项教科书减免额(textbook tax credit)规定:凡符合领取全时间(full time)教育款项的学生,有每月65元的减免额,部分时间(part time)学生减免额为每月20元。

若学生本身不需要使用全部的学费、教育费用和教科书抵减时,剩余的部分可先转给配偶、父母或祖父母,但一个子女可转移的学费、教育费用和教科书费用不能超过5,000

李金萍也指出,子女请家教或补习班的费用是不可以抵扣的;学习钢琴、才艺等非运动型的学费也不可以抵扣,但若才艺音乐学校开具T2202 T2202A的学费收据则可以申报。
问:我在2003年没有收入,不需要交税。所以,我当年的学费不用报税,等以后有收入时再报税可以吗?

答:不妥当。正常的方法是:将学费的税表T2202T2202(A)及时报税,税务局会给你一份Notice of Assessment,列出你还有多少金额没有使用,这些金额可以在自己有收入的时候抵税。也可以转给配偶、父母、祖父母抵税使用。有的纳税人仅用收据或支票存根报税,而没有拿到T2202T2202(A),吃亏很多。因为,按规定,每上一个月的Full-Time 课程,增加$400抵税额;每上一个月的Part-Time,增加$120抵税额。
举例说明:王先生在2003年读Full-Time课程五个月,学费$4,000。同年,又上Part-Time课程两个月,学费$1,000。如果仅以收据,即实际支付金额计算,可以抵税的金额为$5,000正确的抵税金额计算为:$4,000 + 1,000 + 400 x 5 + 120 x 2 = 7,240
若以最低税率22%计算,损失的税款为:(7,240-5000) X 22% = 493
有用,留个记号

AMYER : 2011-08-31#664
回复: 求学选择个人谈及幼教,私人护理,社工,医师行政助理等热门行业介绍

感谢分享!

好妖妖 : 2011-09-12#665
回复: 求学选择个人谈及幼教,私人护理,社工,医师行政助理等热门行业介绍

不错

han168 : 2011-09-15#666
回复: 求学选择个人谈及幼教,私人护理,社工,医师行政助理等热门行业介绍

请教前辈:我现住在NB省,我咨询了一下community college的practical nurse 和 personal support的入学申请感觉好难,排队等着入学的人太多 ,2012年9月学期,已经有200人在排队等着进去了,像我这样年近40的学生,真不知道这条路可不可以走得通??请有经验的同学支个招,有没有什么办法能快速读上这专业啊?在多伦多或者加拿大的其他省份情况怎样?是不是也要排着队申请这专业啊?

meiyangyang : 2011-09-15#667
回复: 求学选择个人谈及幼教,私人护理,社工,医师行政助理等热门行业介绍

明年想去读私人护理,想问一下TZ们,现在学这个还好找工作吗?请了解的朋友说一下,我住在安省一个小城市。

charlenemio : 2011-09-16#668
回复: 求学选择个人谈及幼教,私人护理,社工,医师行政助理等热门行业介绍

我做留学顾问多年,替学生找到专业学校就读.
无论是看护 护士 彩妆 美甲 SPA 幼教 视觉动画…都可以让学生在当地获得高水平的薪资.如果有兴趣想就读一技之长的专业的同学,我们提供联机免费谘询.
长年下来,我们帮助了很多学生,
?迎各位?考 罪士http://tw.myblog.yahoo.com/lpet88/article?mid=89
彩?/美甲http://tw.myblog.yahoo.com/lpet88/article?mid=958
??http://tw.myblog.yahoo.com/lpet88/article?mid=2078
幼教http://tw.myblog.yahoo.com/lpet88/article?mid=640
欢迎您来询问,
MSN: glc_vancouver@hotmail.com
QQ:1502346962

flyingahead : 2011-09-20#669
回复: 求学选择个人谈及幼教,私人护理,社工,医师行政助理等热门行业介绍

看到17页,收益颇丰,mark下。

fenhaitang : 2011-09-22#670
回复: 求学选择个人谈及幼教,私人护理,社工,医师行政助理等热门行业介绍

来家园时间也不短了,今天才发现这么好的一篇帖子。向楼主致敬!

Smilecheese : 2011-09-26#671
回复: 求学选择个人谈及幼教,私人护理,社工,医师行政助理等热门行业介绍

好贴,先收藏了!

暖洋洋2010 : 2011-09-26#672
回复: 求学选择个人谈及幼教,私人护理,社工,医师行政助理等热门行业介绍

收藏了,谢谢楼主

Ajing : 2011-10-03#673
回复: 求学选择个人谈及幼教,私人护理,社工,医师行政助理等热门行业介绍

好贴收藏学习!&送花花

petersonswh : 2011-10-10#674
回复: 求学选择个人谈及幼教,私人护理,社工,医师行政助理等热门行业介绍

vice nice information. Thanks a lot.

1tymlp : 2011-10-26#675
回复: 求学选择个人谈及幼教,私人护理,社工,医师行政助理等热门行业介绍

很棒,受益匪浅!

lisazhang : 2011-11-01#676
回复: 求学选择个人谈及幼教,私人护理,社工,医师行政助理等热门行业介绍

楼主,刚登陆,想学个会计短期的,公立和私立的大专,都分为certificate 和diploma两种,学费和所需时间都差很多的,想问两者在找工作的时候,区别大吗?两者还有哪些不同的地方,能详细的介绍一下吗?

david_yunke : 2011-11-20#677
回复: 求学选择个人谈及幼教,私人护理,社工,医师行政助理等热门行业介绍

"医师行政助理"具体是的工作职责是什么?
这个应该怎么申请?

maggie_li : 2011-11-29#678
回复: 求学选择个人谈及幼教,私人护理,社工,医师行政助理等热门行业介绍

very uesful information from Alexwei....Thank you ^_^

make a mark for myself when someday i need it .

franco : 2011-12-06#679
回复: 求学选择个人谈及幼教,私人护理,社工,医师行政助理等热门行业介绍

请教楼主:2nd career:police foundation方面的就业前景如何?

xiangxiangge : 2011-12-13#680
回复: toronto college介绍,选择,相关问题交流

上次有TZ问到关于税收抵减的问题,为了尽可能让大家明白加拿大在学生学费方面的税收政策,特地找到了一个注册会计师李金萍女士在网上的文章,非常专业。

税款抵减额与所得扣除额不同之处是在於后者是造成所得的减少,而前者是直接抵减税款。


可以申报的税款抵减额包括:基本个人免税额、高龄免税额、配偶免税额、合格扶养人免税额、CPP供款、EI保费、本身的学费和教育费、医疗费用、慈善费用等。


李金萍解释,本身的学费(tuition fees for self)作为抵减额专案,要求付给加拿大的大学或专科以上的教育机构或就业移民部(The Minister of Employment and Immigration)认证的职业训练机构的学费,每个教育机构学费金额在100元以上者,可享受免税额。


可抵减的学费包括入学申请费、图书馆及实验室费、测验费、强制性的电脑服务费、函授课程?的书籍讲义费、证书及文凭费。另外一些附加费,例如体育及保健服务费(但不包含学生协会活动费)也可以抵减,但一般的膳宿费则不可以抵扣。


另外,李金萍也提醒,国外的大学若所暂册的课程最少连续13周的时间,?为全时间课程,?取得学位者,学费也可能可以抵扣,但要视该大学是否为加拿大税务局所认可的。若某国外大学?不在税务局认可的名单?,需要一定的程式申请。


除了学费,教育费(education amount for self)也可申报。李金萍表示,作为全时间学生(full time student),每月可申报400元的教育费免税额。部分时间学生(part time student)至少连续3星期的课程,每月至少12个小时,每月的免税额是120元。2006年又新增的一项教科书减免额(textbook tax credit)规定:凡符合领取全时间(full time)教育款项的学生,有每月65元的减免额,部分时间(part time)学生减免额为每月20元。


若学生本身不需要使用全部的学费、教育费用和教科书抵减时,剩余的部分可先转给配偶、父母或祖父母,但一个子女可转移的学费、教育费用和教科书费用不能超过5,000


李金萍也指出,子女请家教或补习班的费用是不可以抵扣的;学习钢琴、才艺等非运动型的学费也不可以抵扣,但若才艺音乐学校开具T2202 T2202A的学费收据则可以申报。

问:我在2003年没有收入,不需要交税。所以,我当年的学费不用报税,等以后有收入时再报税可以吗?

答:不妥当。正常的方法是:将学费的税表T2202T2202(A)及时报税,税务局会给你一份Notice of Assessment,列出你还有多少金额没有使用,这些金额可以在自己有收入的时候抵税。也可以转给配偶、父母、祖父母抵税使用。有的纳税人仅用收据或支票存根报税,而没有拿到T2202T2202(A),吃亏很多。因为,按规定,每上一个月的Full-Time 课程,增加$400抵税额;每上一个月的Part-Time,增加$120抵税额。
举例说明:王先生在2003年读Full-Time课程五个月,学费$4,000。同年,又上Part-Time课程两个月,学费$1,000。如果仅以收据,即实际支付金额计算,可以抵税的金额为$5,000正确的抵税金额计算为:$4,000 + 1,000 + 400 x 5 + 120 x 2 = 7,240
若以最低税率22%计算,损失的税款为:(7,240-5000) X 22% = 493
很好,收藏

满陇桂雨 : 2011-12-15#681
回复: 求学选择个人谈及幼教,私人护理,社工,医师行政助理等热门行业介绍

好帖。
刚发现,一口气读完。
俺给楼主加评分了。

haohaojunjun : 2011-12-29#682
回复: 求学选择个人谈及幼教,私人护理,社工,医师行政助理等热门行业介绍

up

醒之外 : 2012-01-11#683
回复: 求学选择个人谈及幼教,私人护理,社工,医师行政助理等热门行业介绍

请教楼主:本人发展与教育心理学专业,即将来加,不介意转行,不知道该读什么专业好找工作?我担心原来专业不好就业,请指教

juliezl_us : 2012-01-11#684
回复: 求学选择个人谈及幼教,私人护理,社工,医师行政助理等热门行业介绍

跟lz咨询一下 educational assistant或者 teacher assistant这类似的专业就业情况怎么样 本人是英国的business management的学士学位,但在国内一直在高校里从事教务管理的工作,刚登陆,想继续做这个专业的事。直接就业有困难,就打算读一下college的diploma课程。看了lz的帖子后,想听一下你的建议。谢谢

athena0413 : 2012-01-17#685
回复: 求学选择个人谈及幼教,私人护理,社工,医师行政助理等热门行业介绍

ALEX,你好。看了你的帖子很受用。想向您询问一点建议
我准备今年到加拿大留学,而后准备移民的事情。
我的资料如下:

女,25岁。国内普通二本大学,动植物检疫本科毕业。GPA80分左右。雅思暂时为6分,还准备再考,目标分为6.5分。
毕业后工作三年,其中有将近两年时间在国内某大学的图书馆里做行政老师。

如果我想到加国留学的话,我能怎么选专业呢?(我自己没有特别喜欢的,只是不想再学原专业,动植物检疫,太不合适自己也不感兴趣。)
从毕业后找工作方面来说。。?
从方便以后移民来说?

真心的感谢你的宝贵意见,期待你的回复,谢谢,

tana : 2012-01-18#686
回复: 求学选择个人谈及幼教,私人护理,社工,医师行政助理等热门行业介绍

谢谢分享!

zjniniko : 2012-01-21#687
回复: 求学选择个人谈及幼教,私人护理,社工,医师行政助理等热门行业介绍

楼主您好,请您看看我这种情况是否可以办理移民,谢谢~

本人清华大学艺术设计4年本科毕业,现27岁,目前国内全职工作3年,工作种类为NOC的5241 - Graphic Designers and Illustrators .孩子1岁8个月,老公也是本校本科毕业,毕业后工作1年,现在美国读书。

我想通过去加拿大读书,然后申请移民。自从有了孩子,就想读个幼教类的专业,之前也带过几个4-8岁的小朋友,教他们美术。不知道我这种条件是否可以读这个专业?

还有看您的文章中写到,这种幼教类的,都是没有文凭的,请问有没有有文凭的但是还是这种幼教专业呢?请问如果同样的幼教专业,有文凭的和这种培训类的,哪种更容易就业呢?谢谢~~

我之前听人说,如果通过读书来办理移民有2个方式:
1.读书取得硕士学位(2年)后,不需要工作经验直接申请移民;
2.某些省提名项目可以读研究生文凭课程(1年)+半年的工作经验,然后申请移民。

请问这种幼教专业,属于上面的研究生课程么?谢谢~~

lauter : 2012-01-25#688
回复: 求学选择个人谈及幼教,私人护理,社工,医师行政助理等热门行业介绍

看到本帖Alex和其他很多热心人,感觉暖暖的。

本人的情况是,国内硕士,30岁,现在大学作行政,近2年;之前在另一所大学有近3年兼职(授课)经验;雅思A类6.5,如果考G类,应该可以更高些吧;现在想申请技术移民,仔细研究了下,好像不太符合教师这一项;

或者先申请加国大专,读一年或是两年,申请经验类移民。

期望回复,祝大家新春快乐!

wlm : 2012-01-26#689
回复: 求学选择个人谈及幼教,私人护理,社工,医师行政助理等热门行业介绍

试下远程教育吧。我就是一名兼职Early Childhood Education的远程教育学生,全职daycare老师,半职house wife. 很充实但没太大压力!

学校有入学考试的。

我不清楚别的科目,就ECE来说,在BC ECE Registry的网页上有列有被认可设有ECE专业的院校。

我认为加拿大远程教育的认可度跟全职出来差不了多少。我身边有亲戚朋友都通过远程教育升值,有学会计bachelor degree的(Athabasca University),教育master的(SFU),高校管理 master的(U of Calgary)。

我们都是在职的,对写assignment很有帮助,因为都是理论与实践经验相结合的。

我做留学顾问多年,替学生找到专业学校就读.
无论是看护 护士 彩妆 美甲 SPA 幼教 视觉动画…都可以让学生在当地获得高水平的薪资.如果有兴趣想就读一技之长的专业的同学,我们提供联机免费谘询.
长年下来,我们帮助了很多学生,
?迎各位?考 罪士http://tw.myblog.yahoo.com/lpet88/article?mid=89
彩?/美甲http://tw.myblog.yahoo.com/lpet88/article?mid=958
??http://tw.myblog.yahoo.com/lpet88/article?mid=2078
幼教http://tw.myblog.yahoo.com/lpet88/article?mid=640
欢迎您来询问,
MSN: glc_vancouver@hotmail.com
QQ:1502346962

我是06年申请移民的,现在还在等ME,想请问下,在国内能远程上那里的大学课程吗?我也想学幼教专业或教师专业的课程,MBA有没有教师方面的课程?想1-2年内学完的.有知道的家园朋友请给我信息bcyjs@yahoo.com.cn,谢谢祝大家新年快乐!

46444774 : 2012-01-27#690
回复: 求学选择个人谈及幼教,私人护理,社工,医师行政助理等热门行业介绍

鲜花送上

ellenellen : 2012-01-30#691
回复: 求学选择个人谈及幼教,私人护理,社工,医师行政助理等热门行业介绍

多谢lz热心指点,学习到许多求学方面的知识:wdb37::wdb17:。想请教下:本人英语本科毕业,在高校长期从事教务管理和成人岗位培训工作,业余也教授雅思课程,仍希望以后继续从事教务管理类似工作或是esl英语教师方面的工作,不知先读college的那类专业(最好时间短一点、能做敲门砖就可)有助于将来求职。初来乍到,还望lz细细赐教。先谢了!

牛妹丁她妈 : 2012-02-04#692
回复: 求学选择个人谈及幼教,私人护理,社工,医师行政助理等热门行业介绍

LZ好久没来,我办理工签中,花的时间太久由黑发变成了白发心情沉到谷底。看到你的帖子就像一缕阳光感到些许欣慰。

忐忑 : 2012-02-04#693
回复: 求学选择个人谈及幼教,私人护理,社工,医师行政助理等热门行业介绍

好人+好贴。留个记号

雪是热的 : 2012-02-06#694
回复: 求学选择个人谈及幼教,私人护理,社工,医师行政助理等热门行业介绍

好贴!

lty_1982 : 2012-02-29#695
回复: 求学选择个人谈及幼教,私人护理,社工,医师行政助理等热门行业介绍

楼主好:
我现在登陆多伦多准备3月回国,准备把国内的事情处理完再长登多伦多,同时利用在国内的时间申请college这样行吗?国内应用化学硕士毕业,如果想在这面读个化学相关的college,不知道那个学校比较好?想知道这方面的就业待遇怎么样?如果实在不行转行的话,读什么专业比较好?希望大家多提宝贵意见,小弟万分感谢!

tangrisong : 2012-03-07#696
回复: 求学选择个人谈及幼教,私人护理,社工,医师行政助理等热门行业介绍

好贴,可惜楼主不露面了。还是要顶一下。

Lornalo : 2012-03-29#697
回复: toronto college介绍,选择,相关问题交流

I met a friend today, he is studying in a PSW program and almost done. He has got a $21/hours work and plan to get Register Nurse degree in the past three years. He offer me a useful info that U.S. hospitals often come to Toronto hiring Register nurses and the salary will be over $100000/year with other benefit such as free apartment. To start a career in nurse you need five years study(including at least one years work experience.
LZ, 我有个疑问请教一下你:现在加拿大医院招聘护士有年龄要求吗?估计年轻人会比较吃香吧?如果我现在开始一年的PSW program再加上五年在护士方面的学习和实践,俺都快接近四十啦。你觉得像我这样对医学知识一片空白的人计划重头开始学护理啊当护士的还值得吗?而且我还计划怀宝宝呢:wdb14:给你加分分了:wdb10:

Lornalo : 2012-03-29#698
回复: toronto college介绍,选择,相关问题交流

对啊,刚刚发现发悄悄话给LZ的选项也没了。。。希望有哪位大侠可以挺身而出为新人指点迷津(关于楼上的问题),先谢过了!!

snow_le : 2012-04-04#699
回复: 求学选择个人谈及幼教,私人护理,社工,医师行政助理等热门行业介绍

赞一个,好贴!

爱咖啡滴猫 : 2012-04-06#700
回复: toronto college介绍,选择,相关问题交流

LZ, 我有个疑问请教一下你:现在加拿大医院招聘护士有年龄要求吗?估计年轻人会比较吃香吧?如果我现在开始一年的PSW program再加上五年在护士方面的学习和实践,俺都快接近四十啦。你觉得像我这样对医学知识一片空白的人计划重头开始学护理啊当护士的还值得吗?而且我还计划怀宝宝呢:wdb14:给你加分分了:wdb10:

应该没有,实习的时候碰到很多实习护士都是年纪挺大的,俺最初总以为是资深护士,其实还是学生

nataliehahaha : 2012-06-04#701
回复: toronto college介绍,选择,相关问题交流

介绍一下社会工作者行业
国内没有社工这个行业,不过大家应该清楚这个是干什么的,我们遇到的settlement Counselor, Employment Counselor都是属于这个行业。这个行业对于我们移民来说有一个优势,我们会1门外语,而且是目前最大的移民群体的外语,各个机构都在扩展对于中国移民的服务。这样子就需要我们了。社工为什么要拓展服务呢,大家知道,no profit机构的钱来自于政府,企业或个人捐助,自我经营收入,其中政府最多,而政府的钱是要根据你做了多少事情来得,比如政府今年有7个亿是帮助新移民的,会分配到各个非营利机构,这些机构就要做一些针对新移民的服务来申请经费,同时还要有效果,如果没有政府的funding, 这个机构就要关门了。所以从事这个行业的人往往有marketing、工商管理,教师的背景,然后获得了本地学历。那么这个行业有多大呢,这个行业非常大,和商业基本上快差不多了。那些机构属于no-profit, 医院,学校,就业、安居服务机构,慈善机构,老人服务机构,绿色和平组织,环境和动物保护组织等。他的行业自成体系,你在一般的招聘网站或者agency那里很少看到招聘的信息。需要什么样的人呢,其实和商业一样,需要sales, accounting, administration, customer service, technician.不一样的地方,核心的服务人员是社工,教师,或者医务人员。大家可以去www.charityvillige.com 上去看,这是加拿大最大的非营利机构的招聘网站。
就业前景:不错
薪资水平:16-20左右。
性别:女性居多
个人奋斗方向:如果成为专业social worker, 工资可以达到70000,但是一般需要有social science master degree. 而 bachelor 毕业实际上和college毕业薪资差别不是太大,我个人觉得可以在拿到diploma后工作两年,直接申请master(用国内的学历)+diploma+工作经验,这样不错。
课程介绍
• Introduction to Community Service Worker
• Social Welfare, Diversity and Awareness
• The Family
• Couples, Family and Interpersonal Communication
• Oriented Case Management
• Environmental Issues and Crisis Interventions and
Prevention
• Bereavement and the Elderly
• Respond to Abuse
• Understanding Human Behavior
• Communication Skills
• Counseling Skills
• Report Writing for the CSW
• Field placement

By the end of the course, students will be able to:
• Demonstrate knowledge of the range of social and community service organizations in their own community as well as detailed knowledge of an agency.
• Practice the role of a team member within a social and community service organization.
• Demonstrate beginning level professional skills
• Apply theory and skills learned in the independent study components of this course.


非常感谢楼主的介绍,对这个专业挺感兴趣的,有益于自身,还可以帮助到别人,挺好的,请问楼主知道这个专业去哪个学校读diploma或者degree或者master比较好吗,或者有没有有关社工专业的介绍学校、课程的网站或者比较好的学校的网站可以推荐下?个人觉得如果读diploma会不会上来比较难就业呢?要不要从bachelor开始起步,国内读了两个bachelor,一个是通信,一个是编辑出版学的第二学位,和这个完全没有关系,不能申请社工专业的硕士吧,难道还要读4年的本科?

夏天D : 2012-06-05#702
回复: toronto college介绍,选择,相关问题交流

非常感谢楼主的介绍,对这个专业挺感兴趣的,有益于自身,还可以帮助到别人,挺好的,请问楼主知道这个专业去哪个学校读diploma或者degree或者master比较好吗,或者有没有有关社工专业的介绍学校、课程的网站或者比较好的学校的网站可以推荐下?个人觉得如果读diploma会不会上来比较难就业呢?要不要从bachelor开始起步,国内读了两个bachelor,一个是通信,一个是编辑出版学的第二学位,和这个完全没有关系,不能申请社工专业的硕士吧,难道还要读4年的本科?

如果只是就业可以去读GEORGE BROWN的这个专业,想要以后有上升空间就要读本科或者研究生

liv_wen : 2012-06-05#703
回复: 求学选择个人谈及幼教,私人护理,社工,医师行政助理等热门行业介绍

看大家都是有专业背景的人。我是个英语专业本科出身,虽然在工程公司从事文职兼翻译几年,但说白了我这文职真没什么可提的经验。自己的钱也就够读两年课程的。我想在加拿大换读土木工程大专不知道可行不?还是换个文科专业的二学位读?社会学专业出来就业怎么样?唉~真愁死我了!希望大家能给我点建议呀!

shangguanxiao : 2012-07-09#704
回复: 求学选择个人谈及幼教,私人护理,社工,医师行政助理等热门行业介绍

关于公立和私人职业学校的课程和资质,可以到www.edu.gov.on.ca上去查, 所有经过政府审批的学校,以及私立职业学校的经过政府审批的diploma的课程在上面都可以查到。

首先一个概念,加拿大大专(College)跟我们国内的大专概念大不相同,更偏重于职业培训,注重的是技能和动手实践能力。大专的课程设置比较全面,合理,由浅入深易于接受。除了理论基础课程,更多的是职业培训课程、大学转学分课程和热门职业证书培训。同时,许多大专都与企业合作,甚至为大型企业设置特定专业,而且学院还经常调整专业,开设新兴的专业,尽量为学生就业创造机会,因此,大专生普遍比本科生好找工作,但如果想要做管理工作和科研工作,还得再读大学。

再说说多伦多的college, 多伦多的college分为两种,community college和private college,Community colleg就是俗称的公立大专,像Seneca, Geoge Brown,特点是学科非常全,基本上涵盖各个行业,学分可以被各个大学接受。有1年到3年的certifate, diploma和Degree的课程,也有时间更短的培训。可以在[URL http://www.edu.gov.on.ca/eng/general/list/college.html[/url]上找到On省所有Community 大专,一共24所。学生可以向政府申请OSAP贷款上学,但是大部分课程不能够申请EI学费上学,因为学制比较长,其实有的专业实际课时不多,但是由于Community College 假期和大学类似,时间较长,所以拖得比较长。政府可不愿意你拿着EI学费还休假,不过由于竞争的关系,他们也推出一些适合申请EI学费的专业。学生毕业后的就业率在70%到80%之间。

还有一类叫private college, 也可以叫private career college, 故名思义,是私人办的以职业培训为主的college, 一般规模不大,没有专门的校园。他们不可能开设像community college那么全的课程,所以都选择市场上的热门专业,并且可以较短的时间让学生莆眨玫iploma和Certificate(如果这个行业需要专门的certificate).由于规模不大,因此各个学校都有不同的侧重点,有比较强的专业,因此选择起来要困难一些。课程一般不超过一年,提供diploma,还有很多的课程是专门帮你通过一些证书考试的,有的学校学生可以申请OSAP,但是大部分学校不行。他们主要面向自费学习和EI学费学生。由于没有假期,上课时间灵活(一般每个专业都是滚动开课,所以学生在一年当中的任何一个月都可以入学)。这些学校,就业率是命根子,如果没有很高的就业率,就难招到学生,政府也很难批准EI学生入学。因此他们的专业一定是就业趋势很好的,老师一般是专业知识强并在行业有多年工作经验的,不仅负责教授学生技能,还要帮助学生实习和找工作的指导,因此把老师叫instructor,这是和community College教学非常不同的一点,Community教学还是正规的学校式教育,一个专业,7、8门课,每个老师各教一摊。大部分private college的一个班一个老师带到底,像师傅带徒弟,老师是老板雇的,因此压力更大些。学校也积极和各种社会资源雇主联系给学生提供co-op 和就业机会,因此就业率比community college高,好的能够达到90%以上。现在比较热门的,比如IT的一些专业,社工CSW,牙医助理,幼教助理ECA,药剂师PT,Computer Accounting,护工PSW,医生行政助理MOA等。但是private college 良莠不齐,首先大家可以去政府的网站
http://www.gov.on.ca/ont/portal/!ut/p/.cmd/cs/.ce/7_0_A/.s/7_0_252/_s.7_0_A/7_0_252/_l/en?docid=053263 去搜索,确保这个college是政府审批过的正在经营的college. 然后,看看这个学校的网站,看看有没有感兴趣的课程。

关于如何选择学校的问题,我想留到下次再写,希望能够逐渐补充些资料,给大家一个好的参考。也希望大家和我充分交流,提供不同意见。

6月10日续,如何选择college呢,论坛上这类东西可多了,我谈谈自己看法。我觉得核心就一个,快,时间越短越好。为什么呢,第一点,college 就是一个大专,你不管怎么学,拿什么文凭,到社会上也是一个具备初级技能的毕业生。虽说,seneca等在我们圈子名气响亮,可是并不入本地人眼。我有几个银行和政府的朋友,有本地的也有移民。我问他们关于college的问题,他们一致认为,college是非常初级的,只能从事初级工作。而且他们其实不是很清楚或者很在意college之间有什么差别。而到了大学这一级别,才开始注意学校之间的区别,这和国内差不多。第二点,既然college文凭不怎么样,我为什么还需要呢,因为我要本地教育背景阿。移民都是高学历,被这个卡住了,只好搞个本地文凭,其实大家都在学校里呆了不少年,知道学校里学的东西工作中能用得很少,一旦工作了,还得吃自己的工作经验。那这样,我们上college的目的就明确了。搞个本地文凭,既然如此,同样是diploma, 越快拿到得越好。千万不要去学那些什么2年,三年的课程。1年的本地工作经验足以媲美三年的学校教育。其实还有一点,这里的人并不是不认你在国内的工作和教育经历,只是他们认为如果你能够通过本地的学习获得技能,这样你的经验加上本地获得的技能才能把你的才干发挥出来。

其实短的课程未必不好,紧凑,只教实用的。大家如果仔细看各community大学的课程(我那里三所大学的全套),头三个学期都是基础课,一点用没有。那是给这里的高中生用的。有人说了,不对,我确实感觉学到了很多东西。是的,你主要在头三个学期学到的是英语。有些知识,翻译成中文就是普普通通的东西,一看就懂。但是你的口语恐怕提高不大。工作环境是提高口语的最好地方。所以如果集中学一个快的课,省下的时间去工作,是一个好的选择。
总而言之,时间最重要,同样都是一样的文凭,从哪拿都一样,越快越好。但是还是要找质量好的课,好的专业,这个问题下回讨论。

第三回,关于专业的选择,找热门的技能,两个办法,一个是去www.jobfutures.com上去看,那里可以查到各个专业工作的薪资水平,需要的学历,相关的专业,以及就业趋势。另外一个方法更简单,看看报纸上的各个 carrer college和培训学校的广告,那上面列的专业绝对都是比较好就业的专业,因为就业率是各个学校的生命线。没有前景,华而不实的专业绝对不会落入他们的眼睛。不过也不是绝对,career college主要关注初级技能培训和证书考试,对于有些比如医生,律师之类的可能前景很好,但是需要去大学读master. 我对于技工这块不太了解,不过大家可以去www.serviceontario.ca上面看看,在school & work里面,apprenticeship 是政府大力支持的,有很多的服务和培训。有些学校是专门教授这些的,比如常青藤。community college里面也有相关课程。不过这一块更要当心,有些技能淘汰的很快。其他方面,IT方面硬件是没有什么前途的,软件现在的课程可多了,软件测试,数据分析SAS, 网络安全都算热门。不过如果在国内不是IT人士,学这个恐怕已经晚了。还有一些课程,适合不同背景的人学习,就业前景基本可以,在这里可以介绍一下。

首先是电算化会计,其实学出来bookkeeper,不管你学两年的课程,还是6个月的速成,找工作都差不多,因为会计最需要经验,所以没有经验的只有从基层老老实实干起。在国内干过的,还是好找,那些从来没干过的,学了两年,还是难找。不过现在很多小公司找行政员工,如果同时能做些会计工作竞争力会大很多。

平均薪资:会计$15.78左右,bookkeeper 14.8左右,一开始也就拿个10-12块
就业情况:一直都还可以,比平均情况好。但是由于中国移民学这个的太多,所以现在的竞争也非常激烈

性别:女性占了91%,看来男性没什么机会,但是CFA男性多一些。不过要拿时间熬。光想办法考证是不行的,还要有机会进入事务所或大些的公司混些经验。所以如果希望将来能够发展空间大,那就要宁为牛后,不为鸡首。到大的事务所做做义工,合同工可能是必要的。
建议:学这个行业选择一个6-8个月的速成就可以了。

下一部分:幼教助理 Early Childcare Assistant
ECE o和ECA 介绍,
幼儿教育老师和幼儿教育助理,ECE需要两年学习,需要到community College里面学,ECA有很多6个月的短期课程,其实工资差不多,差1、2块钱。这个行业除了diploma外还要有certificate,基本上你在学习期间就可以考过这个证,你就可以工作了。考试费很便宜:50元左右
英语要求:ECA的英语要求不是太高
行业前景:不错,而且自雇比例颇高,达到41%。所以比较适合需要照顾家庭的女性。
平均薪资:12.87.这个工资实在没有吸引力,但是适合自雇,自己当老板赚多少,这里没有统计,但是自在啊。
性别:还是有2%的男性的!,而且男性的竞争力特强!
课程的情况看第二页的帖子

三:私人护理(personal support worker)
其实如果是需要找个挣钱多一些的职业,这个职业比ECA强,首先工资高,其次有了这个专业的diploma一样可以做ECA的工作,因为课程里面有专们如何照顾孩子的。而且学到的知识其实对自己对家庭都很有好处。只是我们中国人有的时候有偏见,觉得去照顾病人,老人,收拾床铺不太好对家中父老交待。说在国外干什么呢,私人护理,都干些啥,收拾屋子,陪老人聊天,照顾病人。人家会觉得你好好的,跑国外干这个。但是我觉得这个职业不错,听我分析一下
就业前景:很不错。整个世界都进入老年社会了。失业率只有1%!。未来中国也很需要,另外,发达社会越来越重视对于老人,病人,disable包括孩子的照顾和护理。其实不仅是照顾吃穿活动那么简单,还有精神上的照顾,属于体验经济的一类,阳光产业。而且这个行业对于自己的性格磨练很有好处,你会变得很有耐心,细心和爱心,听说对于生老病死的看法也会大大改变。自雇和part time的机会也不错,照顾照顾邻里,有40%的人做part time. 平时做义工的也是这个行业比较多。
薪资:16-18也有到20块的,而且,由于从事一些和病人打交道的工作,一般健康福利会比较不错。
工作环境:其实我去过老人院和老人公寓,环境真得很不错。由于工作中动体力的时间较多,所以一般都不会让你连续工作时间太长的。
语言要求:这个工作一般对语言要求较高,不过有一点,会一门外语绝对是一个财富。
发展空间:这个专业实际上是Nurse Aid,发展当然是朝Rigester Nurse了。热门阿。
性别:女性占91%。 但是:听说急需男性阿,虽说有器械帮助,但是还是有些体力活的,所以经常在老人院里和医院里见到五大三粗的女性。还有,男性也有隐私阿,男同志有的时候也想让同性来照顾一下。顺便说一句,男护士也是非常非常受欢迎。
课程介绍,上这个课的教室很有意思,大家有兴趣可以参观。这个课对于实际操作要求很强,经常要演练急救,比如,突然有人晕倒了,怎么处理,学生们七手八脚的上。
这个课必须要西人教师上,因为西人和我们在处理人的问题上首先观念大大不同。西人把安全放在第一位,首先必须安全。所以什么民间土法,没有经过验证的不用。比如脖子不舒服,趁他不注意给他嘎吱一下。所以同样操作,观念不同,处理的方法不同。
课程介绍太长了,包括,心理学的知识,人际沟通,营养学,老人护理,临终关怀,危机处理,家庭护理,收拾房屋,以及相关的法律法规,当然,实习是非常重要的一环
PT: Pharmacy Technician
就业前景:Very Good, 还是和老年社会有关。
薪资水平:如果在零售,10-15块,如果在社区community health care. 10-20块,医院:19-26块起,有些人成为了药剂师,工资约7万/年起,但是要再继续学习获得degree.
性别:男女均衡
语言:要求较高,学习辛苦,因为有大量东西要记,中国人毕业率高,老外经常吃不消。
自雇:自己做生意的占15%。 年收入较高。
学位:diploma, 同时还有行业的几个certificate
学习时间:40周,要学得抓紧,政府马上要把它改成两年的课程了
课程介绍:太长了

Community Service Worker 介绍,请点,就不用费时找了

好,拜读了!:wdb17:

shangguanxiao : 2012-07-11#705
回复: toronto college介绍,选择,相关问题交流

6月11日,
第三回,关于专业的选择,找热门的技能,两个办法,一个是去http://www.jobfutures.com上去看,那?..看报纸上的各个 carrer college和培训学校的广告,那上面列的专业绝对都是比较好就业的专业,因为就业率是各个学校的生命线。没有前景,华而不实的专业绝对不会落入他们的眼睛。不过也不是绝对,career college主要关注初级技能培训和证书考试,对于有些比如医生,律师之类的可能前景很好,但是需要去大学读master. 我对于技工这块不太了解,不过大家可以去www.serviceontario.ca上面看看,在school & work里面,apprenticeship 是政府大力支持的,有很多的服务和培训。有些学校是专门教授这些的,比如常青藤。community college里面也有相关课程。不过这一块更要当心,有些技能淘汰的很快。其他方面,IT方面硬件是没有什么前途的,软件现在的课程可多了,软件测试,数据分析SAS, 网络安全都算热门。不过如果在国内不是IT人士,学这个恐怕已经晚了。还有一些课程,适合不同背景的人学习,就业前景基本可以,在这里可以介绍一下。
首先是电算化会计,其实学出来bookkeeper,不管你学两年的课程,还是6个月的速成,找工作都差不多,因为会计最需要经验,所以没有经验的只有从基层老老实实干起。在国内干过的,还是好找,那些从来没干过的,学了两年,还是难找。不过现在很多小公司找行政员工,如果同时能做些会计工作竞争力会大很多。
平均薪资:会计$15.78左右,bookkeeper 14.8左右,一开始也就拿个10-12块
就业情况:一直都还可以,比平均情况好。但是由于中国移民学这个的太多,所以现在的竞争也非常激烈
性别:女性占了91%,看来男性没什么机会,但是CFA男性多一些。不过要拿时间熬。光想办法考证是不行的,还要有机会进入事务所或大些的公司混些经验。所以如果希望将来能够发展空间大,那就要宁为牛后,不为鸡首。到大的事务所做做义工,合同工可能是必要的。
建议:学这个行业选择一个6-8个月的速成就可以了。
课程介绍:会计基础,Quickbooks, Simply Accounting, Payroll, ACCPAC accounting system
下一部分:幼教助理 Early Childcare Assistant

学习了!:wdb17:

shangguanxiao : 2012-07-11#706
回复: toronto college介绍,选择,相关问题交流

ECE o和ECA 介绍,
幼儿教育老师和幼儿教育助理,ECE需要两年学习,需要到community College里面学,ECA有很多6个月的短期课程,其实工资差不多,差1、2块钱。这个行业除了diploma外还要有certificate,基本上你在学习期间就可以考过这个证,你就可以工作了。考试费很便宜:50元左右
英语要求:ECA的英语要求不是太高
行业前景:不错,而且自雇比例颇高,达到41%。所以比较适合需要照顾家庭的女性。
平均薪资:12.87.这个工资实在没有吸引力,但是适合自雇,自己当老板赚多少,这里没有统计,但是自在啊。
性别:还是有2%的男性的!,而且男性的竞争力特强!
课程介绍:粘贴另一份帖子里的
课程包括:这个是比较标准的课程。
Sorry no中文资料,安省规定是必须用英语或法语教学
Orientation
This module covers the requirements to set and achieve educational goals and gain exposure to market requirements and opportunities. The student will acquire tools for dealing with the challenges of being a student such as stress, note taking, studying, financial and time management.
Role of Childcare Worker
This module covers the principles of professionalism in the childcare field. It includes manner, dress, attitude and the importance of the Childcare Assistant in a child’s daycare experience and development.
Child Development
This module covers the multifaceted aspects of a child’s development including physical, social, intellectual, emotional and moral components. An in-depth exploration of the complexity of child development is undertaken.
Observation Skills
Course work covers the skills required to observe children effectively. Written observations are made in a variety of settings and circumstances. Students draw from their practical experiences to add to the module. Pitfalls and problems with observation techniques are discussed.
Guiding Children
This module covers concepts of enhancing child based play, encouraging age appropriate behaviors and using a variety of effective guidance techniques. Based on the day-to-day operations of a childcare facility, the theory and practical application of these concepts are observed and discussed.
Communication Skills
The emphases in this module are on developing the skills to communicate in a clear, concise, and articulate manner in both written and oral form. A variety of topics are included: i.e. two-way communication, listening skills, grammar, spelling, written notes and report completion.
Introduction to Technology
This module is designed to introduce the student to basic computer hardware, software. It provides the basics in using computers to complete assignments and essays. Additionally, it provides a good foundation for the module Children and Technology.
Children and Technology
A variety of children’s software programs are examined to evaluate/assess the effectiveness of this medium for child development. Students have an opportunity to review programs and learn about their appropriate application in an early learning environment.
Program Planning
The students are introduced to the diverse components of effective program planning. Incorporated into the program planning are a variety of activities that aid in a child’s overall healthy development. The importance of dramatic play, circle time and problem solving activities are discussed.
Health and Safety
Health and safety standards and applications are studied and discussed in this critically important module. Students learn about healthy environments and how to educate children in managing their own good health habits. Child abuse recognition and prevention procedures are examined and sample policies are reviewed.
Nutrition
This module covers the theory and application of balanced, healthy nutrition for children 0 to 6. Basic food handling principles are discussed, while recipes are reviewed and assessed. Menu planning and compiling a resource of appropriate recipes and menus are incorporated.
Developing a Business in Childcare
This module deals with the demands and trends in daycare centres, starting a daycare centre, government rules, policies and licensing procedure, strategies involved in operating, income potential, daycare management, and marketing the business
Practicum ? 280 hours internship
Field placement provides the students with an opportunity to practice their new skills in a work setting. While on placement students can gain experience in a wider range of Early Childcare Assistant skills, become more self-confident and in some cases receive offers of employment from the placement site.

教材;
TEXT BOOKS & REFERENCE MATERIALS

Keys to Success: How to Achieve Your Goals: 3rd Canadian Edition by Carter Bishop, Kravits and Maurin, Pearson, 2004. ISBN 0-13-120121-2

The Little Brown Compact Handbook: 2nd Canadian Edition by Jane E. Aaron and Murray McArthur, Pearson, 2003. ISBN 0-321-10205-3

Early Childhood Development: A Multicultural Perspective: 3rd Edition by Jeffery trawick-Smith, Merrill Prentice Hall, 2003. ISBN 0-13-046576-3

Skills for Preschool Teachers: 7th Edition by Janice J. Beaty, 2004. ISBN 0-13-048609-4

学习!:wdb17:

shangguanxiao : 2012-07-11#707
回复: toronto college介绍,选择,相关问题交流

介绍一下社会工作者行业
国内没有社工这个行业,不过大家应该清楚这个是干什么的,我们遇到的settlement Counselor, Employment Counselor都是属于这个行业。这个行业对于我们移民来说有一个优势,我们会1门外语,而且是目前最大的移民群体的外语,各个机构都在扩展对于中国移民的服务。这样子就需要我们了。社工为什么要拓展服务呢,大家知道,no profit机构的钱来自于政府,企业或个人捐助,自我经营收入,其中政府最多,而政府的钱是要根据你做了多少事情来得,比如政府今年有7个亿是帮助新移民的,会分配到各个非营利机构,这些机构就要做一些针对新移民的服务来申请经费,同时还要有效果,如果没有政府的funding, 这个机构就要关门了。所以从事这个行业的人往往有marketing、工商管理,教师的背景,然后获得了本地学历。那么这个行业有多大呢,这个行业非常大,和商业基本上快差不多了。那些机构属于no-profit, 医院,学校,就业、安居服务机构,慈善机构,老人服务机构,绿色和平组织,环境和动物保护组织等。他的行业自成体系,你在一般的招聘网站或者agency那里很少看到招聘的信息。需要什么样的人呢,其实和商业一样,需要sales, accounting, administration, customer service, technician.不一样的地方,核心的服务人员是社工,教师,或者医务人员。大家可以去www.charityvillige.com 上去看,这是加拿大最大的非营利机构的招聘网站。
就业前景:不错
薪资水平:16-20左右。
性别:女性居多
个人奋斗方向:如果成为专业social worker, 工资可以达到70000,但是一般需要有social science master degree. 而 bachelor 毕业实际上和college毕业薪资差别不是太大,我个人觉得可以在拿到diploma后工作两年,直接申请master(用国内的学历)+diploma+工作经验,这样不错。
课程介绍
• Introduction to Community Service Worker
• Social Welfare, Diversity and Awareness
• The Family
• Couples, Family and Interpersonal Communication
• Oriented Case Management
• Environmental Issues and Crisis Interventions and
Prevention
• Bereavement and the Elderly
• Respond to Abuse
• Understanding Human Behavior
• Communication Skills
• Counseling Skills
• Report Writing for the CSW
• Field placement

By the end of the course, students will be able to:
• Demonstrate knowledge of the range of social and community service organizations in their own community as well as detailed knowledge of an agency.
• Practice the role of a team member within a social and community service organization.
• Demonstrate beginning level professional skills
• Apply theory and skills learned in the independent study components of this course.

学习了:wdb17:

shangguanxiao : 2012-07-11#708
回复: toronto college介绍,选择,相关问题交流

私立学校的选择和信息,请察看http://www.nacc.ca/ National Association of Career College. 里面有详细的分析。
:wdb17:

shangguanxiao : 2012-07-11#709
回复: toronto college介绍,选择,相关问题交流

需要学什么,虽然很多学校都有相同专业,但是也有很多不同。有一个简单办法,到华人超市,大中报,或者加拿大都市报,或者北美时报,很多很多学校广告,打电话问。如果不想上华人学校,也可以,到地铁站找几份Metro来,打电话问。是西人学校又在中国报纸上做广告的,Yorkville, Canadian Business School, Evergreecollege,基本上在scarborough的都是华人学校。
:wdb17:

shangguanxiao : 2012-07-11#710
回复: toronto college介绍,选择,相关问题交流

关于公立和私人职业学校的课程和资质,可以到www.edu.gov.on.ca上去查, 所有经过政府审批的学校,以及私立职业学校的经过政府审批的diploma的课程在上面都可以查到。
:wdb17:

shangguanxiao : 2012-07-11#711
回复: toronto college介绍,选择,相关问题交流

senica的时间长,中间有假期,实际学习时间我估计在9个月左右,而且开学时间固定,不好选择,批给你的生活费不负责暑假部分,另外,你只看到了学费,公立大学的其他费用较高,一般还有一个统一的杂费,大概在6、7百块钱,还有书本和材料费,估计也要不少,而且这个课程上公立大学也没有多大意思。半年5000块的时间成本较少(早上一个月班工资也有2000块呢),应该每个月都能上课,不过能否批多少贷款未知,因为半年5000+生活费6个月约7200=12000。这个可以去osap网站上去模拟看看自己能拿到多少。对于半年的职业培训,如果经济允许,建议不要申请贷款学习,因为1、半年后就要开始还款,2、凡是申请OSAP贷款学习的学费都贵很多。比如这个课程,如果自费的话包括所有费用3000块左右,现在你申请OSAP,岂不是要多掏2000块! 而且有的学校可以分期付款
3、职业培训的学习一般都比较灵活,你可以选择白天班,晚上班,或者周末班,通过打些工来补贴日常费用。

所以,我觉得贷款上学是不得已的做法。

:wdb17:

shangguanxiao : 2012-07-11#712
回复: toronto college介绍,选择,相关问题交流

很多人问我,公立College和私立College有什么区别,这个问题其实也困扰了我一段时间,我一直都在想一个问题。从公众舆论上看,在加拿大,似乎公立的College享有很好的信誉,光明的就业前景和雄厚的师资,而且有能力招收更多的学生。为什么私立college还有发展的空间。私立college和公立college的立足点差别究竟在哪里?作为私立college的一个教育顾问,我觉得有义务去探讨这个问题。
公立College是政府Funding建立的College, 同时,通过收取学费和其他费用来支持日常的营运。公立College主要有三个经营目标:
1、提供短期的培训,通常少于一年,满足学生学习专项技能或知识的需求,学生毕业后得到Certificate
2、提供大专文凭课程,通常需要2-3年,满足学生准备从事一种职业学习必备知识,技能的需求和继续进行university深造的需求,并且训练学生为未来的社会生活做好准备,学生毕业可以获得代表教育程度的大专文凭。
3、提供Apprenticeship 培训(专业技工培训?),比如木工,面包师,厨师等等
现在有些学校也提供一些快速毕业的大专文凭课程(1年)或者3-4年的Degree课程甚至研究生课程
公立College需要通过提高知名度,招收更多学生获得更多政府支持,改善员工福利,因此竞争也很激烈。
私立College 是私人根据安省私人职业培训法例投资成立的,并获得安省教育厅(MTCU)颁发许可证。许可证每年更新,MTCU每年会对各私人college进行审查,确保其设施,教学质量,教师资质达到要求,私立college每一项培训的收费,课程安排,时间设置也需要经过审批同意,所以在MTCU颁发的证书上不仅能够看到学校的名称,还列出了审批后的各个课程的名称。
私立College经营目标只有一个:提供紧凑的技能培训满足就业市场需求。私立college一般规模要比公立的小得多,名气也差很多,专注于比较少的课程。除了面临公立College的挤压外,私立College之间的竞争也非常激烈。
选择培训首先是选择学校,选择学校比选择课程更重要。我们大概都知道公立College的优势,因此,这里从私立college的特点来入手进行一些比较,由于很多私立学校有一些短期证书培训课程无法和公立College进行比较
1、由于私立college是私人所有,为了生存发展,他们对于就业市场的变化反应更迅速,并不断提供易于就业的课程或者根据需要对课程进行改进。
2、由于私立college专注于提供技能培训,所以学生不需要花大量时间学习基础学术知识,技能传授+实践是职业培训的基本教学方法。
3、私立college对于教师的选择非常慎重,教师必须具有多年行业的工作经验和培训的经验。
4、学生导向,私立college的生存基础是学生的就业,学生的口碑相传是私立学校发展的基石。因此私立college更注重学生的就业率和对课程的评价,对于教师的要求更加严格。
5、私立college的课堂通常都比公立college小,因此和学生有更多的互动和交流。通常学生在整个学习期间由一位老师执教,因此可以给予学生更多的个人指导。
6、私立college提供非常灵活的注册时间表,对大部分课程而言,学生每个月都可以注册开始学习,除了政府公众假期,没有寒暑假。一般有Full timepart time课程,满足不同需求。
7、私立college允许学生反复听课,来巩固自己的学习。

其实,正如我在另一篇文章里讲到的,培训的最大成本不在学费,很多人认为学费贵,实际上一般一年的课程所有费用不过在5000-7000左右,而真正贵的是机会成本,即如果你不工作,你所挣的钱。按照$9一个小时,应该在$18720。这样你一年的学习成本就是$23000---$25000,而两年的成本在$ 46000-$50000左右。对于承担着家庭重担的成年人来说,是一个很大的成本。因此,职业培训的选择和上学不同。学生应该在获得技能的同时,尽量缩短学习时间,这也是选择职业培训的重要原因。
另外私立college对于学生资质的要求其实更高,他们在学生入学前通过交谈和测试,来判断学生的技能,兴趣和学习的动力是否合适所学的专业。因为在这里,学生也是顾客,如果没有给学生的学习提供正确的建议,就是没有尽到学校的职责,耽误学生的职业生涯。
那么确实我看到很多对于私立职业学校的负面的评论,我想一方面,是大家心底里对于私立两个字的不信任,而且市场上也很难找到一个标准来衡量私立学校。另外一些私立学校确实存在不重视教学质量的现象。我在这里需要说的是,大部分私立college非常重视教学质量,教师质量和就业率。以最短的时间获得专业的技能和工作,是选择职业学校最重要的理由。
我希望有时间接下来告诉大家私立college长得什么样,怎么样教学的,以及选择私立学校的问题清单,我去过很多这样的学校做调查,总结了一些,希望对大家有所裨益。(待续)

:wdb17:

山涧的风 : 2012-07-12#713
回复: 求学选择个人谈及幼教,私人护理,社工,医师行政助理等热门行业介绍

最近也在迷茫中,把楼主的帖子用一下午的时间从头到尾看了一遍。可惜楼主最近没怎么出现,不知道加国最近的就业市场跟一两年前变化大不大,楼主的这些分析目前是否还适用?
无论如何,对LZ这样的热心人,赞一个!

刺蛇至上 : 2012-07-13#714
回复: 求学选择个人谈及幼教,私人护理,社工,医师行政助理等热门行业介绍

您好 7月24号我将随父母移民到蒙城 国内专业数控自动化,FANUC系统熟练 学过西门子,手工编程包括宏程序都熟练 软件编程在学校学过用过,理论实操同样不错,市级比赛加工中心全市第一,也参加过很多全国性选拔比赛,2011年夏天毕业,曾工作于轮胎模具加工铣工,石油钻机车间加工中心操作工,学历属于高级,不知道在蒙特利尔怎么发展,我看上的航技要17个月 感觉稍长 有没有培训半年左右就能上岗那种机构,谢谢

amao2514 : 2012-07-14#715
回复: 求学选择个人谈及幼教,私人护理,社工,医师行政助理等热门行业介绍

:wdb17: 看到您的帖子,写得很好,忍不住求教下。

我是MM, 刚来蒙城,PR 身份。在国内是做销售、市场工作的。学的管理类,万精油专业。。本科。

现在刚过来, 英语还可以,法语还得继续学习,已经报了COFI班。

对后面的工作和学习计划还比较迷茫,当然学习也是为了工作服务的。 我的长处是与人交道能力还可以,有亲和力,性格比较好,还喜欢写点东西,文笔还行(当然,是中文的了)。

后面该如何发展,希望您能提点建议。 之前我有考虑过旅游管理或者国际酒店管理, 刚看到您说的社工也挺不错。

我也觉得最好能尽快拿到文凭,多点时间去争取工作机会,后面有需要再进修。


盼复!:wdb17:

daisy13_fri : 2012-08-29#716
回复: 求学选择个人谈及幼教,私人护理,社工,医师行政助理等热门行业介绍

标记下,但愿能有请教的一天!

owenwu528 : 2012-09-07#717
回复: 求学选择个人谈及幼教,私人护理,社工,医师行政助理等热门行业介绍

我登陆后一直非常迷茫 现在想去学习护工PSW 请shangguanxiao分析一下 这个专业是不是饱和了呢 我快40了 男士。感谢!!!!!

owenwu528 : 2012-09-07#718
回复: 求学选择个人谈及幼教,私人护理,社工,医师行政助理等热门行业介绍

楼主在不在 帮忙分析一下啊 感谢啊!!!

owenwu528 : 2012-09-07#719
回复: 求学选择个人谈及幼教,私人护理,社工,医师行政助理等热门行业介绍

那位大侠可以帮我指点迷津啊 求教啊 感谢!

owenwu528 : 2012-09-07#720
回复: 求学选择个人谈及幼教,私人护理,社工,医师行政助理等热门行业介绍

非常想尽快就业 口袋里面没有钱了 学习PSW还要申请贷款。

jsczlry : 2012-09-08#721
回复: 求学选择个人谈及幼教,私人护理,社工,医师行政助理等热门行业介绍

好文章,我要顶

annie777 : 2012-09-17#722
回复: toronto college介绍,选择,相关问题交流

1、申请OSAP,查一下相关贴子吧,注意到加拿大后的银行存款不能超过3000。申请多少因人情况而异。
2,CO-op就是提供实习。人力资源找不到工作,除非英语和当地人一样。

申请时银行存款不能超过3000吗?来时存入银行的钱肯定比这个多,过一段时间等存款到这个标准就可以申请码?

peaceofmind : 2012-09-26#723
回复: toronto college介绍,选择,相关问题交流

三:私人护理(personal support worker)
其实如果是需要找个挣钱多一些的职业,这个职业比ECA强,首先工资高,其次有了这个专业的diploma一样可以做ECA的工作,因为课程里面有专们如何照顾孩子的。而且学到的知识其实对自己对家庭都很有好处。只是我们中国人有的时候有偏见,觉得去照顾病人,老人,收拾床铺不太好对家中父老交待。说在国外干什么呢,私人护理,都干些啥,收拾屋子,陪老人聊天,照顾病人。人家会觉得你好好的,跑国外干这个。但是我觉得这个职业不错,听我分析一下
就业前景:很不错。整个世界都进入老年社会了。失业率只有1%!。未来中国也很需要,另外,发达社会越来越重视对于老人,病人,disable包括孩子的照顾和护理。其实不仅是照顾吃穿活动那么简单,还有精神上的照顾,属于体验经济的一类,阳光产业。而且这个行业对于自己的性格磨练很有好处,你会变得很有耐心,细心和爱心,听说对于生老病死的看法也会大大改变。自雇和part time的机会也不错,照顾照顾邻里,有40%的人做part time. 平时做义工的也是这个行业比较多。
薪资:16-18也有到20块的,而且,由于从事一些和病人打交道的工作,一般健康福利会比较不错。
工作环境:其实我去过老人院和老人公寓,环境真得很不错。由于工作中动体力的时间较多,所以一般都不会让你连续工作时间太长的。
语言要求:这个工作一般对语言要求较高,不过有一点,会一门外语绝对是一个财富。
发展空间:这个专业实际上是Nurse Aid,发展当然是朝Rigester Nurse了。热门阿。
性别:女性占91%。 但是:听说急需男性阿,虽说有器械帮助,但是还是有些体力活的,所以经常在老人院里和医院里见到五大三粗的女性。还有,男性也有隐私阿,男同志有的时候也想让同性来照顾一下。顺便说一句,男护士也是非常非常受欢迎。
课程介绍,上这个课的教室很有意思,大家有兴趣可以参观。这个课对于实际操作要求很强,经常要演练急救,比如,突然有人晕倒了,怎么处理,学生们七手八脚的上。
这个课必须要西人教师上,因为西人和我们在处理人的问题上首先观念大大不同。西人把安全放在第一位,首先必须安全。所以什么民间土法,没有经过验证的不用。比如脖子不舒服,趁他不注意给他嘎吱一下。所以同样操作,观念不同,处理的方法不同。
课程介绍太长了,需要的留下邮件。包括,心理学的知识,人际沟通,营养学,老人护理,临终关怀,危机处理,家庭护理,收拾房屋,以及相关的法律法规,当然,实习是非常重要的一环


====================

虽是旧帖, 对我依然很有帮助, 感谢楼主
:wdb23::wdb37::wdb9::wdb10:

jodi99 : 2012-09-26#724
回复: 求学选择个人谈及幼教,私人护理,社工,医师行政助理等热门行业介绍

一口气把楼爬玩,又累又饿也值得。楼主最近可好?

孤单往事 : 2012-10-14#725
回复: 求学选择个人谈及幼教,私人护理,社工,医师行政助理等热门行业介绍

读完全部帖子,受益匪浅。感谢楼主,如果方便希望您能继续来此传道授业解惑,谢谢!

chuny : 2012-10-25#726
回复: 求学选择个人谈及幼教,私人护理,社工,医师行政助理等热门行业介绍

多谢分享!受益了!

shangguanxiao : 2012-10-31#727
回复: 求学选择个人谈及幼教,私人护理,社工,医师行政助理等热门行业介绍

关于公立和私人职业学校的课程和资质,可以到www.edu.gov.on.ca上去查, 所有经过政府审批的学校,以及私立职业学校的经过政府审批的diploma的课程在上面都可以查到。

首先一个概念,加拿大大专(College)跟我们国内的大专概念大不相同,更偏重于职业培训,注重的是技能和动手实践能力。大专的课程设置比较全面,合理,由浅入深易于接受。除了理论基础课程,更多的是职业培训课程、大学转学分课程和热门职业证书培训。同时,许多大专都与企业合作,甚至为大型企业设置特定专业,而且学院还经常调整专业,开设新兴的专业,尽量为学生就业创造机会,因此,大专生普遍比本科生好找工作,但如果想要做管理工作和科研工作,还得再读大学。

再说说多伦多的college, 多伦多的college分为两种,community college和private college,Community colleg就是俗称的公立大专,像Seneca, Geoge Brown,特点是学科非常全,基本上涵盖各个行业,学分可以被各个大学接受。有1年到3年的certifate, diploma和Degree的课程,也有时间更短的培训。可以在[URL http://www.edu.gov.on.ca/eng/general/list/college.html[/url]上找到On省所有Community 大专,一共24所。学生可以向政府申请OSAP贷款上学,但是大部分课程不能够申请EI学费上学,因为学制比较长,其实有的专业实际课时不多,但是由于Community College 假期和大学类似,时间较长,所以拖得比较长。政府可不愿意你拿着EI学费还休假,不过由于竞争的关系,他们也推出一些适合申请EI学费的专业。学生毕业后的就业率在70%到80%之间。

还有一类叫private college, 也可以叫private career college, 故名思义,是私人办的以职业培训为主的college, 一般规模不大,没有专门的校园。他们不可能开设像community college那么全的课程,所以都选择市场上的热门专业,并且可以较短的时间让学生莆眨玫iploma和Certificate(如果这个行业需要专门的certificate).由于规模不大,因此各个学校都有不同的侧重点,有比较强的专业,因此选择起来要困难一些。课程一般不超过一年,提供diploma,还有很多的课程是专门帮你通过一些证书考试的,有的学校学生可以申请OSAP,但是大部分学校不行。他们主要面向自费学习和EI学费学生。由于没有假期,上课时间灵活(一般每个专业都是滚动开课,所以学生在一年当中的任何一个月都可以入学)。这些学校,就业率是命根子,如果没有很高的就业率,就难招到学生,政府也很难批准EI学生入学。因此他们的专业一定是就业趋势很好的,老师一般是专业知识强并在行业有多年工作经验的,不仅负责教授学生技能,还要帮助学生实习和找工作的指导,因此把老师叫instructor,这是和community College教学非常不同的一点,Community教学还是正规的学校式教育,一个专业,7、8门课,每个老师各教一摊。大部分private college的一个班一个老师带到底,像师傅带徒弟,老师是老板雇的,因此压力更大些。学校也积极和各种社会资源雇主联系给学生提供co-op 和就业机会,因此就业率比community college高,好的能够达到90%以上。现在比较热门的,比如IT的一些专业,社工CSW,牙医助理,幼教助理ECA,药剂师PT,Computer Accounting,护工PSW,医生行政助理MOA等。但是private college 良莠不齐,首先大家可以去政府的网站
http://www.gov.on.ca/ont/portal/!ut/p/.cmd/cs/.ce/7_0_A/.s/7_0_252/_s.7_0_A/7_0_252/_l/en?docid=053263 去搜索,确保这个college是政府审批过的正在经营的college. 然后,看看这个学校的网站,看看有没有感兴趣的课程。

关于如何选择学校的问题,我想留到下次再写,希望能够逐渐补充些资料,给大家一个好的参考。也希望大家和我充分交流,提供不同意见。

6月10日续,如何选择college呢,论坛上这类东西可多了,我谈谈自己看法。我觉得核心就一个,快,时间越短越好。为什么呢,第一点,college 就是一个大专,你不管怎么学,拿什么文凭,到社会上也是一个具备初级技能的毕业生。虽说,seneca等在我们圈子名气响亮,可是并不入本地人眼。我有几个银行和政府的朋友,有本地的也有移民。我问他们关于college的问题,他们一致认为,college是非常初级的,只能从事初级工作。而且他们其实不是很清楚或者很在意college之间有什么差别。而到了大学这一级别,才开始注意学校之间的区别,这和国内差不多。第二点,既然college文凭不怎么样,我为什么还需要呢,因为我要本地教育背景阿。移民都是高学历,被这个卡住了,只好搞个本地文凭,其实大家都在学校里呆了不少年,知道学校里学的东西工作中能用得很少,一旦工作了,还得吃自己的工作经验。那这样,我们上college的目的就明确了。搞个本地文凭,既然如此,同样是diploma, 越快拿到得越好。千万不要去学那些什么2年,三年的课程。1年的本地工作经验足以媲美三年的学校教育。其实还有一点,这里的人并不是不认你在国内的工作和教育经历,只是他们认为如果你能够通过本地的学习获得技能,这样你的经验加上本地获得的技能才能把你的才干发挥出来。

其实短的课程未必不好,紧凑,只教实用的。大家如果仔细看各community大学的课程(我那里三所大学的全套),头三个学期都是基础课,一点用没有。那是给这里的高中生用的。有人说了,不对,我确实感觉学到了很多东西。是的,你主要在头三个学期学到的是英语。有些知识,翻译成中文就是普普通通的东西,一看就懂。但是你的口语恐怕提高不大。工作环境是提高口语的最好地方。所以如果集中学一个快的课,省下的时间去工作,是一个好的选择。
总而言之,时间最重要,同样都是一样的文凭,从哪拿都一样,越快越好。但是还是要找质量好的课,好的专业,这个问题下回讨论。

第三回,关于专业的选择,找热门的技能,两个办法,一个是去www.jobfutures.com上去看,那里可以查到各个专业工作的薪资水平,需要的学历,相关的专业,以及就业趋势。另外一个方法更简单,看看报纸上的各个 carrer college和培训学校的广告,那上面列的专业绝对都是比较好就业的专业,因为就业率是各个学校的生命线。没有前景,华而不实的专业绝对不会落入他们的眼睛。不过也不是绝对,career college主要关注初级技能培训和证书考试,对于有些比如医生,律师之类的可能前景很好,但是需要去大学读master. 我对于技工这块不太了解,不过大家可以去www.serviceontario.ca上面看看,在school & work里面,apprenticeship 是政府大力支持的,有很多的服务和培训。有些学校是专门教授这些的,比如常青藤。community college里面也有相关课程。不过这一块更要当心,有些技能淘汰的很快。其他方面,IT方面硬件是没有什么前途的,软件现在的课程可多了,软件测试,数据分析SAS, 网络安全都算热门。不过如果在国内不是IT人士,学这个恐怕已经晚了。还有一些课程,适合不同背景的人学习,就业前景基本可以,在这里可以介绍一下。

首先是电算化会计,其实学出来bookkeeper,不管你学两年的课程,还是6个月的速成,找工作都差不多,因为会计最需要经验,所以没有经验的只有从基层老老实实干起。在国内干过的,还是好找,那些从来没干过的,学了两年,还是难找。不过现在很多小公司找行政员工,如果同时能做些会计工作竞争力会大很多。

平均薪资:会计$15.78左右,bookkeeper 14.8左右,一开始也就拿个10-12块
就业情况:一直都还可以,比平均情况好。但是由于中国移民学这个的太多,所以现在的竞争也非常激烈

性别:女性占了91%,看来男性没什么机会,但是CFA男性多一些。不过要拿时间熬。光想办法考证是不行的,还要有机会进入事务所或大些的公司混些经验。所以如果希望将来能够发展空间大,那就要宁为牛后,不为鸡首。到大的事务所做做义工,合同工可能是必要的。
建议:学这个行业选择一个6-8个月的速成就可以了。

下一部分:幼教助理 Early Childcare Assistant
ECE o和ECA 介绍,
幼儿教育老师和幼儿教育助理,ECE需要两年学习,需要到community College里面学,ECA有很多6个月的短期课程,其实工资差不多,差1、2块钱。这个行业除了diploma外还要有certificate,基本上你在学习期间就可以考过这个证,你就可以工作了。考试费很便宜:50元左右
英语要求:ECA的英语要求不是太高
行业前景:不错,而且自雇比例颇高,达到41%。所以比较适合需要照顾家庭的女性。
平均薪资:12.87.这个工资实在没有吸引力,但是适合自雇,自己当老板赚多少,这里没有统计,但是自在啊。
性别:还是有2%的男性的!,而且男性的竞争力特强!
课程的情况看第二页的帖子

三:私人护理(personal support worker)
其实如果是需要找个挣钱多一些的职业,这个职业比ECA强,首先工资高,其次有了这个专业的diploma一样可以做ECA的工作,因为课程里面有专们如何照顾孩子的。而且学到的知识其实对自己对家庭都很有好处。只是我们中国人有的时候有偏见,觉得去照顾病人,老人,收拾床铺不太好对家中父老交待。说在国外干什么呢,私人护理,都干些啥,收拾屋子,陪老人聊天,照顾病人。人家会觉得你好好的,跑国外干这个。但是我觉得这个职业不错,听我分析一下
就业前景:很不错。整个世界都进入老年社会了。失业率只有1%!。未来中国也很需要,另外,发达社会越来越重视对于老人,病人,disable包括孩子的照顾和护理。其实不仅是照顾吃穿活动那么简单,还有精神上的照顾,属于体验经济的一类,阳光产业。而且这个行业对于自己的性格磨练很有好处,你会变得很有耐心,细心和爱心,听说对于生老病死的看法也会大大改变。自雇和part time的机会也不错,照顾照顾邻里,有40%的人做part time. 平时做义工的也是这个行业比较多。
薪资:16-18也有到20块的,而且,由于从事一些和病人打交道的工作,一般健康福利会比较不错。
工作环境:其实我去过老人院和老人公寓,环境真得很不错。由于工作中动体力的时间较多,所以一般都不会让你连续工作时间太长的。
语言要求:这个工作一般对语言要求较高,不过有一点,会一门外语绝对是一个财富。
发展空间:这个专业实际上是Nurse Aid,发展当然是朝Rigester Nurse了。热门阿。
性别:女性占91%。 但是:听说急需男性阿,虽说有器械帮助,但是还是有些体力活的,所以经常在老人院里和医院里见到五大三粗的女性。还有,男性也有隐私阿,男同志有的时候也想让同性来照顾一下。顺便说一句,男护士也是非常非常受欢迎。
课程介绍,上这个课的教室很有意思,大家有兴趣可以参观。这个课对于实际操作要求很强,经常要演练急救,比如,突然有人晕倒了,怎么处理,学生们七手八脚的上。
这个课必须要西人教师上,因为西人和我们在处理人的问题上首先观念大大不同。西人把安全放在第一位,首先必须安全。所以什么民间土法,没有经过验证的不用。比如脖子不舒服,趁他不注意给他嘎吱一下。所以同样操作,观念不同,处理的方法不同。
课程介绍太长了,包括,心理学的知识,人际沟通,营养学,老人护理,临终关怀,危机处理,家庭护理,收拾房屋,以及相关的法律法规,当然,实习是非常重要的一环
PT: Pharmacy Technician
就业前景:Very Good, 还是和老年社会有关。
薪资水平:如果在零售,10-15块,如果在社区community health care. 10-20块,医院:19-26块起,有些人成为了药剂师,工资约7万/年起,但是要再继续学习获得degree.
性别:男女均衡
语言:要求较高,学习辛苦,因为有大量东西要记,中国人毕业率高,老外经常吃不消。
自雇:自己做生意的占15%。 年收入较高。
学位:diploma, 同时还有行业的几个certificate
学习时间:40周,要学得抓紧,政府马上要把它改成两年的课程了
课程介绍:太长了

Community Service Worker 介绍,请点,就不用费时找了

请教一下alex, 我听说上college可以拿RMT(这册按摩师)证书,这个好学吗?是得2年艰苦学习吗?学费方面如何?能否拿助学贷款,条件?入学要求什么条件?多伦多都哪里有这个学习的地方?guelph有吗?多谢!

ahwuang : 2012-12-26#728
回复: toronto college介绍,选择,相关问题交流

介绍一下社会工作者行业
国内没有社工这个行业,不过大家应该清楚这个是干什么的,我们遇到的settlement Counselor, Employment Counselor都是属于这个行业。这个行业对于我们移民来说有一个优势,我们会1门外语,而且是目前最大的移民群体的外语,各个机构都在扩展对于中国移民的服务。这样子就需要我们了。社工为什么要拓展服务呢,大家知道,no profit机构的钱来自于政府,企业或个人捐助,自我经营收入,其中政府最多,而政府的钱是要根据你做了多少事情来得,比如政府今年有7个亿是帮助新移民的,会分配到各个非营利机构,这些机构就要做一些针对新移民的服务来申请经费,同时还要有效果,如果没有政府的funding, 这个机构就要关门了。所以从事这个行业的人往往有marketing、工商管理,教师的背景,然后获得了本地学历。那么这个行业有多大呢,这个行业非常大,和商业基本上快差不多了。那些机构属于no-profit, 医院,学校,就业、安居服务机构,慈善机构,老人服务机构,绿色和平组织,环境和动物保护组织等。他的行业自成体系,你在一般的招聘网站或者agency那里很少看到招聘的信息。需要什么样的人呢,其实和商业一样,需要sales, accounting, administration, customer service, technician.不一样的地方,核心的服务人员是社工,教师,或者医务人员。大家可以去www.charityvillige.com 上去看,这是加拿大最大的非营利机构的招聘网站。
就业前景:不错
薪资水平:16-20左右。
性别:女性居多
个人奋斗方向:如果成为专业social worker, 工资可以达到70000,但是一般需要有social science master degree. 而 bachelor 毕业实际上和college毕业薪资差别不是太大,我个人觉得可以在拿到diploma后工作两年,直接申请master(用国内的学历)+diploma+工作经验,这样不错。
课程介绍
• Introduction to Community Service Worker
• Social Welfare, Diversity and Awareness
• The Family
• Couples, Family and Interpersonal Communication
• Oriented Case Management
• Environmental Issues and Crisis Interventions and
Prevention
• Bereavement and the Elderly
• Respond to Abuse
• Understanding Human Behavior
• Communication Skills
• Counseling Skills
• Report Writing for the CSW
• Field placement

By the end of the course, students will be able to:
• Demonstrate knowledge of the range of social and community service organizations in their own community as well as detailed knowledge of an agency.
• Practice the role of a team member within a social and community service organization.
• Demonstrate beginning level professional skills
• Apply theory and skills learned in the independent study components of this course.
mark up and thanks.

lelievre : 2012-12-29#729
回复: 求学选择个人谈及幼教,私人护理,社工,医师行政助理等热

崇拜楼主

木兜脚丫 : 2013-01-08#730
回复: 求学选择个人谈及幼教,私人护理,社工,医师行政助理等热

太好的帖子,感谢楼主!

linda――qin : 2013-01-14#731
回复: 求学选择个人谈及幼教,私人护理,社工,医师行政助理等热

好贴,介绍的很专业详细,是我需要的信息,谢谢LZ!

泡菜坛子 : 2013-02-07#732
回复: 求学选择个人谈及幼教,私人护理,社工,医师行政助理等热

最近正在为学什么而极度纠结,护理据说太脏,大多数人学出来也做不下去,这也是这行这么热门并一直在招人的原因,幼教呢,据说不好找工作,工资低,除非自己开业,没有上升空间。

sgxlmj : 2013-02-15#733
回复: 求学选择个人谈及幼教,私人护理,社工,医师行政助理等热

必须添砖加瓦

chunqiu0509 : 2013-02-18#734
回复: 求学选择个人谈及幼教,私人护理,社工,医师行政助理等热

从第一页一直读完,受益颇多,感谢lz!楼主不来了,为何?

阳光不倒 : 2013-03-11#735
回复: 求学选择个人谈及幼教,私人护理,社工,医师行政助理等热

看了这么多,还是觉得一头雾水,不知道究竟应该选哪条路才是最适合自己的.

huluzhu : 2013-04-02#736
回复: 求学选择个人谈及幼教,私人护理,社工,医师行政助理等热

时隔数年,再来学习,今年登陆,从头再来!

huluzhu : 2013-04-02#737
回复: 求学选择个人谈及幼教,私人护理,社工,医师行政助理等热

终于找到alex牛人的牛贴了

huluzhu : 2013-04-02#738
这个值得关注

介绍一下社会工作者行业
国内没有社工这个行业,不过大家应该清楚这个是干什么的,我们遇到的settlement Counselor, Employment Counselor都是属于这个行业。这个行业对于我们移民来说有一个优势,我们会1门外语,而且是目前最大的移民群体的外语,各个机构都在扩展对于中国移民的服务。这样子就需要我们了。社工为什么要拓展服务呢,大家知道,no profit机构的钱来自于政府,企业或个人捐助,自我经营收入,其中政府最多,而政府的钱是要根据你做了多少事情来得,比如政府今年有7个亿是帮助新移民的,会分配到各个非营利机构,这些机构就要做一些针对新移民的服务来申请经费,同时还要有效果,如果没有政府的funding, 这个机构就要关门了。所以从事这个行业的人往往有marketing、工商管理,教师的背景,然后获得了本地学历。那么这个行业有多大呢,这个行业非常大,和商业基本上快差不多了。那些机构属于no-profit, 医院,学校,就业、安居服务机构,慈善机构,老人服务机构,绿色和平组织,环境和动物保护组织等。他的行业自成体系,你在一般的招聘网站或者agency那里很少看到招聘的信息。需要什么样的人呢,其实和商业一样,需要sales, accounting, administration, customer service, technician.不一样的地方,核心的服务人员是社工,教师,或者医务人员。大家可以去www.charityvillige.com 上去看,这是加拿大最大的非营利机构的招聘网站。
就业前景:不错
薪资水平:16-20左右。
性别:女性居多
个人奋斗方向:如果成为专业social worker, 工资可以达到70000,但是一般需要有social science master degree. 而 bachelor 毕业实际上和college毕业薪资差别不是太大,我个人觉得可以在拿到diploma后工作两年,直接申请master(用国内的学历)+diploma+工作经验,这样不错。
课程介绍
• Introduction to Community Service Worker
• Social Welfare, Diversity and Awareness
• The Family
• Couples, Family and Interpersonal Communication
• Oriented Case Management
• Environmental Issues and Crisis Interventions and
Prevention
• Bereavement and the Elderly
• Respond to Abuse
• Understanding Human Behavior
• Communication Skills
• Counseling Skills
• Report Writing for the CSW
• Field placement

By the end of the course, students will be able to:
• Demonstrate knowledge of the range of social and community service organizations in their own community as well as detailed knowledge of an agency.
• Practice the role of a team member within a social and community service organization.
• Demonstrate beginning level professional skills
• Apply theory and skills learned in the independent study components of this course.

colorful day : 2013-04-19#739
回复: 求学选择个人谈及幼教,私人护理,社工,医师行政助理等热

真是的非常实用的帖子,看来要好好的多研究才可以。是不是大专的比本科更好找工作一些呢?

wanroxy : 2013-04-25#740
回复: 求学选择个人谈及幼教,私人护理,社工,医师行政助理等热

谢谢alex。 受益匪浅

翡翠深蓝 : 2013-04-26#741
回复:

有帮助
:wdb10:

苏打小兔子 : 2013-05-15#742
回复: 求学选择个人谈及幼教,私人护理,社工,医师行政助理等热

请教!本人是马上登陆的一枚26[FONT=宋体]岁大龄女青年,男朋友在加拿大读书,马上登陆,我的目的是移民。本科国内[/FONT]211[FONT=宋体]专业国际经济与贸易,硕士在英国念得[/FONT]accounting[FONT=宋体],回国在一家给政府科技政策规划的研究所做研究和咨询[/FONT]2[FONT=宋体]年。由于不想再念不好找工作的[/FONT]university[FONT=宋体],我读博士没信心,而且也不想再走太[/FONT]academic[FONT=宋体]的路子,这条路在国外也不现实,所以想读个[/FONT]2[FONT=宋体]年的[/FONT]diploma[FONT=宋体],到时候拿三年工签。[/FONT]
[FONT=宋体][/FONT]

[FONT=宋体]但现在在选专业问题上卡壳。有基础的一直在商科,但自己最有优势的经济学在加拿大不容易就业,总不可能给我去大公司或政府当分析员吧,我也没那么高学术和写作水平;[/FONT]Global business[FONT=宋体]的话我看[/FONT]college[FONT=宋体]里面的课程也是大而空,不知道真的能学到什么实际的东西。[/FONT]
[FONT=宋体][/FONT]

会计的话我一没实际工作经验,二没任何证书。就算读2[FONT=宋体]年的accounting[FONT=宋体],我也担心没什么优势。但毕竟自己好歹有个会计硕士,最近也了解了不少关于accounting的专业,seneca有开accounting、accounting and payroll(这个有coop)两个专业。我知道payroll和HR挂钩后可能对communication skill有很大要求。自己虽然在英国读过硕士,无奈当年会计是中国班,提高并不多,雅思口语也是6.5的水平而已。[/FONT][/FONT]

[FONT=宋体]请问,我现在对自己专业定位上有什么建议吗?很迷茫了。感谢帮助![/FONT]

connie_dan : 2013-06-01#743
回复: toronto college介绍,选择,相关问题交流

资料有,但是不知道你需要什么。这个课程的学习,你需要
1、英语至少linc 5以上
2、最好有一定医学,健康,药物方面的背景
我想你需要了解的东西不仅这些,包括就业前景,方向,薪资,课程内容,学费,金融资助,学校选择等。如果你需要了解详情,可以PM我联系方式,我可以提供些建议。

lz可以发一些药剂师助理和医生psw的资料给我吗?本人现在在Montreal,国内医学院本科学历,打算下年三月左右去多伦多找学校

arch13134 : 2013-07-21#744
回复: 求学选择个人谈及幼教,私人护理,社工,医师行政助理等热

楼主,你的帖子也太多人捧场了,我分了两天才看完。

想请教一下你,是否了解petroleum engineer 这个专业如何,薪资超高呢,有十几万一年呢... 而且好像读两年就ok啦。

请问你知道容易入学和毕业吗?具体要学些什么?这种工作是不是要经常出差采油的?当我第一次听到这个专业,还以为眼花,觉得电影里面才会有,原来现实生活也真挺多人做这个的,而且还挺缺人的。

等待楼主回音,谢谢

yaya787 : 2013-07-22#745
回复: toronto college介绍,选择,相关问题交流

谢谢,课程介绍不用麻烦了,小包子可以在COLLEGE的网页上找到,你真是个好人:wdb19:
请教:学习药剂师这个专业,需要有以前的专业背景吗?而且6个月没有背景的是不是就业也会有些压力啊?

rebecca11 : 2013-08-02#746
回复: 求学选择个人谈及幼教,私人护理,社工,医师行政助理等热

有没有平面设计或者桌面输出专业的培训呢?

mandy112 : 2013-09-05#747
回复: toronto college介绍,选择,相关问题交流

介绍一下社会工作者行业
国内没有社工这个行业,不过大家应该清楚这个是干什么的,我们遇到的settlement Counselor, Employment Counselor都是属于这个行业。这个行业对于我们移民来说有一个优势,我们会1门外语,而且是目前最大的移民群体的外语,各个机构都在扩展对于中国移民的服务。这样子就需要我们了。社工为什么要拓展服务呢,大家知道,no profit机构的钱来自于政府,企业或个人捐助,自我经营收入,其中政府最多,而政府的钱是要根据你做了多少事情来得,比如政府今年有7个亿是帮助新移民的,会分配到各个非营利机构,这些机构就要做一些针对新移民的服务来申请经费,同时还要有效果,如果没有政府的funding, 这个机构就要关门了。所以从事这个行业的人往往有marketing、工商管理,教师的背景,然后获得了本地学历。那么这个行业有多大呢,这个行业非常大,和商业基本上快差不多了。那些机构属于no-profit, 医院,学校,就业、安居服务机构,慈善机构,老人服务机构,绿色和平组织,环境和动物保护组织等。他的行业自成体系,你在一般的招聘网站或者agency那里很少看到招聘的信息。需要什么样的人呢,其实和商业一样,需要sales, accounting, administration, customer service, technician.不一样的地方,核心的服务人员是社工,教师,或者医务人员。大家可以去www.charityvillige.com 上去看,这是加拿大最大的非营利机构的招聘网站。
就业前景:不错
薪资水平:16-20左右。
性别:女性居多
个人奋斗方向:如果成为专业social worker, 工资可以达到70000,但是一般需要有social science master degree. 而 bachelor 毕业实际上和college毕业薪资差别不是太大,我个人觉得可以在拿到diploma后工作两年,直接申请master(用国内的学历)+diploma+工作经验,这样不错。
课程介绍
• Introduction to Community Service Worker
• Social Welfare, Diversity and Awareness
• The Family
• Couples, Family and Interpersonal Communication
• Oriented Case Management
• Environmental Issues and Crisis Interventions and
Prevention
• Bereavement and the Elderly
• Respond to Abuse
• Understanding Human Behavior
• Communication Skills
• Counseling Skills
• Report Writing for the CSW
• Field placement

By the end of the course, students will be able to:
• Demonstrate knowledge of the range of social and community service organizations in their own community as well as detailed knowledge of an agency.
• Practice the role of a team member within a social and community service organization.
• Demonstrate beginning level professional skills
• Apply theory and skills learned in the independent study components of this course.

感谢分享!
楼主能否介绍一下多伦多哪些college学social work专业比较好的?学这个专业的master是需要相关工作经验的吗?
谢谢!

zxmm : 2013-10-25#748
回复: toronto college介绍,选择,相关问题交流

越快越好?如果申请经验移民的话,不是说需要学习2年才可以么?

hfjrchu : 2013-10-27#749
回复: toronto college介绍,选择,相关问题交流

:wdb17:

hfjrchu : 2013-10-27#750
回复: toronto college介绍,选择,相关问题交流

考diploma不用这些成绩,参加他们自己的英语测试就行。行就上课,不行也会让你上课,加学一门英语而已。


说的是公校还是私校?我看到的几个公校的program,入学都要求雅思或托福。而且真的考不过学校的语言测试也可以入学?:wdb6:

kalaviuka : 2013-12-16#751
回复: 求学选择个人谈及幼教,私人护理,社工,医师行政助理等热

楼主你好,
我现在比较关注Business Event Planning 这个专业,我在Seneca和http://www.academyoflearning.com这两个...绍,发现挺适合我,我也挺感兴趣,不知道这个专业就业市场怎样呢?薪水又如何呢?麻烦介绍一下!

Kina992 : 2013-12-18#752
回复: 求学选择个人谈及幼教,私人护理,社工,医师行政助理等热

好贴!

kalaviuka : 2013-12-23#753
回复: 求学选择个人谈及幼教,私人护理,社工,医师行政助理等热

顶起来。求回复

libochi : 2014-01-17#754
回复: 求学选择个人谈及幼教,私人护理,社工,医师行政助理等热

时间跨度够长的

libochi : 2014-03-28#755
,,,,

lelievre : 2014-03-29#756
学习学习·

david_yunke : 2014-03-30#757
细读中,有用!

liyancrawford : 2014-05-28#758
回复: toronto college介绍,选择,相关问题交流

麻烦LZ,我想问下多伦多COLLOGE里是否有质量管理方面的专业?是否好就业?我在国内做了十年的质量管理。
你好,我以前也是做质量管理,请问你这个就业怎样

lavender99 : 2014-06-03#759
牙医助理找工作会比较容易 in Canada. 牙医助理 pay is $21 per hour. The program is 10-month long.

lavender99 : 2014-06-03#760
幼教找工作会比较容易 in Canada. Level Three licence may be paid $ 20 per hour. However, the program is two year long.

lavender99 : 2014-06-03#761
私人护理找工作会比较容易 in Canada. The pay is $17 per hour. This program is 5-month long. Some colleges need 6-month long.

lavender99 : 2014-06-03#762
社工, 医师行政助理, 质量管理, CAD及ESTIMATOR, 国际商务管理 国际物流与供应链管理 不 容易找到工作 in Canada.

lavender99 : 2014-06-03#763
回复: toronto college介绍,选择,相关问题交流



那么这么certificate 和diploma找起工作来有什么区别吗??
我看到很多学校的certificate是part-time的,也就说没有入学要求,只要你可以读完十四门功课,就可以拿到这个certificate,没有什么英文十二级要求,数学十一级要求,或者托福,雅思之类的要求.
但是,diploma却有很多的要求.
从这个角度来说,是不是diploma 比certificate更加证明个人有能力呀(我是说在加拿大生活的能力,包括英文,当地文化之类的)
不知道, 这两种东西拿出来的功效是一样的吗??当然,就是为了找份工作, 不知道哪一种更好找工作呀??
现在真的迷惑了,什么是加拿大学习背景?? 一个没有入门要求的certificate是否也是可以的呀??
Certificate job is like a labor job. Diploma job is a professional job.

lavender99 : 2014-06-03#764
回复: 求学选择个人谈及幼教,私人护理,社工,医师行政助理等热门行业介绍

alexwei,请问下pharmacy technician和medical adminstrative assistant哪个专业读出来比较容易就业呢?收入高不?
谢谢
pharmacy technician 读出来比较容易就业, 收入 is $22 per hour in Alberta.

lavender99 : 2014-06-03#765
读护士应该比较好找工作 in Canada, 没钱可以申请贷款. LPN pay is $ 25. 93 per hour. LPN program is two-year long. RN pay is $34.31 per hour. RN program is 4-year long.

lavender99 : 2014-06-03#766
大专的比本科更好找工作一些 in Canada.

lavender99 : 2014-06-03#767
Petroleum Engineer 这个专业薪资超高 in Alberta, 有十万一年 only in Alberta.

dajzh : 2014-06-10#768
Petroleum Engineer 这个专业薪资超高 in Alberta, 有十万一年 only in Alberta.
您就不知道现在石油毕业都找不到工作

dajzh : 2014-06-10#769
不知道在跟帖的,有多少做过调研或者读过其中的一两个专业?

lavender99 : 2014-06-11#770
您就不知道现在石油毕业都找不到工作
It is very difficult for all 石油毕业 people to find a job in 2014. :wdb7:

Lianghj33 : 2014-06-17#771
Mark

lavender99 : 2014-06-27#772
幼教,私人护理,牙科助理,行政助理,会计专业,电工,焊工,钳工,车工,公共汽车司机,买卖房屋中介人员,保险代理,实验室分析员,消毒技师,抽血医务人员,老年公寓管理员,工厂卫生监督安全员。。。。。选择上述职业的移民和留学生都是非常客观地评价自己的实际能力和加拿大的就业现实情况的。

刘张禹彤 : 2014-07-23#773
孩子国内高二,想去加拿大读本科,想了解下物理治疗师的本科毕业是否好就业,以及薪酬如何

lavender99 : 2014-07-25#774
物理理疗师的就业前景目前是很好的。工资是每小时30刀以上的。

刘张禹彤 : 2014-07-27#775
物理治疗师全职的工作听说很少,都是临时的多,如果不能转全职,将来经验类移民有影响吧

lavender99 : 2014-07-29#776
现在和医学有关的工作,都是零时工作。先从Casual 工作干起,工作出色,慢慢熬到temporary 工作,再踏踏实实地工作,慢慢熬到permanent 工作。一步一个脚印地走。

Cally : 2014-08-05#777
好贴

lavender99 : 2014-08-09#778
我刚来加拿大的时候,华人都讨论过学习哪些专业容易找到工作。这个帖子里提到这几个专业是比较容易找到工作的。我亲眼看到身边的英语不是特别出众的移民就是学习了那些专业找到工作的。移民真的需要客观地评价自己的真实实力和水平。那些好高骛远的移民,真的是找不到专业工作的。

lyoo : 2014-08-16#779
留名

daisy13_fri : 2014-09-05#780
现在和医学有关的工作,都是零时工作。先从Casual 工作干起,工作出色,慢慢熬到temporary 工作,再踏踏实实地工作,慢慢熬到permanent 工作。一步一个脚印地走。
Lavender MM你好我想请教下关于移民后读书的student loan的一些问题。这个帖子之前来看过,没想到现在又长了这么长,而且似乎你代替楼主回答了许多园友的问题。这个帖子开头什么地方有过关于student loan的一些介绍,看起来好像申请后可以拿到不少(包括loan和bursary一起),但是我找了几个网站,包括Alberta和Ontario 的OSAP,算下了每月只有不到500刀,还是的low income的才有。不知道你是否了解相关信息或有亲身经验,可否介绍下?
谢谢!

lavender99 : 2014-10-23#781
Lavender MM你好我想请教下关于移民后读书的student loan的一些问题。这个帖子之前来看过,没想到现在又长了这么长,而且似乎你代替楼主回答了许多园友的问题。这个帖子开头什么地方有过关于student loan的一些介绍,看起来好像申请后可以拿到不少(包括loan和bursary一起),但是我找了几个网站,包括Alberta和Ontario 的OSAP,算下了每月只有不到500刀,还是的low income的才有。不知道你是否了解相关信息或有亲身经验,可否介绍下?
谢谢!
我是个特别热情的一个人。我不仅帮助华人回答一些问题。现实生活中,我也帮助黑人和穆斯林人。我帮助黑人把那些专业的学校入学条件和申请政府Funding的内容材料都打印下来,送给黑人。我不知道你目前在哪个省工作和生活。我只了解阿尔伯塔省的一些具体情况,其他省的事情不太清楚的。

daisy13_fri : 2014-10-24#782
我是个特别热情的一个人。我不仅帮助华人回答一些问题。现实生活中,我也帮助黑人和穆斯林人。我帮助黑人把那些专业的学校入学条件和申请政府Funding的内容材料都打印下来,送给黑人。我不知道你目前在哪个省工作和生活。我只了解阿尔伯塔省的一些具体情况,其他省的事情不太清楚的。
谢谢你的回复。我是即将拿到签证,准备明后年去卡城的。去后希望先读书拿个本地文凭。目前的想法是想读physical thareupy assistant,但是查下来发现学费很贵,入学似乎还不容易。还有许多细节也没弄清楚。没有签证之前也不能以新移民的身份去咨询学校,只好到处找资料瞎摸摸。将来可能还有问题请教,先谢谢你!

爱咖啡滴猫 : 2014-11-08#783
物理治疗师全职的工作听说很少,都是临时的多,如果不能转全职,将来经验类移民有影响吧
如果你说的是物理治疗师,不是助理,那么每个医院都会有很多全职的固定职位。问题是,你能申请到吗?多少土生土长的帅哥美女GPA4.0去排队申请呢!

C2C_1227 : 2014-12-29#784
如果你说的是物理治疗师,不是助理,那么每个医院都会有很多全职的固定职位。问题是,你能申请到吗?多少土生土长的帅哥美女GPA4.0去排队申请呢!

I agree it's really hard to get in any physiotheropy program. The schools usually want applicants to have a degree in life science or arts. I don't think it's worth the money and time to take the program. In the hospital I work for, most the the PT`s and OT`s started casual or on call for a year or so, then got permanent part-time positions.

In Vancouver, Ultrasound Tech. (also called Diaglostic Sonography) in BCIT is a very good program. I am on Vancouver Island. The college here doesn`t have the program, and I don`t want to move to Vancouver for it. But I know the pay is really good. Starts at $29 at hospitals. Maybe higher with private clinics but the benefits won`t cover as much as that the hospitals offer.

libochi : 2015-01-03#785
情况会好起来的!

瘦了红颜 : 2015-01-06#786
楼主新年好!
我在国内八年的教学经验,来温哥华两年多了,已通过了大学的语言学习。
BC省有个专门的certificate叫special educationteacher assistant,就业前景是在各公校及幼儿园为残疾、自闭、智障等孩子提供单独辅导。去年去上课发现自己是班里唯一的华人,讨论时语言压力非常大。后来由于身体原因系休学了。现在考虑是否继续这个专业不知楼主那边有没有对应的专业,主要的顾虑是,自己的口语水平真的好难提高,担心自己的就业前景

doremi-doremi : 2015-01-06#787
楼主新年好!
我在国内八年的教学经验,来温哥华两年多了,已通过了大学的语言学习。
BC省有个专门的certificate叫special educationteacher assistant,就业前景是在各公校及幼儿园为残疾、自闭、智障等孩子提供单独辅导。去年去上课发现自己是班里唯一的华人,讨论时语言压力非常大。后来由于身体原因系休学了。现在考虑是否继续这个专业不知楼主那边有没有对应的专业,主要的顾虑是,自己的口语水平真的好难提高,担心自己的就业前景
我对这个专业很感兴趣呢,但不知道多伦多哪里开设了这个专业,查了几个学校好像都没有诶。

昂达 : 2015-03-18#788
感谢楼主分享。码字辛苦了。

GUTU16079 : 2015-03-19#789
回复: toronto college介绍,选择,相关问题交流


专业有两个:国际商务管理 国际物流与供应链管理
毕业之后可以拿研究生毕业证和相应的专业的职业资格证:
加拿大国际贸易协会(FITT)颁发的执业资格证书CITP (Certified International Trade Professional)
加拿大采购管理协会(PMAC)颁发的执业资格证书 CPP (Certified Professional Purchaser)
想毕业后靠着证书留在加拿大,并找到一份工作
有个问题想请教一下:我有个外甥女在国内大学毕业后一直在一家国际货代公司工作,现已工作好几年了,现想到加拿大来发展。我想让她来加后继续从事国际货代的工作,初步打算也是想让她过来先到COLLEGE读一年国际货代。国际货代在加拿大叫什么?COLLEGE有这个专业吗?国际货代算不算加拿大可以移民的职业范围?先谢谢了!

pennycpy : 2015-06-18#790
楼主你好,五月底旅游签证来到多伦多。目前想要在这里读书,像你说的,“时间短、快!”也是我希望的。所以不知道目前我应该如何做,应该先找个语言学校读几个月然后再申请读书签证还是如何比较好呢?说一下我个人的情况:
1、本人女,25,毕业于国内本科院校两年,毕业之后一直从事招聘方面的工作
2、目前由于是旅游签证来的,若希望能在这里读书,可以不回中国申请读书签证吗?
3、曾找过中介,中介说可以先读语言学校,然后去college读一年,拿到工签就可以打工,日后也便于移民,不知道若我走这条路是否可以自己去联系呢?如何联系语言学校和college?
4、像我这种情况,我也不知道读什么专业会对自己日后的找工作比较有利,也希望楼主多给建议,谢谢。

laopoxuxu : 2015-06-18#791
楼主你好,五月底旅游签证来到多伦多。目前想要在这里读书,像你说的,“时间短、快!”也是我希望的。所以不知道目前我应该如何做,应该先找个语言学校读几个月然后再申请读书签证还是如何比较好呢?说一下我个人的情况:
1、本人女,25,毕业于国内本科院校两年,毕业之后一直从事招聘方面的工作
2、目前由于是旅游签证来的,若希望能在这里读书,可以不回中国申请读书签证吗?
3、曾找过中介,中介说可以先读语言学校,然后去college读一年,拿到工签就可以打工,日后也便于移民,不知道若我走这条路是否可以自己去联系呢?如何联系语言学校和college?
4、像我这种情况,我也不知道读什么专业会对自己日后的找工作比较有利,也希望楼主多给建议,谢谢。
除非你经济状况很好,否则还是在国内把雅思考出来直接申请大学,现在留学转移民难度加大了,自己多在网上找找相关信息再做打算,至于专业现在都不好找工作所以前提是选自己不厌烦的专业!

节节草 : 2015-11-17#792
题目里提到的专业是比较容易找到工作的。社工的工资比较高的。一小时是三十刀以上的。

libochi : 2015-11-17#793
:wdb37:

节节草 : 2015-11-19#794
只有在医院里工作的社工工资很高的,社工要帮助那些吸毒的患者,酗酒的患者,无家可归的患者等联系政府的相关救助单位以及帮助他们填写相应的法律文书。

mini5417 : 2017-01-15#795
看完介绍豁然开朗,感谢感谢

monkey12345 : 2017-11-01#796
很谢谢!好贴!

dandanl : 2018-07-11#797
关注!

满陇桂雨 : 2018-07-11#798
只有在医院里工作的社工工资很高的,社工要帮助那些吸毒的患者,酗酒的患者,无家可归的患者等联系政府的相关救助单位以及帮助他们填写相应的法律文书。
本科学历的社工工资高,专科的工资不高。

时艳侠 : 2018-09-25#799
回复: toronto college介绍,选择,相关问题交流

信息非常实用,谢谢/
我9月要去HUMBER读Fundraising Information and Volunteer,应该也属于non-profit机构, 时间是一年半,实际读书只有2学期,最后一学期实习, 所以从时间上讲是不错的, 但是不知道以后的工作是怎样的, 不过听说这个专业整个GTA只有HUMBER开了,所以实习期间会有很多机构上门找人,希望是这样.
另外系主任刚发了个学生名单过来,我好像是唯一的中国人,也可能是唯一的移民,所以很担心自己的英语跟不上,这个专业需要大量writing & presentation
十年后看到这个帖子,不知道现在你怎么样了!我现在就在从事慈善组织的工作,工作内容主要是筹资,现在打算去加国先读书再移民。不知可方便留个联系方式请教你相关信息?

努力前进的包子 : 2019-03-18#800
在曼省读了个文科为了移民,现在pr快下来了,准备到多伦多开始新的生活。看了楼主的信息,nursing or personal care 可能是个比较适合我的专业。我可以专门对付一个人,像幼师要对一群小孩子还要跟家长沟通真是头疼。 Nursing 这个职业父母老后来加拿大,自己更熟悉这个医疗行业,肯定会更好

奔儿 : 2019-03-18#801
不知有哪位大神比较了解社工专业呢?

满陇桂雨 : 2019-03-20#802
不知有哪位大神比较了解社工专业呢?
社工就业面比较广,无家可归收容所,受虐待妇女庇护所,问题青少年中心,戒毒中心,老人院等等。这个专业对语言要求高,学习当中各种读写,大篇大篇的论文,各种心理学生理学社会学相关知识。。。。学员基本上都是母语说英语的。华人学这个的极少,因为语言没优势。
如果对自己英文足够自信可以考虑。
这个专业收入不高,有个朋友说过,他入职近十年,找到相关工作时工资三万多,目前刚刚四万出头。
本科毕业就业面更广一些,工资还是不高。研究生毕业才有点优势,工资提高一些,可是还是和其他同等学历专业人员的工资差一截。
因为爱好可以学社工。因为钱就算了。花同样的精力学别的收入会更高。

瑄小宝 : 2019-04-15#803
想问一些楼主~就college几个来说的话,是怎样一个排名呢,我想读nursing,然后想知道george brown, centennial, humber, seneca这些哪几个好一些。。。我知道george brown现在是不收国际学生

Hui辉 : 2020-05-03#804
在曼省读了个文科为了移民,现在pr快下来了,准备到多伦多开始新的生活。看了楼主的信息,nursing or personal care 可能是个比较适合我的专业。我可以专门对付一个人,像幼师要对一群小孩子还要跟家长沟通真是头疼。 Nursing 这个职业父母老后来加拿大,自己更熟悉这个医疗行业,肯定会更好

请教下读完文科需要找到工作才申请上的PR吗?

Anshanjiangbin : 2020-06-24#805
在大学里学习设计专业,景观设计,室内设计,工业设计,哪个在加拿大好找工作